Final Exam

Ace your homework & exams now with Quizwiz!

Which of the following is/are an appropriate goal(s) for a patient with urinary incontinence? Choose all that apply. 1) Increase the intake of citrus fruits. 2) Maintain daily oral fluids to 8 to 10 servings per day. 3) Limit daily caffeine intake to less than 100 mg. 4) Engage in high-impact, aerobic exercise.

maintain daily oral fluids to 8-10 servings per day and Limit daily caffeine intake to less than 100 mg.

Transient incontinence:

short term incontinence that is expected to resolve spontaneously

Urinary retention d/t neurological problems include:

spinal cord tumors, injury, herniated disks, and viral infections involving perineal nerves.

While conducting a sexual history, the nurse asks a client about protection from sexually transmitted infections (STIs). Which question would be most appropriate for the nurse to ask?

"Have you had any sexually transmitted infections and if so, which ones?"

As a first step in teaching a woman with a spinal cord injury and quadriplegia about her sexual health, the nurse assesses her understanding of her current sexual functioning. Which statement by the client indicates she understands her current ability?

"I can participate in sexual activity but might not experience orgasm."

A 15-year-old girl visits the neighborhood clinic seeking information on "how to keep from getting pregnant." What should the nurse say in response to her request?

"Can you tell me about the precautions you're taking now?"

another word for uti:

cystitis

frequent urination at night:

nocturia

excessive urination:

polyuria

Nurses should obtain information about urinary control from all female patients. T or F

True

Pyelonephritis:

UTI infection progressing superiorly to the ureters or kidneys

Classic signs of a UTI include:

WBCs, pyurina, dysuria, urgency and frequency.

A postmenopausal patient is experiencing dyspareunia. What methods can the nurse recommend she use to diminish the discomfort?

Water-based lubricant

A 3 1/2 year-old child is found masterbating in the family living room. Which response by the parent would best address this behavior?

We do not touch ourselves in public, please go to your room if you want to continue this behavior

ANS: D A wound healing by secondary intention takes longer than one healing by primary intention. The wound is left open until it becomes filled with scar tissue. If the scarring is severe, permanent loss of function often occurs. Wounds that heal by primary intention heal quickly with minimal scarring. Scar tissue contains few pigmented cells and has a lighter color than normal skin.

Which nursing observation would indicate that a wound healed by secondary intention? a. Minimal scar tissue b. Minimal loss of tissue function c. Permanent dark redness at site d. Scarring can be severe.

What is the best technique for obtaining a sterile urine specimen from an indwelling urinary catheter? 1) Use antiseptic wipes to cleanse the meatus prior to obtaining the sample. 2) Briefly disconnect the catheter from the drainage tube to obtain the sample. 3) Withdraw urine through the port using a needleless access device. 4) Obtain the urine specimen directly from the collection bag.

Withdraw urine through the port using a needleless access device

Kidney palpation:

You cannot usually palpate the kidneys. If kidney inflammation is present, percussion of this angle produces pain.

The nurse is discussing contraception with an adolescent patient who asks the nurse: "What if I can't have an orgasm?" What is the nurse's best response? a) "A mature sexual relationship does not require a man and woman to achieve simultaneous orgasm." b) "The ability to achieve orgasm is the only indicator of a person's sexual responsiveness." c) "The larger the penis, the greater the potential for achieving orgasm." d) "Women who have multiple orgasms are promiscuous."

a) "A mature sexual relationship does not require a man and woman to achieve simultaneous orgasm." Explanation: Simultaneous orgasms, or both people attaining orgasm at the same moment, are difficult to achieve. A preoccupation with attaining simultaneous orgasms might disrupt the ultimate intimacy and satisfaction possible during coitus. Multiple orgasms are not abnormal. The size of the penis does not affect whether an orgasm will occur. Having an orgasm is not the only indicator of sexual responsiveness; some individuals may have a pleasurable sexual experience without orgasm.

Which assessment question is most likely to yield clinically meaningful data about a female client's sexual identity? a) "How do you feel about yourself as a woman?" b) "Do you find that your health allows you to enjoy a meaningful sex life?" c) "Are you satisfied with the quality of your relationships right now?" d) "Have you ever had any sexually transmitted infections in the past?"

a) "How do you feel about yourself as a woman?" Explanation: Sexual identity is a broad concept that includes, but supersedes, sexual functioning. However, it is more specific than simply asking about the quality of relationships. Asking an open-ended question about how the client feels about herself as a woman is likely to elicit important insights. Assessing the client's history of STIs does not directly address her sexual identity.

Which of the following statements, if made by an adolescent preparing for abdominal surgery, would indicate to the nurse that the client requires additional instruction? a) "I can have a hamburger and French fries as soon as I wake up." b) "When I can eat again, the best meal would be steak and orange juice." c) "I might be sick to my stomach and throw up after surgery." d) "The better I eat before surgery, the more likely I will heal."

a) "I can have a hamburger and French fries as soon as I wake up." Feedback: Oral fluid and food may be withheld until intestinal motility resumes.

The nurse assessing an adolescent's need for further information should ask which of the following questions? a) "What questions or concerns do you have about your sexual health?" b) "How many sexual partners have you had?" c) "Have you ever been diagnosed with a sexually transmitted disease?" d) "Are you involved in an intimate relationship at this time?"

a) "What questions or concerns do you have about your sexual health?" Explanation: An open-ended, nonthreatening question related to the patient's need for further information should be included while obtaining a sexual history

The nurse is preparing to provide education to a group of high school students on sexually transmitted infection (STI) prevention. The nurse knows that this age group often uses oral-genital stimulation as a way to prevent pregnancy. Which concept should the nurse make sure to convey to the group?

skin-to-skin contact can spread herpes and genital warts

A client has obtained levonorgestrel as emergency contraception. After unprotected intercourse, the client calls the clinic to ask questions about taking the contraceptives. The nurse realizes the client needs further explanation when she makes which statement?

"I can wait up to 4 days after intercourse to start taking these to prevent pregnancy."

A teenage female client tells the nurse that she is having intercourse with her boyfriend. The client asks the nurse about methods to prevent pregnancy. Which statement by the nurse would be most accurate?

"Many unintended pregnancies result from the use of less effective methods of contraception."

The nurse determines that a female client understands how to perform breast self-examination when the client states

"The best time to perform the exam is 1 week after my period."

A 17-year-old girl tells the nurse she uses tampons and asks how she can cut down on odor during her period. What is the nurse's best response?

"The tampon should be changed frequently."

A middle-aged male client is in the clinic for a routine blood pressure check. During the health history he mentions to the nurse that he is concerned that his wife is "unhappy with their love life." Which of the following question is best for the nurse to ask?

"What makes you think she is unhappy? What has happened?"

Individualized goal/outcome statements you might use to evaluate effectieness of interventions for urinary problems:

-will resume normal urination pattern by 2/12 -will discuss feelings about his urostomy -will have no visible blood in urine after 2 days on antibiotics

1. What is the most significant change in kidney function that occurs with aging? 1) Decreased glomerular filtration rate 2) Proliferation of micro blood vessels to renal cortex 3) Formation of urate crystals 4) Increased renal mass

1

7. The nurse measures the urine output of a patient who requires a bedpan to void. Which action should the nurse take first? Put on gloves and: 1) Have the patient void directly into the bedpan. 2) Pour the urine into a graduated container. 3) Read the volume with the bedpan on a flat surface at eye level. 4) Observe color and clarity of the urine in the bedpan.

1

A mother tells the nurse at an annual well-child checkup that her 6-year-old son occasionally wets himself. Which response by the nurse is appropriate? 1) Explain that occasional wetting is normal in children of this age. 2) Tell the mother to restrict her childs activities to avoid wetting. 3) Suggest time out to reinforce the importance of staying dry. 4) Inform the mother that medication is commonly used to control wetting.

1

The nurse identifies the nursing diagnosis Urinary Incontinence (Total) in an older adult patient admitted after a stroke. Urinary Incontinence places the patient at risk for which complication? 1) Skin breakdown 2) Urinary tract infection 3) Bowel incontinence 4) Renal calculi

1

The nurse measures the urine output of a patient who requires a bedpan to void. Which action should the nurse take first? Put on gloves and: 1) Have the patient void directly into the bedpan. 2) Pour the urine into a graduated container. 3) Read the volume with the container on a flat surface at eye level. 4) Observe the color and clarity of the urine in the bedpan

1

18. The nurse assesses a patient's abdomen 4 days after abdominal surgery and notes that bowel sounds are absent. This finding most likely suggest which postoperative complication? 1) Paralytic ileus 2) Small bowel obstruction 3) Diarrhea 4) Constipation

1 Absent bowel sounds on the fourth postoperative day suggests paralytic ileus, a complication associated with abdominal surgery. A small bowel obstruction and diarrhea produce hyperactive bowel sounds. Constipation might be associated with hypoactive bowel sounds. PTS:1DIF:ModerateREF:p. 972 KEY: Nursing process: Diagnosis | Client need: PHSI | Cognitive level: Analysis

5. A patient is diagnosed with an intestinal infection after traveling abroad. The nurse should encourage the intake of which food to promote healing? 1) Yogurt 2) Pasta 3) Oatmeal 4) Broccoli

1 Although the patient may have diarrhea, the goal is not to stop the diarrhea, but to eliminate the pathogens from the digestive tract. The active bacteria in yogurt stimulate peristalsis and promote healing of intestinal infections. Pasta is a low-fiber food that slows peristalsis. It does not promote healing of intestinal infections. Oatmeal stimulates peristalsis, but it does not promote healing of intestinal infections. Broccoli stimulates gas production; it is ineffective against intestinal infections. PTS: 1 DIF: Moderate REF: p. 969 KEY: Nursing process: Interventions | Client need: PHSI | Cognitive level: Application

20. A patient with severe hemorrhoids is incontinent of liquid stool. Which of the following interventions is contraindicated? 1) Apply an indwelling fecal drainage device. 2) Apply an external fecal collection device. 3) Place an incontinence garment on the patient. 4) Place a waterproof pad under the patient's buttocks.

1 An indwelling fecal drainage device is contraindicated for children; for more than 30 consecutive days of use; and for patients who have severe hemorrhoids, recent bowel, rectal, or anal surgery or injury; rectal or anal tumors; or stricture or stenosis. External devices are not typically used for patients who are ambulatory, agitated, or active in bed because the device may be dislodged, causing skin breakdown. External devices cannot be used effectively when the patient has Impaired Skin Integrity because they will not seal tightly. Absorbent products are not contraindicated for this patient unless Impaired Skin Integrity occurs. Even with absorbent products or an external collection device, the nurse should place a waterproof pad under the patient to protect the bed linens. PTS:1DIF:DifficultREF:p. 1001 KEY: Nursing process: Interventions | Client need: SECE | Cognitive level: Application

16. A mother of a school-age child seeks healthcare because her child has had diarrhea after being ill with a viral infection. The patient states that after vomiting for 24 hours, his appetite has returned. Which recommendation should the nurse make to this mother? 1) Consume a diet consisting of bananas, white rice, applesauce, and toast. 2) Drink large quantities of water regularly to prevent dehydration. 3) Take loperamide (an antidiarrheal) as needed to control diarrhea. 4) Increase the consumption of raw fruits and vegetables.

1 The nurse should encourage the patient with diarrhea who has an appetite to consume a diet that consists of bananas, white rice, applesauce, and toast. These foods are easy to digest, provide calories for energy, and help provide a source of calcium. The patient should sip liquids frequently to prevent dehydration; large quantities might worsen diarrhea. Medication, such as loperamide (Imodium), is usually reserved for chronic diarrhea. Raw fruits and vegetables may worsen diarrhea. PTS:1DIF:ModerateREF:p. 978 KEY: Nursing process: Interventions | Client need: PHSI | Cognitive level: Application

12. The nurse is instructing a patient about performing home testing for fecal occult blood. The nurse can conclude that learning occurs if the patient says, "For 3 days prior to testing, I should avoid eating: 1) Beef." 2) Milk." 3) Eggs." 4) Oatmeal."

1 The nurse should instruct the patient to avoid red meat, chicken, fish, horseradish, and certain raw fruits and vegetables for 3 days prior to fecal occult blood testing. PTS: 1 DIF: Moderate REF: pp. 989-990 KEY: Nursing process: Evaluation | Client need: PHSI | Cognitive level: Comprehension

14. Which action should the nurse take to assess a 2-year-old child for pinworms? 1) Press clear cellophane tape against the anal opening at night to obtain a specimen. 2) Collect a freshly passed stool from a diaper using a wooden specimen blade. 3) Place a smear of stool on a slide and add two drops of reagent. 4) Prepare the patient for a flat plate (x-ray) of the abdomen.

1 To assess for pinworms, the nurse should press cellophane tape against the child's anal opening during the night or as soon as he awakens. Remove the tape immediately, and place it on a slide. Perineal swabs may also be necessary for microscopic study. Collecting a fresh stool specimen from a diaper describes the method for an infant or toddler. Placing a smear of stool on a slide and adding a reagent describes fecal occult blood testing. An abdominal flat plate is not a method of assessing for pinworms. PTS: 1 DIF: Moderate REF: p. 975 KEY: Nursing process: Assessment | Client need: PHSI | Cognitive level: Comprehension

Infants & Children in urination:

1) A newborn's kidney's produce 15-60mL of urine per kilogram of body weight per day. 2) Normal specific gravity is 1.008 3) Infants do not have voluntary control of voiding because neuromuscular functioning is immature 4) Must be able to control the external sphincter

1. Which factor(s) place(s) the patient at risk for constipation? Choose all that apply. 1) Sedentary lifestyle 2) High-dose calcium therapy 3) Lactose intolerance 4) Consuming spicy foods

1, 2 Physical activity stimulates peristalsis and bowel elimination. Therefore, those with a sedentary lifestyle commonly experience constipation. High-dose calcium therapy also predisposes a patient to constipation. Lactose intolerance and consuming spicy foods are associated with a nursing diagnosis of Diarrhea, not Constipation. PTS:1DIF:ModerateREF:p. 969 KEY: Nursing process: Diagnosis | Client need: HPM | Cognitive level: Comprehension

17. Which is a key treatment intervention for the patient admitted with diverticulitis? 1) Antacid 2) Antidiarrheal agent 3) Antibiotic therapy 4) NSAIDs

3 A key treatment for diverticulitis (an infected diverticulum) is antibiotic therapy; if antibiotic therapy is ineffective, surgery may be necessary. Antacids, antidiarrheal agents, and NSAIDs are not indicated for treatment of diverticulitis. PTS:1DIF:ModerateREF:p. 970 KEY: Nursing process: Interventions | Client need: PHSI | Cognitive level: Application

Match the assessment technique to the data that should be collected. There may be more than one technique used to collect the data. 1) Auscultation 2) Inspection 3) Palpation 4) Percussion ____ 1. Skin pink, warm, and dry ____ 2. Lung sounds clear ____ 3. Abdomen is tympanic ____ 4. Abdomen soft and nontender

1. ANS: 2 PTS: 1 DIF: Moderate REF: p. 42 KEY: Nursing process: Assessment | Client need: PHSI | Cognitive level: Application 2. ANS: 1 PTS: 1 DIF: Moderate REF: p. 42 KEY: Nursing process: Assessment | Client need: PHSI | Cognitive level: Application 3. ANS: 4 PTS: 1 DIF: Moderate REF: p. 42 KEY: Nursing process: Assessment | Client need: PHSI | Cognitive level: Application 4. ANS: 3 PTS: 1 DIF: Moderate REF: p. 42 KEY: Nursing process: Assessment | Client need: PHSI | Cognitive level: Application

Match the critical thinking attitude on the left with the appropriate example on the right. 1) Reading the instruction manual of a new glucose monitoring machine 2) Asking for help with a procedure because you have not done it before 3) Obtaining the latest research about a new diagnostic procedure even though the articles are difficult to find 4) Questioning the reason for a new staffing policy 5) Realizing your feelings about alternative medicine may interfere with the care you give a patient 6) Asking a patient's feelings about his cancer diagnosis 7) Questioning your feelings when a patient's family requests withholding nutrition for a terminally ill client ____ 1. Independent thinking ____ 2. Intellectual curiosity ____ 3. Intellectual humility ____ 4. Intellectual empathy ____ 5. Intellectual courage ____ 6. Intellectual perseverance

1. ANS: 4 PTS: 1 DIF: Difficult REF: p. 26 KEY: Nursing process: N/A | Client need: SECE | Cognitive level: Application 2. ANS: 1 PTS: 1 DIF: Difficult REF: p. 26 KEY: Nursing process: N/A | Client need: SECE | Cognitive level: Application 3. ANS: 2 PTS: 1 DIF: Difficult REF: p. 26 KEY: Nursing process: N/A | Client need: SECE | Cognitive level: Application 4. ANS: 6 PTS: 1 DIF: Difficult REF: p. 26 KEY: Nursing process: N/A | Client need: SECE | Cognitive level: Application 5. ANS: 7 PTS: 1 DIF: Difficult REF: p. 26 KEY: Nursing process: N/A | Client need: SECE | Cognitive level: Application 6. ANS: 3 PTS: 1 DIF: Difficult REF: p. 26 KEY: Nursing process: N/A | Client need: SECE | Cognitive level: Application

ANS: A, B, C, D A registered dietitian is useful in working with the nurse to determine a meal plan that will support wound healing. An enterostomal or wound care nurse specializes in caring for the needs of the patient with wounds. Physical therapy is concerned about the mobility of the patient and can assist an immobile patient to progress toward mobility and decrease the risk for pressure ulcers. Pressure ulcers take a long time to heal and usually require continued therapy in the home. Case management personnel are useful in obtaining care for the patient outside the home. If the patient has a spiritual need, the chaplain can assist. If the patient has a need associated with medications, the pharmacist can assist. However, chaplains and pharmacists usually are not part of the wound care multidisciplinary team, unless a special need arises.

1. The nurse is caring for a patient with a stage II pressure ulcer and as the coordinator of care understands the need for a multidisciplinary approach. The nurse evaluates the need for several consults. Which of the following should always be included in the consults? (Select all that apply.) a. Registered dietitian b. Enterostomal and wound care nurse c. Physical therapist d. Case management personnel e. Chaplain f. Pharmacist

ANS: B Patients who are confused or disoriented or who have changing levels of consciousness are unable to protect themselves. The patient may feel the pressure but may not understand what to do to relieve the discomfort or to communicate that he or she is feeling discomfort. Impaired sensory perception, impaired mobility, shear, friction, and moisture are other predisposing factors. Shortness of breath, muscular pain, and a diet low in calories and fat are not included among the predisposing factors.

1. The nurse is working on a medical-surgical unit that has been participating in a research project associated with pressure ulcers. The nurse recognizes that the risk factors that predispose a patient to pressure ulcer development include a. A diet low in calories and fat. b. Alteration in level of consciousness. c. Shortness of breath. d. Muscular pain.

What is the normal specific gravity for urine?

1.002-1.030

Which urine specific gravity would be expected in a patient admitted with dehydration? 1) 1.002 2) 1.010 3) 1.025 4) 1.030

1.030

9. The nurse in a long-term care facility is teaching a group of residents about increasing dietary fiber. Which foods should she explain are high in fiber? 1) White bread, pasta, and white rice 2) Oranges, raisins, and strawberries 3) Whole milk, eggs, and bacon 4) Peaches, orange juice, and bananas

2 Oranges, raisins, and strawberries are high in fiber. White bread, pasta, and white rice are carbohydrates. Whole milk, eggs, and bacon are high in cholesterol. Peaches, orange juice, and bananas are sources of potassium. PTS:1DIF:ModerateREF:p. 969 KEY: Nursing process: Interventions | Client need: HPM | Cognitive level: Comprehension

ANS: D A clean surgical incision is an example of a wound with little loss of tissue that heals with primary intention. The skin edges are approximated or closed, and the risk for infection is low. Partial-thickness repairs are done on partial-thickness wounds that are shallow, involving loss of the epidermis and maybe partial loss of the dermis. These wounds heal by regeneration because the epidermis regenerates. Tertiary intention is seen when a wound is left open for several days, and then the wound edges are approximated. Wound closure is delayed until the risk of infection is resolved. A wound involving loss of tissue such as a burn or a pressure ulcer or laceration heals by secondary intention. The wound is left open until it becomes filled with scar tissue. It takes longer for a wound to heal by secondary intention; thus the chance of infection is greater.

10. The nurse is caring for a patient who has experienced a laparoscopic appendectomy. The nurse recalls that this type of wound heals by A.Tertiary intention. B.Secondary intention. C.Partial-thickness repair. D. Primary intention.

ANS: B A wound involving loss of tissue such as a burn or a pressure ulcer or laceration heals by secondary intention. The wound is left open until it becomes filled with scar tissue. It takes longer for a wound to heal by secondary intention; thus the chance of infection is greater. A clean surgical incision is an example of a wound with little loss of tissue that heals by primary intention. The skin edges are approximated or closed, and the risk for infection is low. Partial- thickness repair are done on partial-thickness wounds that are shallow, involving loss of the epidermis and maybe partial loss of the dermis. These wounds heal by regeneration because the epidermis regenerates. Tertiary intention is seen when a wound is left open for several days, and then the wound edges are approximated. Wound closure is delayed until the risk of infection is resolved.

11. The nurse is caring for a patient in the burn unit. The nurse recalls that this type of wound heals by a. Tertiary intention. b. Secondary intention. c. Partial-thickness repair. d. Primary intention.

ANS: D A hematoma is a localized collection of blood underneath the tissues. It appears as swelling, change in color, sensation, or warmth or a mass that often takes on a bluish discoloration. A hematoma near a major artery or vein is dangerous because it can put pressure on the vein or artery and obstruct blood flow. Itching of an incision site can be associated with clipping of hair, dressings, or possibly the healing process. Incisions should be approximated with edges together. After surgery, when nerves in the skin and tissues have been traumatized by the surgical procedure, it is expected that the patient would experience pain.

13. The nurse is caring for a patient who has experienced a total hysterectomy. Which nursing observation would indicate that the patient was experiencing a complication of wound healing? a. The incision site has started to itch. b. The incision site is approximated. c. The patient has pain at the incision site. d. The incision has a mass, bluish in color.

ANS: A occurs is when a wound fails to heal properly and the layers of skin and tissue separate. It involves abdominal surgical wounds and occurs after a sudden strain such as coughing, vomiting, or sitting up in bed. Patients often report feeling as though something has given way. Evisceration is seen when vital organs protrude through a wound opening. A fistula is an abnormal passage between two organs or between an organ and the outside of the body that can be characterized by chronic drainage of fluid. Infection is characterized by drainage that is odorous and purulent.

14. Which of these findings if seen in a postoperative patient should the nurse associate with dehiscence? a. Complaint by patient that something has given way b. Protrusion of visceral organs through a wound opening c. Chronic drainage of fluid through the incision site d. Drainage that is odorous and purulent

ANS: A Normal wound healing requires proper nutrition. Serum proteins are biochemical indicators of malnutrition, and serum albumin is probably the most frequently measured of these parameters. The best measurement of nutritional status is prealbumin because it reflects not only what the patient has ingested, but also what the body has absorbed, digested, and metabolized. Measurement of creatine kinase helps in the diagnosis of myocardial infarcts and has no known role in wound healing. Potassium is a major electrolyte that helps to regulate metabolic activities, cardiac muscle contraction, skeletal and smooth muscle contraction, and transmission and conduction of nerve impulses. Vitamin E is a fat-soluble vitamin that prevents the oxidation of unsaturated fatty acids. It is believed to reduce the risk of coronary artery disease and cancer. Vitamin E has no known role in wound healing.

15. A patient has developed a decubitus ulcer. What laboratory data would be important to gather? a. Serum albumin b. Creatine kinase c. Vitamin E d. Potassium

ANS: C Oxygen fuels the cellular functions essential to the healing process; the ability to perfuse tissues with adequate amounts of oxygenated blood is critical in wound healing. Blood flow through the pulmonary capillaries provides red blood cells for oxygen attachment. Oxygen diffuses from the alveoli into the pulmonary blood; most of the oxygen attaches to hemoglobin molecules within the red blood cells. Red blood cells carry oxygenated hemoglobin molecules through the left side of the heart and out to the peripheral capillaries, where the oxygen detaches, depending on the needs of the tissues. Pulse oximetry measures the oxygen saturation of blood. Assessment of muscular strength and sensation, although useful for fitness and mobility testing, does not provide any data with regard to wound healing. Sleep, although important for rest and for integration of learning and restoration of cognitive function, does not provide any data with regard to wound healing.

16. Which of the following would be the most important piece of assessment data to gather with regard to wound healing? a. Muscular strength assessment b. Sleep assessment c. Pulse oximetry assessment d. Sensation assessment

ANS: A The patient is showing signs and symptoms associated with infection in the wound. It is serious and needs treatment but is not a life-threatening emergency, where care is needed immediately or the patient will suffer long-term consequences. The nurse should complete the assessment; gather all data such as current treatment modalities, medications, vital signs including temperature, and laboratory results such as the most recent complete blood count or white cell count. The nurse can then notify the physician and receive treatment orders for the patient. It is important to notify the charge nurse and consult the wound nurse on the patient's status and on any new orders.

17. The nurse is caring for a patient with a healing stage III pressure ulcer. Upon entering the room, the nurse notices an odor and observes a purulent discharge, along with increased redness at the wound site. What is the next best step for the nurse? a. Complete the head-to-toe assessment, and include current treatment, vital signs, and laboratory results. b. Notify the charge nurse about the change in status and the potential for infection. c. Notify the physician by utilizing Situation, Background, Assessment, and Recommendation (SBAR). d. Notify the wound care nurse about the change in status and the potential for infection.

ANS: C Protein needs are especially increased in supporting the activity of wound healing. The physiological processes of wound healing depend on the availability of protein, vitamins (especially A and C), and the trace minerals of zinc and copper. A balanced diet of fat and carbohydrates, along with protein, vitamins, and minerals, is needed in any diet. Wound healing does not require increased amounts of fats or carbohydrates. Vitamin E has no known role in wound healing.

18. The nurse is collaborating with the dietitian about a patient with a stage III pressure ulcer. After the collaboration, the nurse orders a meal plan that includes increased a. Fat. b. Carbohydrates. c. Protein. d. Vitamin E.

Which daily urine output is within normal limits for a newborn weighing 8 pounds? 1) 288 mL 2) 180 mL 3) 36 mL 4) 18 mL

180 mL

ANS: C The patient's psychological response to any wound is part of the nurse's assessment. Body image changes can influence self-concept. Factors that affect the patient's perception of the wound include the presence of scars, drains, odor from drainage, and temporary or permanent prosthetic devices. The wound is odorous, and a drain is in place. The patient who is asking for a bath and change in linens gives you a clue that he or she may be concerned about the smell in the room. The patient stating that he or she wants to feel better, talking about going home, and caring about what is for dinner could be interpreted as positive statements that indicate progress along the health journey.

19. The nurse is completing an assessment on an individual who has a stage IV pressure ulcer. The wound is odorous, and a drain is currently in place. The nurse determines that the patient is experiencing issues with self-concept when the patient states which of the following? a. "I think I will be ready to go home early next week." b. "I am so weak and tired, I want to feel better." c. "I am ready for my bath and linen change as soon as possible." d. "I am hoping there will be something good for dinner tonight."

A patient is prescribed furosemide (Lasix), a loop diuretic, for treatment of congestive heart failure. The patient is at risk for which electrolyte imbalance associated with use of this drug? 1) Hypocalcemia 2) Hypokalemia 3) Hypomagnesemia 4) Hypophosphatemia

2

The nurse instructs a woman about providing a clean-catch urine specimen. Which of the following statements indicates that the patient correctly understands the procedure? 1) I will be sure to urinate into the hat you placed on the toilet seat. 2) I will wipe my genital area from front to back before I collect the specimen midstream. 3) I will need to lie still while you put in a urinary catheter to obtain the specimen. 4) I will collect my urine each time I urinate for the next 24 hours

2

2. Considering normal developmental and physical maturation in children, for which age would a goal of "Achieves bowel control by the end of this month" be most realistic? 1) 18 months 2) 3 years 3) 4 years 4) 5 years

2 Between ages 2 and 3 years, a child can typically control defecation, thereby making toilet training possible. Nevertheless, some children, especially boys, may not achieve consistent bowel control until somewhat later. PTS:1DIF:ModerateREF:pp. 968-969 KEY: Nursing process: Planning | Client need: HPM | Cognitive level: Comprehension

The nurse is teaching an older female patient how to manage urge incontinence at home. What is the first-line approach to reducing involuntary leakage of urine? 1) Insertion of a pessary 2) Intermittent self-catheterization 3) Bladder training 4) Anticholinergic medication

3

____ 1. When changing a diaper, the nurse observes that a 2-day-old infant has passed a green-black, tarry stool. What should the nurse do? 1) Notify the provider immediately. 2) Do nothing; this is normal. 3) Give the baby sterile water until the mother's milk comes in. 4) Apply a skin barrier cream to the buttocks to prevent irritation.

2 The nurse should do nothing; this is normal. During the first few days of life, a term newborn passes green-black, tarry stools known as meconium. Stools transition to a yellow-green color over the next few days. After that, the appearance of stools depends upon the feedings the newborn receives. Sterile water does nothing to alter this progression. Meconium stools are more irritating to the buttocks than other stools because they are so sticky and the skin usually must be rubbed to cleanse it. However, meconium leads to skin breakdown like a watery stool does. PTS:1DIF:ModerateREF:p. 968 KEY: Nursing process: Interventions | Client need: HPM | Cognitive level: Application

15. The nurse must irrigate the colostomy of a patient who is unable to move independently. How should the nurse position the patient for this procedure? 1) Semi-Fowler's position 2) Left side-lying position 3) Supine with the head of the bed lowered flat 4) Supine with the head of bed raised to 30 degrees

2 The nurse should position an immobile patient in a left side-lying position to irrigate his colostomy. Semi-Fowler's, supine with the bed lowered flat, and the supine position with the head of bed elevated to 30° are not appropriate positions for colostomy irrigation. PTS:1DIF:ModerateREF:p. 1008 KEY: Nursing process: Interventions | Client need: SECE | Cognitive level: Application

11. The healthcare team suspects that a patient has an intestinal infection. Which action should the nurse take to help confirm the diagnosis? 1) Prepare the patient for an abdominal flat plate. 2) Collect a stool specimen that contains 20 to 30 mL of liquid stool. 3) Administer a laxative to prepare the patient for a colonoscopy. 4) Test the patient's stool using a fecal occult test.

2 To confirm the diagnosis of an infection, the nurse should collect a liquid stool specimen that contains 20 to 30 mL of liquid stool. An abdominal flat plate and a fecal occult blood test cannot confirm the diagnosis. Colonoscopy is not necessary to obtain a specimen to confirm the diagnosis. PTS:1DIF:ModerateREF:pp. 974-975 KEY: Nursing process: Interventions | Client need: PHSI | Cognitive level: Application

3. The nurse must administer an enema to an adult patient with constipation. Which of the following would be a safe and effective distance for the nurse to insert the tubing into the patient's rectum? Choose all that apply. 1) 2 in (5.1 cm) 2) 3 in (7.6 cm) 3) 4 in (10.2 cm) 4) 5 in (12.7 cm)

2, 3 When administering an enema, the nurse should insert the tubing about 3 to 4 inches into the patient's rectum. Two inches would not be effective because it would not place the fluid high enough in the rectum. Five inches is too much. PTS:1DIF:ModerateREF:p. 994 KEY: Nursing process: Interventions | Client need: SECE | Cognitive level: Recall Completion

ANS: A Pressure is the main element that causes pressure ulcers. Three pressure-related factors contribute to pressure ulcer development: pressure intensity, pressure duration, and tissue tolerance. When the intensity of the pressure exerted on the capillary exceeds 12 to 32 mm Hg, this occludes the vessel, causing ischemic injury to the tissues it normally feeds. High pressure over a short time and low pressure over a long time cause skin breakdown. Resistance (the ability to remain unaltered by the damaging effect of something), stress (worry or anxiety), and weight (individuals of all sizes, shapes, and ages acquire skin breakdown) are not major causes of pressure ulcers.

2. The nurse is caring for a patient who was involved in an automobile accident 2 weeks ago. The patient sustained a head injury and is unconscious. The nurse is able to identify that the major element involved in the development of a decubitus ulcer is a. Pressure. b. Resistance. c. Stress. d. Weight.

ANS: A, C, D, F Normal wound healing requires proper nutrition. Oxygen and the ability to provide adequate amounts of oxygenated blood are critical for wound healing. Wound infection prolongs the inflammatory phase, delays collagen synthesis, prevents epithelialization, and decreases the production of proinflammatory cytokines, which leads to additional tissue destruction. As patients age, all aspects of wound healing are delayed. Hemorrhage and evisceration are complications of wound healing.

2. The nurse is caring for a patient with wound healing by tertiary intention. Which factors does the nurse recognize as influencing wound healing? (Select all that apply.) a. Nutrition b. Evisceration c. Tissue perfusion d. Infection e. Hemorrhage f. Age

ANS: B Because removal of dressings is painful, if often helps to give an analgesic at least 30 minutes before exposing a wound and changing the dressing. The next sequence of events includes gathering supplies for the dressing change, donning gloves, and avoiding the accidental removal of the drain during the procedure.

21. The nurse is caring for a patient on the medical-surgical unit with a wound that has a drain and a dressing that needs changing. Which of these actions should the nurse take first? a. Don sterile gloves. b. Provide analgesic medications as ordered. c. Avoid accidentally removing the drain. d. Gather supplies.

The nurse notes that a patients indwelling urinary catheter tubing contains sediment and crusting at the meatus. Which action should the nurse take? 1) Notify the provider immediately. 2) Flush the catheter tubing with saline solution. 3) Replace the indwelling urinary catheter. 4) Encourage fluids that increase urine acidity

3

ANS: B Because a drainage system needs to be patent, look for drainage flow through the tubing, as well as around the tubing. A sudden decrease in drainage through the tubing may indicate a blocked drain, and you will need to notify the physician. The hea care lth provider determines the need for drain removal and removes drains. Notifying the charge nurse, although important for communication, is not the next step in providing care for this patient. The evacuator may be compressed when a blockage is present.

22. The nurse is caring for a patient who has a wound drain with a collection device. The nurse notices that the collection device has a sudden decrease in drainage. What would be the nurse's next best step? a. Remove the drain; a drain is no longer needed. b. Call the physician; a blockage is present in the tubing. c. Call the charge nurse to look at the drain. d. As long as the evacuator is compressed, do nothing.

ANS: B A low-air-loss therapy unit is utilized for stage IV pressure ulcers and when prevention or treatment of skin breakdown is needed. If the patient has a stage III or stage IV ulcer or a postoperative myocutaneous flap, the low-air-loss therapy unit wouldbe an appropriate selection. A static air mattress or nonpowered redistribution is utilized for the patient at high risk for skin breakdown. A standard mattress is utilized for an individual who does not have actual or potential altered or impair skin integrity. Lateral rotation is used for treatment and prevention of pulmonary complications associated with mobility.

23. The nurse is caring for a patient who has a stage IV pressure ulcer awaiting plastic surgery consultation. Which of the following specialty beds would be most appropriate? a. Standard mattress b. Nonpowered redistribution air mattress c. Low-air-loss therapy unit d. Lateral rotation

ANS: C Débridement is the removal of nonviable necrotic tissue. Removal of necrotic tissue is necessary to rid the ulcer of a source of infection, to enable visualization of the wound bed, and to provide a clean base for healing. A wound will not move through the phases of healing if the wound is infected. Irrigating the wound with noncytotoxic cleaners will not damage or kill fibroblasts and healing tissue and will help to keep the wound clean once débrided. When treating a pressure ulcer, it is important to monitor and reassess the wound at least every 8 hours. Management of drainage will help keep the wound clean.

24. The nurse is caring for a patient with a pressure ulcer on the left hip. The ulcer is black. The nurse recognizes that the next step in caring for this patient includes a. Monitoring of the wound. b. Irrigation of the wound. c. Débridement of the wound. d. Management of drainage.

ANS: C Clean pressure ulcers with noncytotoxic cleansers such as normal saline, which will not kill fibroblasts and healing tissue. Cytotoxic cleansers such as Dakin's solution, acetic acid, povidone-iodine, and hydrogen peroxide can hinder the healing process and should not be utilized on clean granulating wounds. Consulting a dietitian for the nutritional needs of the patient, utilizing a low-air-loss therapy unit to decrease pressure, and applying hydrogel dressings to provide a moist environment for healing are all orders that would be appropriate.

25. The nurse is caring for a patient with a healing stage III pressure ulcer. The wound is clean and granulating. Which of the following orders would the nurse question? a. Use a low-air-loss therapy unit. b. Consult a dietitian. c. Irrigate with hydrogen peroxide. d. Utilize hydrogel dressing.

ANS: A The nurse continually assesses the skin for signs of ulcer development. Assessment of tissue pressure damage includes visual and tactile inspection of the skin. Observe pressure points such as bony prominences and areas next to treatments such as a binasal cannula and the nares. Assessment of pulses, breath sounds, and bowel sounds is part of a head-to-toe assessment and could influence the function of the body and ultimately skin integrity; however, this assessment is not a specific part of a skin assessment.

26. The nurse is completing an assessment of the skin's integrity, which includes a. Pressure points. b. All pulses. c. Breath sounds. d. Bowel sounds.

ANS: C With use of the Braden scale, the patient receives 3 for slight sensory impairment, 4 for skin being rarely moist, 3 for walks occasionally, 3 for slightly limited mobility, 4 for intake of meals, and 4 for no problem with friction and shear. The total score is

27. The nurse is completing a skin risk assessment utilizing the Braden scale. The patient has some sensory impairment and skin that is rarely moist, walks occasionally, and has slightly limited mobility, along with excellent intake of meals and no apparent problem with friction and shear. What would be the patient's Braden scale total score? a. 15 b. 17 c. 20 d. 23

ANS: B Maintaining adequate pain control and patient comfort increases the patient's willingness and ability to increase mobility, which in turn reduces pressure ulcer risks. It is good to encourage a patient to move about but even better if the patient actually sits up in the chair. Explaining the risk of immobility is important for the patient because it may impact the patient's willingness but not his or her ability. Turning the patient is important for decreasing pressure ulcers but needs to be done every 2 hours, and again does not influence the patient's ability to increase mobility.

28. The nurse is caring for a medical-surgical patient. To decrease the risk of pressure ulcers and encourage the patient's willingness and ability to increase mobility, which intervention is most important for the nurse to complete? a. Encourage the patient to sit up in the chair. b. Provide analgesic medication as ordered. c. Explain the risks of immobility to the patient. d. Turn the patient every 3 hours while in bed.

ANS: C After the assessment is completed and the information that the patient has a stage IV pressure ulcer is gathered, a diagnosis of Impaired skin integrity is selected. Readiness for enhanced nutrition would be selected for an individual with an adequate diet that could be improved. Impaired physical mobility and Chronic pain, as well as the nutrition nursing diagnosis, could well be the nursing diagnoses selected for this patient, but current data in the question strongly support Impaired skin integrity.

29. The nurse is caring for a patient with a stage IV pressure ulcer. The nurse assigns which of the following nursing diagnoses? a. Readiness for enhanced nutrition b. Impaired physical mobility c. Impaired skin integrity d. Chronic pain

The nurse is caring for a patient who underwent a bowel resection 2 hours ago. His urine output for the past 2 hours totals 50 mL. Which action should the nurse take? 1) Do nothing; this is normal postoperative urine output. 2) Increase the infusion rate of the patients IV fluids. 3) Notify the provider about the patients oliguria. 4) Administer the patients routine diuretic dose early

3

6. A nurse is teaching wellness to a women's group. The nurse should explain the importance of consuming at least how much fluid to promote healthy bowel function (assume these are 8-ounce servings)? 1) 3 to 4 servings a day 2) 5 to 6 servings a day 3) 7 to 8 servings a day 4) 9 to 10 servings a day

3 A minimum of 7 to 8 servings of fluid should be consumed each day to promote healthy bowel function. PTS:1DIF:EasyREF:p. 969 KEY: Nursing process: Interventions | Client need: HPM | Cognitive level: Recall

7. A patient with a skin infection is prescribed cephalexin (an antibiotic) 500 mg orally q 12 hours. The patient complains that the last time he took this medication, he had frequent episodes of loose stools. Which recommendation should the nurse make to the patient? 1) Stop taking the drug immediately if diarrhea develops. 2) Take an antidiarrheal agent, such as diphenoxylate. 3) Consume yogurt daily while taking the antibiotic. 4) Increase your intake of fiber until the diarrhea stops.

3 Antibiotics such as cephalexin, given to combat infection, decrease the normal flora in the colon that cause diarrhea. Bacterial populations can be maintained by encouraging the patient to consume yogurt daily while taking the drugs. Diarrhea is a common adverse effect of antibiotics; stopping the drug is not necessary. The patient should not be encouraged to take an antidiarrheal agent at this time. Increasing the intake of fiber combats constipation, not diarrhea. PTS:1DIF:ModerateREF:p. 969 KEY: Nursing process: Interventions | Client need: PHSI | Cognitive level: Application

13. The nurse is instructing a patient about performing home testing for fecal occult blood. The nurse should explain that ingestion of which substance may cause a false-negative fecal occult blood test? 1) Vitamin D 2) Iron 3) Vitamin C 4) Thiamine

3 Ingestion of vitamin C can produce a false-negative fecal occult blood test; ingestion of vitamin D, iron, and thiamine does not. Iron can lead to a false-positive result. PTS: 1 DIF: Moderate REF: pp. 989-990 KEY: Nursing process: Interventions | Client need: PHSI | Cognitive level: Comprehension

21. A patient has a colostomy in the descending (sigmoid) colon and wants to control bowel evacuation and possibly stop wearing an ostomy pouch. To help achieve this goal, nurse should teach the patient to: 1) Call the primary care provider if the stoma becomes pale, dusky, or black. 2) Limit the intake of gas-forming foods such as cabbage, onions, and fish. 3) Irrigate the stoma to produce a bowel movement on a schedule. 4) Avoid returning to the use of an ostomy appliance if he becomes ill.

3 Patients with an ostomy in the descending or sigmoid colon may use colostomy irrigation as a means to control and schedule bowel evacuation and possibly eliminate the need to wear an ostomy pouch. Limiting the intake of gas-forming foods is a good idea from a social perspective; however, it does not help achieve the goal of having regular bowel movements and thus, eliminating the need to wear a pouch. When illness occurs, it may be difficult to control the output, so the patient can use an ostomy appliance. This will not make it more difficult to schedule the bowel movements after the illness passes. PTS:1DIF:ModerateREF:p. 971 KEY: Nursing process: Interventions | Client need: SECE | Cognitive level: Application

4. Which of the following goals is appropriate for a patient with a nursing diagnosis of Constipation? The patient increases the intake of: 1) Milk and cheese. 2) Bread and pasta. 3) Fruits and vegetables. 4) Lean meats.

3 The nurse should encourage the patient to increase his intake of foods rich in fiber because they promote peristalsis and defecation, thereby relieving constipation. Low-fiber foods, such as bread, pasta, and other simple carbohydrates, as well as milk, cheese, and lean meat, slow peristalsis. PTS:1DIF:ModerateREF:p. 978 KEY: Nursing process: Planning | Client need: PHSI | Cognitive level: Application

2. A patient who has been immobile since sustaining injuries in a motor vehicle accident complains of constipation. The nurse encourages him to consume eight to ten 8-ounce servings of fluid daily. Which fluid(s) should the patient avoid because of the diuretic effect? Choose all that apply. 1) Cranberry juice 2) Water 3) Coffee 4) Ginger ale 5) Tea

3, 5 Coffee, tea, and caffeine-containing sodas should be avoided because caffeine promotes diuresis, placing the patient at further risk for constipation. Water is the preferred fluid; however, fruit juices and decaffeinated sodas are also acceptable. PTS:1DIF:ModerateREF:p. 977 KEY: Nursing process: Interventions | Client need: PHSI | Cognitive level: Comprehension

ANS: A, B, C, D Changing positions is important for decreasing the pressure associated with long periods of time in the same position. If the patient is able to feel heat or cold and is mobile, he can protect himself by withdrawing from the source. Knowing toileting habits and any potential for incontinence is important because urine and feces in contact with the skin for long periods can increase skin breakdown. Knowing whether the patient has problems with mobility such as pain will alert the nurse to any potential for decreased movement and increased risk for skin breakdown. Medications and falling are safety risk questions.

3. The nurse is completing a skin assessment on a medical-surgical patient. Which nursing assessment questions should be included in a skin integrity assessment? (Select all that apply.) a. "Can you easily change your position?" b. "Do you have sensitivity to heat or cold?" c. "How often do you need to use the toilet?" d. "Is movement painful?" e. "What medications do you take?" f. "Have you ever fallen?"

ANS: B The presence and duration of moisture on the skin increase the risk of ulcer formation by making it susceptible to injury. Moisture can originate from wound drainage, excessive perspiration, and fecal or urinary incontinence. Bacteria and enzymes in the stool can enhance the opportunity for skin breakdown because the skin is moistened and softened, causing maceration. Eating a balanced diet is important for nutrition, but eating just two thirds of the meal does not indicate that the individual is at risk. A raised red rash on the leg again is a concern and can affect the integrity of the skin, but it is located on the shin, which is not a high-risk area for skin breakdown. Pressure can influence capillary refill, leading to skin breakdown, but this capillary response is within normal limits.

3. Which nursing observation would indicate that the patient was at risk for pressure ulcer formation? a. The patient ate two thirds of breakfast. b. The patient has fecal incontinence. c. The patient has a raised red rash on the right shin. d. The patient's capillary refill is less than 2 seconds.

ANS: A The area on the heel has experienced a decreased supply of blood and oxygen (tissue perfusion), which has resulted in tissue damage. The most appropriate nursing diagnosis with this information is Ineffective tissue perfusion. Risk for infection, Acute pain, and Imbalanced nutrition may be part of this patient's nursing diagnosis, but the data provided do not support this nursing diagnosis.

30. The nurse has collected the following assessment data: right heel with reddened area that does not blanch. What nursing diagnosis would the nurse assign? a. Ineffective tissue perfusion b. Risk for infection c. Imbalanced nutrition: less than body requirements d. Acute pain

C The number one way to decrease the risk of infection by breaking the chain of infection is to wash hands. Encouraging fluid and food intake helps with overall wellness and wound healing, especially protein, but an increase in carbohydrates and fats does not relate to the risk of infection. If the patient will be discharged before the wound is healed, the family will certainly need education on how to care for the patient. Teaching the family how to manage the odor associated with a wound is certainly important, but these interventions do not directly relate to the risk of infection and breaking the chain of the infectious process.

31. The nurse is caring for a patient with a stage III pressure ulcer. The nurse has assigned a nursing diagnosis of Risk for infection. Which intervention would be most important for this patient? a. Teach the family how to manage the odor associated with the wound. b. Discuss with the family how to prepare for care of the patient in the home. c. Encourage thorough handwashing of all individuals caring for the patient. d. Encourage increased quantities of carbohydrates and fats.

ANS: B Assessment and a plan for the patient to optimize the diet are essential. Adequate calories, protein, vitamins, and minerals promote wound healing. The nurse is the coordinator of care, and collaborating with the dietitian would result in planning the best meals for the patient. The respiratory therapist can be consulted when a patient has issues with the respiratory system. Case management can

32. The medical-surgical acute care patient has received a nursing diagnosis of Impaired skin integrity. The nurse consults a a. Respiratory therapist. b. Registered dietitian. c. Chaplain. d. Case manager.

ANS: C Because the patient has an open wound and the skin is no longer intact to protect the tissue, the patient is at increased risk for infection. The nurse will be assessing the patient for signs and symptoms of infection, including an increase in temperature, an increase in white count, and odorous and purulent drainage from the wound. The patient is unconscious and is unable to communicate the signs and symptoms of infection; also, this is an intervention, not a goal for this diagnosis. It is important for the patient's family to be able to demonstrate how to care for the wound and wash their hands, but these statements are interventions, not goals or outcomes for this nursing diagnosis.

33. The nurse is caring for a patient with a stage II pressure ulcer and has assigned a nursing diagnosis of Risk for infection. The patient is unconscious and bedridden. The nurse is completing the plan of care and is writing goals for the patient. What is the best goal for this patient? a. The patient's family will demonstrate specific care of the wound site. b. The patient will state what to look for with regard to an infection. c. The patient will remain free of an increase in temperature and of odorous or purulent drainage from the wound. d. The patient's family members will wash their hands when visiting the patient.

Heat causes vasodilatation and is used to improve blood flow to an injured body part. The application of heat incorrectly when the treatment is too hot, or is applied too long or to the wrong place, can result in a burn for the patient and risk for additional trauma. The skin already has impaired integrity owing to the surgical procedure, and because of this has been at risk for infection since the surgical procedure was performed. This patient is of childbearing age and has had a child. Additional needs for nutrition are present during pregnancy and breastfeeding, but this is an established nursing diagnosis. Data are insufficient to support the nursing diagnosis of Imbalanced nutrition.

34. The nurse is caring for a postpartum patient. The patient has an episiotomy after experiencing birth. The physician has ordered heat to treat this condition, and the nurse is providing this treatment. This patient is at risk for a. Infection. b. Impaired skin integrity. c. Trauma. d. Imbalanced nutrition.

ANS: B Clean dressings as opposed to sterile dressings are recommended for home use. This recommendation is in keeping with principles regarding nosocomial infection, and it takes into account the expense of sterile dressings and the dexterity required for application. The caregiver can use the same no touch technique for dressing changes that is used for changing surface dressings without touching the wound or the surface that might come in contact with the wound. Double bagging is required for the disposal of contaminated dressings. The dressings go in a bag, which is fastened and then placed in the household trash. The ability of the caregiver certainly is a component of the success of home treatment, but it does not influence the cost of supplies.

35. The home health nurse is caring for a patient with impaired skin integrity in the home. The nurse is reviewing dressing changes with the caregiver. Which intervention assists in managing the expenses associated with long-term wound care? a. Sterile technique b. Clean dressings and no touch technique c. Double bagging of contaminated dressings d. Ability of the caregiver

ANS: A Assessment and skin hygiene are two initial defenses for preventing skin breakdown. Avoid soaps and hot water when cleansing the skin. Use gentle cleansers with nonionic surfactants. After bathing, make sure to dry the skin completely, and apply moisturizer to keep the epidermis well lubricated. Absorbent pads and garments are controversial and should be considered only when other alternatives have been exhausted. Positioning the patient reduces pressure and shearing force to the skin and is part of the plan of care but is not one of the initial components. Depending on the needs of the patient, a specialty bed may be needed, but again, this does not provide the initial defense for skin breakdown.

36. The nurse is caring for a patient who has suffered a stroke and has residual mobility problems. The patient is at risk for skin impairment. Which initial interventions should the nurse select to decrease this risk? a. Gentle cleaners and thorough drying of the skin b. Absorbent pads and garments c. Positioning with use of pillows d. Therapeutic beds and mattresses

ANS: C When patients are able to sit up in a chair, make sure to limit the amount of time to 2 hours or less. The chair sitting time should be individualized. In the sitting position, pressure on the ischial tuberosities is greater than in a supine position. Utilize foam, gel, or an air cushion to distribute weight. Longer than 2 hours can increase the chance of ischemia.

37. The nurse is caring for a patient who is at risk for skin impairment. The patient is able to sit up in a chair. The nurse includes this intervention in the plan of care. How long should the nurse schedule the patient to sit in the chair? a. At least 3 hours b. Not longer than 30 minutes c. Less than 2 hours d. As long as the patient remains comfortable

ANS: C When repositioning the patient, obtain assistance and utilize a transfer sliding board under the patient's body to prevent dragging the patient on bed sheets and placing the patient at high risk for shearing and friction injuries. The patient should be placed in a 30-degree lateral position, not supine position. The head of the bed should be elevated less than 30 degrees to prevent pressure ulcer development from shearing forces.

38. The nurse is caring for a patient who is immobile and is at risk for skin impairment. The plan of care includes turning the patient. What is the best method for repositioning the patient? a. Obtain assistance and use the drawsheet to place the patient into the new position. b. Place the patient in a 30-degree supine position. c. Utilize a transfer sliding board and assistance to slide the patient into the new position. d. Elevate the head of the bed 45 degrees.

ANS: A Stage I intact pressure ulcers that resolve slowly without epidermal loss over 7 to 14 days do not require a dressing. This allows visual inspection and monitoring. A transparent dressing could be used to protect the patient from shear but cannot be used in the presence of excessive moisture. A composite film, hydrocolloid, or hydrogel can be utilized on a clean stage II. A hydro colloid, hydrogel covered with foam, calcium alginate, gauze, and growth factors can be utilized with a clean stage III. Hydrogel calcium , alginate, gauze, and growth factors can be utilized with a clean stage IV. An unstageable wound cover with eschar should utilize a dressing of adherent film or gauze with an ordered solution of enzymes. In rare cases when eschar is dry and intact, no dressing is used, but this is an unstaged ulcer.

39. The nurse is staffing a medical-surgical unit that is assigned most of the patients with pressure ulcers. The nurse has become competent in the care of pressure wounds and recognizes that a staged pressure ulcer that does not require a dressing is stage a. I. b. II. c. III. d. IV.

.A patient is admitted with high BUN and creatinine levels, low blood pH, and elevated serum potassium level. Based on these laboratory findings the nurse suspects which diagnosis? 1) Cystitis 2) Renal calculi 3) Enuresis 4) Renal failure

4

A patient complains that she passes urine whenever she sneezes or coughs. How should the nurse document this complaint in the patients healthcare record? 1) Transient incontinence 2) Overflow incontinence 3) Urge incontinence 4) Stress incontinence

4

10. The nurse is assessing a patient who underwent bowel resection 2 days ago. As she auscultates the patient's abdomen, she notes low-pitched, infrequent bowel sounds. How should she document this finding? 1) Hyperactive bowel sounds 2) Abdominal bruit sounds 3) Normal bowel sounds 4) Hypoactive bowel sounds

4 Hypoactive bowel sounds are low-pitched, infrequent, and quiet. An abdominal bruit is a hollow, blowing sound found over an artery, such as the iliac artery. Normal bowel sounds are high pitched with approximately 5 to 35 gurgles occurring every minute. Hyperactive bowel sounds are very high pitched and more frequent than normal bowel sounds. PTS:1DIF:DifficultREF:p. 972 KEY: Nursing process: Assessment | Client need: PHSI | Cognitive level: Application

8. Which collaborative interventions will help prevent paralytic ileus in a patient who underwent right hemicolectomy for colon cancer? 1) Administer morphine 4 mg IV every 2 hours for pain. 2) Administer IV fluids at 125 mL/hr. 3) Insert an indwelling urinary catheter to monitor I&O. 4) Keep the patient NPO until bowel sounds return.

4 Patients who require bowel surgery typically remain NPO until peristalsis returns, helping to prevent paralytic ileus, a complication that can occur after the bowel is surgically manipulated. Administering morphine promotes comfort but may increase the risk of ileus. Administering IV fluids prevents dehydration but does not directly prevent ileus. Inserting an indwelling urinary catheter prevents urine retention and facilitates monitoring postoperative urine output. PTS: 1 DIF: Moderate REF: p. 969; not stated directly in text KEY: Nursing process: Interventions | Client need: PHSI | Cognitive level: Application

3. The nurse educates a patient about the primary risk factors for irritable bowel syndrome. Which behavior by the patient would be evidence of learning? The patient: 1) Reduces her intake of gluten-containing products. 2) Does not consume foods that contain lactose. 3) Consumes only two servings of caffeinated beverages per day. 4) Takes measures to reduce her stress level.

4 Stress is a primary factor in the development of irritable bowel syndrome. Other risk factors include caffeine consumption and lactose intolerance; however, they are not primary risk factors. Celiac disease is associated with gluten intake. PTS:1DIF:ModerateREF:p. 969 KEY: Nursing process: Evaluation | Client need: PHSI | Cognitive level: Application

19. A patient with a colostomy complains to the nurse, "I am noticing really bad odors coming from my pouch." To help control odor, which foods should the nurse advise him to consume? 1) White rice and toast 2) Tomatoes and dried fruit 3) Asparagus and melons 4) Yogurt and parsley

4 Yogurt, cranberry juice, parsley, and buttermilk may help control odor. White rice and toast (also bananas and applesauce) help control diarrhea. Asparagus, peas, melons, and fish are known to cause odor. Tomatoes, pears, and dried fruit are high-fiber foods that might cause blockage in a patient with an ostomy. PTS:1DIF:ModerateREF:p. 987 KEY: Nursing process: Interventions | Client need: SECE | Cognitive level: Application

4. The nurse is caring for a patient with potential skin breakdown. Which components would the nurse include in the skin assessment? (Select all that apply.) a. Mobility b. Hyperemia c. Induration d. Blanching e. Temperature of skin f. Nutritional status

4. The nurse is caring for a patient with potential skin breakdown. Which components would the nurse include in the skin assessment? (Select all that apply.) a. Mobility b. Hyperemia c. Induration d. Blanching e. Temperature of skin f. Nutritional status

ANS: B Explaining the procedure educates the patient regarding the dressing change and involves him in his care, thereby allowing the patient some control in decreasing anxiety. Telling the patient to close his eyes and turning on the television are distractions that do not usually decrease a patient's anxiety. If the family is a support system, asking support systems to leave the room ca actually n increase a patient's anxiety.

40. The nurse is caring for a patient with a wound. The patient appears anxious as the nurse is preparing to change the dressing. What should the nurse do to decrease the patient's anxiety? a. Tell the patient to close his eyes. b. Explain the procedure. c. Turn on the television. d. Ask the family to leave the room.

ANS: C Cleanse surgical or traumatic wounds by applying noncytotoxic solution with sterile gauze or irrigations. Cleanse in a direction from the least contaminated area. Use gentle friction when applying solutions to the skin, and allow irrigation to flow from the least to the most contaminated area.

41. The nurse is cleansing a wound site. As the nurse administers the procedure, what intervention should be included? a. Allowing the solution to flow from the most contaminated to the least contaminated Scrubbing vigorously when applying solutions to the skin Cleansing in a direction from the least contaminated area Utilizing clean gauge and clean gloves to cleanse a site b. c. d.

43. The nurse is caring for a postoperative medial meniscus repair of the right knee. To assist with pain management following the procedure, which intervention should the nurse implement? a. Monitor vital signs every 15 minutes. b. Apply brace to right knee. c. Elevate right knee and apply ice. d. Check pulses in right foot.

43. The nurse is caring for a postoperative medial meniscus repair of the right knee. To assist with pain management following the procedure, which intervention should the nurse implement? a. Monitor vital signs every 15 minutes. b. Apply brace to right knee. c. Elevate right knee and apply ice. d. Check pulses in right foot.

ANS: D The Braden scale is composed of six subscales: sensory perception, moisture, activity, mobility, nutrition, and friction and shear. The total score ranges from 6 to 23, and a lower total score indicates a higher risk for pressure ulcer development. The cutoff score for onset of pressure ulcer risk with the Braden scale in the general adult population is 18. The best sign is a perfect score of 23.

44. The patient has been provided a nursing diagnosis of Risk for skin impairment and has a 15 on the Braden scale upon admission. The nurse has implemented interventions for this nursing diagnosis. Upon reassessment, which Braden score would be the best sign that the risk for skin breakdown is decreasing? a. 12 b. 13 c. 20 d. 23

Average pH of urine:

5-9 average 6

Match the type of special needs assessment with the correct example. 1) Do you perform monthly breast self-exams? 2) Do you live near any industrial manufacturing plants? 3) Who can you talk to when you feel sad? 4) Who do you live with? 5) What is your understanding of your diet? 6) Since you had your stroke, have you had any problems dressing yourself? 7) Do you have a religious preference? ____ 5. Community ____ 6. Family ____ 7. Functional ability ____ 8. Nutrition ____ 9. Psychosocial ____ 10. Wellness ____ 11. Spiritual

5. ANS: 2 PTS: 1 DIF: Easy REF: pp. 43-44 KEY: Nursing process: Assessment | Client need: SECE | Cognitive level: Application 6. ANS: 4 PTS: 1 DIF: Easy REF: pp. 43-44 KEY: Nursing process: Assessment | Client need: SECE | Cognitive level: Application 7. ANS: 6 PTS: 1 DIF: Easy REF: pp. 43-44 KEY: Nursing process: Assessment | Client need: SECE | Cognitive level: Application 8. ANS: 5 PTS: 1 DIF: Easy REF: pp. 43-44 KEY: Nursing process: Assessment | Client need: SECE | Cognitive level: Application 9. ANS: 3 PTS: 1 DIF: Easy REF: pp. 43-44 KEY: Nursing process: Assessment | Client need: SECE | Cognitive level: Application 10. ANS: 1 PTS: 1 DIF: Easy REF: pp. 43-44 KEY: Nursing process: Assessment | Client need: SECE | Cognitive level: Application 11. ANS: 7 PTS: 1 DIF: Moderate REF: pp. 43-44 KEY: Nursing process: Assessment | Client need: SECE | Cognitive level: Recall

ANS: B This would be a stage II pressure ulcer because it presents as partial-thickness skin loss involving epidermis, dermis, or both. The ulcer is superficial and presents clinically as an abrasion, blister, or shallow crater. Stage I is intact skin with nonblanchable redness over a bony prominence. With a Stage III pressure ulcer, subcutaneous fat may be visible, but bone, tendon, and muscles are not exposed. Stage IV involves full-thickness tissue loss with exposed bone, tendon, or muscle.

5. The nurse is admitting an older patient from a nursing home. During the assessment, the nurse notes a shallow open ulcer without slough on the right heel of the patient. This pressure ulcer would be staged as stage a. I. b. II. c. III. d. IV.

ANS: A, B, C, D Before applying a bandage or a binder, the nurse has several responsibilities. The nurse would need to inspect the skin for abrasions, edema, and discoloration or exposed wound edges. The nurse also is responsible for covering exposed wounds or open abrasions with a sterile dressing and assessing the condition of underlying dressings and changing if soiled, as well as assessing the skin of underlying areas that will be distal to the bandage. This checks for signs of circulatory impairment, so that a comparison can be made after bandages are applied. Marking the sites of all abrasions is not necessary. Although it is important for the skin to be clean, and even though it may need to be cleaned with a noncytotoxic cleanser, cleansing with hydrogen peroxide can interfere with wound healing.

5. The nurse is caring for a patient who will have both a large abdominal bandage and an abdominal binder. The nurse's responsibilities and activities before applying the bandage and binder include which of the following? (Select all that apply.) a. Inspecting the skin for abrasions and edema b. Covering exposed wounds c. Assessing condition of current dressings d. Assessing the skin at underlying areas for circulatory impairment e. Marking the sites of all abrasions f. Cleansing the area with hydrogen peroxide

The kidneys produce urine at a rate of?

50-60mL p/hour OR 1500mL per day (may fluctuate to 1000-2000mL)

The kidneys produce urine at approximately?

50-60mL per hour (1500 per day)

An average normal urinary bladder can store how much urine?

500mL (1 pint)

ANS: D When assessing a patient with darkly pigmented skin, proper lighting is essential to accurately complete the first step in assessment—inspection—and the whole assessment process. Natural light or a halogen light is recommended. Fluorescent light sources can produce blue tones on darkly pigmented skin and can interfere with an accurate assessment. Other items that could possibly be used during the assessment include gloves for infection control, a disposable measuring device to measure the size of the wound, and a cotton-tipped applicator to measure the depth of the wound, but these items not the first item used.

6. The nurse is completing a skin assessment on a patient with darkly pigmented skin. Which of the following would be used first to assist in staging an ulcer on this patient? a. Cotton-tipped applicator b. Disposable measuring tape c. Sterile gloves d. Halogen light

ANS: B, D, E, F Optimal outcomes are to prevent injury to skin and tissues, reduce injury to skin, reduce injury to underlying tissues, and restore skin integrity. Asking the patient's perceptions and whether expectations are being met allows one to obtain information regarding the experience, but these are not actual measurable outcomes.

6. The nurse is updating the plan of care for a patient with a stage III pressure ulcer and a nursing diagnosis of Impaired skin integrity. Which of the following outcomes when met indicate progression toward goals? (Select all that apply.) a. Ask whether patient's expectations are being met. b. Prevent injury to the skin and tissues. c. Obtain the patient's perception of interventions. d. Reduce injury to the skin. e. Reduce injury to the underlying tissues. f. Restore skin integrity.

ANS: C Pressure ulcers are full-thickness wounds that extend into the dermis and heal by scar formation because the deeper structures do not regenerate, hence the need for full-thickness repair. The full-thickness repair has three phases: inflammatory, proliferative, and remodeling. A wound heals by primary intention when wounds such as surgical wounds have little tissue loss; the skin edges are approximated or closed, and the risk for infection is low. Partial-thickness repairs are done on partial-thickness wounds that are shallow, involving loss of the epidermis and maybe partial loss of the dermis. These wounds heal by regeneration because the epidermis regenerates. Tertiary intention is seen when a wound is left open for several days, and then the wound edges approximated. Wound closure is delayed until risk of infection is resolved. DIF: Remember REF: 1181-1183 OBJ: Discuss the normal process of wound healing.

7. The nurse is caring for a patient with a stage IV pressure ulcer. The nurse recalls that a pressure ulcer takes time to heal and is an example of a. Primary intention. b. Partial-thickness wound repair. c. Full-thickness wound repair. d. Tertiary intention.

Match the terms from the critical thinking model in your text with the correct example. 1) I wonder if my values about quality of life have affected my thinking. 2) What should I have done differently? 3) I need to talk with the client to make sure the family gave me the correct information. 4) I have been through a situation like this before. 5) There are several interventions that would work in this situation. 6) I need to follow the steps in the procedure manual. ____ 7. Contextual awareness ____ 8. Inquiry ____ 9. Considering alternatives ____ 10. Analyzing assumptions ____ 11. Reflecting skeptically

7.ANS:4PTS:1DIFgrinifficult REF: p. 28-29; High-level question, answer not stated verbatim KEY: Nursing process: N/A | Client need: SECE | Cognitive level: Analysis 8.ANS:3PTS:1DIFgrinifficult REF: p. 28-29; High-level question, answer not stated verbatim KEY: Nursing process: N/A | Client need: SECE | Cognitive level: Analysis 9.ANS:5PTS:1DIFgrinifficult REF: p. 28-29; High-level question, answer not stated verbatim KEY: Nursing process: N/A | Client need: SECE | Cognitive level: Analysis 10.ANS:1PTS:1DIFgrinifficult REF: p. 28-29; High-level question, answer not stated verbatim KEY: Nursing process: N/A | Client need: SECE | Cognitive level: Analysis 11.ANS:2PTS:1DIFgrinifficult REF: p. 28-29; High-level question, answer not stated verbatim KEY: Nursing process: N/A | Client need: SECE | Cognitive level: Analysis

Normal ranges for BUN (blood, urea, nitrogen) and Creatinine:

8-20mg BUN 0.5-1.1mg Creatinine Levels may be increased in: renal failure, kidney obstruction or inflammation, dehydration, excessive protein intake

ANS: A A partial-thickness wound repair has three compartments: the inflammatory response, epithelial proliferation and migration, and re-establishment of the epidermal layers. Epithelial proliferation and migration start at all edges of the wound, allowing for quick resurfacing. Epithelial cells begin to migrate across the wound bed soon after the wound occurs. A wound left open to air resurfaces within 6 to 7 days, whereas a wound that is kept moist can resurface in 4 days. One or 2 days is too soon for this process to occur, moist or dry.

8. The nurse is caring for a patient with a large abrasion from a motorcycle accident. The nurse recalls that if the wound is kept moist, it can resurface in _____ day(s). a. 4 b. 2 c. 1 d. 7

ANS: C Granulation tissue is red, moist tissue composed of new blood vessels, the presence of which indicates progression toward healing. Soft yellow or white tissue is characteristic of slough—a substance that needs to be removed for the wound to heal. Black or brown necrotic tissue is called eschar, which also needs to be removed for a wound to heal. Purulent drainage is indicative of an infection and will need to be resolved for the wound to heal.

9. The nurse is caring for a patient who is experiencing a full-thickness repair. The nurse would expect to see which of the following in this type of repair? a. Eschar b. Slough c. Granulation d. Purulent drainage

What is the resulting problem of older adults (50-80yo) functioning nephron decrease?

A decline in filtration rate which affects the ability to dilute and concentrate urine, does not normally crease problems unless an illness alters fluid balance.

ANS: A After determining that a patient's condition is stable, inspect the wound for bleeding. An abrasion will have limited bleeding, a laceration can bleed more profusely, and a puncture wound bleeds in relation to the size and depth of the wound. Address any bleeding issues. Inspect the wound for foreign bodies; traumatic wounds are dirty and may need to be addressed. Determine the size of the wound. A large open wound may expose bone or tissue and be protected, or the wound may need suturing. When the wound is caused by a dirty penetrating object, determine the need for a tetanus vaccination.

A patient presents to the emergency department with a laceration of the right forearm caused by a fall. After determining that the patient is stable, the next best step is to a. Inspect the wound for bleeding. b. Inspect the wound for foreign bodies. c. Determine the size of the wound. d. Determine the need for a tetanus antitoxin injection.

A female nurse is giving a complete bed bath to a young male patient. She notices the patient has an erection. What should she do? A) Nothing; an erection is not under voluntary control. B) Report and document the incident as sexual harassment. C) Share the information to protect other staff. D) Tell the patient she is offended by his physical response.

A) Nothing; an erection is not under voluntary control.

Parents of a toddler express concern because he is touching his genitals. What should the nurse teach the parents? A) Self-manipulation of genitals is normal behavior. B) Have the child wear clothes that prohibit touching. C) If this bad behavior continues, seek counseling. D) Make him have time out every time it happens.

A) Self-manipulation of genitals is normal behavior.

What term is used to describe painful intercourse? A) dyspareunia B) dysmenorrhea C) impotence D) vulvodynia

A) dyspareunia

What do most nursing interventions pertaining to sexuality involve? A) teaching to promote sexual health B) examinations to identify sexually transmitted infections C) advocacy for those with sexual dysfunctions D) maintaining confidentiality and privacy

A) teaching to promote sexual health

A nurse is educating a preoperative client on how to cough effectively. What can the nurse tell the client to do to facilitate coughing A. "Hold a pillow or folded bath blanket over the incision." B. "Get up and walk before you try to cough." C. "It would be best if you do not cough until you feel better." D. "When you cough, cover your nose and mouth with a tissue."

A. "Hold a pillow or folded bath blanket over the incision." Feedback: Because postoperative coughing is often painful, the client should be taught how to splint the incision by supporting it with a pillow or folded bath blanket.

In order to prevent the possibility of venous stasis, a nurse is teaching a surgical client how to perform leg exercises. Which of the client's following statements indicates a sound understanding of leg exercises? A. "I'll practice these now and try to start them as soon as I can after my surgery." B. "I'll try to do these lying on my stomach so that I can bend my knees more fully." C. "I'll make sure to do these, as long as my doctor doesn't tell me to stay on bed rest after my operation." D. "I'm pretty sure my stomach muscles are strong enough to lift both of my legs off the bed at the same time."

A. "I'll practice these now and try to start them as soon as I can after my surgery." Feedback: Leg exercises should be begun as soon as possible after surgery, unless contraindications exist. Bed rest does not preclude the performance of leg exercises and the legs should be lifted individually, not simultaneously. The client should perform leg exercises in a semi-Fowler's, not prone, position.

Which of the following nursing interventions occurs in the postoperative phase of the surgical experience? A. Airway/oxygen therapy/pulse oximetry B. Teaching deep breathing exercises C. Reviewing the meaning of p.r.n. orders for pain medications D. Putting in IV lines and administering fluids

A. Airway/oxygen therapy/pulse oximetry Feedback: Airway/oxygen therapy/pulse oximetry occur in the postanesthesia unit in the postoperative phase. Teaching deep-breathing exercises and reviewing the meaning of p.r.n. orders for medications occur in the preoperative phase. Putting in IV lines and administering fluids occurs in the intraoperative phase.

A client returning to the floor after orthopedic surgery is complaining of nausea. The nurse is aware that an appropriate intervention is to do which of the following? A. Avoid strong smelling foods. B. Provide clear liquids with a straw. C. Avoid oral hygiene until the nausea subsides. D. Hold all medications.

A. Avoid strong smelling foods. Feedback: Nursing care for a client with nausea includes avoiding strong smelling foods, providing oral hygiene, administering prescribed medications (especially medications ordered for nausea and vomiting), and avoiding use of a straw.

The nurse is providing education to a client regarding pain control after surgery. What time does the nurse inform the client is the best time to request pain medication? A. Before the pain becomes severe B. When the client experiences a pain rating of "10" on a 1-to-10 pain scale C. When there is no pain, but it is time for the medication to be administered D. After the pain becomes severe and relaxation techniques have failed

A. Before the pain becomes severe Feedback: If a pain medication is ordered p.r.n., the client should be instructed to ask for the medication before the pain becomes severe.

The telemetry unit nurse is reviewing laboratory results for a client who is scheduled for an operative procedure later in the day. The nurse notes on the laboratory report that the client has a serum potassium level of 6.5 mEq/L, indicative of hyperkalemia. The nurse informs the physician of this laboratory result because the nurse recognizes hyperkalemia increases the client's operative risk for which of the following? A. Cardiac problems B. Infection C. Bleeding and anemia D. Fluid imbalances

A. Cardiac problems Feedback: Hyperkalemia or hypokalemia increases the client's risk for cardiac problems. A decrease in the hematocrit and hemoglobin level may indicate the presence of anemia or bleeding. An elevated white blood cell count occurs in the presence of infection. Abnormal urine constituents may indicate infection or fluid imbalances.

The nurse is preparing to send a client to the operating room for an exploratory laparoscopy. The nurse recognizes that there is no informed consent for the procedure on the client's chart. The nurse informs the physician who is performing the procedure. The physician asks the nurse to obtain the informed consent signature from the client. What is the nurse's best action to the physician's request? A. Inform the physician that it is his or her responsibility to obtain the signature. B. Obtain the signature and ask another nurse to cosign the signature. C. Inform the physician that the nurse manager will need to obtain the signature. D. Call the house officer to obtain the signature.

A. Inform the physician that it is his or her responsibility to obtain the signature. Feedback: The responsibility for securing informed consent from the client lies with the person who will perform the procedure. The nurse's best action is to inform the physician that it is his or her responsibility to obtain the signature.

A nurse in an outpatient surgical center is teaching a client about what will be necessary for discharge to home. What information should the nurse include about transportation? A. The client is not allowed to drive a car home. B. If the client is not dizzy, driving a car is allowed. C. Only adults over the age of 25 may drive home. D. None; this is not necessary information.

A. The client is not allowed to drive a car home. Feedback: After outpatient surgery, clients may go home when they are no longer dizzy or drowsy, have stable vital signs, and have voided. Clients are not allowed to drive a car home.

What is wrong with the following diagnostic statement? "Impaired Physical Mobility related to laziness and not having appropriate shoes." The statement is 1) Judgmental 2) Too complex 3) Legally questionable 4) Without supportive data

ANS: 1 "Lazy" implies criticism of the client and therefore is judgmental. There need to be several (certainly more than two) etiological factors for the statement to be complex. There is no blame implied or harm resulting, so the statement is not legally questionable. There is no minimum "amount" of supportive data for a diagnosis and the stated etiology related to the nursing diagnosis. No supportive data are given in the stem of the question, so you could not choose "lack of data" as the best answer because all the options lack data as far as you can tell from the information given in the question. In addition, it is not necessary to include supportive data in the diagnostic statement (although some do prefer to use "A.M.B." and include defining characteristics). PTS:1DIF:ModerateREF:p. 74 KEY: Nursing process: Diagnosis | Client need: SECE | Cognitive level: Analysis

Which of the following factors has the greatest positive effect on sleep quality? 1)Sleeping hours in synchrony with the person's circadian rhythm 2)Sleeping in a quiet environment 3)Spending additional time in stage IV of the sleep cycle 4)Napping frequently during the day hours

ANS: 1 A circadian rhythm is a biorhythm based on the day-night pattern in a 24-hour cycle. Sleep quality is best when the time at which the person goes to sleep and awakens is in synchrony with his circadian rhythm. Not all people require a quiet environment for sleep. Time spent in stage IV of the sleep cycle is affected by the total time spent asleep. Napping on and off throughout the day might disrupt the natural circadian rhythm with uninterrupted periods of sleep that cycle through the various stages of the sleep cycle. PTS:1DIF:ModerateREF:p. 1204

Which of the following is an example of a cluster of related cues? 1) Complains of nausea and stomach pain after eating 2) Has a productive cough and states stools are loose 3) Has a daily bowel movement and eats a high-fiber diet 4) Respiratory rate 20 breaths/min, heart rate 85 beats/min, blood pressure 136/84

ANS: 1 A cue is an unhealthy response; a cluster of cues consists of cues related to each other. Productive cough and loose stools are abnormal findings but are not obviously or usually related to each other. Daily bowel movement and high-fiber diet are related but normal responses. The vital signs are also within normal limits. PTS:1DIFgrinifficultREF: pp. 62 KEY: Nursing process: Diagnosis | Client need: PHSI | Cognitive level: Analysis

What makes a nursing history different from a medical history? 1) A nursing history focuses on the patient's responses to the health problem. 2) The same information is gathered; the difference is in who obtains the information. 3) A nursing history is gathered using a specific format. 4) A medical history collects more in-depth information.

ANS: 1 A medical history focuses on the patient's current and past medical/surgical problems. A nursing history focuses on the patient's responses to and perception of the illness/injury or health problem, his coping ability, and resources and support. Nursing history formats vary depending on the patient, the agency, and the patient's needs. Both nursing and medical histories typically use a specific format. A medical history does not necessarily contain more in-depth information. A nursing history can be in-depth, covering a wide range of topics, including biographical data, reason(s) patient is seeking healthcare, history of present illness, patient's perception of health status and expectations for care, past medical history, use of complementary modalities, and review of functional ability associated with activities of daily living. Other topics might deal with nutrition, psychosocial needs, pain assessment, or other special needs topics. PTS:1DIF:ModerateREF:pp. 44-45 KEY: Nursing process: Assessment | Client need: SECE | Cognitive level: Comprehension

A patient is brought to the emergency department after inhaling mercury. The nurse should be alert for which acute adverse effects associated with mercury inhalation? 1) Chest pain, pneumonitis, and inflammation of the mouth 2) Intestinal obstruction and numbness of the hands 3) Hypotension, oliguria, and tingling of the feet 4) Tachycardia, hematuria, and diaphoresis

ANS: 1 Acute adverse effects of mercury inhalation include chest pain, inflammation of mouth, pneumonitis, respiratory damage, wakefulness, muscle weakness, anorexia, headache, and ringing in the ears, Chronic effects include numbness or tingling of the hands, lips, and feet, and personality changes. Intestinal obstruction is an acute effect of mercury ingestion. Hypotension, oliguria, hematuria, and diaphoresis are not acute effects of mercury inhalation. PTS: 1 DIF: Difficult REF: p. 659 KEY: Nursing process: Assessment | Client need: PHSI | Cognitive level: Analysis

A patient suddenly develops right lower-quadrant pain, nausea, vomiting, and rebound tenderness. How should the nurse classify this patient's pain? 1) Acute 2) Chronic 3) Intractable 4) Neuropathic

ANS: 1 Acute pain typically has a short duration and a rapid onset. Chronic pain lasts longer than 6 months and interferes with daily activities. Intractable pain is chronic and highly resistant to relief. Neuropathic pain is a type of chronic pain that occurs from injury to one or more nerves. PTS:1DIF:EasyREF:p. 1092 KEY: Nursing process: Assessment | Client need: PHSI | Cognitive level: Comprehension

Your patient has a deep wound on the right hip, with tunneling at the 8 o'clock position extending 5 cm. The wound is draining large amounts of serosanguineous fluid and contains 100% red beefy tissue in the wound bed. Of the following, which would be an appropriate dressing choice? 1) Alginate dressing 2) Dry gauze dressing 3) Hydrogel 4) Hydrocolloid dressing

ANS: 1 Alginates are highly absorbent and are appropriate for wounds with moderate to large amounts of drainage. They are ideal for wounds with tunneling, as they will conform to fill the tunnel. Gauze and hydrocolloids have limited absorptive ability. Gauze could adhere to the wound bed and cause trauma when removed. A hydrogel would increase the drainage, with the potential of macerating surrounding skin. PTS:1DIF:ModerateREF:p. 1250 KEY: Nursing process: Interventions | Client need: PHSI | Cognitive level: Applicati

How are standardized (model) care plans similar to unit standards of care? Standardized (model) care plans 1) Describe the care needed by patients in defined situations 2) Include specific goals and nursing orders 3) Become a part of the patient's comprehensive care plan 4) Usually describe ideal nursing care

ANS: 1 All of the statements are true for standardized care plans, but only 1 is true of both standardized care plans and unit standards of care. Both describe care needed by patients in defined situations, although unit standards usually describe care for groups of patients (e.g., all women admitted to a labor unit), and standardized care plans are often organized around a particular or all nursing diagnoses commonly occurring with a particular medical diagnosis. Unit standards are more general and do not have goals for each patient. Unit standards are kept on file in a central place on the unit and do not become a part of the care plan. Unit standards describe minimal, not ideal, care. PTS: 1 DIF: Difficult REF: p. 87; requires analysis of text discussion. KEY: Nursing process: Planning | Client need: SECE | Cognitive level: Analysis

What do initial, ongoing, and discharge planning have in common? 1) They are based on assessment and diagnosis. 2) They focus on the patient's perception of his needs. 3) They require input from a multidisciplinary team. 4) They have specific timelines in which to be completed.

ANS: 1 All planning is based on nursing assessment data and identified nursing diagnoses. The patient should have input, but the planning is based on the nursing assessment. The different types of planning are intertwined and may or may not be done at distinct, separate times. Discharge planning often requires a multidisciplinary team, but initial and ongoing planning may not. Initial planning is usually begun after the first patient contact, but there is no specified time for completion; ongoing planning is more or less continuous and is done as the need arises; discharge planning must be done before discharge. PTS:1DIF:ModerateREF: p. 81-82 KEY: Nursing process: Planning | Client need: SECE | Cognitive level: Analysis

What do the nursing assessment models have in common? 1) They assess and cluster data into model categories. 2) They organize assessment data according to body systems. 3) They specify use of the nursing process to collect data. 4) They are based on the ANA Standards of Care.

ANS: 1 All the models categorize or cluster data into functional health patterns, domains, or categories. None of the assessment models clusters data according to body system. Assessment is the first step of the nursing process; the nurse does not use the entire nursing process in data collection. The ANA Standards of Care describe a competent level of clinical nursing practice based on the nursing process; nursing models are not based on the ANA Standards of Care. PTS:1DIFgrinifficultREF:pp. 48

The nurse has been explaining advance directives to a patient. Which response by the patient would indicate that he has correctly understood the information? "An advance directive is a document 1) Specifying your healthcare intentions should you become unable to make self-directed decisions" 2) Identifying the activities considered to be evidence of quality care" 3) Verifying your understanding of the risks and benefits associated with a procedure" 4) Allowing you the autonomy to leave the hospital when you decide, even if it is against medical advice"

ANS: 1 An advance directive is a group of instructions stating the patient's healthcare wishes should he become unable to make decisions. The Patient Care Partnership is a document that helps to ensure that patients receive quality care. An informed consent form verifies the patient's understanding of risks and benefits associated with a procedure. An "against medical advice" form allows the patient to leave the hospital against medical advice and releases the hospital of responsibility for the patient. PTS:1DIF:ModerateREF:p. 364 KEY: Nursing process: Evaluation | Client need: SECE | Cognitive

Which of the following is an example of an ongoing assessment? 1)Taking the patient's temperature 1 hour after giving acetaminophen (Tylenol) 2)Examining the patient's mouth at the time she complains of a sore throat 3)Requesting the patient to rate intensity on a pain scale with the first perception of pain 4)Asking the patient in detail how he will return to his normal exercise activities

ANS: 1 An ongoing assessment occurs when a previously identified problem is being reassessed—for example, taking an hourly temperature when a patient has a fever. Examining the mouth is a focused assessment to explore the patient's complaint of sore throat. Asking for a pain rating is a focused assessment at the first complaint of pain. A detailed interview about exercise is a special needs assessment; there is no way to know if it is initial or ongoing. PTS:1DIF:ModerateREF: p. 41

A patient who was prescribed furosemide (Lasix) is deficient in potassium. Which of the following is an appropriate goal for this patient? The patient will increase his consumption of: 1) Bananas, peaches, molasses, and potatoes. 2) Eggs, baking soda, and baking powder. 3) Wheat bran, chocolate, eggs, and sardines. 4) Egg yolks, nuts, and sardines.

ANS: 1 Foods rich in potassium include bananas, peaches, molasses, meats, avocados, milk, shellfish, dates, figs, and potatoes. Eggs, baking soda, and baking powder have high sodium content. Dairy products, beef, pork, beans, sardines, eggs, chicken, wheat bran, and chocolate are rich in phosphorus. Egg yolks, nuts, sardines, dairy products, broccoli, and legumes are rich in calcium. PTS:1DIF:ModerateREF:p. 908 KEY: Nursing process: Planning | Client need: PHSI | Cognitive level: Application

When applying heat or cold therapy to a wound, what should the nurse do? 1) Leave the therapy on each area no longer than 15 minutes. 2) Leave the therapy on each area no longer than 30 minutes. 3) When using heat, ensure the temperature is at least 135°F (57.2°C) before applying it. 4) When using cold, ensure the temperature is less than 32°F (0°C) before applying it.

ANS: 1 Apply heat or cold therapies intermittently, leaving them on for no more than 15 minutes at a time in an area. This helps prevent tissue injury and also makes the therapy more effective by preventing rebound phenomenon. Temperatures should be kept between 59°F and 113°F (15°C and 45°C), depending on the type of therapy chosen and what is comfortable to the patient. Temperatures colder or warmer than those recommended can damage tissue. PTS:1DIF:EasyREF:p. 1254 KEY: Nursing process: Interventions | Client need: SECE | Cognitive level: Comprehension

The diagnostic label, or patient problem, is used primarily to suggest 1) Client goals 2) Cue clusters 3) Interventions 4) Etiology

ANS: 1 As a general rule, the problem suggests goals for client outcomes. The etiology suggests interventions. Cue clusters support whether the correct nursing diagnosis has been identified. PTS: 1 DIF: Moderate REF: p. 73 KEY: Nursing process: Diagnosis | Client need: SECE | Cognitive level: Recall

The nurse has just been assigned to the clinical care of a newly admitted patient. To know how to best care for the patient, the nurse uses the nursing process. Which step would the nurse probably do first? 1)Assessment 2)Diagnosis 3)Plan outcomes 4)Plan interventions

ANS: 1 Assessment is the first step of the nursing process. The nursing diagnosis is derived from the data gathered during assessment, outcomes from the diagnosis, and interventions from the outcomes. PTS:1DIF:EasyREF: p. 30-31

Which dysrhythmia confirms death? 1) Asystole (absence of heart activity) 2) Pulseless electrical activity 3) Ventricular fibrillation 4) Ventricular tachycardia

ANS: 1 Asystole is a dysrhythmia that commonly serves as a confirmation of death. Pulseless electrical activity, ventricular fibrillation, and ventricular tachycardia are potentially lethal dysrhythmias that may respond to treatment. PTS:1DIF:EasyREF:p. 365 KEY: Nursing process: Assessment | Client need: PHSI | Cognitive level: Comprehension

Which of the following is a criticism of standardized nursing diagnoses developed by NANDA-I? 1) There is little research to support nursing diagnoses labels. 2) A perfect nursing diagnosis must be written for it to be useful. 3) They are not included in all states' nurse practice acts. 4) Other professions do not recognize nursing diagnoses.

ANS: 1 Best practice is evidence-based practice; that is, it is developed through sound, scientific research. Research is currently being conducted, but many of the diagnoses are not research based. A perfect nursing diagnosis is impossible to write, so that is not an issue. Having standardized nursing diagnoses recognized in state practice acts or by other professions has nothing to do with the value of the NANDA-I taxonomy. PTS:1DIFgrinifficultREF: p. 57 KEY: Nursing process: Diagnosis | Client need: SECE | Cognitive level: Recall

How are NANDA-I problem labels and NOC outcome labels alike? Both describe 1) Health status in terms of human responses 2) Patient response before interventions are done 3) Patient response in positive terms 4) A pattern of related cues

ANS: 1 Both NANDA-I and NOC labels are stated as human responses. A NOC label can be used to describe patient responses both before and after intervention—NANDA-I before. NOC statements are neutral to allow for positive, negative, or no change in health status; NANDA-I diagnoses describe both problem responses and positive responses (wellness labels). NANDA-I labels are based on patterns of related cues; NOC labels are based on (linked to) NANDA-I labels. PTS: 1 DIF: Difficult REF: pp. 94; also information about NANDA-I diagnoses from Chapter 4 KEY: Nursing process: Planning | Client need: SECE | Cognitive level: Analysis

Which food provides the body with no usable glucose? 1) Wheat germ 2) Apple 3) White bread 4) White rice

ANS: 1 Dietary fiber, such as wheat germ, contains no usable glucose. Apples, white bread, and white rice all contain carbohydrates, which provide usable glucose. PTS: 1 DIF: Easy REF: p. 902; does not specify wheat germ, just indicates that fiber provides no glucose. KEY: Nursing process: Planning | Client need: HPM | Cognitive level: Comprehension

A client reports taking acetaminophen (Tylenol) to control osteoarthritis. Which instruction should the nurse give the patient requiring long-term acetaminophen use? 1) Caution the patient against combining acetaminophen with alcohol. 2) Explain that acetaminophen increases the risk for bleeding. 3) Advise taking acetaminophen with meals to prevent gastric irritation. 4) Explain that physical dependence may occur with long-term oral use.

ANS: 1 Even in recommended doses, acetaminophen can cause hepatotoxicity in those who consume alcohol. Therefore, the nurse should caution the patient against combining acetaminophen with alcohol. Aspirin, not acetaminophen, increases the risk for bleeding because it inhibits platelet aggregation. Nonsteroidal anti-inflammatory drugs (NSAIDs), not acetaminophen, cause gastric irritation and should be taken with meals. Opioid analgesics, not acetaminophen, can cause physical dependence. PTS:1DIF:ModerateREF:p. 1103 KEY: Nursing process: Interventions | Client need: SECE | Cognitive level: Application

7. The nurse measures the urine output of a patient who requires a bedpan to void. Which action should the nurse take first? Put on gloves and: 1) Have the patient void directly into the bedpan. 2) Pour the urine into a graduated container. 3) Read the volume with the container on a flat surface at eye level. 4) Observe the color and clarity of the urine in the bedpan.

ANS: 1 First, the nurse should put on gloves and have the patient void directly into the bedpan. Next, she should pour the urine into a graduated container, place the measuring device on a flat surface, and read the amount at eye level. She should observe the urine for color, clarity, and odor. Then, if no specimen is required, she should discard the urine in the toilet and clean the container and bedpan. Finally, she should record the amount of urine voided on the patients intake and output record.

The nurse caring in the intensive care unit suspects that one of her patients is experiencing sensory overload. Which findings(s) has/have aroused her suspicion? Choose all that apply. 1) Disorientation 2) Restlessness 3) Hallucinations 4) Depression

ANS: 1, 2 The patient with sensory overload might exhibit disorientation, confusion, restlessness, decreased ability to perform tasks, anxiety, muscle tension, and muscle tension. Sensory deprivation causes irritability, confusion, depression, heart palpitations, hallucinations, and delusions. PTS:1DIF:ModerateREF:p. 1071 KEY: Nursing process: Assessment | Client need: PHSI | Cognitive level: Application

The nurse is interviewing a patient who has a recent onset of migraine headaches. The patient is highly anxious and cannot seem to focus on what the nurse is saying. Which of the following questions would be best for the nurse to use to begin gathering data about the headaches? 1) "When did your migraines begin?" 2) "Tell me about your family history of migraines." 3) "What are the types of things that trigger your headaches?" 4) "Describe what your headaches feel like."

ANS: 1 For someone who is anxious, it is best to use closed questions. (When did your migraines begin?) A closed question can be answered in one or very few words and has a very specific answer. The other questions are open-ended questions. PTS:1DIF:ModerateREF:p. 45 KEY: Nursing process: Assessment | Client need: PSI | Cognitive level: Application

In caring for a patient with comorbidities, the nurse draws upon her knowledge of diabetes and skin integrity. In a spirit of inquiry, she looks up the latest guidelines for providing skin care and includes them in the plan of care. The nurse provides skin care according to the procedural guidelines and begins regular monitoring to evaluate the effectiveness of the interventions. These activities are best described as 1)Full-spectrum nursing 2)Critical thinking 3)Nursing process 4)Nursing knowledge

ANS: 1 Full-spectrum nursing (1) involves the use of critical thinking, nursing knowledge, nursing process, and patient situation. Although the other answers are important for planning and delivering nursing care, they do not reflect all the nurse has demonstrated. PTS:1DIFgrinifficultREF:pp. 32-33; high-level question, answer not stated verbatim

1. What is the most significant change in kidney function that occurs with aging? 1) Decreased glomerular filtration rate 2) Proliferation of micro blood vessels to renal cortex 3) Formation of urate crystals 4) Increased renal mass

ANS: 1 Glomerular filtration rate is the amount of filtrate formed by the kidneys in 1 minute. Renal blood flow progressively decreases with aging primarily because of reduced blood supply through the micro blood vessels of the kidney. A decrease in glomerular filtration is the most important functional deficit caused by aging. Urate crystals are somewhat common in the newborn period. They might indicate that the infant is dehydrated. In older people, they result from too much uric acid in the blood, although this is not related to aging. Renal mass (weight) decreases over time, starting around age 30 to 40.

What is the most significant change in kidney function that occurs with aging? 1) Decreased glomerular filtration rate 2) Proliferation of micro blood vessels to renal cortex 3) Formation of urate crystals 4) Increased renal mass

ANS: 1 Glomerular filtration rate is the amount of filtrate formed by the kidneys in 1 minute. Renal blood flow progressively decreases with aging primarily because of reduced blood supply through the micro blood vessels of the kidney. A decrease in glomerular filtration is the most important functional deficit caused by aging. Urate crystals are somewhat common in the newborn period. They might indicate that the infant is dehydrated. In older people, they result from too much uric acid in the blood, although this is not related to aging. Renal mass (weight) decreases over time, starting around age 30 to 40. PTS:1DIF:DifficultREF:p. 1013 KEY:Nursing process: N/A | Client need: PHSI | Cognitive level: Recall

Health screening activities are designed to: 1) Detect disease at an early stage. 2) Determine treatment options. 3) Assess lifestyle habits. 4) Identify healthcare beliefs.

ANS: 1 Health screening activities are designed to detect disease at an early stage so that treatment can begin before there is an opportunity for disease to spread or become debilitating. PTS: 1 DIF: Moderate REF: p. 879 KEY: Nursing process: Planning | Client need: HPM | Cognitive level: Comprehension

The nurse suspects a 3-year-old child who is coughing vigorously has aspirated a small object. Which action should the nurse take first? 1) Encourage the child to continue coughing. 2) Deliver upward abdominal thrusts with a fisted hand. 3) Deliver five rapid back blows between the shoulder blades. 4) Perform a blind finger sweep of the child's mouth.

ANS: 1 If the nurse suspects aspiration in a child who is coughing vigorously, the nurse should encourage the child to continue coughing. If coughing weakens, the nurse should perform the choking maneuver by administering five rapid back blows alternated with five upward thrusts to the upper abdomen with a fisted hand, just below the rib cage. A blind finger sweep should never be performed because it could push the foreign object into the airway. PTS:1DIF:Moderate REF: ESG, Box 23-5, Rescue Maneuver for Choking: Adult or Child Over Age 12 Months KEY: Nursing process: Interventions | Client need: PHSI | Cognitive level: Analysis

Which action should the nurse take first when the patient has a score of 4 on the sedation rating scale? 1) Stimulate the patient. 2) Prepare to administer naloxone (Narcan). 3) Administer a dose of pain medication. 4) Notify the physician immediately.

ANS: 1 If the patient's score on the sedation rating scale is equal to or greater than 4, the nurse should first stimulate the patient. He should next notify the physician. The nurse should consider administering naloxone, as prescribed, if the patient's respiratory rate is less than 8 breaths/minute; if respirations are shallow with marginal or falling oxygen saturation; or if the patient is unresponsive to stimulation. Before the patient receives another dose of pain medication, the dose should most likely be reduced and other potential causes of sedation should be investigated. PTS:1DIFgrinifficultREF:p. 1106 KEY: Nursing process: Interventions | Client need: SECE | Cognitive level: Application

16. The surgeon orders hourly urine output measurement for a patient after abdominal surgery. The patients urine output has been greater than 60 mL/hour for the past 2 hours. Suddenly the patients urine output drops to almost nothing. What should the nurse do first? 1) Irrigate the catheter with 30 mL of sterile solution. 2) Replace the patients indwelling urinary catheter. 3) Infuse 500 mL of normal saline solution IV over 1 hour. 4) Notify the surgeon immediately.

ANS: 1 If the patients urinary output suddenly ceases, the nurse should irrigate the urinary catheter to assess whether the catheter is blocked. If no blockage is detected, the nurse should notify the surgeon. The surgeon may request that the catheter be changed if irrigation does not help or if the tubing is not kinked. However, the nurse should not change a catheter in the immediate postoperative period without consulting with the surgeon. The surgeon may prescribe an IV fluid bolus if the patient is suspected to have a deficient fluid volume.

13. The student nurse asks the provider if she will prescribe an indwelling urinary catheter for a hospitalized patient who is incontinent. The provider explains that catheters should be utilized only when absolutely necessary because: 1) They are the leading cause of nosocomial infection. 2) They are too expensive for routine use. 3) They contain latex, increasing the risk for allergies. 4) Insertion is painful for most patients.

ANS: 1 Indwelling urinary catheters should not be routinely used for hospitalized patients with incontinence because they are the leading cause of healthcare-acquired infection (nosocomial). The cost of an indwelling urinary catheter should not deter its use if necessary. Latex-free catheters are available for patients with or at risk for latex allergy. Insertion may be somewhat uncomfortable, but it should not be painful.

Three days after a patient had abdominal surgery, the nurse notes a 4-cm periwound erythema and swelling at the distal end of the incision. The area is tender and warm to the touch. Staples are intact along the incision, and there is no obvious drainage. Heart rate is 96 beats/min and oral temperature is 100.8°F (38.2°C). The nurse would suspect that the patient has what kind of complication? 1) Infection at the incisional site 2) Dehiscence of the wound 3) Hematoma under the skin 4) Formation of granulation tissue

ANS: 1 Infection is a complication of wound healing that causes warmth, pain, inflammation of the affected area, and changes in vital signs (i.e., elevated pulse and temperature). Dehiscence is the rupture of a suture line, whereas evisceration is the protrusion of internal organs through the rupture. A hematoma is a collection of blood that forms under the skin. It is usually tender or painful to the touch and is usually swollen. Granulation tissue is new connective tissue and tiny blood vessels that form on the surfaces of a wound during the healing process. It is beefy red in appearance but would not be warm or tender to the touch. PTS:1DIF:ModerateREF:p. 1229 KEY: Nursing process: Diagnosis | Client need: PHSI | Cognitive level: Analysis

In the Leavell and Clark model of health protection, the chief distinction between the levels of prevention is: 1) The point in the disease process at which they occur. 2) Placement on the Wheels of Wellness. 3) The level of activity required to achieve them. 4) Placement in the Model of Change.

ANS: 1 Leavell and Clark identified three levels of activities for health protection: primary, secondary, and tertiary. Interventions are classified according to the point in the disease process in which they occur. PTS:1DIFgrinifficultREF:p. 879 KEY:Nursing process: N/A | Client need: HPM | Cognitive level: Analysis

Physiological changes associated with aging place the older adult especially at risk for which nursing diagnosis? 1) Risk for Falls 2) Risk for Ineffective Airway Clearance (choking) 3) Risk for Poisoning 4) Risk for Suffocation (drowning)

ANS: 1 Loss of muscle strength and joint mobility place older adults at risk for falls. Choking, drowning, and ingesting poisons are primary safety concerns for infants and toddlers. PTS:1DIF:ModerateREF:p. 653 KEY: Nursing process: Diagnosis | Client need: SECE | Cognitive level: Recall

A patient diagnosed with macular degeneration asks the nurse to explain his condition. Which statement by the nurse best describes macular degeneration? 1) "The portion of your eye called the macula, which is responsible for central vision, is damaged." 2) "Your lens became cloudy, causing your blurred vision. This cloudiness will increase over time." 3) "The pressure in the anterior cavity of your eye became elevated, shifting the position of your lens." 4) "There's an irregular curvature of your cornea, causing your blurred vision."

ANS: 1 Macular damage (degeneration) causes diminished central vision. Cataracts are caused by a cloudy lens and result in blurred vision. Glaucoma is pressure in the anterior cavity of the eye, which shifts the lens position. Astigmatism is irregular curvature of the cornea, resulting in blurred vision. PTS:1DIF:ModerateREF:p. 1072 KEY: Nursing process: Interventions | Client need: PHSI | Cognitive level: Application

A patient tells the nurse that since taking a medication he has suffered from excessively dry mouth. Which of the following assessments would be needed in order to plan interventions for that symptom? 1) Asking the patient if foods taste different now 2) Checking the patient's sense of smell 3) Having the patient stand to check for balance 4) Assessing for a history of seizures

ANS: 1 Many medications cause xerostomia (dry mouth), and xerostomia is the most common cause of impaired taste. Impaired sense of smell also affects the sense of taste; however, there is no reason to assume impaired smell in this patient. Balance is related the inner ear and to kinesthetic sense, not to taste and xerostomia. Xerostomia would be related to seizures only if a patient experienced dry mouth as an aura; this would be unusual. Even if this were the case, the information would allow the nurse to plan care for seizures, but not for the symptom of dry mouth. PTS:1DIFgrinifficultREF:p. 1072 KEY: Nursing process: Interventions | Client need: PHSI | Cognitive level: Application

A patient is admitted with an exacerbation of asthma. Which factor places the patient at highest risk for sensory overload? 1) Administering albuterol (a central nervous stimulant) as needed 2) Administering a tranquilizer intravenously every 2 hours as prescribed 3) Delivering oxygen at 6 L/min via nasal cannula 4) Maintaining complete bedrest in a quiet, dimly lit room

ANS: 1 Medications that stimulate the central nervous system, such as albuterol, place the patient at risk for sensory overload. A tranquilizer and a quiet darkened room may help the patient to relax, thus preventing sensory overload. If the patient's oxygen needs are met with oxygen at 6 L/min via nasal cannula, the patient should not experience sensory overload related to oxygen therapy alone. PTS:1DIFgrinifficultREF:p. 1071 KEY: Nursing process: Interventions | Client need: SECE | Cognitive level: Application

The nurse plays music for a child with leukemia who is experiencing pain. Which pain management technique is this nurse using? 1) Distraction 2) Guided imagery 3) Sequential muscle relaxation 4) Hypnosis

ANS: 1 Music is a form of distraction that has been shown to reduce pain and anxiety by allowing the patient to focus on something other than pain. Guided imagery uses auditory and imaginary processes to help the patient to relax. In sequential muscle relaxation, the patient sits and tenses muscles for 15 seconds and then relaxes the muscles while breathing out. This relaxation technique has also been effective for relieving pain. Hypnosis involves the induction of a deeply relaxed state. PTS:1DIF:EasyREF:pp. 1101-1102 KEY: Nursing process: Interventions | Client need: PHSI | Cognitive level: Comprehension

The nurse is individualizing Mr. Wu's plan of care by writing a plan for his nursing diagnosis of Anxiety. Why does the nurse need to write goals/outcomes on the plan of care? Because outcomes describe 1) Desired changes in the patient's health status 2) Specific patient responses to medical interventions 3) Specific nursing behaviors to improve a patient's health 4) Criteria to evaluate the appropriateness of a nursing diagnosis

ANS: 1 Outcomes describe changes in the patient's health status in response to nursing, rather than medical, interventions. Outcomes relate to patient behavior, not nursing behaviors. Outcomes are a measure of the effectiveness of nursing care for a specific nursing diagnosis, not whether the nursing diagnosis is appropriate. PTS:1DIF:ModerateREF: p. 91 KEY: Nursing process: Planning | Client need: SECE | Cognitive level: Comprehension

Patterns of waking behavior that appear during sleep are known as: 1)Parasomnias. 2)Dyssomnias. 3)Insomnia. 4)Hypersomnia.

ANS: 1 Parasomnias are patterns of waking behavior that appear during sleep. Sleepwalking, sleep talking, and bruxism are parasomnias. PTS: 1 DIF: Easy REF: p. 1208

A patient with quadriplegia presents to the outpatient clinic with an ischial wound that extends through the epidermis into the dermis. When documenting the depth of the wound, how would the nurse classify it? 1) Partial-thickness wound 2) Penetrating wound 3) Superficial wound 4) Full-thickness wound

ANS: 1 Partial-thickness wounds extend through the epidermis into the dermis. Superficial wounds involve only the epidermal layer of skin. Full-thickness wounds extend into the subcutaneous tissue and beyond. Penetrating is a descriptor sometimes added to indicate that the wound includes internal organs. PTS:1DIF:EasyREF:pp. 1226-1227 KEY: Nursing process: Assessment | Client need: PHSI | Cognitive level: Application

A patient with a history of mitral valve replacement, hypertension, and type 2 diabetes mellitus undergoes emergency surgery to remove an embolus in her right leg. Which factor contraindicates the use of epidural analgesia in this patient? 1) Anticoagulant therapy 2) Diabetes mellitus 3) Hypertension 4) Embolectomy

ANS: 1 Patients who undergo mitral valve replacement typically require long-term anticoagulant therapy. Anticoagulant therapy is a contraindication for epidural analgesia use because of the risk for spinal hematoma and uncontrolled bleeding. Diabetes and hypertension are not contraindications for epidural analgesia. Epidural analgesia is commonly used after embolectomy because certain anesthetic agents, such as bupivacaine, help prevent vasospasm. PTS:1DIFgrinifficultREF:p. 1108 KEY: Nursing process: Planning | Client need: SECE | Cognitive level: Analysis

. A patient with Parkinson's disease is at risk for which complication? 1) Impaired kinesthesia 2) Macular degeneration 3) Seizures 4) Xerostomia

ANS: 1 Patients with Parkinson's disease are at risk for impaired kinesthesia, placing them at risk for falling. Drooling, not excessive dry mouth (xerostomia), is common with Parkinson's disease. Seizures and macular degeneration are not associated with Parkinson's disease. PTS:1DIF:ModerateREF:pp. 1074-1075 KEY: Nursing process: Assessment | Client need: PHSI | Cognitive level: Application

The nurse notices that a patient has spoon-shaped, brittle nails. This suggests that the patient is experiencing Imbalanced Nutrition: Less Than Body Requirements related to deficiency of which of the following nutrients? 1) Iron 2) Vitamin A 3) Protein 4) Vitamin C

ANS: 1 Patients with iron deficiency may have spoon-shaped, brittle nails. Other abnormal nail findings include dull nails with transverse ridge (protein deficiency); pale, poor blanching, or mottled nails (vitamin A or C deficiency); splinter hemorrhages (vitamin C deficiency); and bruising or bleeding beneath nails (protein or caloric deficiency). PTS:1DIFgrinifficultREF:p. 920 KEY: Nursing process: Diagnosis | Client need: PHSI | Cognitive level: Comprehension

What is the rationale for wrapping petroleum gauze around a chest tube insertion site? 1) Prevents air from leaking around the site 2) Prevents infection at the insertion site 3) Absorbs drainage from the insertion site 4) Protects the tube from becoming dislodged

ANS: 1 Petroleum gauze creates a seal around the insertion site. Collapse of the lung can occur if there is a leak around the insertion site that causes loss of negative pressure within the system. Air leaks are one common cause of loss of negative pressure. PTS:1DIF:EasyREF:p. 1322 KEY: Nursing process: Interventions | Client need: PHSI | Cognitive level: Comprehension

Which assessment finding is considered an age-related change? 1) Presbycusis 2) Hyperopia 3) Increased sensitivity to touch 4) Increased sensitivity to taste

ANS: 1 Presbycusis, the loss of high-frequency tones, is an age-related change. Hyperopia is the ability to see distant objects well; it is not an age-related change. The ability to perceive touch and taste diminishes with age; it does not increase. PTS:1DIF:ModerateREF:p. 1069, 1072 KEY: Nursing process: Assessment | Client need: PHSI | Cognitive level: Comprehension

Pressure ulcers are directly caused by which of the following conditions at the site? 1) Compromised blood flow 2) Edema 3) Shearing forces 4) Inadequate venous return

ANS: 1 Pressure ulcers are caused by unrelieved pressure that compromises blood flow to an area, resulting in ischemia (inadequate blood supply) in the underlying tissue. Friction and shear are extrinsic factors affecting skin integrity, which increases the risk of a client developing a pressure ulcer but is not the direct cause. Inadequate arterial blood flow to an area due to pressure causes the development of a pressure ulcer. Edema leads to compromised skin and tissue integrity, which is more prone to pressure injury. PTS: 1 DIF: Difficult REF: p. 1230 KEY: Nursing process: Planning | Client need: HPM | Cognitive level: Comprehension

How are short-term goals different from long-term goals? Short-term goals 1) Can be met within a few hours or a few days 2) Are developed from the problem side of the nursing diagnosis 3) Must have target times/dates 4) Specify desired client responses to interventions

ANS: 1 Short-term goals may be accomplished in hours or days; long-term goals usually are achieved over weeks, months, or even years. The other statements are true for both short-term and long-term goals. PTS:1DIF:ModerateREF: p. 91 KEY: Nursing process: Planning | Client need: SECE | Cognitive level: Analysis

22. Which daily urine output is within normal limits for a newborn weighing 8 pounds? 1) 288 mL 2) 180 mL 3) 36 mL 4) 18 mL

ANS: 2 A newborn weighing 8 pounds (3.6 kg) should produce 15 to 60 mL of urine per kilogram per day. If the newborn produces 50 mL/kg/day and weighs 3.6 kg, he will produce a total of 180 mL in 24 hours. The other options are not within normal limits and require further assessment.

A patient is brought to the emergency department experiencing leg cramps. He is irritable, his temperature is elevated, and his mucous membranes are dry. Based on these findings, the patient most likely has excess levels of which mineral? 1) Sodium 2) Potassium 3) Phosphorus 4) Magnesium

ANS: 1 Signs and symptoms associated with sodium excess include thirst, fever, dry and sticky tongue and mucous membranes, restlessness, irritability, and seizures. Findings associated with potassium excess include cardiac arrhythmias, weakness, abdominal cramps, diarrhea, anxiety, and paresthesia. Phosphorus excess leads to tetany and seizures. Magnesium excess causes weakness, nausea, and malaise. PTS:1DIFgrinifficultREF:p. 908 KEY: Nursing process: Diagnosis | Client need: PHSI | Cognitive level: Analysis

The primary focus of your interventions for a 6-year-old child who sleepwalks would be to: 1)Maintain patient safety during episodes of somnambulation. 2)Administer and teach about medications to suppress stage IV sleep. 3)Encourage the child to verbalize feelings regarding sleep pattern. 4)Provide a quiet environment for nighttime sleep.

ANS: 1 Sleepwalking places the patient at Risk for Injury because of his lack of awareness of the surroundings. The nurse's primary intervention would be to protect the patient from injury (e.g., falls) while sleepwalking, also called somnambulation. Because the child is only 6 years old, administering and teaching about medications and having him verbalize feelings would not be useful. Providing a quiet environment would likely be ineffective and certainly not the focus of interventions. PTS:1DIF:ModerateREF:p. 1211

Which individuals should receive annual lipid screening? 1) All overweight children 2) All adults 20 years and older 3) Persons with total cholesterol greater than 150 mg/dL 4) Persons with HDL less than 40 mg/dL

ANS: 1 The American Academy of Pediatrics takes a targeted approach, recommending that overweight children receive cholesterol screening, regardless of family history or other risk factors for cardiovascular disease. The American Heart Association recommends that all adults age 20 years or older have a fasting lipid panel at least once every 5 years. If total cholesterol is 200 mg/dL or greater—or HDL is less than 40 mg/dL—frequent monitoring is required. PTS: 1 DIF: Moderate REF: p. 888; Box 27-1 KEY: Nursing process: Assessment | Client need: HPM | Cognitive level: Application

The Joint Commission's national Speak Up® campaign encourages patients to become active and informed participants on the healthcare team. The goal is to: 1) prevent healthcare errors. 2) help control the cost of healthcare. 3) reduce the number of automobile accidents. 4) provide a forum for people without health insurance

ANS: 1 The Joint Commission, with the Centers for Medicare and Medicaid Services, urges patients to take a role in preventing healthcare errors by becoming active, involved, and informed participants on the healthcare team. A reduction in healthcare errors could indirectly reduce healthcare costs, but this is not the intent of the campaign. The campaign has nothing to do with automobile accidents, as might be deduced from the fact that the Joint Commission and Medicare/Medicaid regulate healthcare agencies. The campaign has little relationship to insurance, other than to encourage clients to speak up, ask questions, and know their rights. PTS:1DIFgrinifficultREF:p. 664 KEY: Nursing process: N/A | Client need: SECE | Cognitive level: Comprehension

A nurse admits a patient to the unit after completing a comprehensive interview and physical examination. To develop a nursing diagnosis, the nurse must now 1)Analyze the assessment data 2)Consult standards of care 3)Decide which interventions are appropriate 4)Ask the client's perceptions of her health problem

ANS: 1 The basis of the nursing diagnosis is the assessment data. Standards of care are referred to when establishing nursing interventions. Customizing interventions personalizes nursing care. Asking the patient about her perceptions is a method to validate whether the nurse has chosen the correct nursing diagnosis and would probably have been done during the comprehensive assessment. PTS: 1 DIF: Moderate REF: p. 31

Which drug might the primary care provider prescribe to help facilitate pain management in a client with chronic pain? 1) Selective serotonin reuptake inhibitor 2) Selective norepinephrine reuptake inhibitor 3) Narcotic analgesic 4) Anti-emetic

ANS: 1 The control of depression greatly facilitates pain management, especially for patients experiencing chronic pain. Therefore, the physician may prescribe a selective serotonin uptake inhibitor (antidepressant), such as paroxetine (Paxil), as part of the treatment plan. Selective norepinephrine reuptake inhibitors, such as Atomoxetine (Strattera), are commonly used for attention deficit-hyperactivity disorder. If a narcotic, such as oxycodone (OxyContin), is used for a long time, it may become habit forming (causing mental or physical dependence). Physical dependence may lead to withdrawal side effects when you stop taking the medicine. This is not the first-line therapy for chronic pain. An anti-emetic, such as ondansetron (Zofran), is used to control for nausea and vomiting, which can occur with some analgesic medication. However, the prescriber would more likely change the medication to something the patient tolerates better rather than order an anti-emetic to control the side effect. PTS:1DIFgrinifficultREF:p. 1103; higher-order item, can be inferred from text KEY: Nursing process: Planning | Client need: SECE | Cognitive level: Application

Which of the following characteristics do the various definitions of critical thinking have in common? Critical thinking 1)Requires reasoned thought 2)Asks the questions "why?" or "how?" 3)Is a hierarchical process 4)Demands specialized thinking skills

ANS: 1 The definitions listed in the text as well as definitions in Box 2-1 state that critical thinking requires reasoning or reasoned thinking. Critical thinking is neither linear nor hierarchical. That means that the steps involved in critical thinking are not necessarily sequential, where mastery of one step is necessary to proceed to the next. Critical thinking is a purposeful, dynamic, analytic process that contributes to reasoned decisions and sound contextual judgments. PTS:1DIF:ModerateREF: p. 25; high-level question, answer not stated verbatim KEY: Nursing process: N/A | Client need: SECE | Cognitive level: Analysis

A 73-year-old patient who suffered a stroke is being transferred from the acute care hospital to a nursing home for ongoing care because she is unable to care for herself at home. Which type of loss is this patient most likely experiencing? 1) Environmental loss 2) Internal loss 3) Perceived loss 4) Psychological loss

ANS: 1 This patient is most likely experiencing an environmental loss because she is unable to return to her familiar home setting. Instead, she is being transferred to the new environment of a nursing home. Internal, perceived, and psychological losses are internal and can only be identified by the person experiencing them. PTS:1DIF:EasyREF:p. 358 KEY: Nursing process: Assessment | Client need: PSI | Cognitive level: Application

The patient verbalizes an overwhelming lack of energy. He says, "I still feel exhausted even after I sleep. I feel guilty when I can't keep up with my usual daily activities or sleep during the day. I've been a little depressed lately, too." The patient seems to have difficulty concentrating but has no apparent physical problems. Which of the following diagnoses best describes his health status? 1) Fatigue related to depression 2) Fatigue related to difficulty concentrating 3) Guilt related to lack of energy 4) Chronic confusion related to lack of energy

ANS: 1 The diagnosis that best describes the overall health status is Fatigue. The only cue that might cause Fatigue is depression. You cannot use depression as the problem because it is a medical diagnosis, and it is not a NANDA-I label. The other cues (difficulty concentrating, inability to perform ADLs, and guilt) are symptoms of Fatigue, not etiologies. These diagnoses would lead the nurse to focus on dealing with guilt and confusion, so the source of the Fatigue would not be addressed. PTS: 1 DIF: Difficult REF: pp. 70-73 KEY: Nursing process: Diagnosis | Client need: SECE | Cognitive level: Application

Why is the information obtained from a swab culture of a wound limited? 1) A positive culture does not necessarily indicate infection, because chronic wounds are often colonized with bacteria. 2) A negative culture may not indicate infection, because chronic wounds are often colonized with bacteria. 3) Most wound infections are viral, so the swab culture would not be indicative of a wound infection. 4) A swab culture result does not include bacterial sensitivity information necessary to provide treatment.

ANS: 1 The information obtained from a swab culture is limited because a positive culture may not indicate infection. Chronic wounds are often colonized with bacteria, but this does not require antibiotic treatment. A needle aspiration of the wound would provide more definitive information about whether the wound is infected or not and can be performed by a registered nurse. However, the most accurate wound information is obtained by tissue biopsy performed by a specially trained provider. PTS:1DIF:ModerateREF:p. 1242 KEY: Nursing process: Assessment | Client need: PHSI | Cognitive level: Application

When should the nurse assess pain? 1) Whenever a full set of vital signs is taken 2) During the admission interview 3) Every 4 hours for the first 2 days after surgery 4) Only when the patient complains of pain

ANS: 1 The nurse should assess pain whenever a full set of vital signs is checked. Moreover, the nurse should assess pain on admission of a patient to the facility, even when pain is not the chief complaint. Patients may have chronic pain that has no association with their reason for seeking care. Pain should be assessed more frequently than every 4 hours in the immediate postoperative period. Pain should be reassessed after any treatment is given to evaluate effectiveness of the treatment. Some patients may not complain of pain unless they are specifically asked whether they are in pain. Pain rating scales help to quantify the intensity of pain for the nurse providing analgesia. PTS: 1 DIF: Moderate REF: pp. 1096-1098 KEY: Nursing process: Assessment | Client need: PHSI | Cognitive level: Comprehension

A patient is prescribed morphine sulfate 4 mg intravenously for postoperative pain. Which action should the nurse take before administering the medication? 1) Monitor the patient's respiratory status. 2) Auscultate the patient's heart sounds. 3) Check blood pressure in supine and sitting positions. 4) Monitor the patient for psychological drug dependence.

ANS: 1 The nurse should assess the patient's respiratory status and level of alertness before administering the medication because respiratory depression can be a life-threatening effect. It is not necessary to auscultate heart sounds or to check blood pressure while the patient lies down (supine position) and sits up. Psychological dependence occurs rarely even after long-term prescribed use of morphine. Therefore, it is not necessary to routinely monitor a patient who is receiving morphine for acute postoperative pain for psychological drug dependence. PTS:1DIF:ModerateREF:p. 1104 KEY: Nursing process: Assessment | Client need: SECE | Cognitive level: Application

The nurse is providing nutrition counseling for a patient planning pregnancy. The nurse should emphasize the importance of consuming which nutrient to prevent neural tube defects? 1) Folic acid 2) Calcium 3) Protein 4) Vitamin D

ANS: 1 The nurse should emphasize the importance of consuming folic acid even before conception to prevent neural tube defects from developing. Calcium and protein needs also increase during pregnancy; however, their consumption does not prevent neural tube defects. Vitamin D consumption does not prevent neural tube defects. PTS:1DIF:EasyREF:p. 913 KEY: Nursing process: Interventions | Client need: HPM | Cognitive level: Comprehension

18. A mother tells the nurse at an annual well-child checkup that her 6-year-old son occasionally wets himself. Which response by the nurse is appropriate? 1) Explain that occasional wetting is normal in children of this age. 2) Tell the mother to restrict her childs activities to avoid wetting. 3) Suggest time out to reinforce the importance of staying dry. 4) Inform the mother that medication is commonly used to control wetting

ANS: 1 The nurse should explain that occasional wetting is normal in children during the early school years. The mother should handle the situation calmly and avoid punishing the child. Medications are occasionally prescribed for nocturnal enuresis when the child is older and not sleeping at home, but not for occasional daytime wetting.

A mother brings her 4-month-old infant for a well-baby checkup. The mother tells the nurse that she would like to start bottle feeding her baby because she cannot keep up with the demands of breastfeeding since returning to work. Which response by the nurse is appropriate? 1) "Make sure you give your baby an iron-fortified formula to supplement any stored breast milk you have." 2) "You really need to continue breastfeeding your baby." 3) "Give your baby formula until he is 6 months old; then you can introduce whole milk." 4) "Your baby weighs 14 pounds, so he will require about 36 ounces of formula a day."

ANS: 1 The nurse should not make the mother feel guilty about her decision to stop breastfeeding. Instead, she should provide the mother with instruction about bottle feeding. She can give it to supplement any stored breast milk she might have in supply. She should emphasize the importance of giving the baby iron-fortified formula because fetal iron stores become depleted by 4 to 6 months of age. Infants younger than 1 year of age should not receive regular cow's milk because it may place a strain on the immature kidneys. Because the baby weighs 14 pounds, he will require about 21 ounces of formula a day (not 36 ounces), based on the nutritional recommendations that infants require 80 to 100 mL of formula or breast milk per kilogram of body weight per day. PTS:1DIF:ModerateREF:p. 912 KEY: Nursing process: Interventions | Client need: HPM | Cognitive level: Application

20. Which action should the nurse take when beginning bladder training using scheduled voiding? 1) Offer the patient a bedpan every 2 hours while she is awake. 2) Increase the voiding interval by 30 to 60 minutes each week. 3) Frequently ask the patient if she has the urge to void. 4) Increase the frequency between voiding even if urine leakage occurs.

ANS: 1 The nurse should offer the patient the bedpan or assist the patient to the bathroom every 2 hours while she is awake. You would encourage the patient to get up once during the night to void, but awakening the patient every 2 hours would lead to fatigue. If the patient adheres to the schedule, the voiding interval should be increased by 15 to 30 minutes each week. Simply asking the patient about the urge to void does not help to manage bladder emptying.

A nurse is teaching a group of mothers about first aid. Should poison come in contact with their child's clothing and skin, which action should the nurse instruct the mothers to take first? 1) Remove the contaminated clothing immediately. 2) Flood the contaminated area with lukewarm water. 3) Wash the contaminated area with soap and water and rinse. 4) Call the nearest poison control center immediately.

ANS: 1 The nurse should tell the mother to first remove the contaminated clothing as quickly as possible. Then, flood the contaminated area with lukewarm water. Next, gently wash the area with soap and water and rinse. Have someone call the poison control center. It does not need to be a local poison control center. Additionally, it is most important to remove contact between the skin and poison before doing anything. PTS: 1 DIF: Moderate REF: ESG, Chapter 23, Box 23-2, "Home Care: If Poisoning Occurs at Home" KEY: Nursing process: Interventions | Client need: SECE | Cognitive level: Analysis

Which step should the nurse take first when performing otic irrigation in an adult? 1) Warm the irrigation solution to room temperature. 2) Position the patient so she is sitting with her head tilted away from the affected ear. 3) Straighten the ear canal by pulling up and back on the pinna. 4) Place the tip of the nozzle into the entrance of the ear canal.

ANS: 1 The nurse should warm the irrigation solution to room temperature first. Next, the nurse should assist the patient into a sitting position, with the head tilted away from the affected ear; straighten the ear canal by pulling up and back on the pinna; place the tip of the nozzle into the entrance of the ear canal; and direct the stream of irrigating solution gently along the top of the ear canal toward the back of the patient's head. Then continue irrigating until the canal is clean. PTS: 1 DIF: Easy REF: p. 1086 KEY: Nursing process: Interventions | Client need: SECE | Cognitive level: Analysis

What is typically the most reliable indicator of pain? 1) Patient's self-report 2) Past medical history 3) Description by caregiver(s) 4) Behavioral cues

ANS: 1 The patient's self-report is the most reliable indicator of pain. A patient's facial expression, vocalization, posture or position, or other behaviors do not always accurately indicate the intensity or quality of a patient's experience of pain. The patient might be trying to hide signs of pain in order to be brave or strong. Sociocultural factors can influence a patient's nonverbal expression of pain. Caregivers might not appreciate the extent of pain because pain is an individualized experience. Perception of pain might be heightened if other medical conditions coexist, although this perception is also influenced by other factors, such as past experience with pain and the success or failure of the treatment to produce relief. Emotions, cognitive impairment, developmental stage, communication skills, and mental health disorders, such as depression or anxiety, can influence the perception of pain. PTS:1DIF:ModerateREF:p. 1096 KEY: Nursing process: Assessment | Client need: PHSI | Cognitive level: Application

Which portion of a nutritional assessment must the registered nurse complete? 1) Analyzing the data 2) Obtaining intake and output 3) Weighing the patient 4) Obtaining the history

ANS: 1 The registered nurse should review and interpret (analyze) the data collected as part of a nutritional assessment. The registered nurse can delegate height, weight, and intake and output to nursing assistive personnel. History taking can be safely delegated to the licensed practical nurse. PTS:1DIF:ModerateREF:p. 925 KEY: Nursing process: Interventions | Client need: SECE | Cognitive level: Comprehension

Which structure within the brain is responsible for consciousness and alertness? 1) Reticular activating system 2) Cerebellum 3) Thalamus 4) Hypothalamus

ANS: 1 The reticular activating system, located in the brainstem, controls consciousness and alertness. The cerebellum maintains muscle tone, coordinates muscle movement, and controls balance. The thalamus is a relay system for sensory stimuli. The hypothalamus controls body temperature. PTS:1DIF:EasyREF:p. 1068 KEY:Nursing process: N/A | Client need: HPM | Cognitive level: Recall

The nurse is assessing an intubated patient who returned from coronary artery bypass surgery 3 hours ago. Which assessment finding might indicate that this patient is experiencing pain? 1) Blood pressure 160/82 mm Hg 2) Temperature 100.6°F 3) Heart rate 80 beats/min 4) Oxygen saturation 95%

ANS: 1 This patient has an elevation in blood pressure which is a physiological finding associated with pain. The patient has a mild temperature elevation, which is a common response to surgery. Heart rate and oxygen saturation are within normal limits. PTS: 1 DIF: Moderate REF: pp. 1095-1096 KEY: Nursing process: Assessment | Client need: PHSI | Cognitive level: Application

A 52-year-old man has a triceps skinfold thickness of 18 mm, and his weight exceeds the ideal body weight for his height by 23%. Which nursing diagnosis should the nurse identify for this patient? 1) Imbalanced Nutrition: More Than Body Requirements 2) Risk for Imbalanced Nutrition: More Than Body Requirements 3) Imbalanced Nutrition: Less Than Body Requirements 4) Readiness for Enhanced Nutrition

ANS: 1 This patient has defining characteristics for the nursing diagnosis Imbalanced Nutrition: More Than Body Requirements: triceps skinfold thickness more than 15 mm in men and weight that is 20% over ideal for height and frame. The patient does not have defining characteristics for the other nursing diagnoses. PTS:1DIF:ModerateREF:p. 929 KEY: Nursing process: Diagnosis | Client need: PHSI | Cognitive level: Analysis

During which of the following developmental stages does a person tend to need the most hours of sleep? 1)Toddler 2)Adolescence 3)Middle adulthood 4)Older adulthood

ANS: 1 Toddlers (ages 1 to 3 years) require 12 to 14 hours of sleep in a 24-hour period. Adolescents (ages 12 to 18 years) usually need 8 to 9 hours in a 24-hour period. Middle-aged adults (ages 40 to 65 years) typically require 7 hours in a 24-hour period. Older adults (age 65 years and older) often need 5 to 7 hours of sleep in a 24-hour period. PTS:1DIF:EasyREF:p. 1204

The nurse identifies the nursing diagnosis Urinary Incontinence (Total) in an older adult patient admitted after a stroke. Urinary Incontinence places the patient at risk for which complication? 1) Skin breakdown 2) Urinary tract infection 3) Bowel incontinence 4) Renal calculi

ANS: 1 Urine contains ammonia, which may cause excoriation with prolonged contact with the skin. Bowel incontinence, not urinary incontinence, increases the patient's risk for urinary tract infection. Immobility and high consumption of calcium-containing foods increase the risk for renal calculi. PTS: 1 DIF: Moderate REF: p. 1021 KEY: Nursing process: Planning | Client need: PSI | Cognitive level: Application

5. The nurse identifies the nursing diagnosis Urinary Incontinence (Total) in an older adult patient admitted after a stroke. Urinary Incontinence places the patient at risk for which complication? 1) Skin breakdown 2) Urinary tract infection 3) Bowel incontinence 4) Renal calculi

ANS: 1 Urine contains ammonia, which may cause excoriation with prolonged contact with the skin. Bowel incontinence, not urinary incontinence, increases the patients risk for urinary tract infection. Immobility and high consumption of calcium-containing foods increase the risk for renal calculi.

Which of the following nursing diagnoses is written in correct format? Assume the facts are correct in all of them. 1) Readiness for Enhanced Nutrition 2) Pain related to stating, "On a scale of 1 to 5, it's a 5." 3) Impaired Mobility related to pain A.M.B. hip fracture 4) Risk for Infection related to compromised immunity A.M.B. fever

ANS: 1 Wellness diagnoses (e.g., Readiness for Enhanced . . .) are usually one-part statements. A pain ranking of 5 is a symptom of pain, not an etiology, so it should be preceded by "A.M.B." or "as manifested by." Hip fracture is a medical diagnosis that is causing an etiology of pain; therefore, it should be preceded by "secondary to." Risk diagnoses do not have symptoms, so it is not correct to put anything after "A.M.B." PTS: 1 DIF: Moderate REF: pp. 70-73 KEY: Nursing process: Diagnosis | Client need: SECE | Cognitive level: Application

Which of the following describes the difference between dehiscence and evisceration? 1) With dehiscence, there is a separation of one or more layers of wound tissue; evisceration involves the protrusion of internal viscera from the incision site. 2) Dehiscence is an urgent complication that requires surgery as soon as possible; evisceration is not as urgent. 3) Dehiscence involves the protrusion of internal viscera from the incision site; with evisceration, there is a separation of one or more layers of wound tissue. 4) Dehiscence involves rupture of subcutaneous tissue; evisceration involves damage to dermal tissue.

ANS: 1 With dehiscence there is a separation of one or more layers of wound tissue, whereas evisceration involves the protrusion of internal viscera from the incision site. Evisceration is an urgent complication usually requiring immediate surgical intervention. PTS:1DIF:ModerateREF:pp. 1229-1230 KEY: Nursing process: Assessment | Client need: PHSI | Cognitive level: Analysis

According to William Worden, which task in the grieving process takes longest to achieve? 1) Accepting that the loved one is gone 2) Experiencing the pain from the loss 3) Adjusting to the environment without the deceased 4) Investing emotional energy

ANS: 1 Worden described the tasks a grieving person must achieve. They progress from an initial numbness or denial through experiencing and working through pain and grief and eventually moving on with life. Shock with disbelief is not a Worden task. PTS:1DIF:EasyREF:p. 359 KEY: Nursing process: N/A | Client need: PSI | Cognitive level: Recall

What is the primary goal that the nurse should establish for a patient with an open wound? 1) The wound will remain free of infection throughout the healing process. 2) Client completes antibiotic treatment as ordered. 3) The wound will remain free of scar tissue at healing. 4) Client increases caloric intake throughout the healing process.

ANS: 1 Wounds healing by secondary intention are more prone to infection; therefore, the primary goal would be to prevent infection. Antibiotics may not be necessary, and the nurse can expect the formation of scar tissue in this particular situation. There is no evidence presented that the patient needs to increase caloric intake. PTS:1DIF:ModerateREF:p. 1227 KEY: Nursing process: Planning | Client need: PHSI | Cognitive level: Synthesis

To promote wound healing, the nurse is teaching a patient about choosing foods containing protein. The nurse will evaluate that learning has occurred if the patient recognizes which food(s) as an incomplete protein that should be consumed with a complementary protein? Choose all that apply. 1) Whole grain bread 2) Peanut butter 3) Chicken 4) Eggs

ANS: 1, 2 Incomplete protein foods do not provide all of the essential amino acids necessary for protein synthesis. Therefore, the nurse should inform the patient that whole grain bread and peanut butter should be consumed with a complementary protein. For example, they could be eaten together as a peanut butter sandwich. PTS:1DIF:ModerateREF:p. 901 KEY: Nursing process: Evaluation | Client need: PHSI | Cognitive level: Comprehension

The nurse is teaching a patient about the importance of reducing saturated fats in his diet. The nurse will recognize that learning has occurred if, upon questioning, the patient replies that he should read product labels to eliminate the intake of which saturated fat(s)? Choose all that apply. 1) Palm oil 2) Coconut oil 3) Canola oil 4) Peanut oil

ANS: 1, 2 Palm and coconut oils are sources of saturated fat that are contained in many processed foods. The patient should be encouraged to read product labels to eliminate them from his diet. Olive, canola, and peanut oils are unsaturated fats and should be substituted for saturated fats in the diet. PTS: 1 DIF: Moderate REF: p. 901 KEY: Nursing process: Evaluation | Client need: HPM | Cognitive level: Comprehension

A patient diagnosed with lung cancer who is receiving morphine (MS Contin) complains of constipation. Which instruction(s) by the nurse might help relieve the patient's constipation? Choose all that apply. 1) "Be sure the amount of fruit, vegetables, and fiber in your diet is adequate." 2) "Drink at least eight 8-ounce glasses of water each day." 3) "Avoid using stool softeners because they may become habit forming." 4) "Increase your exercise routine to include 1 hour of exercise a day."

ANS: 1, 2 The nurse should instruct the patient to be sure the amount of fruit, vegetables, and fiber in his diet is adequate, and increase fluid intake to eight, 8-ounce glasses of water per day. Stool softeners may also be used. The patient should also be encouraged to increase exercise; even walking a short distance may be helpful. It is not necessary to increase exercise to 1 hour of exercise a day. The patient may be physically able to walk only short distances. PTS:1DIF:ModerateREF:p. 1105 KEY: Nursing process: Interventions | Client need: PHSI | Cognitive level: Application

The nurse working in an ambulatory care program asks questions about the client's locus of control as a part of his assessment because of which of the following? Choose all that apply. 1) People who feel in charge of their own health are the easiest to motivate toward change. 2) People who feel powerless about preventing illness are least likely to engage in health promotion activities. 3) People who respond to direction from respected authorities often prefer a health promotion program supervised by a health provider. 4) People who feel in charge of their own health are less motivated by health promotion activities.

ANS: 1, 2, 3 Identifying a person's locus of control helps the nurse determine how to approach a client about health promotion. People who feel powerless about preventing illness are least likely to engage in health promotion activities. People who respond to direction from respected authorities often prefer a health promotion program that is supervised by a health provider. Clients who feel in charge of their own health are the easiest to motivate toward positive change. PTS:1DIFgrinifficultREF:p. 888 KEY: Nursing process: Assessment | Client need: PSI | Cognitive level: Comprehension

Select the factors known to affect sleep. Choose all that apply. 1)Age 2)Environment 3)Lifestyle 4)State of health 5)Ethnicity

ANS: 1, 2, 3, 4 Age, environment, lifestyle, and state of health are factors affecting sleep. Many older adults sleep less but require more rest. Alcohol, caffeine, nicotine use, and diet are examples of lifestyle factors that affect sleep. When a person is ill, she may sleep more or find that she cannot sleep because of pain or other factors associated with illness. Changes in environment also affect sleep. PTS:1DIF:EasyREF:pp. 1207-1208

Which of the following actions demonstrate how nurses promote health? 1) Role modeling 2) Educating patients and families 3) Counseling 4) Providing support

ANS: 1, 2, 3, 4 Nurses promote health by acting as role models, counseling, providing health education, and providing and facilitating support. PTS:1DIF:EasyREF:pp. 891-892 KEY: Nursing process: Interventions | Client need: HPM | Cognitive level: Comprehension

The nurse is implementing a wellness program based on data gathered from a group of low-income seniors living in a housing project. He is using the Wheels of Wellness as a model for his planned interventions. Which of the following interventions would be appropriate based on this model? Choose all that apply. 1) Creating a weekly discussion group focused on contemporary news 2) Facilitating a relationship between local pastors and residents of subsidized housing 3) Coordinating a senior tutorial program for local children at the housing center 4) Establishing an on-site healthcare clinic operating one day per week

ANS: 1, 2, 3, 4 The Wheels of Wellness model identifies the following dimensions of health: emotional, intellectual, physical, spiritual, social/family, and occupational. A weekly discussion group stimulates intellectual health. A relationship between local pastors and those living in subsidized housing creates a climate for spiritual health. A tutorial program offered by seniors to local children will facilitate occupational health. An on-site healthcare clinic addresses physical health. PTS:1DIFgrinifficultREF:p. 881 KEY: Nursing process: Interventions | Client need: HPM | Cognitive level: Analysis

Health promotion programs assist a person to advance toward optimal health. Which of the following activities might such programs include? Choose all that apply. 1) Disseminating information 2) Changing lifestyle and behavior 3) Prescribing medications to treat underlying disorders 4) Environmental control programs

ANS: 1, 2, 4 Health promotion programs may be categorized into four types: disseminating information, programs for changing lifestyle and behavior, environmental control programs, and wellness appraisal and health risk assessment programs. Prescribing medications to treat underlying disorders is an activity that fosters health focused at an individual level rather than at a group program level. PTS:1DIF:ModerateREF:p. 881 KEY: Nursing process: Interventions | Client need: HPM | Cognitive level: Recall

Which of the following questions would be effective for obtaining information from a patient? Choose all that apply. 1) "How did this happen to you?" 2) "What was your first symptom?" 3) "Why didn't you seek healthcare earlier?" 4) "When did you start having symptoms?"

ANS: 1, 2, 4 How, what, and when are acceptable lines of questioning. Asking "why" can put the patient on the defensive and may suggest disapproval, limiting the amount of information the patient is willing give. PTS:1DIF:ModerateREF:p. 45-46 KEY: Nursing process: Assessment | Client need: SECE | Cognitive level: Application

A patient has a history of COPD. His pulse oximetry reading is 97%. What other findings would indicate adequate tissue and organ oxygenation? Choose all that apply. 1) Normal urine output 2) Strong peripheral pulses 3) Clear breath sounds bilaterally 4) Normal muscle strength

ANS: 1, 2, 4 To determine adequacy of tissue oxygenation, assess respiration, circulation, and tissue/organ function. Good peripheral circulation is characterized by strong peripheral pulses. Impaired tissue oxygenation to the kidneys would result in abnormal kidney function (e.g., poor urine output). Hypoxic limb tissue would result in abnormal muscle functioning (e.g., muscle weakness and pain with exercise). Adequacy of tissue oxygenation cannot be determined by assessing pulmonary ventilation alone; circulation must also be assessed. PTS:1DIFgrinifficult REF: p. 1300; higher-order item, some of answer implied in text KEY: Nursing process: Assessment | Client need: PHSI | Cognitive level: Comprehension

Which aspects of healthcare are affected by a client's culture? Select all that apply. 1)How the clients views healthcare 2)How the client views illness 3)How the client will pay for healthcare services 4)The types of treatments the client will accept 5)When the client will seek healthcare services 6)The environment where the healthcare services are provided 7)The ease of accessibility of healthcare services

ANS: 1, 2, 4, 5 Culture affects clients' view of health and healthcare. It influences how they will define illness, when they will seek healthcare, and what treatments are acceptable in their culture. How services are paid for is related to economic status. Regardless of culture, anyone can be affected by previous healthcare experiences, the environment in which healthcare is provided, and accessibility of services. PTS:1DIF:ModerateREF:p. 27 KEY:Nursing process: N/A | Client need: SECE | Cognitive level: Recall

Which factors in a health history place a patient at risk for hearing loss? Choose all that apply. 1) Being an older adult 2) Childhood chickenpox 3) Frequent otitis media 4) Diabetes mellitus

ANS: 1, 3 Having had frequent ear infections (otitis media) places a patient at risk for hearing loss because of scarring that may have occurred. Older adults experience a generalized decrease in the number of nerve conduction fibers and structural changes in the ear, which cause hearing loss. Chickenpox and diabetes mellitus do not place the patient at risk for hearing loss. PTS: 1 DIF: Moderate REF: p. 1072 KEY: Nursing process: Assessment | Client need: PHSI | Cognitive level: Comprehension

Goals for Healthy People 2020 include which of the following? Choose all that apply. 1) Eliminate health disparities among various groups. 2) Decrease the cost of healthcare related to tobacco use. 3) Increase the quality and years of healthy life. 4) Decrease the number of inpatient days annually.

ANS: 1, 3 The four overarching goals of Healthy People 2020 are to (1) increase the quality and years of healthy life, free of disease, injury, and premature death; (2) eliminate health disparities and improve health for all groups of people; (3) create physical and social environments for people to live a healthy life; and (4) promote healthy development for people in all stages of life. PTS: 1 DIF: Moderate REF: p. 890 KEY: Nursing process: Planning | Client need: HPM | Cognitive level: Recall

Which instruction(s) should the nurse give to the patient complaining of constipation? Choose all that apply. 1) Drink at least eight glasses of water or non-caffeinated fluid per day. 2) Include a minimum of four servings of meat per day. 3) Consume a high-fiber diet. 4) Exercise as you feel necessary.

ANS: 1, 3 To prevent constipation, the nurse should instruct the patient to consume a high-fiber diet, drink at least eight glasses of water or non-caffeinated fluid per day, exercise regularly, and eat meals on a regular schedule. Caffeine can aggravate constipation. PTS:1DIF:ModerateREF:p. 929 KEY: Nursing process: Interventions | Client need: PHSI | Cognitive level: Comprehension

Which of the following are cues rather than inferences? Choose all correct answers. 1) Ate 50% of his meal 2) Patient feels better today 3) States, "I slept well" 4) White blood cell count 15,000/mm3

ANS: 1, 3, 4 Cues are what the client says and what you observe. "Just the facts." The only inference in the list is "slept well." What did the nurse observe to tell her the client slept well? Those would be cues. If the client states, "I slept well" it is a cue, because it is a fact—that is what the client stated. PTS:1DIF:EasyREF:p. 50 KEY: Nursing process: Assessment | Client need: SECE | Cognitive level: Comprehension

Which of the following factors influence normal lung volumes and capacities? Choose all that apply. 1) Age 2) Race 3) Body size 4) Activity level

ANS: 1, 3, 4 Normal lung volumes and capacities vary with body size, age, and exercise level. Volumes and capacities are higher in men, in large people, and in athletes. Race does not influence normal lung volumes and capacities. PTS:1DIF:EasyREF:p. 1305 KEY: Nursing process: Assessment | Client need: PHSI | Cognitive level: Comprehension

According to Pender's health promotion model, which variables must be considered when planning a health promotion program for a client? Choose all that apply. 1) Individual characteristics and experiences 2) Levels of prevention 3) Behavioral outcomes 4) Behavior-specific cognition and affect

ANS: 1, 3, 4 Pender identified three variables that affect health promotion: individual characteristics and experiences, behavior-specific cognition and affect, and behavioral outcomes. Levels of prevention were identified by Leavell and Clark; three levels relate to health protection. The levels differ based on their timing in the illness cycle. PTS: 1 DIF: Difficult REF: pp. 880-881 KEY: Nursing process: Planning | Client need: HPM | Cognitive level: Comprehension

The nurse is counseling a 17-year-old girl on smoking cessation. The nurse should include which of the following helpful tips in her education? Choose all that apply. 1) "Keep healthy snacks or gum available to chew instead of smoking a cigarette." 2) "Don't tell your friends and family you are trying to quit, until you feel confident that you'll be successful." 3) "Plan a time to quit when you will not have many other demands or stressors in your life." 4) "Reward yourself with an activity you enjoy when you quit smoking."

ANS: 1, 3, 4 People who are trying to quit smoking often are more successful when they are accountable to other people who are encouraging and supportive. Having something to chew (e.g., carrot sticks, gum, nuts, or seeds) can distract from the desire to smoke a cigarette. Setting a date to stop smoking and choosing a time of low stress are two strategies that help people be more successful with smoking cessation. Self-reward for meeting goals is a form of positive reinforcement. PTS:1DIF:ModerateREF:p. 1311, ESG Self-Care: Smoking Cessation Tips KEY: Nursing process: Interventions | Client need: HPM | Cognitive level: Comprehension

Which of the following are examples of nonselective mechanical débridement methods? Choose all that apply. 1) Wet-to-dry dressings 2) Sharp débridement 3) Whirlpool 4) Pulsed lavage

ANS: 1, 3, 4 Wet-to-dry dressings, sharp débridement, and pulsed lavage are all forms of mechanical débridement. They are nonselective forms, which means that healthy tissue as well as devitalized tissue can be removed with their use. Sharp débridement is a selective form of débridement. With sharp débridement, only devitalized tissue is removed. PTS:1DIF:ModerateREF:p. 1248 KEY: Nursing process: Interventions | Client need: SECE | Cognitive level: Comprehension

A nurse is caring for an 80-year-old patient of Chinese heritage. When planning outcomes for this patient, which actions by the nurse would meet the American Nurses Association standards for outcomes identification? Choose all that apply. 1) Developing culturally appropriate outcomes 2) Using the outcomes preprinted on the clinical pathway 3) Choosing the best outcome for the patient, regardless of the costs involved in bringing it about 4) Involving the patient and family in formulating the outcomes

ANS: 1, 4 ANA standard 3 includes "derives culturally appropriate expected outcomes from the diagnosis" and "involves the patient, family . . . in formulating expected outcomes. . . ." It is acceptable for the nurse to use outcomes on a clinical pathway, but these are not individualized; ANA standard 3 says that the nurse "identifies . . . outcomes for a plan individualized to the patient. . . ." The standard also says that the nurse should consider "associated risks, benefits, and costs. . . ." PTS:1DIF:ModerateREF: p. 82 KEY: Nursing process: Planning | Client need: SECE | Cognitive level: Comprehension

For a particular patient, it has become essential to minimize the risk of further damage to the auditory nerve. Which of the following medications may need to be discontinued if the patient is taking them? Choose all that are correct. 1) Furosemide, a diuretic 2) Digoxin, a cardiotonic 3) Famotidine, an antacid 4) Aspirin, an analgesic

ANS: 1, 4 Aspirin and furosemide may cause ototoxicity, leading to auditory nerve impairment. Digoxin and famotidine do not place the patient at risk for auditory nerve impairment. PTS:1DIFgrinifficultREF:p. 1070 KEY: Nursing process: Planning | Client need: SECE | Cognitive level: Comprehension

To be eligible for insurance benefits covering hospice care, a physician must certify that which of the following apply to the patient? Choose all that apply. 1) Life expectancy is not more than 6 months. 2) Life expectancy is not more than 12 months. 3) Condition is expected to improve slightly. 4) Condition is not expected to improve.

ANS: 1, 4 For a patient to be eligible for hospice care insurance benefits, a physician must certify that the patient is not expected to improve or will most likely die within 6 months. PTS:1DIF:ModerateREF:p. 363 KEY: Nursing process: N/A | Client need: SECE | Cognitive level: Recall

Which of the following is/are accurate about nasotracheal suctioning? Choose all that apply. 1) Apply suction for no longer than 10-15 sec during a single pass. 2) Apply suction while inserting and removing the catheter. 3) Reapply oxygen between suctioning passes for ventilator patients. 4) Gently rotate the suction catheter as you remove it.

ANS: 1, 4 Limiting suctioning to 10 seconds or less and reapplying oxygen between suctioning passes prevent hypoxia. Suction should be applied only while withdrawing the catheter, using a continuous rotating motion to prevent trauma to the airway. Endotracheal suctioning is used when the patient is being mechanically ventilated, and most ventilator patients have in-line suctioning, so there is no need to reapply oxygen. PTS: 1 DIF: Moderate REF: p. 1347 KEY: Nursing process: Interventions | Client need: PHSI | Cognitive level: Application

The nurse is teaching a patient about her chest drainage system. Which of the following should the nurse include in the teaching? Choose all that apply. 1) Perform frequent coughing and deep-breathing exercises. 2) Sit up in a chair but do not walk while the drainage system is in place. 3) Get out of bed without assistance as much as possible. 4) Immediately notify the nurse if she experiences increased shortness of breath

ANS: 1, 4 Patients should regularly perform coughing and deep-breathing exercises to promote lung reexpansion. Also to promote lung reexpansion, the nurse should encourage the patient to be as active as her condition permits, rather than telling her not to walk. Chest drainage systems are bulky, but patients with disposable systems can still get out of bed and ambulate. However, the patient will need assistance from one or two staff members to protect and monitor the system and to monitor her responses to activity; she should not get out of bed on her own. If a patient with a chest drainage system becomes acutely short of breath, the patient should immediately notify the nurse so the nurse can check for occlusion of the system, which can result in a tension pneumothorax. PTS:1DIF:ModerateREF:pp. 1313, 1358 KEY: Nursing process: Interventions | Client need: PHSI | Cognitive level: Comprehension

A nurse with a large caseload of patients needs to delegate some assessment tasks to other members of the health team. The nurse is unsure which tasks can be delegated to nursing assistive personnel (NAP) and which are appropriate for a licensed practical nurse (LPN) or a registered nurse (RN). To which sources should the nurse turn for the answer to his question? Choose all that are appropriate. 1) The nurse practice act of his state 2) The American Medical Association guidelines 3) The Code of Ethics for Nurses 4) The American Nurses Association's Scope and Standards of Practice

ANS: 1, 4 State nurse practice acts specify which portions of the assessment can legally be completed by individuals with different credentials. The ANA Scope and Standards of Practice provide a guide for who is ultimately responsible and qualified to collect assessment data. The American Medical Association provides guidelines and standards for physicians, not nurses. The Code of Ethics for Nurses says merely that the nurse should delegate tasks appropriately; it does not speak to credentials of personnel. PTS:1DIF:ModerateREF:p. 40 KEY: Nursing process: Assessment | Client need: SECE | Cognitive level: Comprehension

For a patient with Risk for Imbalanced Nutrition: Less Than Body Requirements related to Impaired Swallowing, which nursing interventions are appropriate? Choose all that apply. 1) Check inside the mouth for pocketing of food after eating. 2) Provide a full liquid diet that is easy to swallow. 3) Remind the patient to raise the chin slightly to prepare for swallowing. 4) Keep the head of the bed elevated for 30 to 45 minutes after feeding.

ANS: 1, 4 The nurse should check for pocketing of food that the patient has not been able to swallow, and should keep the head of the bed elevated for 30 to 45 minutes after feeding. Liquids should be avoided unless thickeners are added. The patient should flex the head forward (tuck the chin) in preparation for swallowing. PTS:1DIF:ModerateREF:pp. 927-928 KEY: Nursing process: Interventions | Client need: SECE | Cognitive level: Application

Why is an accurate description of the location of a wound important? Choose all that apply. 1) Influences the rate of healing 2) Determines the appropriate treatment choice 3) Will affect the frequency of dressing changes 4) Affects patient movement and mobility

ANS: 1, 4 Wounds in highly vascular areas heal more rapidly than wounds in less vascular regions. Wounds that can be stabilized also heal more readily than those in areas of stress. Treatment choices and frequency of dressing changes will be dependent on the condition of the wound, not the location. PTS:1DIF:ModerateREF:p. 1235 KEY: Nursing process: Planning | Client need: PHSI | Cognitive level: Analysis

Which daily urine output is within normal limits for a newborn weighing 8 pounds? 1) 288 mL 2) 180 mL 3) 36 mL 4) 18 mL

ANS: 2 A newborn weighing 8 pounds (3.6 kg) should produce 15 to 60 mL of urine per kilogram per day. If the newborn produces 50 mL/kg/day and weighs 3.6 kg, he will produce a total of 180 mL in 24 hours. The other options are not within normal limits and require further assessment. PTS:1DIF:DifficultREF:p. 1015 KEY: Nursing process: Assessment | Client need: PSI | Cognitive level: Analysis

During a thermometer exchange program at a local hospital, a person drops a mercury thermometer on the floor. Assume the nurse has been trained in cleanup of such a spill. Select all that are appropriate. How should the nurse intervene? 1) Using gloves and a paper towel, place the mercury in a plastic bag, and dispose of it. 2) Notify the hazardous material management team immediately. 3) Evacuate the area immediately. 4) After putting on a gown, gloves, and a mask, clean up the mercury. 5) Wash her hands well after removing the spill. 6) Ventilate the area well for several days.

ANS: 1, 5, 6 The nurse should put on gloves and use a paper towel to pick up the mercury. Then place the mercury, broken thermometer, and soiled paper towel into a plastic bag along with the gloves. Next, the nurse should dispose of the plastic bag, wash her hands, and ventilate the area well. It is not necessary to notify the hazardous material management team or evacuate the area for a spill this small, unless agency policy actually mandates that. The nurse does not need to put on a gown and mask to dispose of the mercury. PTS: 1 DIF: Moderate REF: pp. 674-675 KEY: Nursing process: Interventions | Client need: SECE | Cognitive level: Application

Of the following, which is the best choice for performing wound irrigation? 1) Water jet irrigation 2) 35-cc syringe with a 19-gauge angiocatheter 3) 5-cc syringe with a 23-gauge needle 4) Bulb syringe

ANS: 2 A 35-cc syringe with a 19-gauge angiocatheter is the best choice for irrigation because it will deliver the irrigation solution at approximately 8 psi. The water jet irrigation unit and 5-cc syringe with a 23-gauge needle would deliver the solution above the recommended pressure range of 4 to 15 psi. A bulb syringe is not an appropriate choice because there is an increased risk of aspirating drainage from the wound. PTS: 1 DIF: Moderate REF: p. 1246 KEY: Nursing process: Interventions | Client need: SECE | Cognitive level: Application

For which patient would it be most important to perform a comprehensive discharge plan? 1) A teen who is a first-time mother, single, and lives with her parents 2) An older adult who has had a stroke affecting the left side of his body and lives alone 3) A middle-aged man who has had outpatient surgery on his knee and requires crutches 4) A young woman who was admitted to the hospital for observation following an accident

ANS: 2 A comprehensive discharge plan should be developed for older adults and anyone who has complex needs, including self-care deficits. The other patients do not have the complex needs of the older adult patient who has had a stroke that affects body function. PTS:1DIF:ModerateREF:p. 83 KEY: Nursing process: Planning | Client need: PHSI | Cognitive level: Analysis

A few nurses on a unit have proposed to the nurse manager that the process for documenting care on the unit be changed. They have described a completely new system. Why is it important for the nurse manager to have a critical attitude? It will help the manager to 1)Consider all the possible advantages and disadvantages 2)Maintain an open mind about the proposed change 3)Apply the nursing process to the situation 4)Make a decision based on past experience with documentation

ANS: 2 A critical attitude enables the person to think fairly and keep an open mind. PTS:1DIF:ModerateREF:pp. 26 KEY: Nursing process: N/A | Client need: SECE | Cognitive level: Comprehension

The most obvious reason for using a framework when assessing a patient is to 1) Prioritize assessment data 2) Organize and cluster data 3) Separate subjective and objective data 4) Identify primary from secondary data

ANS: 2 A framework is used to organize and cluster data to find patterns. During the assessment phase, the nurse is collecting and recording data, not prioritizing the data. A framework includes subjective and objective data as well as primary and secondary data; it does not help you to separate them. PTS:1DIF:EasyREF:p. 48 KEY: Nursing process: Assessment | Client need: SECE | Cognitive level: Recall

A patient has left-sided weakness because of a recent stroke. Which type of special needs assessment would it be most important to perform? 1) Family 2) Functional 3) Community 4) Psychosocial

ANS: 2 A functional assessment is most important because of discharge needs (e.g., self-care ability at home) and patient safety. A family and community assessment would be helpful to evaluate support systems, and a psychosocial assessment would be helpful to evaluate a patient's understanding of and coping with his recent stroke. Remember that special needs assessments are lengthy and time-consuming, so they should be used only when in-depth information is needed about a topic. PTS:1DIF:ModerateREF:pp. 43 KEY: Nursing process: Assessment | Client need: SECE | Cognitive level: Analysis

4. Which medication class will the primary care provider most likely prescribe to increase urine output in the patient admitted with congestive heart failure? 1) Thiazide diuretic 2) Loop diuretic 3) MAO inhibitor 4) Anticholinergic

ANS: 2 A loop diuretic [e.g., Furosemide (Lasix)] increases urine elimination. It works by limiting the reabsorption of water in the renal tubules and is used to reduce congestion in the cardiopulmonary circulation. A thiazide diuretic is used to treat high blood pressure by reducing the amount of sodium and water in the blood vessels. An MAO inhibitor [e.g., phenelzine (Nardil)] is an antidepressant that is used after other medications have proven unsuccessful in lifting symptoms of serious depression. Anticholinergics [e.g., ipratropium (Atrovent)] relax smooth muscle in the airways. Also known as antispasmodics, they reduce airway constriction experienced by those with asthma, for example.

Which medication class will the primary care provider most likely prescribe to increase urine output in the patient admitted with congestive heart failure? 1) Thiazide diuretic 2) Loop diuretic 3) MAO inhibitor 4) Anticholinergic

ANS: 2 A loop diuretic [e.g., Furosemide (Lasix)] increases urine elimination. It works by limiting the reabsorption of water in the renal tubules and is used to reduce congestion in the cardiopulmonary circulation. A thiazide diuretic is used to treat high blood pressure by reducing the amount of sodium and water in the blood vessels. An MAO inhibitor [e.g., phenelzine (Nardil)] is an antidepressant that is used after other medications have proven unsuccessful in lifting symptoms of serious depression. Anticholinergics [e.g., ipratropium (Atrovent)] relax smooth muscle in the airways. Also known as antispasmodics, they reduce airway constriction experienced by those with asthma, for example. is a cholesterol-lowering drug. Although high cholesterol is a leading factor for heart disease, the medication is used to reduce cholesterol in the blood—not to promote diuresis to reduce the demand on the heart and backflow into the lungs. PTS:1DIF:ModerateREF:p. 1016; not stated directly in the text and requires critical thinking KEY: Nursing process: Planning | Client need: SECE | Cognitive level: Application

Which of the following is an example of a problem that nurses can treat independently? 1) Hemorrhage 2) Nausea 3) Fracture 4) Infection

ANS: 2 A nursing diagnosis (or nursing problem) is a human response to a disease, injury, or other stressor that nurses can identify, prevent, or treat independently. Nausea is the only problem that meets that criterion; all others are medical or collaborative problems. PTS:1DIF:ModerateREF: pp. 57 KEY: Nursing process: Diagnosis | Client need: PHSI | Cognitive level: Application

Which patient is most likely experiencing positive nitrogen balance? A patient admitted: 1) With third-degree burns of his legs. 2) In the sixth month of a healthy pregnancy. 3) From a nursing home who has been refusing to eat. 4) With acute pancreatitis.

ANS: 2 A positive nitrogen balance typically exists during pregnancy when new tissues are being formed. Patients with burns, malnutrition, and serious illness commonly experience negative nitrogen balance because tissues are lost. PTS:1DIF:ModerateREF:p. 902 KEY: Nursing process: Assessment | Client need: PHSI | Cognitive level: Application

How does a risk nursing diagnosis differ from a possible nursing diagnosis? 1) A risk diagnosis is based on data about the patient. 2) A possible diagnosis is based on partial (or incomplete) data. 3) Nurses collect the data to support risk diagnoses. 4) A possible diagnosis becomes an actual diagnosis when symptoms develop.

ANS: 2 A possible nursing diagnosis is based on nursing knowledge, intuition, and experience and does not have enough data to support it; it is based on incomplete data. A risk diagnosis describes a problem that may develop in a vulnerable client if nursing care is not initiated to prevent it; it is made when risk factors are present in the data. Nurses collect data to support both risk and possible diagnoses; therefore, this statement does not differentiate them. A risk diagnosis becomes an actual diagnosis when symptoms develop. PTS:1DIF:ModerateREF:p. 60 KEY: Nursing process: Diagnosis | Client need: SECE | Cognitive level: Analysis

What is the purpose of using a sleep diary? 1)Identify sleep-rest patterns over a 1-year period. 2)Note the trend in sleep-wakefulness patterns over a 2-week period. 3)Note typical sleep habits and most common daily routines. 4)Examine the patterns of sleep during the night and naps during the day.

ANS: 2 A sleep diary provides specific information about the patient's sleep-wakefulness patterns over a certain period of time. It allows identification of trends in sleep-wakefulness patterns and associates specific behaviors interfering with sleep. The diary is typically kept for 14 days. PTS: 1 DIF: Moderate REF: p. 1212

According to the Uniform Determination of Death Act, which bodily function must be lost to declare death? 1) Consciousness 2) Brain stem function 3) Cephalic reflexes 4) Spontaneous respirations

ANS: 2 According to the Uniform Determination of Death Act, death can be declared when there is a loss of brain stem function. Higher-brain death occurs when there is a loss of consciousness, cephalic reflexes, and spontaneous respirations. PTS:1DIF:ModerateREF:p. 362 KEY: Nursing process: Assessment | Client need: PSI | Cognitive level: Recall

What is a common characteristic of aging skin? 1) Increased permeability to moisture 2) Diminished sweat gland activity 3) Reduced oxygen-free radicals 4) Overproduction of elastin

ANS: 2 Aging skin tends to be drier. Sweat gland activity is diminished. The skin's connective tissue, collagen, and elastin are reduced, which means the skin loses firmness and so wrinkles. Skin aging also occurs with exposure to oxygen-free radicals that are waste products from chemical reactions in the body as well as with exposure to certain food and environmental sources. An infant's skin is thinner and more permeable to moisture in the environment. PTS:1DIF:ModerateREF:p. 1224 KEY: Nursing process: Assessment | Client need: PHSI | Cognitive level: Comprehension

For which of the following purposes is a graphic flow sheet superior to other methods of recording data? 1) Easy documentation of routine vital signs 2) Seeing the patterns of a patient's fever 3) Describing the symptoms accompanying a rising temperature 4) Checking to make sure vital signs were taken

ANS: 2 All are benefits of the graphic flow sheet, but to easily and graphically see trends over time, the graphic flow sheet is superior to other methods of documentation. For the other options, other kinds of flow sheets would be equally effective. PTS:1DIF:ModerateREF:p. 50 KEY: Nursing process: N/A | Client need: SECE | Cognitive level: Analysis

Which patient is at most risk for experiencing difficult grieving? 1) The middle-aged woman whose grandmother died of advanced Parkinson's disease 2) The young adult with three small children whose wife died suddenly in an accident 3) The middle-aged person whose spouse suffered a slow, painful death 4) The older adult whose spouse died of complications of chronic renal disease

ANS: 2 Although it is impossible to predict with certainty and the grieving process is highly individual and personal, in general those who suffer a sudden loss typically have more difficult grieving than those who have had the time to prepare for the death. Family and friends of persons with chronic illnesses (e.g., cancer) have usually had time to emotionally prepare for the death, initiate the funeral and burial arrangements, and begin the grieving process before the death occurs. PTS: 1 DIF: Moderate REF: p. 360 KEY: Nursing process: Diagnosis | Client need: PSI | Cognitive level: Analysis

. A patient had abdominal surgery. The incision has been closed by primary intention, and the staples are intact. To provide more support to the incision site and decrease the risk of dehiscence, it would be appropriate to apply which of the following? 1) Steri-Strips 2) Abdominal binder 3) T-binder 4) Paper tape

ANS: 2 An abdominal binder provides added support to an incision site and decreases the risk of wound dehiscence. A T-binder is used in the perineal area. Steri-Strips and paper tape would not be needed for an approximated incision that has intact staples, sutures, or surgical glue. PTS:1DIF:EasyREF:p. 1253 KEY: Nursing process: Interventions | Client need: SECE | Cognitive level: Application

Based only on Maslow's hierarchy of needs, which nursing diagnosis should have the highest priority? 1) Self-care Deficit 2) Risk for Aspiration 3) Impaired Physical Mobility 4) Disturbed Sensory Perception

ANS: 2 Highest priority is given to problems that are life threatening or that could be destructive to the client. Safety is most basic in Maslow's hierarchy. Even though Risk for Aspiration is not an actual problem, it poses the most immediate life-threatening risk to the client, and nursing interventions must be performed to prevent it from becoming an actual problem. PTS:1DIF:ModerateREF:pp. 64-65 KEY: Nursing process: Diagnosis | Client need: PHSI | Cognitive level: Analysis

A patient tells you that she has trouble falling asleep at night, even though she is very tired. A review of symptoms reveals no physical problems and she takes no medication. She has recently quit smoking, is trying to eat healthier foods, and has started a moderate-intensity exercise program. Her sleep history reveals no changes in bedtime routine, stress level, or environment. Based on this information, the most appropriate nursing diagnosis would be Disturbed Sleep Pattern related to: 1)Increased exercise. 2)Nicotine withdrawal. 3)Caffeine intake. 4)Environmental changes.

ANS: 2 Based on the information given, the patient is not experiencing significant stress or change in sleep routine or environment, which commonly lead to insomnia. People who use nicotine tend to have more difficulty falling asleep and are more easily aroused than those who are nicotine free. People who stop smoking often experience temporary sleep disturbances during the withdrawal period. PTS:1DIF:ModerateREF:p. 1208

A patient with trigeminal neuralgia is prescribed a mechanical soft diet. This diet places the patient at risk for which complication? 1) Dehydration 2) Constipation 3) Hyperglycemia 4) Diarrhea

ANS: 2 Because of its lack of fiber, a mechanical soft diet places the patient at risk for constipation. It does not place the patient at risk for dehydration, hyperglycemia, or diarrhea. PTS:1DIF:ModerateREF:p. 917 KEY: Nursing process: Assessment | Client need: PHSI | Cognitive level: Comprehension

Skin integrity and wound healing are compromised in the client who takes blood pressure medications because antihypertensives: 1) Can cause cellular toxicity. 2) Increase the risk of ischemia. 3) Delay wound healing. 4) Predispose to hematoma formation.

ANS: 2 Blood pressure medications decrease the amount of pressure required to occlude blood flow to an area, creating a risk for ischemia. Chemotherapeutic agents delay wound healing because of their cellular toxicity. Anticoagulants can lead to extravasation of blood into subcutaneous tissue, predisposing to hematoma formation with minimal pressure or injury. PTS:1DIFgrinifficultREF:p. 1224 KEY: Nursing process: Assessment | Client need: PHSI | Cognitive level: Comprehension

What do critical thinking and the nursing process have in common? 1)They are both linear processes used to guide one's thinking. 2)They are both thinking methods used to solve a problem. 3)They both use specific steps to solve a problem. 4)They both use similar steps to solve a problem.

ANS: 2 Critical thinking and the nursing process are ways of thinking that can be used in problem solving (although critical thinking can be used beyond problem-solving applications). Neither method of thinking is linear. The nursing process has specific steps; critical thinking does not. PTS:1DIFgrinifficultREF: p. 31

Which side effects associated with opioid use may improve after taking a few doses of the drug? 1) Constipation 2) Drowsiness 3) Dry mouth 4) Difficulty with urination

ANS: 2 Drowsiness as well as nausea are side effects of opioid therapy that commonly improve after a few doses are administered. Other side effects include constipation, vomiting, dry mouth, and difficulty with urination. These side effects do not typically lessen with use. PTS:1DIF:ModerateREF:p. 1104 KEY: Nursing process: Planning | Client need: SECE | Cognitive level: Comprehension

Which of the following examples includes both objective and subjective data? 1)The client's blood pressure is 132/68 and her heart rate is 88. 2)The client's cholesterol is elevated, and he states he likes fried food. 3)The client states she has trouble sleeping and that she drinks coffee in the evening. 4)The client states he gets frequent headaches and that he takes aspirin for the pain.

ANS: 2 Elevated cholesterol is objective, and "states he likes fried food" is subjective. Objective data can be observed by someone other than the patient (e.g., from physical assessments or lab and diagnostic tests). Subjective data are information given by the client. Blood pressure and heart rate measurements are both objective. "States . . . trouble sleeping and . . . drinks coffee . . ." are both subjective. States ". . . frequent headaches and . . . takes aspirin . . ." are both subjective. PTS:1DIF:ModerateREF:pp. 40 KEY: Nursing process: Assessment | Client need: PHSI | Cognitive level: Analysis

The nurse has been teaching a parent about stimuli to develop her infant's auditory nervous system. Which behavior by a parent toward the child provides evidence that learning occurred? 1) Cuddling 2) Speaking 3) Feeding 4) Soothing

ANS: 2 Exposure to voices, music, and ambient sound helps develop the infant's auditory nervous system. Cuddling, feeding, and soothing provide comfort and pleasure and teach the infant about his external environment. PTS:1DIF:EasyREF:p. 1069 KEY: Nursing process: Evaluation | Client need: HPM | Cognitive level: Recall

21. A patient is prescribed furosemide (Lasix), a loop diuretic, for treatment of congestive heart failure. The patient is at risk for which electrolyte imbalance associated with use of this drug? 1) Hypocalcemia 2) Hypokalemia 3) Hypomagnesemia 4) Hypophosphatemia

ANS: 2 Furosemide is a loop diuretic, which causes potassium to pass into the urine. This drug increases the risk for hypokalemia (low potassium); it does not cause hypocalcemia (low calcium in the blood), hypomagnesemia (low blood magnesium), or hypophosphatemia (low blood phosphorous).

A patient is prescribed furosemide (Lasix), a loop diuretic, for treatment of congestive heart failure. The patient is at risk for which electrolyte imbalance associated with use of this drug? 1) Hypocalcemia 2) Hypokalemia 3) Hypomagnesemia 4) Hypophosphatemia

ANS: 2 Furosemide is a loop diuretic, which causes potassium to pass into the urine. This drug increases the risk for hypokalemia (low potassium); it does not cause hypocalcemia (low calcium in the blood), hypomagnesemia (low blood magnesium), or hypophosphatemia (low blood phosphorous). PTS:1DIF:ModerateREF:p. 1016, not stated directly in the text and requires critical thinking KEY: Nursing process: Assessment | Client need: SECE | Cognitive level: Comprehension

Which is one of the greatest concerns with heavy and chronic use of alcohol in teens and young adults? 1) Liver damage 2) Unintentional death 3) Tobacco use 4) Obesity

ANS: 2 Heavy and chronic use of alcohol and use of illicit drugs increase the risk of disease and injuries and intentional death (suicide and homicide). Although alcohol as a depressant slows metabolism, chronic alcohol use is more likely associated with poor nutrition, which may or may not lead to obesity. Chronic alcohol use causes damage to liver cells over time in the later years. Alcohol intake is often associated with tobacco and recreational drug use; however, the risk of unintentional injury, such as car accident, suicide, or violence, is more concerning than smoking. PTS:1DIF:EasyREF:p. 879 KEY: Nursing process: Planning | Client need: HPM | Cognitive level: Comprehension

Which intervention by the nurse is most appropriate when she notices that her dying patient has developed a "death rattle"? 1) Perform nasotracheal suctioning of secretions. 2) Turn the patient on his side and raise the head of the bed. 3) Insert a nasopharyngeal airway as needed. 4) Administer morphine sulfate intravenously.

ANS: 2 If a "death rattle" occurs, turn the patient on his side, and elevate the head of the bed. Nasotracheal suctioning and inserting a nasopharyngeal airway are ineffective against a "death rattle" and may cause the patient unnecessary discomfort. The patient may require IV morphine sulfate to treat pain, but it does not help stop a "death rattle." This narcotic analgesic can also reduce the respiratory drive, leading to hypoventilation and respiratory depression or arrest. PTS:1DIF:ModerateREF:p. 376 KEY: Nursing process: Interventions | Client need: PHSI | Cognitive level: Application

A 78-year-old patient is being seen in the emergency department. The nurse observes his gait and balance appear to be slightly unsteady. What assessment should the nurse perform next? 1) Perform the Get Up and Go Test. 2) Ask the patient if he has fallen in the past year. 3) Refer the patient for a comprehensive fall evaluation. 4) Administer the Timed Up and Go Test.

ANS: 2 If a patient's gait or balance is unsteady, the nurse should question the patient for a history of falls. If the patient reports a single fall, the nurse should do the Get Up and Go Test. If the patient has difficulty with that test, or is unsteady with it, the nurse should perform a follow-up assessment of gait and balance by having the person close the eyes for a few seconds wile standing in place; stand with eyes closed while the nurse pushes gently on the sternum; walk, stop, turn around, return to the chair, and sit in the chair without using his arms for support. Physicians and advanced practitioners perform the Timed Up and Go Test; it is recommended annually for patients 65 years or older. PTS:1DIFgrinifficultREF:p. 661 KEY: Nursing process: Assessment | Client need: SECE | Cognitive level: Application

What is wrong with the format of this diagnostic statement: Possible Risk for Constipation related to irregular defecation habits A.M.B. statement that "When I'm busy, I can't always take the time to go to the bathroom." 1) Possible nursing diagnoses do not have signs and symptoms. 2) A nursing diagnosis is either a possible risk or a risk, not both. 3) Constipation is a medical diagnosis. 4) The etiology is actually a defining characteristic.

ANS: 2 If there are risk factors, it is not a possible diagnosis, it is a risk diagnosis. It is possible to have a "possible risk for" diagnosis. The patient with possible diagnoses may have symptoms, just not enough to support the diagnosis. Constipation is a nursing diagnosis, and the etiology is a defining characteristic for a risk diagnosis because it contributes to the problem. In risk diagnoses, the etiology consists of the risk factors. PTS: 1 DIF: Moderate REF: pp. 60 KEY: Nursing process: Diagnosis | Client need: SECE | Cognitive level: Analysis

You are caring for an adult patient with a tracheostomy who is being mechanically ventilated. His pulse oximetry reading is 85%, heart rate is 113, and respiratory rate is 30. The patient is very restless. His respirations are labored, and you hear gurgling sounds. You auscultate crackles and rhonchi in both lungs. What is the most appropriate action to take? 1) Call the respiratory therapist to check the ventilator settings. 2) Provide endotracheal suctioning. 3) Provide tracheostomy care. 4) Notify the physician of the patient's signs of fluid overload.

ANS: 2 Increased pulse and respiratory rates, decreased oxygen saturation, gurgling sounds during respiration, auscultation of adventitious breath sounds, and restlessness are signs that indicate the need for suctioning. Airways are suctioned to remove secretions and maintain patency. The patient's symptoms should subside once the airway is cleared. PTS: 1 DIF: Moderate REF: p. 1318, 1342 KEY: Nursing process: Interventions | Client need: PHSI | Cognitive level: Application

A client arrives in the emergency department, pale and breathing rapidly. He immediately becomes unconscious and collapses to the floor. The nurse rapidly assesses the patient and decides the first series of actions that are needed. This scenario demonstrates 1) Formal planning 2) Informal planning 3) Ongoing planning 4) Initial planning

ANS: 2 Informal planning is performed while doing other nursing process steps and is not written; this nurse is forming a plan in her mind. The end product of formal planning is a holistic plan of care that addresses the patient's unique problems and strengths; this nurse has no time to create a holistic plan of care. Ongoing planning refers to changes made in the plan as you evaluate the patient's responses to care; no care has been given at this point. Initial planning does indeed begin with the first patient contact. However, it refers to the development of the initial comprehensive plan or care; this nurse does not have enough data for a comprehensive plan, nor does she have time to make such a plan at the moment. PTS:1DIF:EasyREF:p. 81 KEY: Nursing process: Planning | Client need: SECE | Cognitive level: Analysis

How is NOC different from the Omaha System? 1) NOC can be used to write health restoration outcomes. 2) NOC can be used in all specialty and practice areas. 3) NOC can be used for individuals, families, or groups. 4) NOC formulates goals based on nursing diagnoses.

ANS: 2 NOC was developed for all specialty and practice areas. The Omaha System was developed for community health nursing. Both address health restoration and can be used for individuals, family, or groups (community). Both base goals on nursing diagnoses, although Omaha does not use the NANDA-I taxonomy. PTS:1DIF:ModerateREF: p. 95; answer based on analysis of text discussion | V1, p. 98; answer based on analysis of text discussion KEY: Nursing process: Planning | Client need: SECE | Cognitive level: Analysis

Throughout the course of his illness, a patient has denied its seriousness, even though his health professionals have explained prognosis of death very clearly. Physiologic signs now indicate that he will probably die within a short period of time, but he is still firmly in a state of emotional denial. The patient says to the nurse, "Tell my wife to stop hovering and go home. I'm going to be fine." How should the nurse respond? 1) "Your physical signs indicate that you will likely not live more than a few more days." 2) "You seem very sure that you are not going to die. Please tell me more about what you are feeling." 3) "It seems to me you would be feeling some anger and wondering why all this is happening to you." 4) "It would be best for your family if you were able to work through this and come to accept the reality of your situation."

ANS: 2 Not all patients go through all the traditional stages of grieving. It is not the nurse's responsibility to move patients sequentially through each stage of the dying and grieving process with the goal that everyone ends life accepting death. It is a nursing responsibility to accept and support people "where they are" and help them to express their feelings. Nurses need to understand patients, not change them. In this situation, denial may be very important to this patient, as an emotional defense and coping strategy. "You seem sure . . . tell me . . . what you are feeling" restates what the patient has said (indicating understanding) and encourages expression of feelings—both are supportive. Even though moving him through stages is not the goal in this situation, support does facilitate that. Telling the patient that his physical signs indicate that death is imminent is presenting truth and reality; however, the exact time of death is not always predictable. Forecasting the hour of death can have negative impact on the family as they anticipate the event with emotion and exhaustion. Presenting reality is appropriate in certain circumstances earlier in the dying process, but not in this situation because it has already been tried with no change in the patient. Presenting reality does not support the patient's needs at this time. Saying "It seems to me you would be feeling some anger . . ." is directed toward moving the patient from denial and suggesting he should feel something he has not yet expressed. This is not therapeutic. Saying "It would be best for your family . . ." presumes that the nurse knows more about what is "best" for the patient's family than the patient himself. This statement is also judgmental. PTS: 1 DIF: Difficult REF: pp. 362-363 KEY: Nursing process: Interventions | Client need: PSI | Cognitive level: Application

Direct visualization of the urethra, bladder, and ureteral orfices by insertion of a scope, May be used to obtain biopsies and treat pathology of visualized areas:

Cytoscopy

Which is the most commonly reported incident in hospitals? 1) Equipment malfunction 2) Patient falls 3) Laboratory specimen errors 4) Treatment delays

ANS: 2 Patient falls are by far the most common incident reported in hospitals and long-term care facilities. Although equipment (e.g., infusion pump) malfunctions, missed or incorrectly identified laboratory specimen collection, and treatment delays sometimes occur, they do not occur as frequently as patient falls. PTS:1DIF:EasyREF:p. 657 KEY: Nursing process: N/A | Client need: SECE | Cognitive level: Comprehension

A middle-aged patient with a history of alcohol abuse is admitted with acute pancreatitis. This patient will most likely be deficient in which nutrients? 1) Iron 2) B vitamins 3) Calcium 4) Phosphorus

ANS: 2 Patients who regularly abuse alcohol may be deficient in many nutrients; however, they are commonly deficient in the B vitamins and folic acid. Vitamin A deficiency can be associated with night blindness in heavy drinkers; vitamin D deficiency leads to softening of the bones. Because some alcoholics are deficient in vitamins A, C, D, E, and K and the B vitamins, they experience delayed wound healing. In particular, because vitamin K, the vitamin needed for blood clotting, is commonly deficient in those who regularly abuse alcohol, those patients can have delayed clotting, resulting in excess bleeding. Deficiencies of other vitamins involved in brain function can cause severe neurological damage. PTS:1DIF:ModerateREF:p. 915 KEY: Nursing process: Assessment | Client need: PHSI | Cognitive level: Application

In which process do peripheral nerves carry the pain message to the dorsal horn of the spinal cord? 1) Transduction 2) Transmission 3) Perception 4) Modulation

ANS: 2 Peripheral nerves carry the pain message to the dorsal horn of the spinal cord during a process known as transmission. In a process called transduction, specialized nociceptors convert potentially damaging mechanical, thermal, and chemical stimuli into electrical activity that leads to the experience of pain. Perception involves the recognition of pain by the frontal cortex of the brain. During modulation, pain signals can be facilitated or inhibited, and the perception of pain can be changed. PTS:1DIFgrinifficultREF:p. 1092 KEY:Nursing process: NA | Client need: PHSI | Cognitive level: Recall

A patient comes to the clinic complaining of a taste disturbance. Which medication that the patient is currently prescribed is most likely responsible for this disturbance? 1) Furosemide, a diuretic 2) Phenytoin, an anticonvulsant 3) Glyburide, an antidiabetic 4) Heparin, an anticoagulant

ANS: 2 Phenytoin is a medication that has a high incidence of associated taste disturbance. Furosemide, glyburide, and heparin are not implicated in taste disturbances. PTS:1DIF:ModerateREF:p. 1073 KEY: Nursing process: Assessment | Client need: SECE | Cognitive level: Application

The nurse checks a patient's pupils using a penlight. Which receptors is the nurse stimulating? 1) Chemoreceptors 2) Photoreceptors 3) Proprioceptors 4) Mechanoreceptors

ANS: 2 Photoreceptors located in the retina of the eyes detect visible light. Proprioceptors in the skin, muscles, tendons, ligaments, and joint capsules coordinate input to enable an individual to sense the position of the body in space. Chemoreceptors are located in the taste buds and epithelium of the nasal cavity. They play a role in taste and smell. Thermoreceptors in the skin detect variations in temperature. Mechanoreceptors in the skin and hair follicles detect touch, pressure, and vibration. PTS:1DIF:EasyREF:p. 1068 KEY: Nursing process: Assessment | Client need: PHSI | Cognitive level: Application

The nurse is caring for a patient who is experiencing dyspnea. Which of the following positions would be most effective if incorporated into the patient's care? 1) Supine 2) Head of bed elevated 80° 3) Head of bed elevated 30° 4) Lying on left side

ANS: 2 Position affects ventilation. An upright or elevated position pulls abdominal organs down, thus allowing maximum diaphragm excursion and lung expansion. PTS:1DIF:EasyREF:p. 1303 KEY: Nursing process: Interventions | Client need: PHSI | Cognitive level: Application

Which laboratory test result most accurately reflects a patient's nutritional status? 1) Albumin 2) Prealbumin 3) Transferrin 4) Hemoglobin

ANS: 2 Prealbumin levels fluctuate daily and give the best indication of the patient's immediate nutritional status. Albumin level is not as accurate because the half-life of albumin is 18 to 21 days, causing a delay in detection of nutritional problems. Transferrin, a protein that binds to iron, has a half-life of 8 to 9 days; therefore, it allows for faster detection of protein deficiency than does albumin. However, transferrin is not as fast as prealbumin. Hemoglobin level reflects iron intake or blood loss. PTS:1DIF:ModerateREF:p. 925 KEY: Nursing process: Assessment | Client need: PHSI | Cognitive level: Recall

A mother of three young children is newly diagnosed with breast cancer. She is intensely committed to fighting the cancer. She believes she can control her cancer to some degree with a positive attitude and feelings of inner strength. Which of the following traits is she demonstrating that is linked to health and healing? 1) Invincibility 2) Hardiness 3) Baseline strength 4) Vulnerability

ANS: 2 Research has also demonstrated that in the face of difficult life events, some people develop hardiness rather than vulnerability. Hardiness is a quality in which an individual experiences high levels of stress yet does not fall ill. There are three general characteristics of the hardy person: control (belief in the ability to control the experience), commitment (feeling deeply involved in the activity producing stress), and challenge (the ability to view the change as a challenge to grow). These traits are associated with a strong resistance to negative feelings that occur under adverse circumstances. PTS:1DIF:ModerateREF:p. 887 KEY: Nursing process: Planning | Client need: HPM | Cognitive level: Application

While performing a physical assessment, the student nurse tells her instructor that she cannot palpate her patient's bladder. Which statement by the instructor is best? "You should: 1) Try to palpate it again; it takes practice but you will locate it." 2) Palpate the patient's bladder only when it is distended by urine." 3) Document this abnormal finding on the patient's chart." 4) Immediately notify the nurse assigned to the care of your patient."

ANS: 2 The bladder is not palpable unless it is distended by urine. It is not difficult to palpate the bladder when distended. The nurse should document her finding, but it is not an abnormal finding. It is not necessary to notify the nurse assigned to the patient. PTS:1DIF:EasyREF:p. 1014 KEY: Nursing process: Assessment | Client need: PSI | Cognitive level: Application

When providing postmortem care, the nurse places dentures in the mouth and closes the eyes and mouth of the patient within 2 to 4 hours after death. Why is the timing of the action so important? 1) To prevent blood from settling in the head, neck, and shoulders 2) To perform these actions more easily before rigor mortis develops 3) To set the mouth in a natural position for viewing by the family 4) To prevent discoloration caused by blood settling in the facial area

ANS: 2 Rigor mortis develops 2 to 4 hours after death; therefore, the nurse should place dentures in the mouth and close the patient's eyes and mouth before that time. The nurse should place a pillow under the head and shoulders to prevent blood from settling there and causing discoloration. Closing the patient's mouth and tying a strip of soft gauze under the chin and around the head keeps the mouth set in a natural position for a viewing later. Closing the eyes after death creates a peaceful resting appearance when the body is later viewed but has nothing to do with setting the mouth. Placing dentures in the mouth and closing the eyes and mouth do not prevent discoloration in the facial area. PTS: 1 DIF: Moderate REF: p. 378 KEY: Nursing process: Interventions | Client need: PHSI | Cognitive level: Comprehension

A patient underwent abdominal surgery for a ruptured appendix. The surgeon did not surgically close the wound. The wound healing process described in this situation is: 1) Primary intention healing. 2) Secondary intention healing. 3) Tertiary intention healing. 4) Approximation healing.

ANS: 2 Secondary intention healing occurs when a wound is left open, and it heals from the inner layer to the surface by filling in with beefy red granulation tissue. Primary intention healing occurs when a wound is surgically closed. Tertiary intention healing occurs when a wound that was previously left open to heal by secondary intention is closed by joining the margins of granulation tissue. Approximation is another word for the joining of wound edges. PTS:1DIF:ModerateREF:p. 1227 KEY: Nursing process: Assessment | Client need: PHSI | Cognitive level: Comprehension

The nurse is preparing to admit a patient from the emergency department. The transferring nurse reports that the patient is obese. The nurse has been overweight at one time and works very hard now to maintain a healthy weight. She immediately thinks, "I know I tend to feel negatively about obese people; I figure if I can stop eating, they should be able to. I must remember how very difficult that is and be very careful not to be judgmental of this patient." This best illustrates 1)Theoretical knowledge 2)Self-knowledge 3)Using reliable resources 4)Use of the nursing process

ANS: 2 Self-knowledge is self-understanding—awareness of one's beliefs, values, biases, and so on. That best describes the nurse's awareness that her bias can affect her patient care. Theoretical knowledge consists of information, facts, principles, and theories in nursing and related disciplines; it consists of research findings and rationally constructed explanations of phenomena. Using reliable resources is a critical thinking skill. The nursing process is a problem-solving process consisting of the steps of assessing, diagnosing, planning outcomes, planning interventions, implementing, and evaluating. The nurse has not yet met this patient, so she could not have begun the nursing process. PTS:1DIFgrinifficultREF:pp. 30; high-level question, answer not stated verbatim

When using sterile technique to perform tracheostomy care of a new tracheostomy, which of the following is correct? 1) You will need a single pair of sterile gloves. 2) Place the patient in semi-Fowler's position, if possible. 3) Clean the stoma under the faceplate with hydrogen peroxide. 4) Cut a slit in sterile 4 × 4 gauze halfway through to make a dressing

ANS: 2 Semi-Fowler's position promotes lung expansion and prevents back strain for the nurse. You will need two pairs of sterile gloves: one pair for dressing removal, and a clean pair for the rest of the procedure. You should clean the stoma under the faceplate with sterile saline. Never cut a 4 × 4 gauze for the dressing because lint and fibers from the cut edge could enter the trachea and cause respiratory distress. PTS: 1 DIF: Easy REF: p. 1338 KEY: Nursing process: Interventions | Client need: SECE | Cognitive level: Application

While assessing a new wound, the nurse notes red, watery drainage. What type of drainage will the nurse document this as? 1) Sanguineous 2) Serosanguineous 3) Serous 4) Purosanguineous

ANS: 2 Serosanguineous drainage, a combination of bloody and serous drainage, is most commonly seen with new wounds. Serous drainage is straw colored, and sanguineous drainage is bloody. Purosanguineous drainage is pus that is red tinged. PTS:1DIF:EasyREF:p. 1229 KEY: Nursing process: Assessment | Client need: PHSI | Cognitive level: Comprehension

When making rounds on the night shift, the nurse observes her patient to be in deep sleep. His muscles are very relaxed. When he arouses as the nurse changes the IV tubing, he is confused. What stage of sleep was the patient most likely experiencing? 1)Stage V 2)Stage IV 3)Stage III 4)REM

ANS: 2 Stage IV is the deepest sleep. In this stage, the delta waves are highest in amplitude, slowest in frequency, and highly synchronized. The body, mind, and muscles are very relaxed. It is difficult to awaken someone in stage IV sleep; if awakened, the person may appear confused and react slowly. During this stage, the body releases human growth hormone, which is essential for repair and renewal of brain and other cells. PTS:1DIF:EasyREF:p. 1206

The Joint Commission requires which type of assessment to be performed on all patients? 1)Functional ability 2)Pain 3)Cultural 4)Wellness

ANS: 2 The Joint Commission requires that pain and nutrition assessment be performed on all patients. Other special needs assessments should be performed when cues indicate there are risk factors. PTS:1DIF:ModerateREF:p. 39 KEY: Nursing process: Assessment | Client need: PHSI | Cognitive level: Analysis

2. While performing a physical assessment, the student nurse tells her instructor that she cannot palpate her patients bladder. Which statement by the instructor is best? You should: 1) Try to palpate it again; it takes practice but you will locate it. 2) Palpate the patients bladder only when it is distended by urine. 3) Document this abnormal finding on the patients chart. 4) Immediately notify the nurse assigned to the care of your patient.

ANS: 2 The bladder is not palpable unless it is distended by urine. It is not difficult to palpate the bladder when distended. The nurse should document her finding, but it is not an abnormal finding. It is not necessary to notify the nurse assigned to the patient.

Patients may be deficient in which vitamin during the winter months? 1) A 2) D 3) E 4) K

ANS: 2 The body can synthesize vitamin D from a cholesterol compound in the skin when exposed to adequate sunlight. People at risk for vitamin D deficiency are those who spend little time outdoors, older people, and people who live in an institution (e.g., a nursing home). The deficiency can also occur in the winter at northern and southern latitudes, in people who keep their bodies covered (e.g., traditional Muslim women), and in those who use sunscreen. Also, because breast milk contains only small amounts of vitamin D, breastfed infants who are not exposed to enough sunlight are at risk of the deficiency and rickets. There is no seasonal tie to deficiencies in the other fat-soluble vitamins, A, E, and K. PTS: 1 DIF: Easy REF: p. 905 KEY: Nursing process: Assessment | Client need: HPM | Cognitive level: Recall

The nursing diagnosis is Impaired Memory related to fluid and electrolyte imbalances A.M.B. inability to recall recent events. Which of the following goals/outcomes must be included on the care plan? 1) Checks current medications for mind-altering side effects 2) Demonstrates use of techniques to help with memory loss 3) Drinks at least 1500 cc of fluid per day 4) Takes electrolyte supplements with meals

ANS: 2 The essential goal/outcome is aimed at the problem response Impaired Memory. The other goals in this question address the etiology. PTS:1DIF:ModerateREF: p. 93-94 KEY: Nursing process: Planning | Client need: PHSI | Cognitive level: Application

The nurse developed a care plan for a patient to help prevent Impaired Skin Integrity. She has made sure that nursing assistive personnel change the patient's position every 2 hours. In the evaluation phase of the nursing process, which of the following would the nurse do first? 1)Determine whether she has gathered enough assessment data. 2)Judge whether the interventions achieved the stated outcomes. 3)Follow up to verify that care for the nursing diagnosis was given. 4)Decide whether the nursing diagnosis was accurate for the patient's condition.

ANS: 2 The evaluation phase judges whether the interventions were effective in achieving the desired outcomes and helped to alleviate the nursing diagnosis. This must be done before examining the nursing process steps and revising the care plan. PTS:1DIF:ModerateREF: p. 31

The nurse is admitting to the medical-surgical unit an older adult woman with a diagnosis of pulmonary hypertension and right-sided heart failure. The patient is complaining of shortness of breath, and the nurse observes conversational dyspnea. What is the first action the nurse should take? 1) Review and implement the primary care provider's prescriptions for treatments. 2) Perform a quick physical examination of breathing, circulation, and oxygenation. 3) Gather a thorough medical history, including current symptoms, from the family. 4) Administer oxygen to the patient through a nasal cannula.

ANS: 2 The first action the nurse should take is to make a quick assessment of the adequacy of breathing, circulation, and oxygenation in order to determine the type of immediate intervention required. The nurse's assessment should include simple questions about current symptoms. A more thorough medical history can be gathered once the patient's oxygenation needs are addressed. Following a quick assessment, the nurse should then review and implement physician's orders. Administering oxygen is not appropriate without knowing what treatments the primary care provider has prescribed. PTS:1DIF:ModerateREF:pp. 1301-1302 KEY: Nursing process: Interventions | Client need: PHSI | Cognitive level: Application

11. Which outcome is appropriate for the patient who underwent urinary diversion surgery and creation of an ileal conduit for invasive bladder cancer? 1) Patient will resume his normal urination pattern by (target date). 2) Patient will perform urostomy self-care by (target date). 3) Patient will perform self-catheterization by (target date). 4) Patients urine will remain clear with sufficient volume.

ANS: 2 The most appropriate outcome for this patient is the patient will perform urostomy self-care by a specific date. The patient with an ileal conduit is unable to resume a normal urination pattern; urine, along with mucus, drains continuously from the stoma site, so the urine will not be clear. Also, the phrase sufficient volume is too vague for an outcome statement. The patient with a continent urostomy inserts a catheter into the stoma to drain urine.

Which of the following is an example of the most basic motivation in Maslow's hierarchy of needs? 1)Experiencing loving relationships 2)Having adequate housing 3)Receiving education 4)Living in a crime-free neighborhood

ANS: 2 The most basic needs are centered on physiological survival—shelter (housing), food, and water. All other options are for higher needs. The order from most basic to highest level is physiologic, safety and security, love and belonging, esteem, and self-actualization. Loving relationships fall under the love and belonging category. Education is a form of self-actualization. Living in a crime-free neighborhood meets the need for safety and security. PTS:1DIF:ModerateREF:p. 48 KEY: Nursing process: Assessment | Client need: PHSI | Cognitive level: Application

Which of the following patient goals is most appropriate when managing the patient dying of cancer? The patient will 1) Request pain medication when needed 2) Report or demonstrate satisfactory pain control 3) Use only nonpharmacological measures to control pain 4) Verbalize understanding that it may not be possible to control his pain

ANS: 2 The most important goal is that the patient will report or demonstrate satisfactory pain control. The nurse should administer pain medication on a regular schedule to ensure satisfactory pain control; pain may not be controlled if medication is administered on an "as needed" basis. Nonpharmacologic measures can be a helpful adjunct in controlling pain, but they are not likely to be adequate for pain associated with cancer. Effective pain-control medications are available and can be administered by several routes; it should be possible to control the pain. PTS: 1 DIF: Moderate REF: p. 369 KEY: Nursing process: Planning | Client need: PHSI | Cognitive level: Application

A patient who underwent a left above-the-knee amputation complains of pain in his left foot. The nurse should document this finding as what type of pain? 1) Psychogenic 2) Phantom 3) Referred 4) Radiating

ANS: 2 The nurse should document this finding as phantom pain. Phantom pain is pain that is perceived to originate in an area that has been amputated. Psychogenic pain refers to pain experienced by a person which does not match the symptoms or the apparent source of pain. It is thought to arise from psychological factors and is disproportional to the painful stimuli. Referred pain occurs in an area distant from the original site. Radiating pain starts at the source but extends to other locations. PTS:1DIF:EasyREF:p. 1091 KEY: Nursing process: Assessment | Client need: PHSI | Cognitive level: Comprehension

Which nursing diagnosis is most appropriate for the patient who returns from the postanesthesia care unit after undergoing right hemicolectomy surgery for colon cancer? 1) Acute pain secondary to surgery 2) Acute pain (abdominal) secondary to surgery for colon cancer 3) Chronic pain secondary to cancer diagnosis 4) Chronic pain (abdominal) secondary to abdominal surgery

ANS: 2 The nurse should identify a diagnosis by specifying the location of the pain and any precipitating or etiological factors. This patient is experiencing acute abdominal pain that is related to his surgery for colon cancer; therefore, a nursing diagnosis that specifies the surgery is the most appropriate diagnosis for this patient. In addition, options listing chronic pain are incorrect because the pain is acute, not chronic. PTS: 1 DIF: Moderate REF: p. 1100 KEY: Nursing process: Diagnosis | Client need: PHSI | Cognitive level: Application

14. A patient who sustained a spinal cord injury will perform intermittent self-catheterization after discharge. After discharge teaching, which statement by the patient would indicate correct understanding of the procedure? 1) I will need to replace the catheter weekly. 2) I can use clean, rather than sterile, technique at home. 3) I will remember to inflate the catheter balloon after insertion. 4) I will dispose of the catheter after use and get a new one each time.

ANS: 2 The nurse should inform the patient that clean technique can be used after discharge. The patient should wash his hands before the procedure, then wash the reusable catheter in soap and water, and rinse and store it in a clean, dry place. It is not necessary for the patient to use a new catheter for each catheterization. The patient should use a straight catheter; therefore, a balloon is not inflated after insertion. Straight catheters are removed immediately after use.

A patient with a history of chronic obstructive pulmonary disease has a living will that states he does not want endotracheal intubation and mechanical ventilation as a means of respiratory resuscitation. As the patient's condition deteriorates, the patient asks whether he can change his decision. Which response by the nurse is best? 1) "I'll call your physician right away so he can discuss this with you." 2) "You have the right to change your decision about treatment at any time." 3) "Are you sure you want to change your decision?" 4) "We must follow whatever is written in your living will."

ANS: 2 The nurse should inform the patient that he has the right to change his decision about treatment at any time. Next, the nurse should notify the physician of the patient's decision so that the physician can speak to the patient and revise the treatment plan as needed. Questioning the patient's decision is judgmental. The patient has the right to change his living will at any time. The medical team should not follow the living will if the patient changes his decision about what is in it. PTS:1DIF:ModerateREF:p. 364 KEY: Nursing process: Interventions | Client need: SECE | Cognitive level: Application

A patient prescribed a nonsteroidal anti-inflammatory drug (NSAID), naproxen (Aleve, Naprosyn), for treatment of arthritis complains of stomach upset. What should the nurse instruct the patient to do? 1) Notify the prescriber immediately. 2) Take the medication with food. 3) Take the medication with 8 ounces of water. 4) Take the medication before bedtime.

ANS: 2 The nurse should instruct the patient to take the medication with food to lessen gastric irritation. Taking the medication with 8 ounces of water will not decrease gastric irritation. Taking the medication just before bedtime may cause gastric reflux, increasing gastric irritation. Although indigestion is an unwanted side effect of naproxen, it is not an emergency that requires the prescriber to be notified immediately. However, prior to giving naproxen, be sure the patient has not had ulcers, stomach bleeding, or severe kidney or liver problems. If so, the patient is not a candidate for treatment with naproxen. PTS:1DIF:ModerateREF:p. 1103 KEY: Nursing process: Interventions | Client need: SECE | Cognitive level: Application

The nurse administers acetaminophen 325 mg and codeine 30 mg orally to a patient complaining of a severe headache. When should the nurse reassess the patient's pain? 1) 15 minutes after administration 2) 60 minutes after administration 3) 90 minutes after administration 4) Immediately before the next dose is due

ANS: 2 The nurse should reassess pain in the patient who received an oral pain medication 30 to 60 minutes after administration. The nurse should reassess the patient receiving IV medications 10 to 15 minutes after administration. The nurse should not wait until just before the patient can receive another dose. The patient may require additional pain medication before the next dose is due. PTS:1DIF:ModerateREF:p. 1100 KEY: Nursing process: Evaluation | Client need: SECE | Cognitive level: Application

A patient with a history of falling continually attempts to get out of bed unassisted despite frequent reminders to call for help first. Which action should the nurse take first? 1) Apply a cloth vest restraint. 2) Encourage a family member to stay with the patient. 3) Administer lorazepam (an antianxiety medication). 4) Keep the patient's bed side rails up.

ANS: 2 The nurse should use one-to-one supervision with this patient to maintain the patient's safety. One way to accomplish this is by encouraging a family member to stay with the patient. Restraints should be used only when all other less-restrictive measures have failed and are absolutely necessary to prevent injury to the patient. Restraints have been shown to jeopardize patient safety. It is not appropriate to administer sedation for the purpose of keeping the patient in bed; this is a form of restraint. Keeping the side rails up is also a form of restraint and increases the risk for falling. PTS: 1 DIF: Moderate REF: p. 673 KEY: Nursing process: Interventions | Client need: SECE | Cognitive level: Application

Which statement about the nursing process is correct? 1)It was developed from the ANA Standards of Care. 2)It is a problem-solving method to guide nursing activities. 3)It is a linear process with separate, distinct steps. 4)It involves care that only the nurse will give.

ANS: 2 The nursing process is a problem-solving process that guides nursing actions. The ANA organizes its Standards of Care around the nursing process, but the process was not developed from the standards. The nursing process is cyclical and involves care the nurses give or delegate to other members of the healthcare team. PTS:1DIF:EasyREF: p. 31

Chest percussion and postural drainage would be an appropriate intervention for which of the following conditions? 1) Congestive heart failure 2) Pulmonary edema 3) Pneumonia 4) Pulmonary embolus

ANS: 3 Chest physiotherapy moves secretions to the large, central airways for expectoration or suctioning. This treatment is not effective for conditions that do not involve the development of airway secretions, including congestive heart failure, pulmonary edema, and pulmonary embolus. PTS:1DIF:ModerateREF:p. 1313 KEY: Nursing process: Interventions | Client need: PHSI | Cognitive level: Comprehension

A patient's wife tells the nurse that she wants to be with her husband when he dies. The patient's respirations are irregular, and he is congested. The wife tells the nurse that she would like to go home to shower but that she is afraid her husband might die before she returns. Which response by the nurse is best? 1) "Certainly, go ahead; your husband will most likely hold on until you return." 2) "Your husband could live for days or a few hours; you should do whatever you are comfortable with." 3) "You need to take care of yourself; go home and shower, and I'll stay at his bedside while you are gone." 4) "Don't worry. Your husband is in good hands; I'll look out for him."

ANS: 2 The patient is exhibiting signs that typically occur days to a few hours before death. The nurse should provide information to the wife so she can make an informed decision about whether to leave her husband's bedside. The nurse should not offer false reassurance by stating that the patient will most likely be fine until the wife's return. The nurse should not offer her opinion by telling the wife that she needs to take care of herself. It is also unrealistic for the nurse to stay with the patient until his wife returns. The nurse would be minimizing the wife's concern by telling her not to worry because her husband is in good hands. The issue for the family member is not trust in the competency of the healthcare provider but rather wanting to be present with her spouse at the time of death. PTS:1DIF:ModerateREF:pp. 367-368 KEY: Nursing process: Interventions | Client need: PSI | Cognitive level: Application

A home health patient previously lived with her sister for more than 20 years. Although it has been over a year since her sister died, the patient tells the nurse, "It's no worse now, but I never feel any relief from this overwhelming sadness. I still can't sleep a full night. The house is a mess; I feel too tired, even to take a bath. But, sometimes at night, she comes to me and I can see her plain as can be." The patient's clothing is not clean and her hair is not combed. She is apparently not eating adequately. What can the nurse conclude? The patient is probably 1) Grieving longer than usual because of the closeness of the relationship with her sister 2) Experiencing a depressive disorder rather than simply grieving the loss of her sister 3) Feeling guilt and worthlessness because her sister died and she is still alive 4) Interpreting the holiday as a trigger event, which is causing her to hallucinate event, which is causing her to hallucinate

ANS: 2 The patient is likely experiencing a depressive disorder. Her symptoms include unrelieved, overwhelming sadness; insomnia; difficulty carrying out ADLs; fatigue; and visual hallucinations. Note that her sadness is pervasive, not created by a trigger event (holiday). Of those symptoms, insomnia is common to both grief and depression, but the other symptoms are signs of depressive disorder. There is, of course, no "correct" timeline for what constitutes "longer than usual" grieving; however, the patient's symptoms are typical of depression, not grief. She has not said she feels guilty or worthless, and there is nothing from which the nurse could infer that. She has specifically said that the holiday has not made her feel any worse—that is, it has not been a trigger event. PTS: 1 DIF: Difficult REF: pp. 367-368 KEY: Nursing process: Diagnosis | Client need: PSI | Cognitive level: Application

A patient had a bowel resection 5 days ago. Which request by the patient might alert the nurse that the patient has a history of substance abuse? 1) Oral pain medication once every 6 to 8 hours 2) Patient-controlled analgesic 3) Oral pain medications instead of the IM form 4) Only nonpharmacological pain measures

ANS: 2 The patient underwent surgery 5 days ago; if there are no complications, it is unlikely that he would require frequent dosing of analgesic. The nurse should recognize this behavior as a possible indicator of current substance abuse or addiction. Requesting oral pain medications every 6 to 8 hours is a typical behavior for a patient 5 days after surgery. Requesting an oral form of the drug does not indicate substance abuse. PTS:1DIF:EasyREF:pp. 1109-1110 KEY: Nursing process: Assessment | Client need: PHSI | Cognitive level: Application

Which of the following is an example of theoretical knowledge? 1)A nurse uses sterile technique to catheterize a patient. 2)Room air has an oxygen concentration of 21%. 3)Glucose monitoring machines should be calibrated daily. 4)An irregular apical heart rate should be compared with the radial pulse.

ANS: 2 Theoretical knowledge consists of research findings, facts, principles, and theories. The oxygen concentration of room air is a scientific fact. The others are examples of practical knowledge—what to do and how to do it. PTS:1DIF:ModerateREF:p. 30; high-level question, answer not stated verbatim

An older adult receiving hospice care has dementia as a result of metastasis to the brain. His bone cancer has progressed to an advanced stage. Why might the client fail to request pain medication as needed? The client: 1) Experiences less pain than in earlier stages of cancer. 2) Cannot communicate the character of his pain effectively. 3) Recalls pain at a later time than when it occurs. 4) Relies on caregiver to provide pain relief without asking.

ANS: 2 There is no evidence to suggest that patients with dementia and other forms of cognitive impairment do not experience pain. It is most likely that they cannot effectively communicate the intensity or quality of pain and are therefore at risk for underassessment of pain and inadequate pain relief. Be aware of behavioral cues indicating pain rather than relying on verbal report. Failure to request pain medication is not likely a result of hesitation to ask for it out of habit or reliance on others; rather, it is likely due to inability to effectively express to the caregiver that analgesia is needed. PTS:1DIF:ModerateREF:p. 1095 KEY: Nursing process: Planning | Client need: PHSI | Cognitive level: Analysis

. A man was involved in a motor vehicle accident yesterday. He is to be sedated for over 2 weeks while breathing with the assistance of a mechanical ventilator. Which of the following would be an appropriate nursing diagnosis for him at this time? 1) Risk for Infection related to subcutaneous injuries 2) Risk for Impaired Skin Integrity related to immobility 3) Impaired Tissue Integrity related to ventilator dependency 4) Impaired Skin Integrity related to ventilator dependency

ANS: 2 This patient is at Risk for Impaired Skin Integrity because he is being kept in a sedated state. Thus, he is unable to turn himself to relieve pressure. There is no mention of subcutaneous injuries, ruling out Risk for Infection related to subcutaneous injuries. Impaired Tissue Integrity and Impaired Skin Integrity are also incorrect because there is no supporting evidence for these nursing diagnoses. PTS:1DIF:ModerateREF:pp. 1235-1237 KEY: Nursing process: Diagnosis | Client need: SECE | Cognitive level: Analysis

After sustaining a stroke, the patient lacks attention to the right side of his body. Which nursing diagnosis best describes the patient's problem? 1) Disturbed Sensory Perception 2) Unilateral Neglect 3) Risk for Peripheral Vascular Dysfunction 4) Acute Confusion

ANS: 2 This patient lacks attention to the right side of his body after sustaining a stroke; therefore, the most appropriate nursing diagnosis is Unilateral Neglect. The patient may also have Disturbed Sensory Perception, Risk for Peripheral Vascular Dysfunction, and Acute Confusion, but they are not the most appropriate for the defined problem. PTS:1DIFgrinifficultREF:p. 1079 KEY: Nursing process: Diagnosis | Client need: PHSI | Cognitive level: Analysis

8. The nurse instructs a woman about providing a clean-catch urine specimen. Which of the following statements indicates that the patient correctly understands the procedure? 1) I will be sure to urinate into the hat you placed on the toilet seat. 2) I will wipe my genital area from front to back before I collect the specimen midstream. 3) I will need to lie still while you put in a urinary catheter to obtain the specimen. 4) I will collect my urine each time I urinate for the next 24 hours.

ANS: 2 To obtain a clean-catch urine specimen, the nurse should instruct the patient to cleanse the genital area from front to back and collect the specimen midstream. This follows the principle of going from clean to dirty. The nurse should have the ambulatory patient void into a hat (container for collecting the urine of an ambulatory patient) when monitoring urinary output, but not when obtaining a clean-catch urine specimen. A urinary catheter is required for a sterile urine specimen, not a clean-catch specimen. A 24-hour urine collection may be necessary to evaluate some disorders, but a clean-catch specimen is a one-time collection.

After undergoing dural puncture while receiving epidural pain medication, a patient complains of a headache. Which action can help alleviate the patient's pain? 1) Encourage the client to ambulate to promote flow of spinal fluid. 2) Offer caffeinated beverages to constrict blood vessels in his head. 3) Encourage coughing and deep breathing to increase CSF pressure. 4) Restrict oral fluid intake to prevent excess spinal pressure.

ANS: 2 Treatment for a headache that occurs as a result of dural puncture consists of bedrest, analgesics as prescribed, and liberal hydration. Caffeine and a dark, quiet environment may also be helpful. Headaches will be more severe when the patient is sitting upright or ambulating. Fluid volume deficit will also aggravate a "spinal headache" after epidural anesthesia. PTS:1DIFgrinifficultREF:p. 1108 KEY: Nursing process: Interventions | Client need: PHSI | Cognitive level: Application

A patient has received a radiation implant. The patient is weak and needs help even to turn in bed. Which action should the nurse take when caring for this patient? 1) Avoid giving the patient a complete bed bath. 2) Limit the amount of time spent with the patient. 3) Allow extra time for the patient to express feelings. 4) Do not allow anyone to visit the patient.

ANS: 2 When caring for a patient with a radiation implant, the nurse should organize nursing care to limit the amount of time with the patient to limit radiation exposure. The nurse must meet the patient's personal hygiene needs by bathing the patient, if necessary. The nurse should encourage the patient to express her feelings; however, she should limit her contact with the patient. Pregnant women should not visit the patient; however, others may visit as long as they uphold the principles of time, distance, and shielding. PTS: 1 DIF: Moderate REF: p. 660 KEY: Nursing process: Interventions | Client need: SECE | Cognitive level: Comprehension

When providing safety education to the mother of a toddler, you would inform the mother that, based on the child's developmental stage, he is at high risk for which of the following factors that influence oxygenation? Choose all that apply. 1) Frequent, serious respiratory infections 2) Airway obstruction from aspiration of small objects 3) Drowning in small amounts of water around the home 4) Development of asthma

ANS: 2, 3 As a toddler's respiratory and immune systems mature, the risk for frequent and serious infections is less than in infanthood. Most children recover from upper respiratory infections without difficulty. Toddlers' airways are relatively short and small and may be easily obstructed, and they often put objects in their mouth as part of exploring their environment, thus increasing their risk for aspiration and airway obstruction. In addition, toddlers are at high risk for drowning in very small amounts of water around the home (e.g., in a bucket of water or toilet bowl). The risk for developing asthma is not significantly influenced by the child's developmental stage. PTS:1DIF:ModerateREF:p. 1297 KEY: Nursing process: Interventions | Client need: HPM | Cognitive level: Comprehension

Where in the body is glucose stored? Choose all that apply. 1) Brain 2) Liver 3) Skeletal muscles 4) Smooth muscles

ANS: 2, 3 Human beings store glucose in liver and skeletal muscle tissue as glycogen. Glycogen is converted back into glucose to meet energy needs. PTS:1DIF:ModerateREF:p. 902 KEY:Nursing process: N/A | Client need: HPM | Cognitive level: Recall

1. Which of the following is/are an appropriate goal(s) for a patient with urinary incontinence? Choose all that apply. 1) Increase the intake of citrus fruits. 2) Maintain daily oral fluids to 8 to 10 servings per day. 3) Limit daily caffeine intake to less than 100 mg. 4) Engage in high-impact, aerobic exercise.

ANS: 2, 3 The nurse should encourage lifestyle changes such as limiting caffeine intake to fewer than 100 mg per day; limiting intake of alcohol, artificial sweeteners, spicy foods, and citrus fruit; and maintaining daily oral fluid intake to 8 to 10 servings per day. High-impact exercise can be associated with stress incontinence for those with weakened pelvic muscles that support the bladder and urethra.

Which point(s) should the nurse include when teaching safety precautions to a mother of a toddler? Select all that apply. 1) Make sure the child sleeps on his back at night. 2) Keep the telephone number of the poison control center accessible. 3) Use a front-facing car seat placed in the back seat of the car. 4) Keep syrup of ipecac on hand in case of accidental poisoning.

ANS: 2, 3 The nurse should teach the mother of a toddler to keep the telephone number of the poison control center accessible because toddlers are at risk for accidental poisonings. Toddlers should also have front-facing car seats. Syrup of ipecac is no longer recommended to induce emesis after poisonings. Infants, not toddlers, should sleep on their backs to prevent sudden infant death syndrome. PTS: 1 DIF: Moderate REF: pp. 664, 670 KEY: Nursing process: Interventions | Client need: SECE | Cognitive level: Application

What happens when the bladder contains 200-450mL of urine?

the distension activates stretch receptors in the bladder wall. These receptors send sensory impulses to the voiding reflex center in the spinal cord- detrusor muscle contracts and internal sphincter relaxes, thus causing urination.

Obesity is associated with higher risk for which of the following conditions that affect the pulmonary and cardiovascular systems? Choose all that apply. 1) Reduced alveolar-capillary gas exchange 2) Lower respiratory tract infections 3) Sleep apnea 4) Hypertension

ANS: 2, 3, 4 Obesity causes multiple health problems, many of which affect the lungs, heart, and circulation. Large abdominal fat stores press upward on the diaphragm, preventing full chest expansion and leading to hypoventilation and dyspnea on exertion. The risk for respiratory infection increases because lower lung segments are poorly ventilated, and secretions are not removed effectively. When an obese person lies down, chest expansion is limited even more. Excess neck girth and fat stores in the upper airway often lead to obstructive sleep apnea. Obesity also increases the risk of developing atherosclerosis and hypertension. Obesity does not cause reduced alveolar-capillary gas exchange. PTS: 1 DIF: Easy REF: p. 1299 KEY: Nursing process: Assessment | Client need: HPM | Cognitive level: Comprehension

The World Health Organization's definition of health includes which of the following? Choose all that apply. 1) Absence of disease 2) Physical well-being 3) Mental well-being 4) Social well-being

ANS: 2, 3, 4 The World Health Organization defines health as a state of complete physical, mental, and social well-being, not merely the absence of disease or infirmity. PTS:1DIF:EasyREF:p. 878 KEY:Nursing process: N/A | Client need: HPM | Cognitive level: Recall

Which of the following might be a warning sign that a child needs professional help after the death of a loved one? Choose all that apply. 1) Interest in his usual activities 2) Extended regression 3) Withdrawal from friends 4) Inability to sleep 5) Intermittent sadness

ANS: 2, 3, 4 The warning signs that may indicate the need for professional help include inability to sleep, extended regression, loss of interest in daily activities, and withdrawal from friends. Interest in usual activities is a sign of coping; intermittent expressions of sadness and anger are to be expected, even over a long period of time, so they would not indicate a need for professional help. PTS:1DIF:EasyREF:p. 380 KEY: Nursing process: Assessment | Client need: PSI | Cognitive level: Analysis

Which action(s) can the nurse take to prevent sensory overload? Choose all that apply. 1) Leave the television on to block out other noises. 2) Minimize unnecessary light in the patient's room. 3) Plan care to provide uninterrupted periods of sleep. 4) Speak calmly with a moderate voice volume.

ANS: 2, 3, 4 To prevent sensory overload, minimize unnecessary light, plan care to provide uninterrupted periods of sleep, and speak to the patient in a moderate tone of voice using a calm and confident manner. Television can be used to provide sensory stimuli, but not to prevent sensory overload. When used, it should not be left on indiscriminately. PTS:1DIF:ModerateREF:p. 1081 KEY: Nursing process: Interventions | Client need: PHSI | Cognitive level: Application

Which of the following are cues? Select all that apply. 1) Taking a brisk walk five times a week 2) Using laxatives to have a bowel movement 3) Needing more sleep than usual 4) Decreasing the amount of fat in the diet 5) Weighing less than indicated by developmental norms

ANS: 2, 3, 5 Cues are a deviation from norms, such as changes in usual health behavior, indications of delayed growth and development, changes in behaviors, or nonproductive or dysfunctional behavior. PTS:1DIF:ModerateREF:p. 61-62 KEY: Nursing process: Diagnosis | Client need: SECE | Cognitive level: Application

Select the process(es) that occur(s) during the inflammatory phase of wound healing. Choose all that apply. 1) Granulation 2) Hemostasis 3) Epithelialization 4) Inflammation

ANS: 2, 4 During the inflammatory phase of wound healing, hemostasis and inflammation occur. After an injury, blood vessels constrict to limit blood loss, and platelets migrate to the site and aggregate to stop bleeding. Together, this results in hemostasis. Inflammation follows as a defense against infection at the wound site. PTS: 1 DIF: Moderate REF: p. 1228 KEY:Nursing process: Assessment | Client need: PHSI | Cognitive level: Recall

A mother expresses concern that her 7-year-old has episodes of nocturnal enuresis approximately 3 to 4 times per week. The nurse's best response would be which of the following? Choose all that apply. 1)"Your daughter's bladder is still developing at this point in her life." 2)"Be patient; most children outgrow enuresis." 3)"Wake your daughter every 4 hours to use the bathroom." 4)"You might consider purchasing protective pads for the bed." 5)"Try a bed alarm to wake her when she starts wetting the bed at night."

ANS: 2, 4 Enuresis is nighttime incontinence past the stage at which toilet training has been well established. Most incidents occur during NREM sleep when the child is difficult to arouse. As the great majority of children outgrow enuresis, the best strategy is patience. In the meantime, protecting the mattress from moisture and odor will help reduce frustration and embarrassment. A bed alarm can be used for older children (typically older than age 10 or 12) who are resistant to other behavioral strategies. PTS:1DIF:ModerateREF:p. 1211

6. The nurse is caring for a patient who underwent a bowel resection 2 hours ago. His urine output for the past 2 hours totals 50 mL. Which action should the nurse take? 1) Do nothing; this is normal postoperative urine output. 2) Increase the infusion rate of the patients IV fluids. 3) Notify the provider about the patients oliguria. 4) Administer the patients routine diuretic dose early.

ANS: 3 50 mL in 2 hours is not normal output. The kidneys typically produce 60 mL of urine per hour. Therefore, the nurse should notify the provider when the patient shows diminished urine output (oliguria). Patients who undergo abdominal surgery commonly require increased infusions of IV fluid during the immediate postoperative period. The nurse cannot provide increased IV fluids without a providers order. The nurse should not administer any medications before the scheduled time without a prescription. The provider may hold the patients scheduled dose of diuretic if he determines that the patient is experiencing deficient fluid volume.

A patient who underwent surgery 24 hours ago is prescribed a clear liquid diet. The patient asks for something to drink. Which item may the nurse provide for the patient? 1) Tea with cream 2) Orange juice 3) Gelatin 4) Skim milk

ANS: 3 A clear liquid diet consists of water; tea (without cream); coffee; broth; clear juices, such as apple, grape, or cranberry; popsicles; carbonated beverages; and gelatin. Skim milk, tea with cream, and orange juice are included in a full liquid diet. PTS:1DIF:EasyREF:p. 917 KEY: Nursing process: Interventions | Client need: PHSI | Cognitive level: Comprehension

Urge incontinence:

the involuntary loss of larger amounts of urine accompanied by a strong urge to void

A patient has a stage II pressure ulcer on her right buttock. The ulcer is covered with dry, yellow slough that tightly adheres to the wound. What is the best treatment the nurse could recommend for treating this wound? 1) Dry gauze dressing changed twice daily 2) Nonadherent dressing with daily wound care 3) Hydrocolloid dressing changed as needed 4) Wet-to-dry dressings changed three times a day

ANS: 3 A hydrocolloid dressing would conform to this area and form a protective layer against friction and bacterial invasion. It would also promote autolytic débridement of the slough and absorb the exudate from the autolysis. Dry gauze and nonadherent dressing (e.g., Telfa) would cover the wound but would not aid in removing the slough. A wet-to-dry dressing is a form of mechanical débridement. It would aid in removing the slough but is nonselective; therefore, it could cause damage to healthy tissue as well. PTS:1DIFgrinifficultREF:pp. 1233, 1251, 1277; synthesis required KEY: Nursing process: Interventions | Client need: PHSI | Cognitive level: Application

Which expected outcome is best for the patient with a nursing diagnosis of Acute Pain related to movement and secondary to surgical resection of a ruptured spleen and possible inadequate analgesia? 1) The patient will verbalize a reduction in pain after receiving pain medication and repositioning. 2) The patient will rest quietly when undisturbed. 3) On a scale of 0 to 10, the patient will rate pain as a 3 while in bed or as a 4 during ambulation. 4) The patient will receive pain medication every 2 hours as prescribed.

ANS: 3 A low pain rating is the best expected outcome for the patient with a nursing diagnosis of Acute Pain secondary to surgical resection of a ruptured spleen and possible inadequate analgesia because it is specific and measurable. The patient verbalizing reduced pain is not specific enough. The nurse needs to know how much pain relief is achieved. A numeric score gives a clearer indication of the effectiveness of analgesia. The patient might have experienced a reduction in pain, but his pain level might still be intolerable. Saying the patient's pain is relieved because he is resting quietly does not address the pain relief while he is awake. Some patients will sleep in an attempt to cope with pain, so this outcome could lead to inaccurate evaluation. Providing pain medication is a nursing intervention, not an expected outcome. PTS:1DIFgrinifficultREF:p. 1100 KEY: Nursing process: Planning | Client need: PHSI | Cognitive level: Analysis

A person who is deprived of REM sleep for several nights in succession will usually experience: 1)Extended NREM sleep. 2)Paradoxical sleep. 3)REM rebound. 4)Insomnia.

ANS: 3 A person who is deprived of REM sleep for several nights will usually experience REM rebound. The person will spend a greater amount of time in REM sleep on successive nights, generally keeping the total amount of REM sleep constant over time. PTS:1DIF:ModerateREF:p. 1205

Which intervention takes priority for the patient receiving hospice care? 1) Turning and repositioning the patient every 2 hours 2) Assisting the patient out of bed into a chair twice a day 3) Administering pain medication to keep the patient comfortable 4) Providing the patient with small frequent, nutritious meals

ANS: 3 A priority intervention for the hospice team is administering pain medications to keep the patient comfortable. Turning the patient to prevent skin breakdown and promote comfort is also important, but it does not take priority over administering pain medications. The patient may not be able to eat meals or get out of bed into the chair and may tolerate only small amounts at a meal. During the dying process, bowel activity reduces and digestion is minimal, which often results in nausea or food intolerance. Additionally, the body's need for nutrition and hydration is reduced as the body begins the desiccation process. PTS:1DIFgrinifficultREF:p. 363 KEY: Nursing process: Interventions | Client need: PHSI | Cognitive level: Application

What intervention would be most appropriate for a wound with a beefy red wound bed? 1) Mechanical débridement 2) Autolytic débridement 3) Dressing to keep the wound moist and clean 4) Removal of devitalized tissue and a sterile dressing

ANS: 3 A red wound indicates active healing, and the best treatment is gentle cleansing and a dressing that will ensure a clean, moist wound environment. Débridement is not necessary in this situation because there is no devitalized tissue present. PTS: 1 DIF: Moderate REF: p. 1228 KEY: Nursing process: Interventions | Client need: PHSI | Cognitive level: Application

Which of the following is an example of an active listening behavior? 1) Taking frequent notes 2) Asking for more details 3) Leaning toward the patient 4) Sitting with legs crossed

ANS: 3 Active listening behaviors include leaning toward the patient; facing the patient; open, relaxed posture without crossing arms or legs; and maintaining eye contact. Taking frequent notes makes it difficult to keep eye contact. Asking for more details may seem like idle curiosity. Sitting with legs crossed may indicate to the patient that you are not open to her. PTS:1DIF:EasyREF:p. 47 KEY: Nursing process: Assessment | Client need: SECE | Cognitive level: Comprehension

The patient shows the necessary defining characteristics, and the nurse has diagnosed Decisional Conflict related to unclear personal values and beliefs. What essential action should the nurse take to help ensure the accuracy of this diagnosis? 1) Ask a more experienced nurse to confirm it. 2) Have a social worker interview the patient. 3) Ask the patient to confirm the diagnosis. 4) Read about Decisional Conflict in the NANDA-I handbook.

ANS: 3 After identifying problems and etiologies (which this nurse has done), the nurse should verify them with the patient to help ensure that her conclusions are accurate. If the patient does not agree that he has Decisional Conflict, the nurse might interview him more to clarify the meaning of the data. Certainly the nurse could ask another nurse's opinion, but that is not essential. It would make no sense to have a social worker interview the patient unless the situation remains unclear even after confirming with the client. If the nurse did have adequate theoretical knowledge of Decisional Conflict for this patient, she should have been informed by reading the NANDA-I handbook before making the diagnosis. If the patient does not confirm the diagnosis, and the nurse concludes the diagnosis is in error, she might then reread the NANDA-I guide. PTS: 1 DIF: Moderate REF: V1, p. 56 | p. 63 KEY: Nursing process: Diagnosis | Client need: SECE | Cognitive level: Application

The patient is diagnosed with obstructive sleep apnea. Identify the symptoms you would expect the client to exhibit. Choose all that apply. 1)Bruxism 2)Enuresis 3)Daytime fatigue 4)Snoring 5)Drooling

ANS: 3, 4 Obstructive sleep apnea is caused by partial airway occlusion (usually by the tongue or palate) during sleep. The patient experiences interrupted sleep as he arouses frequently to clear the airway. As a result, the patient has episodes of snoring and daytime fatigue. PTS:1DIF:ModerateREF:p. 1210

Which of the following is the best approach to validate a clinical inference? 1) Have another nurse evaluate it. 2) Have the physician evaluate it. 3) Have sufficient supportive data. 4) Have the client's family confirm it.

ANS: 3 All clinical inferences should be well supported by data. The more reliable data you gather, the more certain you can be that your inference is accurate. Because inferences are nursing diagnoses, it would be inappropriate to have a physician evaluate them. Although another experienced nurse could evaluate the inference, it still needs to be supported by sound and sufficient data. Even clients can validate clinical inferences in some situations, but adequate supporting data are still needed. Keep in mind that the client's data might or might not be sufficient to "prove" the inference. PTS: 1 DIF: Easy REF: p. 63 KEY: Nursing process: Diagnosis | Client need: SECE | Cognitive level: Recall

A patient comes to the urgent care clinic because he stepped on a rusty nail. What type of assessment would the nurse perform? 1) Comprehensive 2) Ongoing 3) Initial focused 4) Special needs

ANS: 3 An initial focused assessment is performed during a first exam for specific abnormal findings. A comprehensive assessment is holistic and is usually done upon admission to a healthcare facility. An ongoing assessment follows up after an initial database is completed or a problem is identified. A special needs assessment is performed when there are cues that more in-depth assessment is needed. PTS:1DIF:ModerateREF: pp. 42-43 KEY: Nursing process: Assessment | Client need: PHSI | Cognitive level: Application

The nurse administers an antitussive/expectorant cough preparation to a patient with bronchitis. Which of the following responses indicates to the nurse that the medication is effective? 1) The amount of sputum the patient expectorates decreases with each dose administered. 2) Cough is completely suppressed, and she is able to sleep through the night. 3) Dry, unproductive cough is reduced, but her voluntary coughing is more productive. 4) Involuntary coughing produces large amounts of thick yellow sputum.

ANS: 3 Antitussives are cough suppressants that reduce the frequency of an involuntary, dry, nonproductive cough. Antitussives are useful for adults when coughing is unproductive and frequent, leading to throat irritation or interrupted sleep. Expectorants help make coughing more productive. The goal of an antitussive/expectorant combination is to reduce the frequency of dry, unproductive coughing while making voluntary coughing more productive. PTS:1DIFgrinifficultREF:p. 1310 KEY: Nursing process: Evaluation | Client need: PHSI | Cognitive level: Application

During the day shift, a patient's temperature measures 97°F (36.1°C) orally. At 2000, the patient's temperature measures 102°F (38.9°C). What effect does this rise in temperature have on the patient's basal metabolic rate? 1) Increases the rate by 7% 2) Decreases the rate by 14% 3) Increases the rate by 35% 4) Decreases the rate by 28%

ANS: 3 Basal metabolic rate increases 7% for each degree Fahrenheit (0.56°C); therefore, this patient's temperature rise is an increase of 35%. PTS:1DIFgrinifficultREF:p. 910 KEY: Nursing process: Assessment | Client need: PHSI | Cognitive level: Application

A patient's 2:1 parenteral nutrition container infuses before the pharmacy prepares the next container. This places the patient at risk for which complication? 1) Sepsis 2) Pneumothorax 3) Hypoglycemia 4) Thrombophlebitis

ANS: 3 Because of the high glucose content of 2:1 parenteral nutrition, any interruption in therapy places the patient at risk for hypoglycemia. A PN of this type should not be discontinued abruptly, but rather over several (as many as 48) hours to prevent a sudden drop in blood sugar. Hypoglycemia is unlikely to occur with a 3:1 solution (containing lipids), as the final concentration of glucose is less than 10%. Sepsis is a complication that can occur if a break in aseptic technique occurs during therapy. Pneumothorax can occur as a result of central venous catheter insertion. Central venous catheters are typically employed for parenteral nutrition. Thrombophlebitis is a complication of central venous catheter use. PTS: 1 DIF: Difficult REF: pp. 959-960 KEY: Nursing process: Diagnosis | Client need: PHSI | Cognitive level: Analysis

When teaching a patient about the healing process of an open wound after surgery, which of the following points would the nurse make? 1) The patient will need to take antibiotics until the wound is completely healed. 2) Because the patient's wound was left open, the wound will likely become infected. 3) The patient will have more scar tissue formation than for a wound closed at surgery. 4) The patient should expect to remain hospitalized until complete wound healing occurs.

ANS: 3 Because the wound edges are not approximated, more scar tissue will form. Although open wounds are more prone to infection, this is not an expected outcome, and antibiotics would not necessarily be needed. A patient with an open wound should not expect an extended hospital stay if wound care can be provided in the home or an outpatient setting. PTS:1DIFgrinifficultREF:p. 1227 KEY: Nursing process: Interventions | Client need: PSI | Cognitive level: Application

How are critical pathways and standardized nursing care plans similar? Both 1) Specify daily, or even hourly, outcomes and interventions 2) Prescribe minimal care needed to meet recommended lengths of stay 3) Describe care common to all patients with a certain condition or situation 4) Emphasize medical problems and interventions

ANS: 3 Both critical pathways and standardized care plans are preplanned documents; they describe care common to all patients who have a certain condition (e.g., all patients who have a heart attack need some of the same interventions). The other statements are true of critical pathways but not of standardized nursing care plans. PTS:1DIFgrinifficultREF: pp. 86-87; high-level question, answer not given verbatim KEY: Nursing process: Planning | Client need: SECE | Cognitive level: Analysis

Which of the following provide the most reliable data about the effectiveness of airway suctioning? 1) The amount, color, consistency, and odor of secretions 2) The patient's tolerance for the procedure 3) Breath sounds, vital signs, and pulse oximetry before and after suctioning 4) The number of suctioning passes required to clear secretions

ANS: 3 Breath sounds, vital signs, and oxygen saturation levels before and after suctioning provide data about the effectiveness of suctioning. Information about the amount and appearance of secretions provides data about the likelihood of airway infection and/or inflammation. Data about the patient's tolerance of suctioning provide information about the patient's overall condition. The number of suctioning passes required to clear the secretions provides information about the amount of secretions present. PTS: 1 DIF: Moderate REF: p. 1351 KEY: Nursing process: Evaluation | Client need: PHSI | Cognitive level: Application

From what stage of sleep are people typically most difficult to arouse? 1)NREM, alpha waves 2)NREM, sleep spindles 3)NREM, delta waves 4)REM

ANS: 3 Contrary to previous beliefs, stages III and IV of NREM (delta wave) are the deepest stages of sleep—not REM. It is difficult to awaken someone in stage IV slow wave NREM sleep, and if she is awakened, the person may appear confused and react slowly. Stage I NREM is a light sleep from which the sleeper can easily be awakened. Stage II (sleep spindles) is also light sleep; the sleeper in this stage is easily roused. REM sleep is the stage at which most dream activity occurs, as well as more spontaneous awakenings. PTS:1DIF:ModerateREF:p. 1205

Which of the following is the most important reason for nurses to be critical thinkers? 1)Nurses need to follow policies and procedures. 2)Nurses work with other healthcare team members. 3)Nurses care for clients who have multiple health problems. 4)Nurses have to be flexible and work variable schedules.

ANS: 3 Critical thinking is essential for client care, particularly when the care is complex, involving numerous health issues. Following policies and procedures does not necessarily require critical thinking, and working with others or being flexible and working different schedules do not necessarily require critical thinking. PTS:1DIF:ModerateREF: p. 26-27; high-level question, answer not stated verbatim

Mr. Jackson is terminally ill with metastatic cancer of the colon. His family notices that he is suddenly more focused and coherent. They are questioning whether he is really going to die. The nurse recognizes that a sudden surge of activity may occur 1) Moments before death 2) Days to hours before death 3) 1 to 2 weeks before death 4) 1 to 3 months before death

ANS: 3 Days to hours before death, patients commonly experience a surge of energy that brings mental clarity and a desire to speak with family. One to 3 months before death, the dying person begins to withdraw from the world by sleeping more and eating less. One to 2 weeks before death, the body loses its ability to maintain itself, and body systems begin to deteriorate. Near the time of death, the dying person does not respond to touch or sound and cannot be awakened. PTS:1DIF:ModerateREF:p. 367 KEY: Nursing process: Assessment | Client need: PHSI | Cognitive level: Application

Which of the following is the main difference between sleep and rest? 1)In sleep, the body may respond to external stimuli. 2)Short periods of sleep do not restore the body as much as do short periods of rest. 3)Sleep is characterized by an altered level of consciousness. 4)The metabolism slows less during sleep than during rest.

ANS: 3 During rest, the mind remains active and conscious; sleep is characterized by altered consciousness. Sleep is a cyclic state of decreased motor activity and perception. A sleeping person is unaware of the environment and does respond selectively to certain external stimuli. However, at rest, the body is disturbed by all external stimuli; sleep restores the body more than does rest. The metabolism decreases more during sleep than during rest. PTS:1DIF:EasyREF:p. 1202

The school nurse at a local elementary school is performing physical fitness assessments on the third-grade children. When assessing students' cardiorespiratory fitness, the most appropriate test is to have the students: 1) Step up and down on a 12-inch bench. 2) Perform the sit-and-reach test. 3) Run a mile without stopping, if they can. 4) Perform range-of-motion exercises.

ANS: 3 Field tests for running are good for children and can be utilized when assessing cardiorespiratory fitness. The step test is appropriate for adults. The 12-inch bench height is too high for young children. The sit-and-reach test as well as range-of-motion exercises would be appropriate when assessing flexibility. PTS:1DIF:ModerateREF:pp. 884-885 KEY: Nursing process: Assessment | Client need: HPM | Cognitive level: Application

Which polysaccharide is stored in the liver? 1) Insulin 2) Ketones 3) Glycogen 4) Glucose

ANS: 3 Humans store glucose in the liver as polysaccharides, known as glycogen. Glycogen can then be converted back into glucose to meet energy needs through a process known as glycogenolysis. If fats must be used for energy, they are converted directly into ketones. Insulin is a pancreatic hormone that promotes the movement of glucose into cells. PTS: 1 DIF: Moderate REF: p. 902 KEY: Nursing process: N/A | Client need: PHSI | Cognitive level: Recall

A child is brought to the emergency department after swallowing liquid cleanser. He is awake and alert and able to swallow. Which action should the nurse take first? 1) Administer a dose of syrup of ipecac. 2) Administer activated charcoal immediately. 3) Give water to the child immediately. 4) Call the nearest poison control center.

ANS: 3 If the child is awake and able to swallow, and the child has swallowed a household chemical, give one-half glassful of water immediately. After giving the water, call the poison control center. The American Academy of Pediatrics does not advise giving syrup of ipecac. Emergency departments have stopped using ipecac in favor of activated charcoal, which binds to poison in the stomach and prevents it from entering the bloodstream. Continued vomiting caused by syrup of ipecac may later result in the child being unable to tolerate activated charcoal or other poison treatments. No one can tell how much a child vomits, and therefore, no one would know if all the poison was eliminated from the stomach. There is also potential for misuse by bulimics. The poison control center may recommend activated charcoal, depending upon the agent ingested. PTS: 1 DIF: Difficult REF: ESG, Chapter 23, Table 23-2, "Sources and Medical Treatment for Commonly Ingested Poisons" KEY: Nursing process: Interventions | Client need: PHSI | Cognitive level: Analysis

Which intervention should be included in the plan of care for a patient in the end-stage death process? 1) Encourage the patient to accept as much help as possible. 2) Avoid administering laxatives. 3) Wet the lips and mouth frequently. 4) Administer pain medication on an as-needed basis.

ANS: 3 If the patient is unable to take fluids, prevent dryness and cracking of lips and mucous membranes by wetting the lips and mouth frequently. Encourage the patient to be as independent as possible. Administer laxatives if constipation occurs. Administer pain medications on a regular schedule instead of waiting for the patient to request them. PTS:1DIF:ModerateREF:p. 375 KEY: Nursing process: Interventions | Client need: PHSI | Cognitive level: Application

The nurse is planning care for a patient. She is using a standardized care plan for Impaired Walking related to left-side weakness. Which of the following activities will the nurse perform when individualizing the plan for the patient? 1) Validate conflicting data with the patient. 2) Transcribe medical orders. 3) State the frequency for ambulation. 4) Perform a comprehensive assessment.

ANS: 3 Individualizing the care plan means identifying specific problems, outcomes, and interventions and the frequency of those interventions to meet the patient's needs. Validating data ensures your assessment is accurate. Transcribing orders is a part of developing and implementing the care plan but not of individualizing the plan. Performing an assessment is the beginning step to developing a care plan. Assessment helps you to know the ways in which a standardized plan needs to be individualized. PTS:1DIF:ModerateREF: p. 90 KEY: Nursing process: Planning | Client need: SECE | Cognitive level: Application

The muscle strength of a woman weighing 132 pounds who is able to lift 72 pounds would be recorded as 0.55. The nurse explains this to the client as the 1) Ratio of weight lifted divided by body weight 2) Measure of weight pushed divided by BMI 3) Ability of a muscle to perform repeated movements 4) Ability to move a joint through its range of motion

ANS: 3 Muscle strength measures the amount of weight a muscle (or group of muscles) can move at one time. This is recorded as a ratio of weight pushed (or lifted) divided by body weight. A woman weighing 132 pounds who is able to lift 72 pounds has a ratio of 72 divided by 132, or 0.55. PTS:1DIF:ModerateREF:pp. 884-885 KEY: Nursing process: Assessment | Client need: HPM | Cognitive level: Application

A patient states that many of his friends told him to ask for Valium or Ativan to help him sleep while hospitalized. The nurse knows that nonbenzodiazepines (such as Ambien) are often preferred over benzodiazepines (Ativan or Valium). Why is this? 1)Benzodiazepines are eliminated from the body faster than are nonbenzodiazepines, so they do not provide a full night of sleep. 2)Nonbenzodiazepines cause daytime sleepiness, allowing people to rest throughout the day. 3)Benzodiazepines produce daytime sleepiness and alter the sleep cycle. 4)Nonbenzodiazepines remain in the body longer than do benzodiazepines.

ANS: 3 Nonbenzodiazepines (such as Ambien) have a short half-life, which means that they are eliminated from the body quickly and do not cause daytime sleepiness. Ativan is a long-acting benzodiazepine and remains in the body longer than Ambien, often causing daytime sleepiness. PTS: 1 DIF: Moderate REF: p. 1215

Nondirective interviewing is a useful technique because it 1) Allows the nurse to have control of the interview 2) Is an efficient way to interview a patient 3) Facilitates open communication 4) Helps focus patients who are anxious

ANS: 3 Nondirective interviewing helps build rapport and facilitates open communication. Because it puts the patient in control, it can be very time-consuming (inefficient) and produce information that is not relevant. Directive interviewing should be used to focus anxious patients. PTS:1DIF:EasyREF:p. 45 KEY: Nursing process: Assessment | Client need: PSI | Cognitive level: Recall

Which of the following most accurately describes nursing diagnoses? A nursing diagnosis 1) Supports the nurse's diagnostic reasoning 2) Supports the client's medical diagnosis 3) Identifies a client's response to a health problem 4) Identifies a client's health problem

ANS: 3 Nursing diagnoses are statements that nurses use to describe a client's physical, mental, emotional, spiritual, and social response to disease, injury, or other stressor. Diagnostic reasoning is used to identify the appropriate nursing diagnosis; it is not meant to "support" the diagnosis. A health problem is a condition that requires intervention to promote wellness or prevent illness; it is sometimes, but not always, a nursing diagnosis. Nursing diagnoses are not medical diagnoses. PTS:1DIF:ModerateREF: p. 57 KEY: Nursing process: Diagnosis | Client need: SECE | Cognitive level: Analysis

Which of the following is an example of an open-ended question? 1)Have you had surgery before? 2)When was your last menstrual period? 3)What happens when you have a headache? 4)Do you have a family history of heart disease?

ANS: 3 Open-ended questions such as "What happens when you have a headache?" are broad so as to encourage the patient to elaborate. The questions about surgery, menstrual period, and family history can all be answered with a "yes," "no," or short, specific answer (a date). PTS:1DIF:ModerateREF:pp. 45 KEY: Nursing process: Assessment | Client need: SECE | Cognitive level: Application

Which of the following is the best example of an outcome statement? The patient will 1) Use the incentive spirometer when awake 2) Walk two times during day and evening shifts 3) Maintain oxygen saturation above 92% while performing ADLs each morning 4) Tolerate 10 sets of range-of-motion exercises with physical therapy

ANS: 3 Outcome statements should have specific performance criteria and a target time; "maintain oxygen saturation" is the only one that meets those criteria. The incentive spirometer goal should say how many times the incentive spirometer should be used each hour as well as the volume. The walking goal should state how far the patient should walk. In the range-of-motion goal, tolerate is a vague word and is difficult to measure, and the outcome needs to specify how often. PTS:1DIF:ModerateREF: p. 91-92 KEY: Nursing process: Planning | Client need: SECE | Cognitive level: Analysis

A patient who sustained rib fractures in a motor vehicle accident is complaining that his pain medication is ineffective. Inadequate pain control places this patient at risk for which complication? 1) Metabolic alkalosis 2) Pneumothorax 3) Pneumonia 4) Hemothorax

ANS: 3 Pain associated with rib fractures causes splinting. Splinting often causes the patient to breathe shallowly and avoid deep coughing to expel sputum, which can lead to pneumonia. Rib fractures can also lead to complications such as pneumothorax and hemothorax; however, they do not result from inadequate pain control. Respiratory acidosis, not metabolic alkalosis, may result from the shallow breaths caused by pain and restricted chest wall movement with splinting. PTS:1DIF:ModerateREF:p. 1096 KEY: Nursing process: Planning | Client need: PHSI | Cognitive level: Application

Which nutritional goal is appropriate for a patient newly diagnosed with hypertension? The patient will: 1) Limit his intake of protein. 2) Avoid foods containing gluten. 3) Restrict his use of sodium. 4) Limit his intake of potassium-rich foods.

ANS: 3 Patients with hypertension should limit their intake of sodium. Those with liver disease should control their protein intake. Patients with renal disease must limit their intake of potassium-rich foods. Patients with celiac disease should avoid foods containing gluten. PTS: 1 DIF: Easy REF: p. 917 KEY: Nursing process: Planning | Client need: PHSI | Cognitive level: Application

Which of the following is an example of practical knowledge? (Assume all are true.) 1)The tricuspid valve is between the right atrium and ventricle of the heart. 2)The pancreas does not produce enough insulin in type 1 diabetes. 3)When assessing the abdomen, you should auscultate before palpating. 4)Research shows pain medication given intravenously acts faster than by other routes.

ANS: 3 Practical knowledge is knowing what to do and how to do it, such as how to do an assessment. The others are examples of theoretical knowledge, anatomy (tricuspid valve), fact (type 1 diabetes), and research (IV pain medication). PTS:1DIF:ModerateREF:p. 30; high-level question, answer not stated verbatim

A 55-year-old man suffered a myocardial infarction (heart attack) 3 months ago. During his hospitalization, he had stents inserted in two sites in the coronary arteries. He was also placed on a cholesterol-lowering agent and two antihypertensives. What type of care is he receiving? 1) Primary prevention 2) Secondary prevention 3) Tertiary prevention 4) Health promotion

ANS: 3 Primary prevention activities are designed to prevent or slow the onset of disease. Activities such as eating healthy foods, exercising, wearing sunscreen, obeying seat-belt laws, and getting immunizations are examples of primary-level interventions. Secondary prevention activities detect illness so it can be treated in the early stages. Tertiary prevention focuses on stopping the disease from progressing and returning the individual to the pre-illness phase. The patient has an established disease and is receiving care to stop the disease from progressing. PTS:1DIFgrinifficultREF:p. 879 KEY:Nursing process: Planning | Client need: PSI | Cognitive level: Application

A patient with morbid obesity was enrolled in a weight loss program last month and has attended four weekly meetings. But now he believes he no longer needs to attend meetings because he has "learned what to do." He informs the nurse facilitator about his decision to quit the program. What should the nurse tell him? 1) "By now you have successfully completed the steps of the change process. You should be able to successfully lose the rest of the weight on your own." 2) "Although you have learned some healthy habits, you will need at least another 6 weeks before you can quit the program and have success." 3) "You have done well in this program. However, it is important to continue in the program to learn how to maintain weight loss. Otherwise, you are likely to return to your previous lifestyle." 4) "You have entered the 'determination stage' and are ready to make positive changes that you can keep for the rest of your life. If you need additional help, you can come back at a later time."

ANS: 3 Prochaska and Diclemente identified four stages of change: the contemplation stage, the determination stage, the action stage, and the maintenance stage. This patient demonstrates behaviors typical of the action stage. If a participant exits a program before the end of the maintenance stage, relapse is likely to occur as the individual resumes his previous lifestyle. PTS:1DIF:ModerateREF:pp. 881-882 KEY: Nursing process: Interventions | Client need: PSI | Cognitive level: Application

While addressing a community group, the nurse explains the importance of replacing saturated fats in the diet with mono- and polyunsaturated fats. She emphasizes that doing so greatly reduces the risk of which complication? 1) Kidney failure 2) Liver failure 3) Stroke 4) Lung cancer

ANS: 3 Replacing saturated fats in the diet with mono- and polyunsaturated fats reduces the risk of heart disease, atherosclerosis, and stroke, not kidney failure, liver failure, or lung cancer. PTS:1DIF:ModerateREF:p. 903 KEY: Nursing process: Interventions | Client need: HPM | Cognitive level: Application

Which essential oil might the nurse trained in aromatherapy use to uplift and stimulate a patient? 1) Lavender 2) Roman chamomile 3) Rosemary 4) Ylang-ylang

ANS: 3 Rosemary is very stimulating and uplifting. Lavender, Roman chamomile, and Ylang-ylang are used to promote relaxation. PTS:1DIF:ModerateREF:p. 1073 KEY: Nursing process: Interventions | Client need: PHSI | Cognitive level: Comprehension

Which of the following is an example of self-knowledge? The nurse thinks, "I know that I 1)Should take the client's apical pulse for 1 minute before giving digoxin" 2)Should follow the client's wishes even though it is not what I would want" 3)Have religious beliefs that may make it difficult to take care of some clients" 4)Need to honor the client's request not to discuss his health concern with the family"

ANS: 3 Self-knowledge is being aware of your religious and cultural beliefs and values. Taking the pulse is an example of practical knowledge. Following client wishes and honoring client requests are examples of ethical knowledge. PTS:1DIFgrinifficultREF: p. 30; high-level question, answer not stated verbatim | V1, p. 32; high-level question, answer not stated verbatim

The nurse is caring for a patient with dementia who becomes agitated every evening. Which intervention by the nurse is best for calming this patient? 1) Encouraging family members to visit only during the day 2) Applying wrist restraints during periods of agitation 3) Playing soft, calming music during the evening 4) Administering lorazepam (a tranquilizer)

ANS: 3 Soft, calming music is sometimes helpful for patients with dementia. Encouraging a family member to sit with the patient might have a calming effect, but the option does not provide for that during the evening when the patient is symptomatic. Applying bilateral wrist restraints might further agitate the patient. Lorazepam will provide sedation but might cause further confusion. PTS:1DIF:ModerateREF:p. 1081 KEY: Nursing process: Interventions | Client need: PHSI | Cognitive level: Analysis

15. The nurse notes that a patients indwelling urinary catheter tubing contains sediment and crusting at the meatus. Which action should the nurse take? 1) Notify the provider immediately. 2) Flush the catheter tubing with saline solution. 3) Replace the indwelling urinary catheter. 4) Encourage fluids that increase urine acidity.

ANS: 3 The catheter needs to be changed when sediment collects in the tubing or catheter and crusting at the meatus occurs. It is not necessary to notify the provider immediately. The nurse should not flush the catheter tubing. The patient should be encouraged to consume fluids that increase urine acidity to prevent urinary tract infection; however, it will not help clear the catheter tubing of sediment.

Which client does the nurse recognize as being at greatest risk for pressure ulcers? 1) Infant with skin excoriations in the diaper region 2) Young adult with diabetes in skeletal traction 3) Middle-aged adult with quadriplegia 4) Older adult requiring use of assistive device for ambulation

ANS: 3 The client at greatest risk for pressure sores is the one with a lack of sensory perception at the site (e.g., quadriplegia). The infant with disruption to the skin from diaper rash is at risk for skin infection but not for a pressure sore. The young adult with diabetes is at increased risk for delayed wound healing but not likely for a pressure sore because he would shift weight in bed and respond to discomfort of pressure on a bony site. The older adult is normally at risk for pressure injury, but when mobile, even with an assistive device, the risk is minimal. PTS:1DIF:ModerateREF:p. 1224 KEY: Nursing process: Assessment | Client need: PHSI | Cognitive level: Analysis

Which nursing diagnosis is written in the correct format when using the NANDA-I taxonomy? 1) Bowel Obstruction related to recent abdominal surgery A.M.B. nausea, vomiting, and abdominal pain 2) Inability to Ingest Food related to imbalanced nutrition: less than body requirements A.M.B. inadequate food intake, weight less than 20% under ideal body weight 3) Impaired Skin Integrity related to physical immobility A.M.B. skin tear over sacral area 4) Caregiver Role Strain related to alienation from family and friends A.M.B 24-hour care responsibilities

ANS: 3 The components of NANDA-I nursing diagnosis might include the following four parts: diagnostic label, defining characteristics, related factors, and risk factors. "Impaired Skin Integrity . . ." has the problem statement, etiology, and symptoms. For "Bowel Obstruction . . ." the problem is a medical diagnosis. The cause-and-effect order of "Inability to Ingest Food . . ." is incorrect; it starts with the etiology. The etiology and symptoms (A.M.B.) of "Caregiver Role Strain . . ." are reversed (alienation from family and friends are the symptoms that support the diagnosis). PTS: 1 DIF: Difficult REF: pp. 70-71 KEY: Nursing process: Diagnosis | Client need: SECE | Cognitive level: Application

The nurse is beginning discharge planning for an older adult with left-side weakness. All of the following are important, but which action is most important in ensuring that the discharge plan is successful? 1) Start planning at admission. 2) Involve the family members. 3) Get patient input when making the plan. 4) Involve the multidisciplinary team.

ANS: 3 The discharge plan may be developed in a timely manner and involve the family and a multidisciplinary team, but if the patient does not agree with the plan, it will not be successful. PTS: 1 DIF: Moderate REF: pp. 83| p. 87 KEY: Nursing process: Planning | Client need: SECE | Cognitive level: Analysis

A patient has just had a chest tube inserted to dry-seal suction drainage. Which of the following is a correct nursing intervention for maintenance? 1) Keep the head of the bed flat for 6 hours. 2) Immobilize the patient's arm on the affected side. 3) Keep the drainage system lower than the insertion site. 4) Drain condensation into the humidifier when it collects in the tubing.

ANS: 3 The drainage system must be below the insertion site to prevent fluid flowing back into the pleural cavity and compromising the patient's respiratory status. Maintain patient in semirecumbent position (head of bed elevated 30 to 45 degrees), not flat. This is extremely important to promote lung expansion, reduce gastric reflux, and prevent ventilator-associated pneumonia (VAP), if the person is being mechanically ventilated. Patients being mechanically ventilated are at high risk for developing VAP, which is associated with high mortality rates. Mouth rinses and mouthwashes are a part of the recommended routine for preventing VAP. They also provide comfort and preserve integrity of the mucous membranes. Encourage the patient to move the arm on the affected side; if he cannot, perform passive range-of-motion. You should check the ventilator tubing frequently for condensation, and drain the fluid into a collection device or waste receptacle because condensation in the ventilator tubing can cause resistance to airflow. Moreover, the patient can aspirate it if it backflows down into the endotracheal tube. The fluid should not be drained into the humidifier because the patient's secretions may have contaminated it. PTS: 1 DIF: Difficult REF: p. 1353 KEY: Nursing process: Interventions | Client need: SECE | Cognitive level: Application

Which of the following explains why it is important to have the correct etiology for a nursing diagnosis? The etiology 1) Is the cause of the problem 2) Cannot always be observed 3) Directs nursing care 4) Is an inference

ANS: 3 The etiology directs nursing interventions. If the incorrect etiology is given, the nursing care would not be appropriate for the client. The other statements are true but not a reason for the importance of the etiology being correct. PTS:1DIFgrinifficultREF:pp. 63 KEY: Nursing process: Diagnosis | Client need: SECE | Cognitive level: Analysis

Which nursing diagnosis is written in the correct format? 1) Imbalanced Nutrition: Less than Body Requirements related to body weight less than 20% under ideal weight 2) Ineffective Airway Clearance related to increased respiratory rate and irregular rhythm 3) Impaired Swallowing related to absent gag reflex 4) Excess Fluid Volume related to 3 lb weight gain in 24 hours

ANS: 3 The etiology should describe what is causing or contributing to the problem. The etiologies for Ineffective Airway Clearance, Impaired Airway Swallowing, and Excess Fluid Volume describe signs or symptoms rather than causal factors. PTS: 1 DIF: Difficult REF: V1, p. 64 | pp. 70-73 KEY: Nursing process: Diagnosis | Client need: SECE | Cognitive level: Analysis

Which of the following is true for goals/outcomes for collaborative problems? 1) They are monitored only by other disciplines. 2) They are usually sensitive to nursing interventions. 3) They state that a complication will not occur. 4) They state only broad performance criteria.

ANS: 3 The goal for a collaborative problem is always that the complication will not occur. Other disciplines may be involved in helping to prevent the problem, but nurses still monitor for the complication. The outcomes to collaborative problems are not affected by nursing interventions alone. Goals for collaborative problems are specific to the medical condition/treatment. PTS:1DIF:ModerateREF: pp. 93 KEY: Nursing process: Planning | Client need: SECE | Cognitive level: Analysis

A female client experienced a pelvic fracture in a motor vehicle accident several months earlier and recovery has been slow. Among the challenges presented by this event is the fact that sexual activity causes a dull ache in her pelvis. Which nursing diagnosis is most appropriate for this client?

Alteration in Comfort: Pain

What happens when a person is dehydrated in the kidneys and bladder?

the kidneys conserve water when a person is dehydrated, when fluid intake is inadequate. This causes the urine to be concentrated and low in volume

For the client with a stage IV pressure ulcer, what would an applicable patient goal/outcome be? 1) Client will maintain intact skin throughout hospitalization. 2) Client will limit pressure to wound site throughout treatment course. 3) Wound will close with no evidence of infection within 6 weeks. 4) Wound will improve prior to discharge as evidenced by a decrease in drainage

ANS: 3 The goal for any wound is for healing to take place with no complications (such as infection). Intact skin throughout hospitalization is not realistic with a stage IV pressure ulcer. Limiting pressure to a wound site is incorrect because total pressure relief must be provided to the area. Improved wound drainage before discharge is not a realistic expectation for a stage IV pressure ulcer. PTS:1DIFgrinifficultREF:p. 1234 KEY: Nursing process: Planning | Client need: SECE | Cognitive level: Synthesis

23. The nurse is teaching an older female patient how to manage urge incontinence at home. What is the first-line approach to reducing involuntary leakage of urine? 1) Insertion of a pessary 2) Intermittent self-catheterization 3) Bladder training 4) Anticholinergic medication

ANS: 3 The goal of bladder training is to enable the patient to hold increasingly greater volumes of urine in the bladder and to increase the interval between voiding. This involves patient teaching, scheduled voiding, and self-monitoring using a voiding diary. In addition to teaching the mechanisms of urination, teach distraction and relaxation strategies to help inhibit the urge to void. Other techniques include deep breathing and guided imagery. A pessary is an incontinence device that is inserted into the vagina to reduce organ prolapse or pressure on the bladder. Clean, intermittent self-catheterization is a good option for managing incontinence that is resistant to conservative measure, such as bladder training, Kegel exercises, lifestyle modification, and medication. Anticholinergic medication can be highly effective for improving urinary incontinence. However, more conservative measures, such as timed voiding and Kegel exercises, are recommended first.

Which situation is the most conducive to conducting a successful interview of an elderly woman whose husband and two children are in the hospital room visiting and watching television? The woman is alert and oriented. 1) Provide enough chairs so the family and you are able to sit facing the client. 2) Introduce yourself and ask, "Dear, what name do you prefer to go by?" before asking any questions. 3) After the family leaves, ask the client if she is comfortable and willing to answer a few questions. 4) Ask the client if you can talk with her while her family is watching the television.

ANS: 3 The interview should be done when the client is comfortable and there are no distractions. Endearing terms are inappropriate unless the client prefers them. Family members may offer information that may or may not be pertinent and may distract from the interview. The presence of family members may also inhibit full disclosure of information by the client. PTS:1DIFgrinifficultREF:p. 46-47 KEY: Nursing process: Assessment | Client need: PSI | Cognitive level: Application

19. Which task can the nurse safely delegate to the nursing assistive personnel? 1) Palpating the bladder of a patient who is unable to void 2) Administering a continuous bladder irrigation 3) Providing indwelling urinary catheter care 4) Obtaining the patients history and physical assessment

ANS: 3 The nurse can safely delegate indwelling urinary catheter care to nursing assistive personnel who are adequately trained to do so. Palpating the bladder, administering continuous bladder irrigation, and obtaining the patients history and physical assessment involve the critical thinking skills of a professional nurse.

Which pain management task can be safely delegated to nursing assistive personnel? 1) Assessing the quality and intensity of the patient's pain 2) Evaluating the effectiveness of pain medication 3) Providing a therapeutic back massage 4) Administering oral dose of acetaminophen

ANS: 3 The nurse can safely delegate providing a back massage for the patient in pain. However, the nurse should never delegate the responsibility of assessing the patient's pain, monitoring the patient's response to pain management strategies, administering medications (including over-the-counter preparations), or evaluating the pain management plan. PTS:1DIF:ModerateREF:p. 1100 KEY: Nursing process: Interventions | Client need: SECE | Cognitive level: Application

The mother of a preschool child dies suddenly of a ruptured cerebral aneurysm. What recommendation should the nurse make to the family regarding how to most therapeutically care for the child? 1) Take the child to the funeral even if he is frightened. 2) Notify the physician immediately if the child shows signs of regression. 3) Spend as much time as possible with the child. 4) Provide distraction whenever the child begins to express feelings of sadness.

ANS: 3 The nurse should advise the family to spend as much time as possible with the child. If the child is frightened about attending the funeral, he should not be forced to attend. Signs of regression are a normal reaction to the loss of a loved one, especially a parent. The child should be encouraged to express his feelings and fears. PTS:1DIF:ModerateREF:p. 380 KEY: Nursing process: Interventions | Client need: PSI | Cognitive level: Application

A patient develops a respiratory rate 6 breaths/minute after receiving IV hydromorphone (Dilaudid) 2 mg. Which medication should the nurse anticipate administering to this patient after notifying the prescriber of this side effect? 1) Physostigmine (Antilirium) 2) Flumazenil (Romazicon) 3) Naloxone (Narcan) 4) Protamine sulfate

ANS: 3 The nurse should anticipate administering naloxone to reverse the respiratory depression associated with opioid use. Flumazenil reverses the central nervous system depressant effects of benzodiazepines. Physostigmine reverses the effects of anticholinergic drugs. Protamine sulfate is the antidote for heparin. PTS:1DIF:ModerateREF:p. 1104 KEY: Nursing process: Planning | Client need: SECE | Cognitive level: Application

Which intervention is appropriate for the patient with a nursing diagnosis of Disturbed Sensory Perception: Gustatory? 1) Limit oral hygiene to one time a day. 2) Teach the patient to combine foods in each bite. 3) Assess for sores or open areas in the mouth. 4) Instruct the patient to avoid salt substitutes.

ANS: 3 The nurse should assess for sores or open areas in the mouth and provide frequent oral hygiene. The nurse should also teach the patient to eat foods separately to allow the taste of food to be distinguishable. Seasonings, salt substitutes, spices, or lemon may improve the taste of foods, so the patient should not avoid them. PTS:1DIF:ModerateREF:p. 1083 KEY: Nursing process: Interventions | Client need: PHSI | Cognitive level: Application

How should the nurse respond to a family immediately after a patient dies? 1) Ask the family to leave the patient's room so postmortem care can be performed. 2) Leave tubes and IV lines in place until the family has the opportunity to view the body. 3) Express sympathy to the family (e.g., "I am sorry for your loss"). 4) Tell the family that they will have limited time with their loved one.

ANS: 3 The nurse should express sympathy to the family immediately after the patient's death. She should give the family as much time as they need with their loved one and take care to present the body in a restful pose. If family members are not present at the time of death, remove tubes and IV lines before they see the body, unless an autopsy is planned or the death is being investigated by the coroner. The body should not be removed from the patient care area until the family is ready. PTS:1DIF:ModerateREF:p. 378 KEY: Nursing process: Interventions | Client need: PSI | Cognitive level: Application

9. What position should the patient assume before the nurse inserts an indwelling urinary catheter? 1) Modified Trendelenburg 2) Prone 3) Dorsal recumbent 4) Semi-Fowlers

ANS: 3 The nurse should have the patient lie supine with knees flexed, feet flat on the bed (dorsal recumbent position). If the patient is unable to assume this position, the nurse should help the patient to a side-lying position. Modified Trendelenburg position is used for central venous catheter insertion. Prone position is sometimes used to improve oxygenation in patients with adult respiratory distress syndrome. Semi-Fowlers position is used to prevent aspiration in those receiving enteral feedings.

The nurse is teaching a client who sustained an ankle injury about cold application. Which instruction should the nurse include in the teaching plan? 1) Place the cold pack directly on the skin over the ankle. 2) Apply the cold pack to the ankle for 30 minutes at a time. 3) Check the skin frequently for extreme redness. 4) Keep the cold pack in place for at least 24 hours.

ANS: 3 The nurse should instruct the patient to cover the cold pack with a washcloth, towel, or fitted sheet before applying it to the ankle to prevent tissue damage. A cold pack should be applied intermittently for the first 24 hours, leaving it in place for no longer than 15 minutes at a time. The patient should check the skin frequently and discontinue the treatment immediately if redness or other signs of tissue irritation occur. PTS:1DIF:ModerateREF:p. 1101 KEY: Nursing process: Interventions | Client need: SECE | Cognitive level: Application

A patient in the emergency department is angry, yelling, cursing, and waving his arms when the nurse comes to the treatment cubicle. Which action(s) by the nurse is(are) advisable? 1) Reassure the patient by entering the room alone. 2) Ask the patient if he is carrying any weapons. 3) Stay between the patient and the door; keep the door open. 4) Make eye contact while stating firmly "I will not tolerate cursing and threats."

ANS: 3 The nurse should keep the door open and position herself so that the patient cannot block her exit from the room (stay between the patient and the door). The nurse should not enter a room alone with an angry patient. The progression to physical violence is first anxiety, then verbal aggression, and finally physical aggression. The nurse's first priority in this situation is her own safety and the safety of others in the environment. The object is to relieve the patient's anxiety and not respond to anger with anger. Questioning about weapons, or being firm and defending against verbal aggression will likely provoke even more anger from the patient. The nurse must be calm and reassuring. PTS:1DIFgrinifficultREF:p. 674 KEY: Nursing process: N/A | Client need: PSI | Cognitive level: Application

The nurse must irrigate the ear of a 4-year-old child. How should the nurse pull the pinna to straighten the child's ear canal? 1) Up and back 2) Straight back 3) Down and back 4) Straight upward

ANS: 3 The nurse should straighten the ear canal of a small child by pulling the pinna down and back. To straighten the ear canal of an adult, the nurse should pull the pinna up and outward. PTS: 1 DIF: Moderate REF: p. 1086 KEY: Nursing process: Interventions | Client need: PHSI | Cognitive level: Application

A 62-year-old man with emphysema says, "My doctor wants me to quit smoking. It's too late now, though; I already have lung problems." Which of the following would be the best response to his statement? 1) "You should quit so your family does not get sick from exposure to secondhand smoke." 2) "You will need to use oxygen, but remember it is a fire hazard to smoke with oxygen in your home." 3) "Once you stop smoking, your body will begin to repair some of the damage to your lungs." 4) "You should ask your primary care provider for a prescription for a nicotine patch to help you quit."

ANS: 3 The nurse's response should focus on correcting the patient's misinformation rather than on convincing him to stop smoking. Once a person stops smoking, the body begins to repair the damage. During the first few days, the person will cough more as the cilia begin to clear the airways. Then the coughing subsides, and breathing becomes easier. Even long-time smokers can benefit from smoking cessation. The suggestions that the patient's family will become ill and that oxygen is a fire hazard appear to be scare tactics, which can be seen as coercive, and would not be effective in motivating the patient to stop smoking. Although asking the primary care provider for a prescription may help the patient to stop smoking, it does not address his incorrect belief that it is "too late" for him to do so. PTS:1DIF:ModerateREF:p. 1299 KEY: Nursing process: Interventions | Client need: PHSI | Cognitive level: Application

A patient complains of an impaired sense of smell. Which cranial nerve might have been affected? 1) Trigeminal 2) Glossopharyngeal 3) Olfactory 4) Vagus

ANS: 3 The olfactory nerve is responsible for the sense of smell. Damage to this nerve causes an impaired sense of smell. The trigeminal nerve transmits stimuli from the face and head. The glossopharyngeal nerve is responsible for taste. The vagus nerve is responsible for sensations of the throat, larynx, and thoracic and abdominal viscera. PTS: 1 DIF: Moderate REF: p. 1072 KEY: Nursing process: Assessment | Client need: PHSI | Cognitive level: Comprehension

A patient has an area of nonblanchable erythema on his coccyx. The nurse has determined this to be a stage I pressure ulcer. What would be the most important treatment for this patient? 1) Transparent film dressing 2) Sheet hydrogel 3) Frequent turn schedule 4) Enzymatic débridement

ANS: 3 The patient should be placed on a turn schedule to relieve the pressure. If pressure is not relieved, the wound will worsen. A stage I wound is not open, so a dressing is not warranted. Enzymatic débridement is used to remove slough or eschar in an open wound. A transparent film dressing would protect the area. However, the primary treatment is to relieve the source of pressure. PTS:1DIF:ModerateREF:p. 1244 KEY: Nursing process: Interventions | Client need: SECE | Cognitive level: Analysis

Which nursing diagnosis has the highest priority for a patient with impaired tactile perception? 1) Self-Care Deficit: Dressing and Grooming 2) Impaired Adjustment 3) Risk for Injury 4) Activity Intolerance

ANS: 3 The patient with impaired tactile perception is unable to perceive touch, pressure, heat, cold, or pain, placing him at risk for injury. Self-Care Deficit, Impaired Adjustment, and Activity Intolerance are also likely to be appropriate for this patient but are not as high a priority as Risk for Injury. Risk for Injury is directly related to safety, which must always be a priority. PTS: 1 DIF: Moderate REF: p. 1072 KEY: Nursing process: Diagnosis | Client need: PHSI | Cognitive level: Analysis

What is the function of the stratum corneum? 1) Provides insulation for temperature regulation 2) Provides strength and elasticity to the skin 3) Protects the body against the entry of pathogens 4) Continually produces new skin cells

ANS: 3 The stratum corneum is the outermost layer of the epidermis and is composed of numerous thicknesses of dead cells. Functioning as a barrier to the environment, it restricts water loss, prevents entry of fluids into the body, and protects the body against the entry of pathogens and chemicals. The subcutaneous layer is composed of adipose and connective tissue that provide insulation, protection, and an energy reserve (adipose). The dermis is composed of irregular fibrous connective tissue that provides strength and elasticity to the skin. The stratum germinativum is the innermost layer of the skin that produces new cells, pushing older cells toward the skin surface. PTS:1DIF:ModerateREF:p. 1223 KEY: Nursing process: Assessment | Client need: PHSI | Cognitive level: Comprehension

The nurse uses his hands to direct energy fields surrounding the patient's body. After this intervention, the patient states that his pain has lessened. How should the nurse document the intervention? 1) Tactile distraction was performed and appeared effective in reducing pain. 2) Guided imagery was effective to relax the patient and reduce the pain. 3) Therapeutic touch was performed; patient verbalized lessening of pain after treatment. 4) Sequential muscle relaxation was performed; patient states pain is less.

ANS: 3 Therapeutic touch focuses on the use of hands to direct energy fields surrounding the body. The nurse should document use of therapeutic touch and its effectiveness in the progress notes after performing the procedure. Tactile distraction involves activities such as massage, hugging a favorite toy, holding a loved one, or stroking a pet. Guided imagery uses auditory and imaginary processes to help the patient to relax. In sequential muscle relaxation, the patient sits and tenses muscles for 15 seconds and then relaxes the muscle while breathing out. This relaxation technique is often effective for relieving pain. PTS:1DIF:ModerateREF:p. 1102 KEY: Nursing process: Evaluation | Client need: PHSI | Cognitive level: Application

A patient dying of heart failure has changed his choice about his end-of-life treatment measures several times. He says, "I just can't make up my mind about it." Which nursing diagnosis is most appropriate for this patient? 1) Deficient Knowledge 2) Spiritual Distress 3) Decisional Conflict 4) Death Anxiety

ANS: 3 This patient is experiencing Decisional Conflict related to his end-of-life treatment measures. Deficient Knowledge, Spiritual Distress, or Death Anxiety may be the etiology of his changing decisions, but his indecision about his treatment option clearly identifies his Decisional Conflict. PTS:1DIF:ModerateREF:pp. 367-368; high-level question, not stated verbatim in text | V2, pp. 168-169; high-level question, not stated verbatim in text KEY: Nursing process: Nursing diagnosis | Client need: PSI | Cognitive level: Analysis

A patient who sustained a leg laceration in an industrial accident is brought to the emergency department. The area around the laceration is red, swollen, and tender. Which substance is responsible for causing this response? 1) Histamine 2) Prostaglandin 3) Bradykinin 4) Serotonin

ANS: 3 Tissue damage causes the release of the substances histamine, bradykinin, and prostaglandin. Bradykinin triggers the release of inflammatory chemicals that cause the injured area to become red, swollen, and tender. Serotonin is a neurotransmitter and is not involved in the inflammatory response. PTS:1DIFgrinifficultREF:p. 1092 KEY: Nursing process: NA | Client need: PHSI | Cognitive level: Application

24. What is the best technique for obtaining a sterile urine specimen from an indwelling urinary catheter? 1) Use antiseptic wipes to cleanse the meatus prior to obtaining the sample. 2) Briefly disconnect the catheter from the drainage tube to obtain the sample. 3) Withdraw urine through the port using a needleless access device. 4) Obtain the urine specimen directly from the collection bag.

ANS: 3 To obtain a specimen from an indwelling catheter, insert the needleless access device with a 20- or 30-mL syringe into the specimen port, and aspirate to withdraw the amount of urine you need. Wiping the meatus with an antiseptic material helps to minimize contamination for a clean-catch voided specimen, not a sample collected from a closed system such as an indwelling catheter system. Never disconnect the catheter from the drainage tube to obtain a sample. Interrupting the system creates a portal of entry for pathogens, thereby increasing the risk of contamination. Do not take the specimen from the collection bag because that urine may be several hours old.

How should the nurse classify pain that a patient with lung cancer is experiencing? 1) Radiating 2) Deep somatic 3) Visceral 4) Referred

ANS: 3 Visceral pain is commonly experienced in the abdominal cavity, cranium, or thorax. Lung cancer produces visceral pain. Radiating pain starts at the source and extends to other locations. Deep somatic pain is typically caused by fracture, sprain, arthritis, and bone cancer. Referred pain occurs in an area distant from the original site. PTS:1DIF:ModerateREF:p. 1091 KEY: Nursing process: Assessment | Client need: PHSI | Cognitive level: Comprehension

A patient with type 1 diabetes mellitus is admitted with hyperglycemia and associated acidosis. The presence of which alternative fuel in the body is responsible for the acidosis? 1) Glycogen 2) Insulin 3) Ketones 4) Proteins

ANS: 3 When fats are converted to ketones for use as alternative fuel, as in diabetic ketoacidosis when glucose cannot by used by the cells, the acidity of the blood rises, leading to the acidosis. Glycogen is converted to glucose to meet energy needs. Insulin, a pancreatic hormone, promotes the movement of glucose into cells for use. Proteins would not be used for fuel as long as fats were available. PTS:1DIFgrinifficultREF:pp. 902, 925 KEY: Nursing process: Assessment | Client need: PHSI | Cognitive level: Comprehe

Of the following interventions, which is/are likely to reduce the risk of postoperative atelectasis? Choose all that apply. 1) Administer bronchodilators. 2) Apply low-flow oxygen. 3) Encourage coughing and deep breathing. 4) Administer pain medication.

ANS: 3, 4 Pain alters the rate and depth of respirations. Often, patients in pain breathe shallowly, which puts them at risk for atelectasis. Regularly assess all patients for pain. Once you have medicated the patient, reassess breath sounds, and encourage the patient to cough and breathe deeply. This will help to open air sacs and mobilize secretions in the airways. PTS: 1 DIF: Moderate REF: pp. 1303, 1313 ; critical-thinking item that requires synthesis of information KEY: Nursing process: Interventions | Client need: PHSI | Cognitive level: Synthesis

Which intervention is appropriate for a client receiving palliative care? Choose all that apply. 1) Surgical insertion of a device to decrease the workload of the heart in a patient awaiting heart transplantation 2) Administering IV dopamine to raise blood pressure of a patient with end-stage lung cancer 3) Providing moisturizing eye drops to an unconscious patient whose eyes are dry 4) Administering a medication to relieve the nausea of a patient with end-stage leukemia

ANS: 3, 4 Palliative care focuses on relieving symptoms for patients whose disease process no longer responds to treatment. Providing moisturizing eye drops and administering antinausea medication in a patient with end-stage leukemia are examples of palliative care. Surgical insertion of a device to decrease heart workload and administering dopamine are aggressive treatment measures. PTS:1DIF:ModerateREF:p. 363 KEY: Nursing process: Interventions | Client need: PHSI | Cognitive level: Application

. For an unconscious patient, which of the following interventions are necessary to provide for patient safety? Choose all that apply. 1) Talk to the patient as you provide care. 2) Incorporate more touch in the plan of care. 3) Give frequent eye care if blink reflex is absent. 4) Keep the side rails up and bed in low position.

ANS: 3, 4 Safety measures are a priority for unconscious clients. Keep the bed in low position when you are not at the bedside, and keep the side rails up. If the patient's blink reflex is absent or her eyes do not close totally, you may need to give frequent eye care to keep secretions from collecting along the lid margins. The eyes may be patched to prevent corneal drying, and lubricating eye drops may be ordered. It is important to talk to the patient because the sense of hearing may still be intact. This provides some stimulation and may help with reality orientation. Providing touch will also help prevent sensory deficit; however, it is not a safety measure. PTS:1DIFgrinifficultREF:p. 1085 KEY: Nursing process: Interventions | Client need: PHSI | Cognitive level: Application

What are two risk assessment tools used in the United States to evaluate a patient's risk for pressure ulcers? Choose all that apply. 1) Pressure ulcer healing chart 2) PUSH tool 3) Braden scale 4) Norton scale

ANS: 3, 4 The Braden scale is a tool used to predict the risk of developing a pressure sore. Evaluation is based on six areas (indicators): sensory perception, moisture, activity, mobility, nutrition, and friction or shear. The Norton scale is another tool used to assess the risk for pressure ulcers based on the patient's physical condition, mental state, activity, mobility, and incontinence. These are the two most used risk assessment tools in the United States. Both of these tools are used to identify persons at high risk of pressure ulcer development. The PUSH tool provides a comprehensive means of reporting the progression of a pressure ulcer. Surface area, exudate, and type of wound tissue are scored and totaled. The Pressure Ulcer Healing Chart is part of the PUSH tool, which is used to monitor the progression of a pressure ulcer. PTS: 1 DIF: Moderate REF: p. 1235 KEY: Nursing process: Assessment | Client need: PHSI | Cognitive level: Recall

. The nurse would know care for a stage II pressure ulcer is achieving the desired goal when: 1) The ulcer is completely healed with minimal scarring. 2) The patient reports no pain at the site. 3) A minimal amount of drainage is noted. 4) The wound bed contains 100% granulated tissue.

ANS: 4 A healing wound contains granulating tissue. Although pain and drainage are indicators of inflammation, infection, and bleeding, no pain or drainage at the wound site does not indicate proper healing is occurring. A wound can heal leaving a scar. PTS:1DIFgrinifficult REF: p. 1227; higher-order item, answer can be derived from text KEY: Nursing process: Evaluation | Client need: PHSI | Cognitive level: Application

How does nursing diagnosis differ from a medical diagnosis? A nursing diagnosis is 1)Terminology for the client's disease or injury 2)A part of the client's medical diagnosis 3)The client's presenting signs and symptoms 4)A client's response to a health problem

ANS: 4 A nursing diagnosis is the client's response to actual or potential health problems. PTS:1DIF:ModerateREF: p. 31 KEY: Nursing process: Diagnosis | Client need: SECE | Cognitive level: Recall

A nursing instructor is guiding nursing students on best practices for interviewing patients. Which of the following comments by a student would indicate the need for further instruction? 1) "My patient is a young adult, so I plan to talk to her without her parents in the room." 2) "Because my patient is old enough to be my grandfather, I will call him 'Mr.'" 3) "When reading my patient's health record, I thought of a few questions to ask." 4) "When I give my patient his pain medication, I will have time to ask questions."

ANS: 4 A patient should be comfortable when interviewing. The pain medication should have time to work before considering interviewing the patient, so asking questions when giving the medication is not a good idea. It is appropriate to interview patients without family/friends around. In nearly every culture, calling a patient Mr. or Mrs. shows respect and is therefore correct. Reading the patient's health record is appropriate preparation for an interview. PTS:1DIF:ModerateREF:p. 46 KEY: Nursing process: Evaluation | Client need: SECE | Cognitive level: Application

An elderly man lost his wife a year ago to cardiovascular disease. During a healthcare visit, he tells the nurse he has begun adjusting to life without his wife. According to John Bowlby, which stage of grief does this comment most likely indicate? 1) Shock and numbness 2) Yearning and searching 3) Disorganization and despair 4) Reorganization

ANS: 4 According to Bowlby, a person adjusts to life without the deceased during the reorganization phase. During the shock and numbness phase, the person experiences disorientation and a feeling of helplessness. The person wants to be reconnected with the deceased during the yearning and searching phase. The person feels pain and the emotions of grief during the disorganization and despair phase. PTS:1DIF:ModerateREF:p. 359 KEY: Nursing process: Assessment | Client need: PSI | Cognitive level: Application

Of the following recommended interviewing techniques, which one is the most basic? (That is, without that intervention, the others will all be less effective.) 1)Beginning with neutral topics 2)Individualizing your approach 3)Minimizing note taking 4)Using active listening

ANS: 4 All are important techniques, but active listening focuses the attention on the patient and lets her know you are trying to understand her needs. The interviewer is more likely to get the patient to open up. Patients will forgive you for most errors in technique, but if they think you are not listening, that can negatively affect your relationship. PTS:1DIFgrinifficultREF:47

A client informs the nurse that he has quit smoking because his father died from lung cancer 3 months ago. Based on his motivation, smoking cessation should be recognized as an example of which of the following? 1) Healthy living 2) Health promotion 3) Wellness behaviors 4) Health protection

ANS: 4 Although health promotion and health protection may involve the same activities, their difference lies in the motivation for action. Health protection is motivated by a desire to avoid illness. Health promotion is motivated by the desire to increase wellness. Smoking cessation may also be a wellness behavior and may be considered a step toward healthy living; however, neither of these addresses motivation for action. PTS:1DIF:ModerateREF:p. 879 KEY: Nursing process: Assessment | Client need: HPM | Cognitive level: Comprehension

A patient hospitalized in a long-term rehabilitation facility is immobile and requires mechanical ventilation with a tracheostomy. She has a pressure area on her coccyx measuring 5 cm by 3 cm. The area is covered with 100% eschar. What would the nurse identify this as? 1) Stage II pressure ulcer 2) Stage III pressure ulcer 3) Stage IV pressure ulcer 4) Unstageable pressure ulcer

ANS: 4 An eschar is a black, leathery covering made up of necrotic tissue. An ulcer covered in eschar cannot be classified using a staging method because it is impossible to determine the depth. PTS: 1 DIF: Moderate REF: p. 1234 KEY: Nursing process: Assessment | Client need: PHSI | Cognitive level: Application

Which of the following describes the most important use of nursing diagnosis? (All statements are true.) 1) Differentiates the nurse's role from that of the physician 2) Identifies a body of knowledge unique to nursing 3) Helps nursing develop a more professional image 4) Describes the client's needs for nursing care

ANS: 4 Benefits to nurses and nursing are that nursing diagnoses differentiate the nurse's role, they identify a unique body of nursing knowledge, and some think they help nursing to develop a more professional image. However, the primary goal of nursing is to serve the good of the patient. Therefore, the most important use of a diagnosis is to specifically identify the client's needs for quality nursing care. PTS: 1 DIF: Moderate REF: p. 56 KEY: Nursing process: Diagnosis | Client need: Safe-care environment | Cognitive level: Analysis

A patient has underlying cardiac disease and requires careful monitoring of his fluid balance. He also has a draining wound. Which of the following methods for evaluating his wound drainage would be most appropriate for assessing fluid loss? 1) Draw a circle around the area of drainage on a dressing. 2) Classify drainage as less or more than the previous drainage. 3) Weigh the patient at the same time each day on the same scale. 4) Weigh dressings before they are applied and after they are removed.

ANS: 4 By weighing the dressing before it is applied and after it is removed, the nurse can accurately determine the amount of drainage. Weighing the patient daily would evaluate his overall fluid balance but is not sensitive to fluid loss through the wound. Marking a circle around the wound is useful for determining the extent of drainage seeping out of a wound, but it does not provide information how much fluid is draining. PTS:1DIF:ModerateREF:p. 1238 KEY: Nursing process: Evaluation | Client need: PHSI | Cognitive level: Synthesis

Which of the following blood levels normally provides the primary stimulus for breathing? 1) pH 2) Oxygen 3) Bicarbonate 4) Carbon dioxide

ANS: 4 Carbon dioxide (CO2) level provides the primary stimulus to breathe. High CO2 levels stimulate breathing to eliminate the excess CO2. A secondary, although important, drive to breathe is hypoxemia. Low blood O2 levels stimulate breathing to bring more oxygen into the lungs. PTS:1DIF:ModerateREF:p. 1296 KEY: Nursing process: Assessment | Client need: PHSI | Cognitive level: Recall

A patient who sustained a head injury in a motor vehicle accident has damage to the temporal lobe. This injury places the patient at risk for which type of hearing loss? 1) Otosclerosis 2) Conduction deafness 3) Presbycusis 4) Central deafness

ANS: 4 Central deafness results from damage to the auditory areas in the temporal lobes. Otosclerosis is hardening of the bones of the middle ear, especially the stapes. Conduction deafness results when one of the structures that transmits vibrations is affected. Presbycusis is a progressive sensorineural loss associated with aging. PTS:1DIF:ModerateREF:p. 1072 KEY: Nursing process: Diagnosis | Client need: PHSI | Cognitive level: Application

Which of the following describes the difference between a collaborative problem and a medical diagnosis? 1) A collaborative problem is treated by the nurse; a physician is responsible for the treatment of a medical problem. 2) A collaborative problem is a nursing diagnosis that requires specific orders from a physician; a medical diagnosis directs all nursing care. 3) A collaborative problem has the potential to become an actual nursing diagnosis; a medical diagnosis rarely changes. 4) A collaborative problem requires intervention by the nurse and physician or other professional; a medical diagnosis requires intervention by a physician.

ANS: 4 Collaborative problems are physiological complications a client may be at risk for due to her medical diagnosis, medical treatment, or diagnostic studies. A collaborative problem requires monitoring by the nurse and intervention by a physician. A medical diagnosis requires interventions (medications, treatments) by the physician. Medical diagnoses do not direct all nursing care. Collaborative problems have the potential to become medical, not nursing, diagnoses. PTS:1DIF:ModerateREF: pp. 58-59 KEY: Nursing process: Diagnosis | Client need: SECE | Cognitive level: Analysis

Which of the following outcome statements contains the best example of performance criteria? The patient will 1) Turn herself in bed frequently while awake 2) Understand how to use crutches by day 2 3) State that pain is decreased after being medicated 4) Eat 75% of each meal without complaint of nausea

ANS: 4 Performance criteria should be specific and measurable. "75% of each meal" is specific and measurable. "Frequently" is vague. You cannot observe whether someone "understands." "Decreased" is vague; a numerical pain rating would be better. PTS:1DIF:ModerateREF: p. 92 KEY: Nursing process: Planning | Client need: SECE | Cognitive level: Application

Which client has the greatest need for comprehensive discharge planning? 1) A woman who has just given birth to her second child and lives with her husband and 18-month-old daughter 2) A man who has been readmitted for exacerbation of his chronic obstructive pulmonary disease 3) A 12-year-old boy who had outpatient surgery on his knee and lives with his mother 4) A woman who was just diagnosed with renal failure and has started peritoneal dialysis

ANS: 4 Comprehensive discharge planning should be done for patients who have a newly diagnosed chronic disease or have complex needs. The other patients may require discharge planning but not as comprehensive as someone with a new diagnosis with complex treatment. PTS:1DIFgrinifficultREF: p. 83 KEY: Nursing process: Planning | Client need: SECE | Cognitive level: Analysis

Which of the following interventions would help to prevent or relieve persistent nausea? 1) Assess for signs of dehydration. 2) Provide dietary supplements. 3) Have the patient sit in an upright position for 30 minutes after eating. 4) Immediately remove any food that the patient cannot eat

ANS: 4 Dehydration can occur as a result of continued nausea and vomiting, so the nurse should assess for it. However, this intervention does not prevent nausea. Dietary supplements might help to prevent malnutrition. However, they do not prevent nausea; in fact, they often cause nausea. Having the patient sit upright helps to prevent respiratory aspiration should the patient vomit; it does not prevent or relieve nausea. Odors (even pleasant ones) and even the sight of food can cause nausea, so any uneaten food should be removed immediately from the room. PTS:1DIFgrinifficultREF:p. 928 KEY: Nursing process: Interventions | Client need: SECE | Cognitive level: Application

The nurse will know that the plan of care for the diabetic client with severe peripheral neuropathy is effective if the client: 1) begins an aggressive exercise program. 2) follows a diet plan of 1,200 calories per day. 3) is fitted for deep-depth diabetic footwear. 4) remains free of foot wounds.

ANS: 4 Diabetic clients experiencing difficulty with blood sugar control are prone to the development of peripheral neuropathy, which results in decreased sensation in the feet and lower extremities. Decreased sensation in the feet places the client at increased risk for development of wounds or pressure ulcers in the feet. The nurse will know his plan of care is effective when the client's feet remain free of wounds. An aggressive exercise program would not be appropriate for a client with severely diminished sensation in the feet. Similarly, a 1,200-calorie diet would be inadequate for most clients. Being fitted for diabetic footwear is an intervention rather than a goal. PTS:1DIFgrinifficultREF:p. 1232; higher-order item implied from text KEY: Nursing process: Evaluation | Client need: PHSI | Cognitive level: Synthesis

What emotional response is typical during the Rando's confrontation phase of the grieving process? 1) Anger and bargaining 2) Shock with disbelief 3) Denial 4) Emotional upset

ANS: 4 During the confrontation phase, the person faces the loss and experiences emotional upset. In the avoidance phase, the person experiences shock, disbelief, denial, anger, and bargaining. During the accommodation phase, the person begins to live with the loss, feel better, and resume routine activities. PTS:1DIF:ModerateREF:p. 359 KEY: Nursing process: Assessment | Client need: PSI | Cognitive level: Recall

17. A patient is admitted with high BUN and creatinine levels, low blood pH, and elevated serum potassium level. Based on these laboratory findings the nurse suspects which diagnosis? 1) Cystitis 2) Renal calculi 3) Enuresis 4) Renal failure

ANS: 4 Elevated BUN, creatinine, and serum potassium levels and low blood pH are signs of renal failure. Cystitis is an infection of the bladder and would not result in abnormal renal function. Renal calculi typically produce blood in the urine but do not lead to marked renal dysfunction and failure. Enuresis is involuntary urination, particularly common in children, and does not produce renal dysfunction. The cause of enuresis is often emotional, developmental, or trauma related.

A patient is admitted with high BUN and creatinine levels, low blood pH, and elevated serum potassium level. Based on these laboratory findings the nurse suspects which diagnosis? 1) Cystitis 2) Renal calculi 3) Enuresis 4) Renal failure

ANS: 4 Elevated BUN, creatinine, and serum potassium levels and low blood pH are signs of renal failure. Cystitis is an infection of the bladder and would not result in abnormal renal function. Renal calculi typically produce blood in the urine but do not lead to marked renal dysfunction and failure. Enuresis is involuntary urination, particularly common in children, and does not produce renal dysfunction. The cause of enuresis is often emotional, developmental, or trauma related. PTS:1DIF:DifficultREF:p. 1023 KEY: Nursing process: Diagnosis | Client need: PSI | Cognitive level: Analysis

The nurse working in the emergency department is preparing heat therapy for one of the patients in the unit. Which one is it most likely to be? Choose all that apply. 1) Is actively bleeding 2) Has swollen, tender insect bite 3) Has just sprained her ankle 4) Has lower back pain

ANS: 4 Heat therapy is used to relieve stiffness and discomfort commonly associated with musculoskeletal soreness. Heat causes dilation of the blood vessels and improves delivery of oxygen and nutrients to the tissues. It promotes relaxation and is used to aid in the healing process. Applying heat promotes vasodilation and reduces blood thickness (viscosity) and leaky capillaries, all of which would be harmful to the patient who is actively bleeding. It can lead to a drop in blood pressure. Heat should not be applied to a site with inflammation (insect bite or acute joint injury with swelling) because it can increase edema to the site. A good application for heat therapy is to promote comfort and relaxation to the patient experiencing back pain. PTS:1DIF:ModerateREF:p. 1254 KEY: Nursing process: Interventions | Client need: PHSI | Cognitive level: Application

Why is it important to obtain information about nutritional and herbal supplements as well as about complementary and alternative therapies? 1) To determine what type of therapies are acceptable to the client 2) To identify whether the client has a nutrition deficiency 3) To help you to understand cultural and spiritual beliefs 4) To identify potential interaction with prescribed medication and therapies

ANS: 4 Herbs and nutritional supplements can interact with prescription medications, and complementary and alternative treatments can interfere with conventional therapies. Physical assessment and laboratory tests are needed to assess a nutritional deficiency. To identify cultural and spiritual beliefs and well as what therapies are acceptable to the client, you need more than just information about nutritional and herbal supplements. PTS:1DIFgrinifficultREF: p. 45

Where and when can you palpate a bladder?

In the suprapubic region only when it is full or distended

Which patient teaching would be most therapeutic for someone with sleep disturbance? 1)Give yourself at least 60 minutes to fall asleep. 2)Avoid eating carbohydrates before going to sleep. 3)Catch up on sleep by napping or sleeping in when possible. 4)Do not go to bed feeling upset about a conflict.

ANS: 4 Intense emotion before bedtime can interfere with rest and sleep. Lying awake longer than 30 minutes is counterproductive. Eating a small amount of a complex carbohydrate can aid in falling asleep. Avoid simple sugars because sucrose can lead to a short-term energy boost instead of relaxation. Taking naps during the day and sleeping late on some mornings can actually exacerbate a sleep disturbance. It's better to establish a consistent routine for wake and sleep. The extra sleep during the day can interfere with the body's readiness for sleep at night. PTS:1DIF:EasyREF:pp. 1207-1208

The nurse administering pain medication every 4 hours is an example of which aspect of patient care? 1)Assessment data 2)Nursing diagnosis 3)Patient outcome 4)Nursing intervention

ANS: 4 Interventions are activities that will help the patient achieve a goal, such as administering pain-relieving medication. An example of assessment data might be, "Patient reports pain is a 5 on a 1 to 10 scale." The nursing diagnosis would be Pain. The nurse might define the patient outcome in this scenario as, "The patient will state the level of pain is less than 4." PTS:1DIF:ModerateREF:p. 31; high-level question, answer not stated verbatim KEY: Nursing process: Interventions | Client need: SECE | Cognitive level: Application

A patient has anemia. An appropriate goal for that the patient would be for him to increase his intake of which nutrient? 1) Calcium 2) Magnesium 3) Potassium 4) Iron

ANS: 4 Iron deficiency causes anemia; therefore, the nurse should encourage the patient with anemia to increase his intake of iron. Increasing calcium intake helps prevent osteoporosis. Magnesium supplementation may decrease the risk of hypertension and coronary artery disease in women. Potassium is essential for muscle contraction, acid-base balance, and blood pressure control. PTS:1DIF:EasyREF:p. 907 KEY: Nursing process: Planning | Client need: PHSI | Cognitive level: Application

After instructing a mother about nutrition for a preschool-age child, which statement by the mother would indicate correct understanding of the topic? 1) "I usually use dessert only as a reward for eating other foods." 2) "I will hide vegetables in casseroles and stews to get my child to eat them." 3) "I do not give my child snacks; they simply spoil his appetite for meals." 4) "I know that lifelong food habits are developed during this stage of life."

ANS: 4 Lifelong food habits are developed during the preschool stage of life. Therefore, the mother should widen the variety of foods she introduces to her child. Desserts should not be used as rewards for eating other foods. This practice can shape an attitude about food that can lead to eating disorders later in life. Preschool-age children often refuse combined foods such as casseroles and stews. Because they are active, preschoolers require nutritious between-meal snacks. PTS: 1 DIF: Moderate REF: p. 913 KEY: Nursing process: Evaluation | Client need: HPM | Cognitive level: Application

During a health history, a patient whose wife died unexpectedly 6 months ago in a motor vehicle accident admits that he drinks at least six bourbon and waters every night before going to bed. Which type of grief does this best illustrate? 1) Delayed 2) Chronic 3) Disenfranchised 4) Masked

ANS: 4 Masked grief occurs when the person is grieving, but it may look as though something else is occurring; in this case, the person is abusing alcohol. Delayed grief occurs when grief is put off until a later time. Chronic grief begins as normal grief but continues long term with little resolution of feelings or ability to rejoin normal life. Disenfranchised grief is experienced when a loss is not socially supported. PTS: 1 DIF: Moderate REF: p. 361 KEY: Nursing process: Diagnosis | Client need: PSI | Cognitive level: Analysis

A patient reports that he uses music therapy to help control his chronic pain. Music therapy works by prompting the release of endogenous opioids during which stage of the pain process? 1) Perception 2) Transduction 3) Transmission 4) Modulation

ANS: 4 Music therapy can prompt the release of endogenous opioids during the modulation stage, which is the stage of the pain process where the perception of pain changes. It is not during the perception (recognizing the pain sensation), transmission (relaying the pain message), or transduction (converting potentially damaging stimuli into electrical activity leading to pain sensation). PTS:1DIF:ModerateREF:pp. 1093, 1100; synthesis of information required KEY:Nursing process: Planning | Client need: PHSI | Cognitive level: Recall

3. Which urine specific gravity would be expected in a patient admitted with dehydration? 1) 1.002 2) 1.010 3) 1.025 4) 1.030

ANS: 4 Normal urine specific gravity ranges from 1.010 to 1.025. Specific gravity less than 1.010 indicates fluid volume excess, such as when the patient has fluid overload (too much IV fluid) or when the kidneys fail to concentrate urine. Specific gravity greater than 1.025 is a sign of deficient fluid volume that occurs, for example, as a result of blood loss or dehydration.

Which urine specific gravity would be expected in a patient admitted with dehydration? 1) 1.002 2) 1.010 3) 1.025 4) 1.030

ANS: 4 Normal urine specific gravity ranges from 1.010 to 1.025. Specific gravity less than 1.010 indicates fluid volume excess, such as when the patient has fluid overload (too much IV fluid) or when the kidneys fail to concentrate urine. Specific gravity greater than 1.025 is a sign of deficient fluid volume that occurs, for example, as a result of blood loss or dehydration. PTS:1DIF:ModerateREF:p. 1015 KEY: Nursing process: Assessment | Client need: PSI | Cognitive level: Application

The nurse administers intravenous morphine sulfate to a patient for pain control. She will need to monitor her patient for which of the following adverse effects? 1) Decreased heart rate 2) Muscle weakness 3) Decreased urine output 4) Respiratory depression

ANS: 4 Opioids are potent respiratory depressants. Patients receiving opioids should be monitored for decreased rate and depth of respirations. PTS:1DIF:ModerateREF:pp. 1299, 1305; critical-thinking and synthesis required KEY: Nursing process: Evaluation | Client need: PHSI | Cognitive level: Comprehension

A patient complains to the nurse that since taking a medication he has suffered from excessively dry mouth. What term should the nurse use to document this complaint? 1) Exophthalmos 2) Anosmia 3) Insomnia 4) Xerostomia

ANS: 4 The nurse should document excessively dry mouth as xerostomia. Exophthalmos is abnormal bulging of the eyeballs that commonly occurs with thyrotoxicosis. Anosmia is losing the sense of smell. Insomnia is inability to sleep. PTS:1DIF:ModerateREF:p. 1072 KEY: Nursing process: Interventions | Client need: PHSI | Cognitive level: Comprehension

A patient had a CVA (stroke) 2 days ago, resulting in decreased mobility to her left side. During the assessment, the nurse discovers a stage I pressure area on the patient's left heel. What is the initial treatment for this pressure ulcer? 1) Antibiotic therapy for 2 weeks 2) Normal saline irrigation of the ulcer daily 3) Débridement to the left heel 4) Elevation of the left heel off the bed

ANS: 4 Pressure ulcers are caused by pressure to an area that restricts blood flow, causing ischemia to underlying tissue. The primary treatment is to relieve the pressure, thus improving blood flow. Elevating the patient's left heel off the bed would relieve pressure to this area. PTS:1DIF:ModerateREF:p. 1231 KEY: Nursing process: Interventions | Client need: PHSI | Cognitive level: Application

While a patient is receiving hygiene care, her chest tube becomes disconnected from the water-seal chest drainage system (CDU). Which action should the nurse take immediately? 1) Clamp the chest tube close to the insertion site. 2) Set up a new drainage system, and connect it to the chest tube. 3) Have the patient take and hold a deep breath while the nurse reconnects the tube to the CDU. 4) Place the disconnected end nearest the patient into a bottle of sterile water.

ANS: 4 Recollapse of the lung can occur because of loss of negative pressure within the system. This is commonly caused by air leaks, disconnections, or cracks in the bottles or chambers. If any of these occur, the nurse should immediately place the disconnected end nearest the patient into a bottle of sterile water or saline to a depth of 2 cm to serve as an emergency water seal until a new system can be connected. Do not clamp the chest tube because this can rapidly lead to a tension pneumothorax. A new drainage system should be set up to decrease the risk of infection, but the immediate action is to place the disconnected end into a bottle of sterile water. PTS:1DIF:ModerateREF:p. 1322 KEY: Nursing process: Interventions | Client need: PHSI | Cognitive level: Analysis

A client developed a stage IV pressure ulcer to his sacrum 6 weeks ago, and now the ulcer appears to be a shallow crater involving only partial skin loss. What would the nurse now classify the pressure ulcer as? 1) Stage I pressure ulcer, healing 2) Stage II pressure ulcer, healing 3) Stage III pressure ulcer, healing 4) Stage IV pressure ulcer, healing

ANS: 4 Reverse staging is not done because as the ulcer heals with granulation tissue and becomes shallower, the lost muscle, subcutaneous fat, and dermis are not replaced. Pressure ulcers maintain their original staging classification throughout the healing process but are accompanied by the modifier healing. PTS: 1 DIF: Moderate REF: p. 1232 KEY: Nursing process: Assessment | Client need: PHSI | Cognitive level: Analysis

A 6-year-old boy is admitted to the hospital for a surgical procedure associated with a hospital stay. When the nurse asks his mother about the boy's sleep patterns, she says, "Sometimes he will get out of bed, walk into the kitchen, and get the cereal out of the cabinet. Then he just turns around and goes back to bed." The nurse explains that he is sleepwalking. The best nursing diagnosis for Tad would be: 1)Risk for Insomnia related to sleepwalking. 2)Risk for Fatigue related to sleepwalking. 3)Disturbed Sleep Pattern related to dyssomnia. 4Risk for Injury related to sleepwalking.

ANS: 4 Sleepwalking occurs during stages III and IV of NREM sleep. The sleeper leaves the bed and walks about with little awareness of surroundings. He may perform what appear to be conscious motor activities but does not wake up and has no memory of the event on awakening. The boy is at high risk for injury when sleepwalking because of his lack of awareness of his surroundings. Insomnia is a medical diagnosis rather than a nursing diagnosis. Certainly his sleep pattern is disturbed; however, there is little in the way of independent actions that the nurse could take for either the problem or etiology of this diagnosis, so it would not be useful. The boy does not awaken while sleepwalking and is not likely to experience fatigue from the event. PTS:1DIF:ModerateREF:p. 1213

Which of the following is a benefit of standardized care plans, as defined in your text? Standardized care plans 1) Apply to every patient on a particular unit 2) Include both medical and nursing orders 3) Specify patient outcomes for each day 4) Help ensure that important interventions are not overlooked

ANS: 4 Standardized care plans help promote consistency of care and ensure that important interventions are not forgotten. They are not likely to apply to every patient on a unit because they are usually single-problem plans or are used with a particular medical diagnosis. Unlike protocols, they do not include medical orders. Unlike critical pathways, they do not specify predicted patient outcomes for each day. PTS:1DIF:ModerateREF: p. 86 KEY: Nursing process: Planning | Client need: SECE | Cognitive level: Recall

10. A patient complains that she passes urine whenever she sneezes or coughs. How should the nurse document this complaint in the patients healthcare record? 1) Transient incontinence 2) Overflow incontinence 3) Urge incontinence 4) Stress incontinence

ANS: 4 Stress incontinence is an involuntary loss of urine that occurs with increased intra-abdominal pressure. Activities that typically produce the symptom include sneezing, coughing, laughing, lifting, and exercise. Transient incontinence is a short-term incontinence that is expected to resolve spontaneously. It is typically caused by urinary tract infection or medications, such as diuretics. Overflow incontinence is the loss of urine when the bladder becomes distended; it is commonly associated with fecal impaction, enlarged prostate, and neurological conditions. Urge incontinence is the involuntary loss of urine associated with a strong urge to void.

A patient complains that she passes urine whenever she sneezes or coughs. How should the nurse document this complaint in the patient's healthcare record? 1) Transient incontinence 2) Overflow incontinence 3) Urge incontinence 4) Stress incontinence

ANS: 4 Stress incontinence is an involuntary loss of urine that occurs with increased intra-abdominal pressure. Activities that typically produce the symptom include sneezing, coughing, laughing, lifting, and exercise. Transient incontinence is a short-term incontinence that is expected to resolve spontaneously. It is typically caused by urinary tract infection or medications, such as diuretics. Overflow incontinence is the loss of urine when the bladder becomes distended; it is commonly associated with fecal impaction, enlarged prostate, and neurological conditions. Urge incontinence is the involuntary loss of urine associated with a strong urge to void. PTS:1DIF:ModerateREF:p. 1018 KEY: Nursing process: Diagnosis | Client need: PSI | Cognitive level: Analysis

Functions of the kidneys:

the kidneys filter metabolic wastes, toxins, excess ions, and water from the blood stream and excrete them as urine. They also regulate blood volume, blood pressure, electrolyte levels, and acid-base balance by selectively absorbing water and other substances.

A patient with Raynaud's disease receives no symptomatic relief with diltiazem (Cardizem). Which surgical intervention might be a treatment option for this patient to help provide symptomatic relief? 1) Cordotomy 2) Rhizotomy 3) Neurectomy 4) Sympathectomy

ANS: 4 Sympathectomy severs the pathways to the sympathetic nervous system. The procedure improves vascular blood supply and eliminates vasospasm. It is effective for treatment of pain associated with vascular disorders, such as Raynaud's disease. Cordotomy interrupts pain and temperature sensation below the tract that is severed. This procedure is commonly performed to relieve trunk and leg pain. Rhizotomy interrupts the anterior or posterior nerve route located between the ganglion and the cord. It is commonly used to treat head and neck pain. Neurectomy is used to eliminate intractable localized pain. The pathways of peripheral or cranial nerves are interrupted to block pain transmission. PTS:1DIF:ModerateREF:p. 1108 KEY: Nursing process: Planning | Client need: PHSI | Cognitive level: Recall

A patient has a contaminated right hip wound that requires dressing changes twice daily. The surgeon informs the nurse that when the wound "heals a little more" he will suture it closed. The nurse recognizes that the surgeon is using which form of wound healing? 1) Primary intention 2) Regenerative healing 3) Secondary intention 4) Tertiary intention

ANS: 4 Tertiary intention is a technique used when a wound is clean contaminated or "dirty" (potentially infected). Initially, the wound is allowed to heal by secondary intention, and when there is no evidence of edema, infection, or foreign matter, granulating tissue is brought together and the wound edges are sutured closed. PTS:1DIF:ModerateREF:p. 1227 KEY: Nursing process: Assessment | Client need: PHSI | Cognitive level: Comprehension

What is the role of the Joint Commission in regard to patient assessment? The Joint Commission 1)States what assessments are collected by individuals with different credentials 2)Regulates the time frames for when assessments should be completed 3)Identifies how data are to be collected and documented 4)Sets standards for what and when to assess the patients

ANS: 4 The Joint Commission sets detailed standards regarding what and when to assess but does not address credentials. Nurse practice acts specify what data are collected and by whom. Agency policy may set time frames for when assessments should be done and how they should be documented. Nursing knowledge identifies "how" data are to be collected. PTS:1DIF:ModerateREF:p. 39

You are admitting a 54-year-old patient with chronic obstructive pulmonary disease (COPD). The physician prescribes O2 at 24% FIO2. What is the most appropriate oxygen delivery method for this patient? 1) Nonrebreather mask 2) Nasal cannula 3) Partial rebreather mask 4) Venturi mask

ANS: 4 The Venturi mask is capable of delivering 24% to 50% FIO2. The cone-shaped adapter at the base of the mask allows a precise FIO2 to be delivered. This is very useful for patients with chronic lung disease. Rebreather masks are used when high concentrations of oxygen are required. A nasal cannula administers oxygen in liters per minute and does not allow administration of a precise FIO2. PTS:1DIFgrinifficultREF:p. 1335 KEY: Nursing process: Interventions | Client need: PHSI | Cognitive level: Application

Which organ relies almost exclusively on glucose for energy? 1) Liver 2) Heart 3) Pancreas 4) Brain

ANS: 4 The brain relies almost exclusively on glucose for energy. The heart and liver do not. The pancreas produces insulin for glucose utilization but does not use glucose. PTS:1DIF:Easy REF: p. 902; ESG, Chapter 28, Supplemental Materials, Dietary Reference Intake: Macronutrients KEY:Nursing process: N/A | Client need: HPM | Cognitive level: Recall

Which is a major factor regulating sleep? 1)Electrical impulses transmitted to the cerebellum 2)Level of sympathetic nervous system stimulation 3)Amount of sleep a person has become accustomed to 4)Amount of light received through the eyes

ANS: 4 The circadian rhythm is a biorhythm based on the day-night pattern in a 24-hour cycle. A person's circadian rhythm is regulated by a cluster of cells in the hypothalamus of the brainstem that respond to changing levels of light. A major factor in regulating sleep is the amount of light received through the eyes—not the typical amount of sleep the person has within a 24-hour period. The autonomic nervous system (rather than central nervous system) controls the involuntary processes of the body, such as sleep, digestion, immune function, and so on. PTS:1DIF:ModerateREF:p. 1204

When making a diagnosis using NANDA-I, which of the following provides support for the diagnostic label you choose? 1) Etiology 2) Related factors 3) Diagnostic label 4) Defining characteristics

ANS: 4 The defining characteristics are the signs and symptoms that must be present to support any given nursing diagnosis. The etiology and related factors are the causes or contributing factors to the problem. The diagnostic label is the name NANDA-I has given the problem; it is chosen based on the presence of defining characteristics. PTS:1DIF:EasyREF:p. 68 KEY: Nursing process: Diagnosis | Client need: SECE | Cognitive level: Recall

What is the primary difference between acute and chronic wounds? Chronic wounds: 1) Are full-thickness wounds, but acute wounds are superficial. 2) Result from pressure, but acute wounds result from surgery. 3) Are usually infected, whereas acute wounds are contaminated. 4) Exceed the typical healing time, but acute wounds heal readily.

ANS: 4 The length of time for healing is the determining factor when classifying a wound as acute or chronic. Acute wounds are expected to be of short duration. Wounds that exceed the anticipated length of recovery are classified as chronic wounds. PTS:1DIF:EasyREF:p. 1225 KEY: Nursing process: Assessment | Client need: PHSI | Cognitive level: Analysis

Which aspect of restraint use can the nurse delegate to the nursing assistive personnel? 1) Assessing the patient's status 2) Determining the need for restraint 3) Evaluating the patient's response to restraints 4) Applying and removing the restraints

ANS: 4 The nurse can delegate applying and removing the restraints, skin care, and checking for skin breakdown. The nurse responsible for care of the patient must assess the patient's need for restraint and the patient's status and must evaluate the patient's response to restraints. PTS: 1 DIF: Moderate REF: p. 675 KEY: Nursing process: Interventions | Client need: SECE | Cognitive level: Recall

A 15-year-old male patient is experiencing nocturnal emissions. What nursing intervention would be appropriate for this patient? A) Ask the parents to consult with a specialist. B) Tell the patient to limit physical activity in the evening. C) Ask the primary care provider to perform a physical examination. D) No intervention is necessary as this is a normal phenomenon.

D) No intervention is necessary as this is a normal phenomenon.

Which of the following instructions is most important for the nurse to include when teaching a mother of a 3-year-old about protecting her child against accidental poisoning? 1) Store medications on countertops out of the child's reach. 2) Purchase medication in child-resistant containers 3) Take medications in front of the child, and explain that they are for adults only. 4) Never leave the child unattended around medications or cleaning solutions.

ANS: 4 The nurse should instruct the mother to avoid leaving her child unattended around medications or cleaners even for a moment. Medications should never be stored on kitchen counters or bathroom surfaces because children love to explore and climb and can get into them. The nurse should explain that medications should not be taken in front of the child because children imitate adult behavior. The nurse should reinforce that although child-resistant containers are a deterrent, they are not foolproof because many toddlers and preschoolers can open them. PTS:1DIF:ModerateREF:p. 665 KEY: Nursing process: Interventions | Client need: SECE | Cognitive level: Application

Which instruction should the nurse be certain to include when providing discharge teaching for a patient who has a serious visual deficit? 1) Install blinking lights to alert an incoming phone call. 2) Have gas appliances inspected regularly to detect gas leaks. 3) Wear properly fitting shoes and socks. 4) Avoid using throw rugs on the floors.

ANS: 4 The nurse should instruct the visually impaired patient to avoid using throw rugs on the floors at home. She should instruct the patient with a hearing deficit to install blinking lights to alert him to an incoming phone call. She should instruct the patient with an olfactory deficit to have gas appliances inspected regularly to detect leaks. The patient with a tactile deficit should be instructed to use properly fitting shoes and socks. PTS:1DIF:ModerateREF:p. 1084 KEY: Nursing process: Planning | Client need: PHSI | Cognitive level: Application

When should the nurse make systematic observations about a patient? 1)When the patient has specific complaints 2)With the first assessment of the shift 3)Each time the nurse gives medications to the patient 4)Each time the nurse interacts with the patient

ANS: 4 The nurse should make observations about the patient each time she enters the room or interacts with the patient to gain ongoing data about the patient. PTS:1DIF:EasyREF:p. 41-42 KEY: Nursing process: Assessment | Client need: SECE | Cognitive level: Application

After receiving ibuprofen (Motrin) 800 mg orally for right hip pain, the patient states that his pain is 8 out of 10 on the numerical pain scale. Which action should the nurse take? 1) Use nonpharmacological therapy while waiting 3 more hours before the next dose. 2) Administer an additional 800 mg oral dose of ibuprofen right away. 3) Do nothing because the patient's facial expression indicates he is comfortable. 4) Notify the prescriber that the current pain management plan is ineffective.

ANS: 4 The nurse should notify the prescriber that the current pain management plan is ineffective. The nurse should not delay treatment for 3 hours when the next dose of medication is due. The nurse cannot administer an extra dose of ibuprofen without a prescriber's order to do so. Ibuprofen 800 mg is a maximum dose for most individuals. The nurse should not assume that the patient is not in pain simply because he appears comfortable; pain is what the patient states it is. PTS:1DIF:ModerateREF:pp. 1110-1111 KEY: Nursing process: Interventions | Client need: SECE | Cognitive level: Application

12. Which intervention should the nurse take first to promote micturition in a patient who is having difficulty voiding? 1) Insert an indwelling urinary catheter. 2) Notify the provider immediately. 3) Insert an intermittent, straight catheter. 4) Pour warm water over the patients perineum.

ANS: 4 The nurse should perform independent nursing measures, such as pouring warm water over the patients perineum before notifying the provider. If nursing measures fail, the nurse should notify the provider. The provider may order an indwelling urinary catheter or a straight catheter to relieve the patients urinary retention.

Which intervention is helpful when caring for a patient with impaired vision? 1) Suggest the patient use bright overhead lighting. 2) Advise the patient to avoid wearing sunglasses when outdoors. 3) Do not offer large-print books, as this may embarrass the patient. 4) Place the patient's eyeglasses within easy reach.

ANS: 4 The nurse should place the patient's eyeglasses within easy reach and make sure that they are clean and in good repair. The patient should have sufficient light but avoid bright light, which might cause glare. The patient should be encouraged to wear sunglasses, visors, or hats with brims when outdoors. A magnifying lens or large-print books may be helpful. PTS:1DIF:ModerateREF:p. 1082 KEY: Nursing process: Interventions | Client need: PHSI | Cognitive level: Application

Despite less-restrictive interventions, a patient's behavior escalates, requiring emergency application of restraints. Which of the following must the nurse do in this situation? 1) Obtain a physician's order before applying restraints. 2) Monitor the patient's status every 4 hours while restrained. 3) Release the restraints and check circulation every hour. 4) Continually reevaluate the patient's need for restraint.

ANS: 4 The patient must be continually monitored, and the need for restraint must be continually reevaluated. As a rule, a medical order should be obtained before applying restraints. However, in an emergency, the nurse is permitted to apply restraints for behavior management, but a physician or advanced practice nurse must then evaluate the patient within 1 hour of restraint application. The order for restraint must be renewed daily. The restraints must be released at least every 2 hours, and circulation must be checked. PTS: 1 DIF: Difficult REF: pp. 679-681 KEY: Nursing process: Interventions | Client need: SECE | Cognitive level: Application

The expected outcome (goal) for a patient with Disturbed Sleep Pattern is that she will: 1)Limit exercise to 1 hour per day early in the day. 2)Consume only one caffeinated beverage per day. 3)Demonstrate effective guided imagery to aid relaxation. 4)Verbalize that she is sleeping better and feels less fatigued.

ANS: 4 The patient would verbalize that she is sleeping better and feels less fatigued. The expected outcome (goal) is based on the nursing diagnosis, and its achievement should reflect resolution of the problem. The other options are outcomes that demonstrate only that the patient took certain actions. They would not, if achieved, demonstrate that the problem of Disturbed Sleep Pattern had been resolved. PTS:1DIF:ModerateREF:p. 1213

An adolescent male tells the nurse that he is afraid his penis will be damaged because he masturbates every day. The nurses response is based on what knowledge? A) Masturbation is not a normal activity. B) Only adult men masturbate. C) Masturbation may delay puberty. D) Self-stimulation is a normal activity.

D) Self-stimulation is a normal activity.

Which nursing intervention should be included in the plan of care for a patient dying of cancer? 1) Encourage at least one family member to remain at the bedside at all times. 2) Follow-up with other healthcare team members during weekly meetings. 3) Avoid discussing the dying process with family (to reduce sadness). 4) Encourage family members to participate in care of the patient when possible

ANS: 4 The plan of care should include encouraging family members to help with the patient's care when they are able. Family members should also be encouraged to take care of themselves. They often need to be encouraged to take breaks to eat and rest. Provide them with anticipatory guidance about the stages of death so they know what to expect. Follow up promptly (not weekly) with other healthcare team members to address family concerns. PTS: 1 DIF: Moderate REF: pp. 369-371 KEY: Nursing process: Interventions | Client need: PSI | Cognitive level: Application

You are caring for a young adult patient with an intracranial hemorrhage secondary to a closed head injury. During your assessment, you notice that the patient's respirations follow a cycle progressively increasing in depth, then progressively decreasing in depth, followed by a period of apnea. Which of the following appropriately describes this respiratory pattern? 1) Biot's breathing 2) Kussmaul's respirations 3) Sleep apnea 4) Cheyne-Stokes respirations

ANS: 4 This respiratory pattern is known as Cheyne-Stokes respirations. It is often associated with damage to the medullary respiratory center or high intracranial pressure due to brain injury. PTS:1DIF:EasyREF:p. 1303 KEY: Nursing process: Assessment | Client need: PHSI | Cognitive level: Comprehension

Your patient has multiple open wounds that require treatment. When performing dressing changes, you should: 1) Remove all of the soiled dressings before beginning wound treatment. 2) Cleanse wounds from most contaminated to least contaminated. 3) Treat wounds on the patient's side first, then the front and back of the patient. 4) Irrigate wounds from least contaminated to most contaminated.

ANS: 4 To avoid the possibility of cross-contamination, the wound with the least amount of contamination should be treated first, progressing to the wound with the most contamination. PTS:1DIF:ModerateREF:p. 1249 KEY: Nursing process: Interventions | Client need: SECE | Cognitive level: Synthesis

Which of the following is an example of data that should be validated? 1)The client's weight measures 185 lb at the clinic. 2)The client's liver function test results are elevated. 3)The client's blood pressure is 160/94 mm Hg; he states that that is typical for him. 4)The client states she eats a low-sodium diet and reports eating processed food.

ANS: 4 Validation should be done when the client's statements are inconsistent (processed foods are generally high in sodium). Validation is not necessary for laboratory data when you suspect an error has been made in the results. Personal information that patients might be embarrassed about, such as weight, is best validated with a scale. PTS:1DIF:ModerateREF: p. 47 KEY: Nursing process: Assessment | Client need: PHSI | Cognitive level: Application

A nursing instructor asked his nursing students to discuss their experiences with charting assessment data. Which comment by the student indicates the need for further teaching? 1) "I find it difficult to avoid using phrases like, 'The patient tolerated the procedure well.'" 2) "It's confusing to have to remember which abbreviations this hospital allows." 3) "I need to work on charting assessments and interventions right after they are done." 4) "My patient was really quiet and didn't say much, so I charted that he acted depressed."

ANS: 4 When charting data, chart only what was observed, not what it meant. Inferences should not be made about a patient's behavior during data collection ("he acted depressed"); so that response reflects the student's lack of knowledge and need for teaching. Chart specific data, not vague phrases; the student is acknowledging the importance of this. There are no universally accepted phrases, just agency-approved abbreviations; the student is acknowledging the need to use agency-approved abbreviations. The student is correct that charting should be completed as soon after data collection as possible. PTS:1DIF:ModerateREF:p. 50 KEY: Nursing process: Evaluation | Client need: SECE | Cognitive level: Application

The nurse notes that the electrical cord on an IV infusion pump is cracked. Which action by the nurse is best? 1) Continue to monitor the pump to see if the crack worsens. 2) Place the pump back on the utility room shelf. 3) A small crack poses no danger so continue using the pump. 4) Clearly label the pump and send it for repair.

ANS: 4 Whenever an electrical safety hazard is suspected or visible, the nurse should label the malfunctioning equipment and send it for repair or inspection. Continuing to use the IV infusion pump or any other equipment places the patient at risk for injury. Placing the pump back on the shelf places other healthcare team members at risk for electrical injury if they attempt to use the equipment. PTS:1DIF:EasyREF:p. 673 KEY: Nursing process: Interventions | Client need: SECE | Cognitive level: Recall

The client's weight is appropriate for his height. His laboratory values and other assessments reflect normal nutritional status. However, he has told the nurse, "I probably eat a little too much red meat. And what is this I hear about needing omega 3 oils in my diet? I don't like to take supplements, and I think I could really improve my nutrition." Which of the following nursing diagnoses should the nurse use? 1) Balanced Nutrition 2) Possible Imbalanced Nutrition: Less Than Body Requirements 3) Risk for Imbalanced Nutrition: Less Than Body Requirements 4) Readiness for Enhanced Nutrition

ANS: 4 You will use a wellness diagnosis when a person's present level of wellness is effective and when the person wants to move to a higher level of wellness—in this case, a higher level of nutrition. The format for a wellness diagnosis is "Readiness for Enhanced . . ." Use a possible diagnosis when you have enough data to suspect a problem but need more data to support a diagnosis. Use a risk diagnosis when there are risk factors for a problem. PTS: 1 DIF: Moderate REF: p. 72 KEY: Nursing process: Diagnosis | Client need: SECE | Cognitive level: Application

An OR nurse is participating in an interdisciplinary audit of infection control practices in the surgical department. The nurse should know that a basic guideline for maintaining surgical asepsis is what? A) Sterile surfaces or articles may touch other sterile surfaces. B) Sterile supplies can be used on another patient if the packages are intact. C) The outer lip of a sterile solution is considered sterile. D) The scrub nurse may pour a sterile solution from a nonsterile bottle.

Ans: A Feedback: Basic guidelines for maintaining sterile technique include that sterile surfaces or articles may touch other sterile surfaces only. The other options each constitute a break in sterile technique.

Match the assessment model with the intended use for that model. 1) Categorizes nursing diagnoses, client outcomes, and nursing interventions 2) Assesses the client's ability to achieve balance (homeostasis) 3) Identifies deficits in activities of daily living that require nursing assistance 4) Formulates a model for nursing assessment and diagnosis but is not a theory 5) Categorizes nursing diagnoses ____ 12. Gordon's Functional Health Patterns ____ 13. NANDA Nursing Diagnosis Taxonomy II ____14.Taxonomy of Nursing Practice ____ 15. Roy's Adaptation model ____ 16. Orem's Self-Care model

ANS: 4 PTS: 1 DIF: Moderate REF: p. 49 KEY: Nursing process: Assessment | Client need: SECE | Cognitive level: Recall 13. ANS: 5 PTS: 1 DIF: Moderate REF: p. 49 KEY: Nursing process: Assessment | Client need: SECE | Cognitive level: Recall 14. ANS: 1 PTS: 1 DIF: Moderate REF: p. 49 KEY: Nursing process: Assessment | Client need: SECE | Cognitive level: Recall 15. ANS: 2 PTS: 1 DIF: Moderate REF: p. 49 KEY: Nursing process: Assessment | Client need: SECE | Cognitive level: Recall 16. ANS: 3 PTS: 1 DIF: Moderate REF: p. 49 KEY: Nursing process: Assessment | Client need: SECE | Cognitive level: Recall

Using Maslow's hierarchy of needs, rank the following nursing diagnoses in order of importance, beginning with the highest-priority diagnosis. 1) Anxiety 2) Risk for infection 3) Disturbed body image 4) Sleep deprivation

ANS: 4, 2, 1, 3 In Maslow's hierarchy, physiologic needs and safety are the highest priority. Sleep is a basic physiologic need. Infection can threaten physical health. In this question, infection is not present; therefore, there is just a risk for it. Sleep Deprivation is an immediate problem that affects general physical, mental, and emotional health. Neither Anxiety nor Disturbed Body image is a physiologic or safety need. Anxiety is a more immediate need than Disturbed Body Image, so it probably deserves a higher ranking. Remind students that the ranking would depend on the severity of each problem, which is not known by the labels alone. PTS:1DIFgrinifficultREF:pp. 64 KEY: Nursing process: Diagnosis | Client need: SECE | Cognitive level: Analysis

A clinic nurse is teaching a client prior to surgery. The client does not seem to comprehend the teaching, forgets a lot of what is said, and asks the same questions again and again. What action by the nurse is best? a. Assess the client for anxiety. b. Break the information into smaller bits. c. Give the client written information. d. Review the information again.

ANS: A Anxiety can interfere with learning and cooperation. The nurse should assess the client for anxiety. The other actions are appropriate too, and can be included in the teaching plan, but effective teaching cannot occur if the client is highly anxious. DIF: Applying/Application REF: 233 KEY: Preoperative nursing| anxiety| client education MSC: Integrated Process: Nursing Process: Assessment NOT: Client Needs Category: Psychosocial Integrity

An older client is hospitalized after an operation. When assessing the client for postoperative infection, the nurse places priority on which assessment? a. Change in behavior b. Daily white blood cell count c. Presence of fever and chills d. Tolerance of increasing activity

ANS: A Older people have an age-related decrease in immune system functioning and may not show classic signs of infection such as increased white blood cell count, fever and chills, or obvious localized signs of infection. A change in behavior often signals an infection or onset of other illness in the older client. DIF: Applying/Application REF: 221 KEY: Preoperative nursing| infection| older adult MSC: Integrated Process: Nursing Process: Assessment NOT: Client Needs Category: Health Promotion and Maintenance

A student is caring for clients in the preoperative area. The nurse contacts the surgeon about a client whose heart rate is 120 beats/min. After consulting with the surgeon, the nurse administers a beta blocker to the client. The student asks why this was needed. What response by the nurse is best? a. "A rapid heart rate requires more effort by the heart." b. "Anesthesia has bad effects if the client is tachycardic." c. "The client may have an undiagnosed heart condition." d. "When the heart rate goes up, the blood pressure does too."

ANS: A Tachycardia increases the workload of the heart and requires more oxygen delivery to the myocardial tissues. This added strain is not needed on top of the physical and emotional stress of surgery. The other statements are not accurate. DIF: Applying/Application REF: 219 KEY: Preoperative nursing| tachycardia| beta blocker MSC: Integrated Process: Teaching/Learning NOT: Client Needs Category: Physiological Integrity: Physiological Adaptation

The perioperative nurse manager and the postoperative unit manager are concerned about the increasing number of surgical infections in their hospital. What action by the managers is best? a. Audit charts to see if the Surgical Care Improvement Project (SCIP) outcomes were met. b. Encourage staff on both units to provide peer pressure to adhere to hand hygiene policy. c. Hold educational meetings with the nursing and surgical staff on infection prevention. d. Monitor staff on both units for consistent adherence to established hand hygiene practices

ANS: A The SCIP project contains core measures that are mandatory for all surgical clients and focuses on preventing infection, serious cardiac events, and venous thromboembolism. The managers should start by reviewing charts to see if the guidelines of this project were implemented. The other actions may be necessary too, but first the managers need to assess the situation. DIF: Applying/Application REF: 216 KEY: Preoperative nursing| Surgical Care Improvement Project (SCIP)| infection MSC: Integrated Process: Nursing Process: Implementation NOT: Client Needs Category: Safe and Effective Care Environment: Management of Care

A nurse is concerned that a preoperative client has a great deal of anxiety about the upcoming procedure. What action by the nurse is best? a. Ask the client to describe current feelings. b. Determine if the client wants a chaplain. c. Reassure the client this surgery is common. d. Tell the client there is no need to be anxious.

ANS: A The nurse needs to conduct further assessment of the client's anxiety. Asking open-ended questions about current feelings is an appropriate way to begin. The client may want a chaplain, but the nurse needs to do more for the client. Reassurance can be good, but false hope is not, and simply reassuring the client may not be helpful. Telling the client not to be anxious belittles the client's feelings. DIF: Applying/Application REF: 222 KEY: Preoperative nursing| anxiety| support MSC: Integrated Process: Nursing Process: Assessment NOT: Client Needs Category: Psychosocial Integrity

A woman tells a nurse, My husband wants to have sex when I have my period. Is that safe? What is an appropriate answer? A) No, the flow of blood could be slowed down. B) No, it will tend to make your cramps worse. C) Yes, but be sure to douche after sex. D) Yes, there is no reason not to have sex then.

D) Yes, there is no reason not to have sex then.

A client is on the phone when the nurse brings a preoperative antibiotic before scheduled surgery. The circulating nurse has requested the antibiotic be started. The client wants the nurse to wait before starting it. What response by the nurse is most appropriate? a. Explain the rationale for giving the medicine now. b. Leave the room and come back in 15 minutes. c. Provide holistic client care and come back later. d. Tell the client you must start the medication now.

ANS: A The preoperative antibiotic must be given within 60 minutes of the surgical start time to ensure the proper amount is in the tissues when the incision is made. The nurse should explain the rationale to the client for this timing. The other options do not take this timing into consideration and do not give the client the information needed to be cooperative. DIF: Applying/Application REF: 234 KEY: Preoperative nursing| antibiotic| Surgical Care Improvement Project (SCIP) MSC: Integrated Process: Nursing Process: Implementation NOT: Client Needs Category: Physiological Integrity: Pharmacological and Parenteral Therapies

nurse assesses a client in the preoperative holding area and finds brittle nails and hair, dry skin turgor, and muscle wasting. What action by the nurse is best? a. Consult the surgeon about a postoperative dietitian referral. b. Document the findings thoroughly in the client's chart. c. Encourage the client to eat more after recovering from surgery. d. Refer the client to Meals on Wheels after discharge.

ANS: A This client has signs of malnutrition, which can impact recovery from surgery. The nurse should consult the surgeon about prescribing a consultation with a dietitian in the postoperative period. The nurse should document the findings but needs to do more. Encouraging the client to eat more may be helpful, but the client needs a professional nutritional assessment so that the appropriate diet and supplements can be ordered. The client may or may not need Meals on Wheels after discharge. DIF: Applying/Application REF: 222 KEY: Preoperative nursing| malnutrition| nutrition MSC: Integrated Process: Communication and Documentation NOT: Client Needs Category: Safe and Effective Care Environment: Management of Care

A nurse cares for a client with an increased blood urea nitrogen (BUN)/creatinine ratio. Which action should the nurse take first? a. Assess the client's dietary habits. b. Inquire about the use of nonsteroidal anti-inflammatory drugs (NSAIDs). c. Hold the client's metformin (Glucophage). d. Contact the health care provider immediately.

ANS: A An elevated BUN/creatinine ratio is often indicative of dehydration, urinary obstruction, catabolism, or a high-protein diet. The nurse should inquire about the client's dietary habits. Kidney damage related to NSAID use most likely would manifest with elevations in both BUN and creatinine, but no change in the ratio. The nurse should obtain more assessment data before holding any medications or contacting the provider. DIF: Applying/Application REF: 1355 KEY: Urinary/renal system| assessment/diagnostic examination| nutritional requirements MSC: Integrated Process: Nursing Process: Assessment NOT: Client Needs Category: Safe and Effective Care Environment: Management of Care

A nurse reviews the allergy list of a client who is scheduled for an intravenous urography. Which client allergy should alert the nurse to urgently contact the health care provider? a. Seafood b. Penicillin c. Bee stings d. Red food dye

ANS: A Clients with seafood allergies often have severe allergic reactions to the standard dyes used during intravenous urography. The other allergies have no impact on the client's safety during an intravenous urography. DIF: Applying/Application REF: 1361 KEY: Urinary/renal system| assessment/diagnostic examination| allergies MSC: Integrated Process: Nursing Process: Assessment NOT: Client Needs Category: Physiological Integrity: Reduction of Risk Potential

A 59-year-old male patient is scheduled for a hemorrhoidectomy. The OR nurse should anticipate assisting the other team members with positioning the patient in what manner? A) Dorsal recumbent position B) Trendelenburg position C) Sims position D) Lithotomy position

Ans: D Feedback: The lithotomy position is used for nearly all perineal, rectal, and vaginal surgical procedures. The Sims or lateral position is used for renal surgery and the Trendelenburg position usually is used for surgery on the lower abdomen and pelvis. The usual position for surgery, called the dorsal recumbent position, is flat on the back, but this would be impracticable for rectal surgery.

A nurse cares for a client with diabetes mellitus who is prescribed metformin (Glucophage) and is scheduled for an intravenous urography. Which action should the nurse take first? a. Contact the provider and recommend discontinuing the metformin. b. Keep the client NPO for at least 6 hours prior to the examination. c. Check the client's capillary artery blood glucose and administer prescribed insulin. d. Administer intravenous fluids to dilute and increase the excretion of dye.

ANS: A Metformin can cause lactic acidosis and renal impairment as the result of an interaction with the dye. This drug must be discontinued for 48 hours before the procedure and not started again after the procedure until urine output is well established. The client's health care provider needs to provide alternative therapy for the client until the metformin can be resumed. Keeping the client NPO, checking the client's blood glucose, and administering intravenous fluids should be part of the client's plan of care, but are not the priority, as the examination should not occur while the client is still taking metformin. DIF: Applying/Application REF: 1361 KEY: Urinary/renal system| diabetes mellitus| medication safety MSC: Integrated Process: Nursing Process: Implementation NOT: Client Needs Category: Physiological Integrity: Reduction of Risk Potential

A nurse contacts the health care provider after reviewing a client's laboratory results and noting a blood urea nitrogen (BUN) of 35 mg/dL and a creatinine of 1.0 mg/dL. For which action should the nurse recommend a prescription? a. Intravenous fluids b. Hemodialysis c. Fluid restriction d. Urine culture and sensitivity

ANS: A Normal BUN is 10 to 20 mg/dL. Normal creatinine is 0.6 to 1.2 mg/dL (males) or 0.5 to 1.1 mg/dL (females). Creatinine is more specific for kidney function than BUN, because BUN can be affected by several factors (dehydration, high-protein diet, and catabolism). This client's creatinine is normal, which suggests a non-renal cause for the elevated BUN. A common cause of increased BUN is dehydration, so the nurse should anticipate giving the client more fluids, not placing the client on fluid restrictions. Hemodialysis is not an appropriate treatment for dehydration. The lab results do not indicate an infection; therefore, a urine culture and sensitivity is not appropriate. DIF: Applying/Application REF: 1355 KEY: Urinary/renal system| assessment/diagnostic examination| hydration MSC: Integrated Process: Nursing Process: Analysis NOT: Client Needs Category: Physiological Integrity: Pharmacological and Parenteral Therapies

A nurse reviews laboratory results for a client who was admitted for a myocardial infarction and cardiogenic shock 2 days ago. Which laboratory test result should the nurse expect to find? a. Blood urea nitrogen (BUN) of 52 mg/dL b. Creatinine of 2.3 mg/dL c. BUN of 10 mg/dL d. BUN/creatinine ratio of 8:1

ANS: A Shock leads to decreased renal perfusion. An elevated BUN accompanies this condition. The creatinine should be normal because no kidney damage occurred. A low BUN signifies overhydration, malnutrition, or liver damage. A low BUN/creatinine ratio indicates fluid volume excess or acute renal tubular acidosis. DIF: Applying/Application REF: 1359 KEY: Urinary/renal system| assessment/diagnostic examination| shock MSC: Integrated Process: Nursing Process: Analysis NOT: Client Needs Category: Physiological Integrity: Reduction of Risk Potential

A client is clearly uncomfortable and anxious in the preoperative holding room waiting for emergent abdominal surgery. What actions can the nurse perform to increase comfort? (Select all that apply.) a. Allow the client to assume a position of comfort. b. Allow the client's family to remain at the bedside. c. Give the client a warm, non-caffeinated drink. d. Provide warm blankets or cool washcloths as desired. e. Pull the curtains around the bed to provide privacy

ANS: A, B, D, E There are many nonpharmacologic comfort measures the nurse can employ, such as allowing the client to remain in the position that is most comfortable, letting the family stay with the client, providing warmth or cooling measures as requested by the client, and providing privacy. The client in the preoperative holding area is NPO, so drinks should not be provided. DIF: Applying/Application REF: 233 KEY: Preoperative nursing| comfort MSC: Integrated Process: Nursing Process: Implementation NOT: Client Needs Category: Physiological Integrity: Basic Care and Comfort

Overflow incontinence:

the loss of urine in combination with a distended bladder. causes include: fecal impaction, neuro disorders, enlarged prostate

After delegating to an unlicensed assistive personnel (UAP) the task of completing a bladder scan examination for a client, the nurse evaluates the UAP's performance. Which action by the UAP indicates the nurse must provide additional instructions when delegating this task? a. Selecting the female icon for all female clients and male icon for all male clients b. Telling the client, "This test measures the amount of urine in your bladder." c. Applying ultrasound gel to the scanning head and removing it when finished d. Taking at least two readings using the aiming icon to place the scanning head

ANS: A The UAP should use the female icon for women who have not had a hysterectomy. This allows the scanner to subtract the volume of the uterus from readings. If a woman has had a hysterectomy, the UAP should choose the male icon. The UAP should explain the procedure to the client, apply gel to the scanning head and clean it after use, and take at least two readings. DIF: Applying/Application REF: 1360 KEY: Urinary/renal system| delegation| supervision| unlicensed assistive personnel (UAP) MSC: Integrated Process: Communication and Documentation NOT: Client Needs Category: Safe and Effective Care Environment: Management of Care

A student nurse asks why older adults are at higher risk for complications after surgery. What reasons does the registered nurse give? (Select all that apply.) a. Decreased cardiac output b. Decreased oxygenation c. Frequent nocturia d. Mobility alterations e. Inability to adapt to changes

ANS: A, B, C, D Older adults have many age-related physiologic changes that put them at higher risk of falling and other complications after surgery. Some of these include decreased cardiac output, decreased oxygenation of tissues, nocturia, and mobility alterations. They also have a decreased ability to adapt to new surroundings, but that is not the same as being unable to adapt. DIF: Understanding/Comprehension REF: 220 KEY: Preoperative nursing| older adult MSC: Integrated Process: Teaching/Learning NOT: Client Needs Category: Health Promotion and Maintenance

A nurse reviews a client's laboratory results. Which results from the client's urinalysis should the nurse identify as normal? (Select all that apply.) a. pH: 6 b. Specific gravity: 1.015 c. Protein: 1.2 mg/dL d. Glucose: negative e. Nitrate: small f. Leukocyte esterase: positive

ANS: A, B, D The pH, specific gravity, and glucose are all within normal ranges. The other values are abnormal. DIF: Remembering/Knowledge REF: 1356 KEY: Urinary/renal system| assessment/diagnostic examination MSC: Integrated Process: Nursing Process: Assessment NOT: Client Needs Category: Physiological Integrity: Reduction of Risk Potential

A nurse prepares a client for a percutaneous kidney biopsy. Which actions should the nurse take prior to this procedure? (Select all that apply.) a. Keep the client NPO for 4 to 6 hours. b. Obtain coagulation study results. c. Maintain strict bedrest in a supine position. d. Assess for blood in the client's urine. e. Administer antihypertensive medications.

ANS: A, B, E Prior to a percutaneous kidney biopsy, the client should be NPO for 4 to 6 hours. Coagulation studies should be completed to prevent bleeding after the biopsy. Blood pressure medications should be administered to prevent hypertension before and after the procedure. There is no need to keep the client on bedrest or assess for blood in the client's urine prior to the procedure; these interventions should be implemented after a percutaneous kidney biopsy. DIF: Applying/Application REF: 1362 KEY: Urinary/renal system| assessment/diagnostic examination MSC: Integrated Process: Nursing Process: Implementation NOT: Client Needs Category: Physiological Integrity: Reduction of Risk Potential

A nurse plans care for an older adult client. Which interventions should the nurse include in this client's plan of care to promote kidney health? (Select all that apply.) a. Ensure adequate fluid intake. b. Leave the bathroom light on at night. c. Encourage use of the toilet every 6 hours. d. Delegate bladder training instructions to the unlicensed assistive personnel (UAP). e. Provide thorough perineal care after each voiding. f. Assess for urinary retention and urinary tract infection.

ANS: A, B, E, F The nurse should ensure that the client receives adequate fluid intake and has adequate lighting to ambulate safely to the bathroom at night, encourage the client to use the toilet every 2 hours, provide thorough perineal care after each voiding, and assess for urinary retention and urinary tract infections. The nurse should not delegate any teaching to the UAP, including bladder training instructions. The UAP may participate in bladder training activities, including encouraging and assisting the client to the bathroom at specific times. DIF: Understanding/Comprehension REF: 1351 KEY: Urinary/renal system| older adult| safety| unlicensed assistive personnel (UAP) MSC: Integrated Process: Nursing Process: Implementation NOT: Client Needs Category: Health Promotion and Maintenance

A nurse is caring for several clients prior to surgery. Which medications taken by the clients require the nurse to consult with the physician about their administration? (Select all that apply.) a. Metformin (Glucophage) b. Omega-3 fatty acids (Sea Omega 30) c. Phenytoin (Dilantin) d. Pilocarpine hydrochloride (Isopto Carpine) e. Warfarin (Coumadin)

ANS: A, C, D, E Although the client will be on NPO status before surgery, the nurse should check with the provider about allowing the client to take medications prescribed for diabetes, hypertension, cardiac disease, seizure disorders, depression, glaucoma, anticoagulation, or depression. Metformin is used to treat diabetes; phenytoin is for seizures; pilocarpine is for glaucoma, and warfarin is an anticoagulant. The omega-3 fatty acids can be held the day of surgery. DIF: Analyzing/Analysis REF: 228 KEY: Preoperative nursing| medications| NPO MSC: Integrated Process: Nursing Process: Analysis NOT: Client Needs Category: Physiological Integrity: Pharmacological and Parenteral Therapies

A student nurse is caring for clients on the postoperative unit. The student asks the registered nurse why malnutrition can lead to poor surgical outcomes. What responses by the nurse are best? (Select all that apply.) a. "A malnourished client will have fragile skin." b. "Malnourished clients always have other problems." c. "Many drugs are bound to protein in the body." d. "Protein stores are needed for wound healing." e. "Weakness and fatigue are common in malnutrition."

ANS: A, C, D, E Malnutrition can lead to poorer surgical outcomes for several reasons, including fragile skin that might break down, altered pharmacokinetics, poorer wound healing, and weakness or fatigue that can interfere with recovery. Malnutrition can exist without other comorbidities. DIF: Understanding/Comprehension REF: 222 KEY: Preoperative nursing| nutrition| malnutrition MSC: Integrated Process: Teaching/Learning NOT: Client Needs Category: Physiological Integrity: Physiological Adaptation

A young woman has been diagnosed with human papilloma virus (HPV). As a result, she will be at increased risk for which of the following? A) infertility B) genital warts C) vaginal bleeding D) cervical cancer

D) cervical cancer

A nurse assesses a client recovering from a cystoscopy. Which assessment findings should alert the nurse to urgently contact the health care provider? (Select all that apply.) a. Decrease in urine output b. Tolerating oral fluids c. Prescription for metformin d. Blood clots present in the urine e. Burning sensation when urinating

ANS: A, D The nurse should monitor urine output and contact the provider if urine output decreases or becomes absent. The nurse should also assess for blood in the client's urine. The urine may be pink-tinged, but gross bleeding or blood clots should not be present. If bleeding is present, the nurse should urgently contact the provider. Tolerating oral fluids is a positive outcome and does not need intervention. Metformin would be a concern if the client received dye; no dye is used in a cystoscopy procedure. The client may experience a burning sensation when urinating after this procedure; this would not require a call to the provider. DIF: Applying/Application REF: 1352 KEY: Urinary/renal system| assessment/diagnostic examination MSC: Integrated Process: Nursing Process: Implementation NOT: Client Needs Category: Physiological Integrity: Reduction of Risk Potential

A client waiting for surgery is very anxious. What intervention can the nurse delegate to the unlicensed assistive personnel (UAP)? a. Assess the client's anxiety. b. Give the client a back rub. c. Remind the client to turn. d. Teach about postoperative care.

ANS: B A back rub reduces anxiety and can be delegated to the UAP. Once teaching has been done, the UAP can remind the client to turn, but this is not related to relieving anxiety. Assessing anxiety and teaching are not within the scope of practice for the UAP. DIF: Applying/Application REF: 233 KEY: Preoperative nursing| anxiety| unlicensed assistive personnel (UAP) MSC: Integrated Process: Nursing Process: Implementation NOT: Client Needs Category: Safe and Effective Care Environment: Management of Care

A client has been given hydroxyzine (Atarax) in the preoperative holding area. What action by the nurse is most important for this client? a. Document giving the drug. b. Raise the siderails on the bed. c. Record the client's vital signs. d. Teach relaxation techniques.

ANS: B All actions are appropriate for a preoperative client. However, for client safety, the nurse should raise the siderails on the bed because hydroxyzine can make the client sleepy. DIF: Applying/Application REF: 234 KEY: Preoperative nursing| safety| hydroxyzine MSC: Integrated Process: Nursing Process: Implementation NOT: Client Needs Category: Physiological Integrity: Pharmacological and Parenteral Therapies

. A circulating nurse provides care in a surgical department that has multiple surgeries scheduled for the day. The nurse should know to monitor which patient most closely during the intraoperative period because of the increased risk for hypothermia? A) A 74-year-old woman with a low body mass index B) A 17-year-old boy with traumatic injuries C) A 45-year-old woman having an abdominal hysterectomy D) A 13-year-old girl undergoing craniofacial surgery

Ans: A Feedback: Elderly patients are at greatest risk during surgical procedures because they have an impaired ability to increase their metabolic rate and impaired thermoregulatory mechanisms, which increase susceptibility to hypothermia. The other patients are likely at a lower risk.

An inpatient nurse brings an informed consent form to a client for an operation scheduled for tomorrow. The client asks about possible complications from the operation. What response by the nurse is best? a. Answer the questions and document that teaching was done. b. Do not have the client sign the consent and call the surgeon. c. Have the client sign the consent, then call the surgeon. d. Remind the client of what teaching the surgeon has done

ANS: B In order to give informed consent, the client needs sufficient information. Questions about potential complications should be answered by the surgeon. The nurse should notify the surgeon to come back and answer the client's questions before the client signs the consent form. The other actions are not appropriate. DIF: Applying/Application REF: 226 KEY: Preoperative nursing| informed consent MSC: Integrated Process: Communication and Documentation NOT: Client Needs Category: Safe and Effective Care Environment: Management of Care

A client has a great deal of pain when coughing and deep breathing after abdominal surgery despite having pain medication. What action by the nurse is best? a. Call the provider to request more analgesia. b. Demonstrate how to splint the incision. c. Have the client take shallower breaths. d. Tell the client a little pain is expected.

ANS: B Splinting an incision provides extra support during coughing and activity and helps decrease pain. If the client is otherwise comfortable, no more analgesia is required. Shallow breathing can lead to atelectasis and pneumonia. The client should know some pain is normal and expected after surgery, but that answer alone does not provide any interventions to help the client. DIF: Applying/Application REF: 230 KEY: Preoperative nursing| nonpharmacologic pain management| splinting MSC: Integrated Process: Teaching/Learning NOT: Client Needs Category: Physiological Integrity: Basic Care and Comfort

A nurse is giving a client instructions for showering with special antimicrobial soap the night before surgery. What instruction is most appropriate? a. "After you wash the surgical site, shave that area with your own razor." b. "Be sure to wash the area where you will have surgery very thoroughly." c. "Use a washcloth to wash the surgical site; do not take a full shower or bath." d. "Wash the surgical site first, then shampoo and wash the rest of your body."

ANS: B The entire proposed surgical site needs to be washed thoroughly and completely with the antimicrobial soap. Shaving, if absolutely necessary, should be done in the operative suite immediately before the operation begins, using sterile equipment. The client needs a full shower or bath (shower preferred). The client should wash the surgical site last; dirty water from shampooing will run over the cleansed site if the site is washed first. DIF: Applying/Application REF: 228 KEY: Preoperative nursing| client education| skin preparation MSC: Integrated Process: Teaching/Learning NOT: Client Needs Category: Physiological Integrity: Reduction of Risk Potential

A postoperative client has an abdominal drain. What assessment by the nurse indicates that goals for the priority client problems related to the drain are being met? a. Drainage from the surgical site is 30 mL less than yesterday. b. There is no redness, warmth, or drainage at the insertion site. c. The client reports adequate pain control with medications. d. Urine is clear yellow and urine output is greater than 40 mL/hr.

ANS: B The priority client problem related to a surgical drain is the potential for infection. An insertion site that is free of redness, warmth, and drainage indicates that goals for this client problem are being met. The other assessments are normal, but not related to the drain. DIF: Evaluating/Synthesis REF: 230 KEY: Preoperative nursing| drains MSC: Integrated Process: Nursing Process: Evaluation NOT: Client Needs Category: Physiological Integrity: Reduction of Risk Potential

A nurse is giving a preoperative client a dose of ranitidine (Zantac). The client asks why the nurse is giving this drug when the client has no history of ulcers. What response by the nurse is best? a. "All preoperative clients get this medication." b. "It helps prevent ulcers from the stress of the surgery." c. "Since you don't have ulcers, I will have to ask." d. "The physician prescribed this medication for you."

ANS: B Ulcer prophylaxis is common for clients undergoing long procedures or for whom high stress is likely. The nurse is not being truthful by saying all clients get this medication. If the nurse does not know the information, it is appropriate to find out, but this is a common medication for which the nurse should know the rationale prior to administering it. Simply stating that the physician prescribed the medication does not give the client any useful information. DIF: Understanding/Comprehension REF: 234 KEY: Preoperative nursing| histamine blocker| ulcers MSC: Integrated Process: Communication and Documentation NOT: Client Needs Category: Physiological Integrity: Pharmacological and Parenteral Therapies

A nurse cares for a client who has elevated levels of antidiuretic hormone (ADH). Which disorder should the nurse identify as a trigger for the release of this hormone? a. Pneumonia b. Dehydration c. Renal failure d. Edema

ANS: B ADH increases tubular permeability to water, leading to absorption of more water into the capillaries. ADH is triggered by a rising extracellular fluid osmolarity, as occurs in dehydration. Pneumonia, renal failure, and edema would not trigger the release of ADH. DIF: Understanding/Comprehension REF: 1355 KEY: Urinary/renal system| assessment/diagnostic examination| hydration MSC: Integrated Process: Nursing Process: Assessment NOT: Client Needs Category: Physiological Integrity: Physiological Adaptation

A nurse recently hired to the preoperative area learns that certain clients are at higher risk for venous thromboembolism (VTE). Which clients are considered at high risk? (Select all that apply.) a. Client with a humerus fracture b. Morbidly obese client c. Client who underwent a prolonged surgical procedure d. Client with severe heart failure e. Wheelchair-bound client

ANS: B, C, D, E All surgical clients should be assessed for VTE risk. Those considered at higher risk include those who are obese; are over 40; have cancer; have decreased mobility, immobility, or a spinal cord injury; have a history of any thrombotic event, varicose veins, or edema; take oral contraceptives or smoke; have decreased cardiac output; have a hip fracture; or are having total hip or knee surgery. Prolonged surgical time increases risk due to mobility and positioning needs. DIF: Remembering/Knowledge REF: 231 KEY: Preoperative nursing| venous thromboembolism prevention MSC: Integrated Process: Teaching/Learning NOT: Client Needs Category: Physiological Integrity: Reduction of Risk Potential

A nurse is conducting a health history for a woman at a local clinic. The woman tells the nurse, I feel like I am a man trapped in this womans body. What term is given to this feeling? A) heterosexual B) homosexual C) bisexual D) transsexual

D) transsexual

A nurse reviews a female client's laboratory results. Which results from the client's urinalysis should the nurse recognize as abnormal? a. pH 5.6 b. Ketone bodies present c. Specific gravity of 1.020 d. Clear and yellow color

ANS: B Ketone bodies are by-products of incomplete metabolism of fatty acids. Normally no ketones are present in urine. Ketone bodies are produced when fat sources are used instead of glucose to provide cellular energy. A pH between 4.6 and 8, specific gravity between 1.005 and 1.030, and clear yellow urine are normal findings for a female client's urinalysis. DIF: Remembering/Knowledge REF: 1356 KEY: Urinary/renal system| assessment/diagnostic examination MSC: Integrated Process: Nursing Process: Assessment NOT: Client Needs Category: Physiological Integrity: Reduction of Risk Potential

A nurse reviews the health history of a client with an oversecretion of renin. Which disorder should the nurse correlate with this assessment finding? a. Alzheimer's disease b. Hypertension c. Diabetes mellitus d. Viral hepatitis

ANS: B Renin is secreted when special cells in the distal convoluted tubule, called the macula densa, sense changes in blood volume and pressure. When the macula densa cells sense that blood volume, blood pressure, or blood sodium levels are low, renin is secreted. Renin then converts angiotensinogen into angiotensin I. This leads to a series of reactions that cause secretion of the hormone aldosterone. This hormone increases kidney reabsorption of sodium and water, increasing blood pressure, blood volume, and blood sodium levels. Inappropriate or excessive renin secretion is a major cause of persistent hypertension. Renin has no impact on Alzheimer's disease, diabetes mellitus, or viral hepatitis. DIF: Understanding/Comprehension REF: 1346 KEY: Urinary/renal system| health screening MSC: Integrated Process: Nursing Process: Assessment NOT: Client Needs Category: Physiological Integrity: Physiological Adaptation

A nurse cares for a client with a urine specific gravity of 1.018. Which action should the nurse take? a. Evaluate the client's intake and output for the past 24 hours. b. Document the finding in the chart and continue to monitor. c. Obtain a specimen for a urine culture and sensitivity. d. Encourage the client to drink more fluids, especially water.

ANS: B This specific gravity is within the normal range for urine. There is no need to evaluate the client's intake and output, obtain a urine specimen, or increase fluid intake. DIF: Applying/Application REF: 1356 KEY: Urinary/renal system| assessment/diagnostic examination MSC: Integrated Process: Nursing Process: Analysis NOT: Client Needs Category: Physiological Integrity: Reduction of Risk Potential

A nurse assesses clients on the medical-surgical unit. Which clients are at risk for kidney problems? (Select all that apply.) a. A 24-year-old pregnant woman prescribed prenatal vitamins b. A 32-year-old bodybuilder taking synthetic creatine supplements c. A 56-year-old who is taking metformin for diabetes mellitus d. A 68-year-old taking high-dose nonsteroidal anti-inflammatory drugs (NSAIDs) for chronic back pain e. A 75-year-old with chronic obstructive pulmonary disease (COPD) who is prescribed an albuterol nebulizer

ANS: B, C, D Many medications can affect kidney function. Clients who take synthetic creatine supplements, metformin, and high-dose or long-term NSAIDs are at risk for kidney dysfunction. Prenatal vitamins and albuterol nebulizers do not place these clients at risk. DIF: Applying/Application REF: 1356 KEY: Urinary/renal system| health screening MSC: Integrated Process: Nursing Process: Assessment NOT: Client Needs Category: Physiological Integrity: Pharmacological and Parenteral Therapies

A nurse working in the preoperative holding area performs which functions to ensure client safety? (Select all that apply.) a. Allow small sips of plain water. b. Check that consent is on the chart. c. Ensure the client has an armband on. d. Have the client help mark the surgical site. e. Allow the client to use the toilet before giving sedation

ANS: B, C, D, E Providing for client safety is a priority function of the preoperative nurse. Checking for appropriately completed consent, verifying the client's identity, having the client assist in marking the surgical site if applicable, and allowing the client to use the toilet prior to sedating him or her are just some examples of important safety measures. The preoperative client should be NPO, so water should not be provided. DIF: Applying/Application REF: 216 KEY: Preoperative nursing| safety MSC: Integrated Process: Nursing Process: Implementation NOT: Client Needs Category: Safe and Effective Care Environment: Safety and Infection Control

A new perioperative nurse is receiving orientation to the surgical area and learns about the Surgical Care Improvement Project (SCIP) goals. What major areas do these measures focus on preventing? (Select all that apply.) a. Hemorrhage b. Infection c. Serious cardiac events d. Stroke e. Thromboembolism

ANS: B, C, E The SCIP project includes core measures to prevent infection, serious cardiac events, and thromboembolic events such as deep vein thrombosis. DIF: Remembering/Knowledge REF: 216 KEY: Preoperative nursing| Surgical Care Improvement Project (SCIP)| core measures MSC: Integrated Process: Teaching/Learning NOT: Client Needs Category: Safe and Effective Care Environment: Safety and Infection Control

A nursing instructor is teaching students about different surgical procedures and their classifications. Which examples does the instructor include? (Select all that apply.) a. Hemicolectomy: diagnostic b. Liver biopsy: diagnostic c. Mastectomy: restorative d. Spinal cord decompression: palliative e. Total shoulder replacement: restorative

ANS: B, E A diagnostic procedure is used to determine cell type of cancer and to determine the cause of a problem. An example is a liver biopsy. A restorative procedure aims to improve functional ability. An example would be a total shoulder replacement or a spinal cord decompression (not palliative). A curative procedure either removes or repairs the causative problem. An example would be a mastectomy (not restorative) or a hemicolectomy (not diagnostic). A palliative procedure relieves symptoms but will not cure the disease. An example is an ileostomy. A cosmetic procedure is done to improve appearance. An example is rhinoplasty (a "nose job"). DIF: Remembering/Knowledge REF: 218 KEY: Preoperative nursing| surgical procedures MSC: Integrated Process: Teaching/Learning NOT: Client Needs Category: Physiological Integrity: Physiological Adaptation

43. The nurse is caring for a postoperative medial meniscus repair of the right knee. To assist with pain management following the procedure, which intervention should the nurse implement? a. Monitor vital signs every 15 minutes. b. Apply brace to right knee. c. Elevate right knee and apply ice. d. Check pulses in right foot.

ANS: C Cleanse surgical or traumatic wounds by applying noncytotoxic solution with sterile gauze or irrigations. Cleanse in a direction from the least contaminated area. Use gentle friction when applying solutions to the skin, and allow irrigation to flow from the least to the most contaminated area.

A preoperative nurse is reviewing morning laboratory values on four clients waiting for surgery. Which result warrants immediate communication with the surgical team? a. Creatinine: 1.2 mg/dL b. Hemoglobin: 14.8 mg/dL c. Potassium: 2.9 mEq/L d. Sodium: 134 mEq/L

ANS: C A potassium of 2.9 mEq/L is critically low and can affect cardiac and respiratory status. The nurse should communicate this laboratory value immediately. The creatinine is at the high end of normal, the hemoglobin is normal, and the sodium is only slightly low (normal low being 136 mEq/L), so these values do not need to be reported immediately. DIF: Applying/Application REF: 223 KEY: Preoperative nursing| laboratory values MSC: Integrated Process: Communication and Documentation NOT: Client Needs Category: Physiological Integrity: Reduction of Risk Potential

A client in the preoperative holding room has received sedation and now needs to urinate. What action by the nurse is best? a. Allow the client to walk to the bathroom. b. Delegate assisting the client to the nurse's aide. c. Give the client a bedpan or urinal to use. d. Insert a urinary catheter now instead of waiting.

ANS: C Although possibly uncomfortable or embarrassing for the client, the client should not be allowed out of bed after receiving sedation. The nurse should get the client a bedpan or urinal. The client may or may not need a urinary catheter. DIF: Applying/Application REF: 234 KEY: Preoperative nursing| sedation| safety MSC: Integrated Process: Nursing Process: Implementation NOT: Client Needs Category: Safe and Effective Care Environment: Safety and Infection Control

A nurse works on the postoperative floor and has four clients who are being discharged tomorrow. Which one has the greatest need for the nurse to consult other members of the health care team for post-discharge care? a. Married young adult who is the primary caregiver for children b. Middle-aged client who is post knee replacement, needs physical therapy c. Older adult who lives at home despite some memory loss d. Young client who lives alone, has family and friends nearby

ANS: C The older adult has the most potentially complex discharge needs. With memory loss, the client may not be able to follow the prescribed home regimen. The client's physical abilities may be limited by chronic illness. This client has several safety needs that should be assessed. The other clients all have evidence of a support system and no known potential for serious safety issues. DIF: Applying/Application REF: 221 KEY: Preoperative nursing| discharge planning| older adult MSC: Integrated Process: Communication and Documentation NOT: Client Needs Category: Safe and Effective Care Environment: Management of Care

A client that is post-menopausal reports painful intercourse for the last two months. When performing an assessment of the client, what data should the nurse obtain?

the use of antihistamines, tranquilizers, or alcohol

A nurse cares for a client who is recovering from a closed percutaneous kidney biopsy. The client states, "My pain has suddenly increased from a 3 to a 10 on a scale of 0 to 10." Which action should the nurse take first? a. Reposition the client on the operative side. b. Administer the prescribed opioid analgesic. c. Assess the pulse rate and blood pressure. d. Examine the color of the client's urine.

ANS: C An increase in the intensity of pain after a percutaneous kidney biopsy is a symptom of internal hemorrhage. A change in vital signs can indicate that hemorrhage is occurring. Before other actions, the nurse must assess the client's hemodynamic status. DIF: Applying/Application REF: 1363 KEY: Urinary/renal system| assessment/diagnostic examination| hemorrhage MSC: Integrated Process: Nursing Process: Implementation NOT: Client Needs Category: Safe and Effective Care Environment: Management of Care

A nurse teaches a client who is recovering from a urography. Which instruction should the nurse include in this client's discharge teaching? a. "Avoid direct contact with your urine for 24 hours until the radioisotope clears." b. "You may have some dribbling of urine for several weeks after this procedure." c. "Be sure to drink at least 3 liters of fluids today to help eliminate the dye faster." d. "Your skin may become slightly yellow from the dye used in this procedure."

ANS: C Dyes used in urography are potentially nephrotoxic. A large fluid intake will help the client eliminate the dye rapidly. Dyes used in urography are not radioactive, the client should not experience any dribbling of urine, and the dye should not change the color of the client's skin. DIF: Applying/Application REF: 1361 KEY: Urinary/renal system| assessment/diagnostic examination| hydration MSC: Integrated Process: Teaching/Learning NOT: Client Needs Category: Health Promotion and Maintenance

A nurse assesses a client with renal insufficiency and a low red blood cell count. The client asks, "Is my anemia related to the renal insufficiency?" How should the nurse respond? a. "Red blood cells produce erythropoietin, which increases blood flow to the kidneys." b. "Your anemia and renal insufficiency are related to inadequate vitamin D and a loss of bone density." c. "Erythropoietin is usually released from the kidneys and stimulates red blood cell production in the bone marrow." d. "Kidney insufficiency inhibits active transportation of red blood cells throughout the blood."

ANS: C Erythropoietin is produced in the kidney and is released in response to decreased oxygen tension in the renal blood supply. Erythropoietin stimulates red blood cell production in the bone marrow. Anemia and renal insufficiency are not manifestations of vitamin D deficiency. The kidneys do not play a role in the transportation of red blood cells or any other cells in the blood. DIF: Remembering/Knowledge REF: 1349 KEY: Urinary/renal system| assessment/diagnostic examination MSC: Integrated Process: Nursing Process: Assessment NOT: Client Needs Category: Physiological Integrity: Physiological Adaptation

A nurse obtains a sterile urine specimen from a client's Foley catheter. After applying a clamp to the drainage tubing distal to the injection port, which action should the nurse take next? a. Clamp another section of the tube to create a fixed sample section for retrieval. b. Insert a syringe into the injection port and aspirate the quantity of urine required. c. Clean the injection port cap of the drainage tubing with povidone-iodine solution. d. Withdraw 10 mL of urine and discard it; then withdraw a fresh sample of urine.

ANS: C It is important to clean the injection port cap of the catheter drainage tubing with an appropriate antiseptic, such as povidone-iodine solution or alcohol. This will help prevent surface contamination before injection of the syringe. The urine sample should be collected directly from the catheter; therefore, a second clamp to create a sample section would not be appropriate. Every sample from the catheter is usable; there is the need to discard the first sample. DIF: Understanding/Comprehension REF: 1357 KEY: Urinary/renal system| assessment/diagnostic examination MSC: Integrated Process: Nursing Process: Implementation NOT: Client Needs Category: Safe and Effective Care Environment: Safety and Infection Control

A preoperative nurse is assessing a client prior to surgery. Which information would be most important for the nurse to relay to the surgical team? a. Allergy to bee and wasp stings b. History of lactose intolerance c. No previous experience with surgery d. Use of multiple herbs and supplements

ANS: D Some herbs and supplements can interact with medications, so this information needs to be reported as the priority. An allergy to bee and wasp stings should not affect the client during surgery. Lactose intolerance should also not affect the client during surgery but will need to be noted before a postoperative diet is ordered. Lack of experience with surgery may increase anxiety and may require higher teaching needs, but is not the priority over client safety. DIF: Applying/Application REF: 228 KEY: Preoperative nursing| herbs and supplements| medication interactions MSC: Integrated Process: Communication and Documentation NOT: Client Needs Category: Physiological Integrity: Pharmacological and Parenteral Therapies

The nurse is speaking to a client regarding his sexual history. The client states, "I was born with a penis but I should have had a vagina." The nurse would document the client's gender identity as:

transsexual

Cystometry:

urodynamic testing of bladder function, measures bladder pressure and volume

A nurse reviews the urinalysis of a client and notes the presence of glucose. Which action should the nurse take? a. Document findings and continue to monitor the client. b. Contact the provider and recommend a 24-hour urine test. c. Review the client's recent dietary selections. d. Perform a capillary artery glucose assessment.

ANS: D Glucose normally is not found in the urine. The normal renal threshold for glucose is about 220 mg/dL, which means that a person whose blood glucose is less than 220 mg/dL will not have glucose in the urine. A positive finding for glucose on urinalysis indicates high blood sugar. The most appropriate action would be to perform a capillary artery glucose assessment. The client needs further evaluation for this abnormal result; therefore, documenting and continuing to monitor is not appropriate. Requesting a 24-hour urine test or reviewing the client's dietary selections will not assist the nurse to make a clinical decision related to this abnormality. DIF: Applying/Application REF: 1348 KEY: Urinary/renal system| assessment/diagnostic examination| capillary artery blood glucose MSC: Integrated Process: Nursing Process: Assessment NOT: Client Needs Category: Physiological Integrity: Reduction of Risk Potential

A nurse cares for a client with a urine specific gravity of 1.040. Which action should the nurse take? a. Obtain a urine culture and sensitivity. b. Place the client on restricted fluids. c. Assess the client's creatinine level. d. Increase the client's fluid intake.

ANS: D Normal specific gravity for urine is 1.005 to 1.030. A high specific gravity can occur with dehydration, decreased kidney blood flow (often because of dehydration), and the presence of antidiuretic hormone. Increasing the client's fluid intake would be a beneficial intervention. Assessing the creatinine or obtaining a urine culture would not provide data necessary for the nurse to make a clinical decision. DIF: Applying/Application REF: 1356 KEY: Urinary/renal system| assessment/diagnostic examination| hydration MSC: Integrated Process: Nursing Process: Implementation NOT: Client Needs Category: Safe and Effective Care Environment: Management of Care

A nurse reviews the urinalysis results of a client and notes a urine osmolality of 1200 mOsm/L. Which action should the nurse take? a. Contact the provider and recommend a low-sodium diet. b. Prepare to administer an intravenous diuretic. c. Obtain a suction device and implement seizure precautions. d. Encourage the client to drink more fluids.

ANS: D Normal urine osmolality ranges from 300 to 900 mOsm/L. This client's urine is more concentrated, indicating dehydration. The nurse should encourage the client to drink more water. Dehydration can be associated with elevated serum sodium levels. Although a low-sodium diet may be appropriate for this client, this diet change will not have a significant impact on urine osmolality. A diuretic would increase urine output and decrease urine osmolality further. Low serum sodium levels, not elevated serum levels, place the client at risk for seizure activity. These options would further contribute to the client's dehydration or elevate the osmolality. DIF: Applying/Application REF: 1359 KEY: Urinary/renal system| assessment/diagnostic examination MSC: Integrated Process: Nursing Process: Implementation NOT: Client Needs Category: Physiological Integrity: Basic Care and Comfort

A nurse cares for a client who is having trouble voiding. The client states, "I cannot urinate in public places." How should the nurse respond? a. "I will turn on the faucet in the bathroom to help stimulate your urination." b. "I can recommend a prescription for a diuretic to improve your urine output." c. "I'll move you to a room with a private bathroom to increase your comfort." d. "I will close the curtain to provide you with as much privacy as possible."

ANS: D The nurse should provide privacy to clients who may be uncomfortable or have issues related to elimination or the urogenital area. Turning on the faucet and administering a diuretic will not address the client's concern. Although moving the client to a private room with a private bathroom would be nice, this is not realistic. The nurse needs to provide as much privacy as possible within the client's current room. DIF: Applying/Application REF: 1363 KEY: Urinary/renal system| patient-centered care MSC: Integrated Process: Caring NOT: Client Needs Category: Psychosocial Integrity

True/false 1. Nurses should obtain information about urinary control from all female patients.

ANS: T All women, especially older women and those who have experienced childbirth, should be screened for different types of urinary incontinence.

While teaching sex education to a group of high school students, the nurse talks about forms of birth control. Which of the following could the nurse say is 100% effective if practiced correctly and consistently?

Abstinence

The nurse is preparing an elderly patient for a scheduled removal of orthopedic hardware, a procedure to be performed under general anesthetic. For which adverse effect should the nurse most closely monitor the patient? A) Hypothermia B) Pulmonary edema C) Cerebral ischemia D) Arthritis

Ans: A Feedback: Inadvertent hypothermia may occur as a result of a low temperature in the OR, infusion of cold fluids, inhalation of cold gases, open body wounds or cavities, decreased muscle activity, advanced age, or the pharmaceutical agents used (e.g., vasodilators, phenothiazines, general anesthetics). The anesthetist monitors for pulmonary edema and cerebral ischemia. Arthritis is not an adverse effect of surgical anesthesia.

The OR nurse acts in the circulating role during a patient's scheduled cesarean section. For what task is this nurse solely responsible? A) Performing documentation B) Estimating the patient's blood loss C) Setting up the sterile tables D) Keeping track of drains and sponges

Ans: A Feedback: Main responsibilities of the circulating nurse include verifying consent; coordinating the team; and ensuring cleanliness, proper temperature and humidity, lighting, safe function of equipment, and the availability of supplies and materials. The circulating nurse monitors aseptic practices to avoid breaks in technique while coordinating the movement of related personnel as well as implementing fire safety precautions. The circulating nurse also monitors the patient and documents specific activities throughout the operation to ensure the patient's safety and well-being. Estimating the patient's blood loss is the surgeon's responsibility; setting up the sterile tables is the responsibility of the first scrub; and keeping track of the drains and sponges is the joint responsibility of the circulating nurse and the scrub nurse.

The perioperative nurse has completed the presurgical assessment of an 82-year-old female patient who is scheduled for a left total knee replacement. When planning this patient's care, the nurse should address the consequences of the patient's aging cardiovascular system. These include an increased risk of which of the following? A) Hypervolemia B) Hyponatremia C) Hyperkalemia D) Hyperphosphatemia

Ans: A Feedback: The aging heart and blood vessels have decreased ability to respond to stress. Reduced cardiac output and limited cardiac reserve make the elderly patient vulnerable to changes in circulating volume and blood oxygen levels. There is not an increased risk for hypopnea, hyperkalemia, or hyperphosphatemia because of an aging cardiovascular system.

A patient waiting in the presurgical holding area asks the nurse, "Why exactly do they have to put a breathing tube into me? My surgery is on my knee." What is the best rationale for intubation during a surgical procedure that the nurse should describe? A) The tube provides an airway for ventilation. B) The tube protects the patient's esophagus from trauma. C) The patient may receive an antiemetic through the tube. D) The patient's vital signs can be monitored with the tube.

Ans: A Feedback: The anesthetic is administered and the patient's airway is maintained through an intranasal intubation, oral intubation, or a laryngeal mask airway. The tube also helps protect aspiration of stomach contents. The tube does not protect the esophagus. Because the tube goes into the lungs, no medications are given through the tube. The patient's vital signs are not monitored through the tube.

The circulating nurse in an outpatient surgery center is assessing a patient who is scheduled to receive moderate sedation. What principle should guide the care of a patient receiving this form of anesthesia? A) The patient must never be left unattended by the nurse. B) The patient should begin a course of antiemetics the day before surgery. C) The patient should be informed that he or she will remember most of the procedure. D) The patient must be able to maintain his or her own airway.

Ans: A Feedback: The patient receiving moderate sedation should never be left unattended. The patient's ability to maintain his or her airway depends on the level of sedation. The administration of moderate sedation is not a counter indication for giving an antiemetic. The patient receiving moderate sedation does not remember most of the procedure.

The OR nurse is participating in the appendectomy of a 20 year-old female patient who has a dangerously low body mass index. The nurse recognizes the patient's consequent risk for hypothermia. What action should the nurse implement to prevent the development of hypothermia? A) Ensure that IV fluids are warmed to the patient's body temperature. B) Transfuse packed red blood cells to increase oxygen carrying capacity. C) Place warmed bags of normal saline at strategic points around the patient's body. D) Monitor the patient's blood pressure and heart rate vigilantly.

Ans: A Feedback: Warmed IV fluids can prevent the development of hypothermia. Applying warmed bags of saline around the patient is not common practice. The patient is not transfused to prevent hypothermia. Blood pressure and heart rate monitoring are important, but do not relate directly to the risk for hypothermia.

An intraoperative nurse is applying interventions that will address surgical patients' risks for perioperative positioning injury. Which of the following factors contribute to this increased risk for injury in the intraoperative phase of the surgical experience? Select all that apply. A) Absence of reflexes B) Diminished ability to communicate C) Loss of pain sensation D) Nausea resulting from anesthetic E) Reduced blood pressure

Ans: A, B, C Feedback: Loss of pain sense, reflexes, and ability to communicate subjects the intraoperative patient to possible injury. Nausea and low blood pressure are not central factors that contribute to this risk, though they are adverse outcomes.

When creating plans of nursing care for patients who are undergoing surgery using general anesthetic, what nursing diagnoses should the nurse identify? Select all that apply. A) Disturbed sensory perception related to anesthetic B) Risk for impaired nutrition: less than body requirements related to anesthesia C) Risk of latex allergy response related to surgical exposure D) Disturbed body image related to anesthesia E) Anxiety related to surgical concerns

Ans: A, C, E Feedback: Based on the assessment data, some major nursing diagnoses may include the following: anxiety related to surgical or environmental concerns, risk of latex allergy response due to possible exposure to latex in the OR environment, risk for perioperative positioning injury related to positioning in the OR, risk for injury related to anesthesia and surgical procedure, or disturbed sensory perception (global) related to general anesthesia or sedation. Malnutrition and disturbed body image are much less likely.

The OR will be caring for a patient who will receive a transsacral block. For what patient would the use of a transsacral block be appropriate for pain control? A) A middle-aged man who is scheduled for a thoracotomy B) An older adult man who will undergo an inguinal hernia repair C) A 50-year-old woman who will be having a reduction mammoplasty D) A child who requires closed reduction of a right humerus fracture

Ans: B Feedback: A transsacral block produces anesthesia for the perineum and lower abdomen. Both a thoracotomy and breast reduction are in the chest region, and a transsacral block would not provide pain control for these procedures. A closed reduction of a right humerus is a procedure on the right arm, and a transsacral block would not provide pain control.

The anesthetist is coming to the surgical admissions unit to see a patient prior to surgery scheduled for tomorrow morning. Which of the following is the priority information that the nurse should provide to the anesthetist during the visit? A) Last bowel movement B) Latex allergy C) Number of pregnancies D) Difficulty falling asleep

Ans: B Feedback: Due to the increased number of patients with latex allergies, it is essential to identify this allergy early on so precautions can be taken in the OR. The anesthetist should be informed of any allergies. This is a priority over pregnancy history, insomnia, or recent bowel function, though some of these may be relevant.

The nurse knows that elderly patients are at higher risk for complications and adverse outcomes during the intraoperative period. What is the best rationale for this phenomenon? A) The elderly patient has a more angular bone structure than a younger person. B) The elderly patient has reduced ability to adjust rapidly to emotional and physical stress. C) The elderly patient has impaired thermoregulatory mechanisms, which increase susceptibility to hyperthermia. D) The elderly patient has an impaired ability to decrease his or her metabolic rate.

Ans: B Feedback: Factors that affect the elderly surgical patient in the intraoperative period include the following: impaired ability to increase metabolic rate and impaired thermoregulatory mechanisms increase susceptibility to hypothermia. Bone loss (25% in women, 12% in men) necessitates careful manipulation and positioning during surgery. Reduced ability to adjust rapidly to emotional and physical stress influences surgical outcomes and requires meticulous observation of vital functions. Older adults do not have more angular bones than younger people.

The patient's surgery is nearly finished and the surgeon has opted to use tissue adhesives to close the surgical wound. This requires the nurse to prioritize assessments related to what complication? A) Hypothermia B) Anaphylaxis C) Infection D) Malignant hyperthermia

Ans: B Feedback: Fibrin sealants are used in a variety of surgical procedures, and cyanoacrylate tissue adhesives are used to close wounds without the use of sutures. These sealants have been implicated in allergic reactions and anaphylaxis. There is not an increased risk of malignant hyperthermia, hypothermia, or infection because of the use of tissue adhesives.

The intraoperative nurse is implementing a care plan that addresses the surgical patient's risk for vomiting. Interventions that address the potential for vomiting reduce the risk of what subsequent surgical complication? A) Impaired skin integrity B) Hypoxia C) Malignant hyperthermia D) Hypothermia

Ans: B Feedback: If the patient aspirates vomitus, an asthma-like attack with severe bronchial spasms and wheezing is triggered. Pneumonitis and pulmonary edema can subsequently develop, leading to extreme hypoxia. Vomiting can cause choking, but the question asks about aspirated vomitus. Malignant hyperthermia is an adverse reaction to anesthesia. Aspirated vomitus does not cause hypothermia. Vomiting does not result in impaired skin integrity.

The surgical patient is a 35-year-old woman who has been administered general anesthesia. The nurse recognizes that the patient is in stage II (the excitement stage) of anesthesia. Which intervention would be most appropriate for the nurse to implement during this stage? A) Rub the patient's back. B) Restrain the patient. C) Encourage the patient to express feelings. D) Stroke the patient's hand.

Ans: B Feedback: In stage II, the patient may struggle, shout, or laugh. The movements of the patient may be uncontrolled, so it is essential the nurse help to restrain the patient for safety. None of the other listed actions protects the patient's safety.

Verification that all required documentation is completed is an important function of the intraoperative nurse. The intraoperative nurse should confirm that the patient's accompanying documentation includes which of the following? A) Discharge planning B) Informed consent C) Analgesia prescription D) Educational resources

Ans: B Feedback: It is important to review the patient's record for the following: correct informed surgical consent, with patient's signature; completed records for health history and physical examination; results of diagnostic studies; and allergies (including latex). Discharge planning records and prescriptions are not normally necessary. Educational resources would not be included at this stage of the surgical process.

The OR nurse is taking the patient into the OR when the patient informs the operating nurse that his grandmother spiked a 104°F temperature in the OR and nearly died 15 years ago. What relevance does this information have regarding the patient? A) The patient may be experiencing presurgical anxiety. B) The patient may be at risk for malignant hyperthermia. C) The grandmother's surgery has minimal relevance to the patient's surgery. D) The patient may be at risk for a sudden onset of postsurgical infection.

Ans: B Feedback: Malignant hyperthermia is an inherited muscle disorder chemically induced by anesthetic agents. Identifying patients at risk is imperative because the mortality rate is 50%. The patient's anxiety is not relevant, the grandmother's surgery is very relevant, and all patients are at risk for hypothermia.

An OR nurse is teaching a nursing student about the principles of surgical asepsis as a requirement in the restricted zone of the operating suite. What personal protective equipment should the nurse wear at all times in the restricted zone of the OR? A) Reusable shoe covers B) Mask covering the nose and mouth C) Goggles D) Gloves

Ans: B Feedback: Masks are worn at all times in the restricted zone of the OR. Shoe covers are worn one time only; goggles and gloves are worn as required, but not necessarily at all times

To promote comfort and optimal respiratory expansion for a client with chronic obstructive pulmonary disease during sexual intimacy, the nurse can suggest that the couple:

use pillows to raise the affected partner's head and upper torso.

While the surgical patient is anesthetized, the scrub nurse hears a member of the surgical team make an inappropriate remark about the patient's weight. How should the nurse best respond? A) Ignore the comment because the patient is unconscious. B) Discourage the colleague from making such comments. C) Report the comment immediately to a supervisor. D) Realize that humor is needed in the workplace.

Ans: B Feedback: Patients, whether conscious or unconscious, should not be subjected to excess noise, inappropriate conversation, or, most of all, derogatory comments. The nurse must act as an advocate on behalf of the patient and discourage any such remarks. Reporting to a supervisor, however, is not likely necessary.

A patient is scheduled for surgery the next day and the different phases of the patient's surgical experience will require input from members of numerous health disciplines. How should the patient's care best be coordinated? A) By planning care using a surgical approach B) By identifying the professional with the most knowledge of the patient C) By implementing an interdisciplinary approach to care D) By using the nursing process to guide all aspects of care and treatment

Ans: C Feedback: An interdisciplinary approach involving the surgeon, anesthesiologist or anesthetist, and nurse is best. This is superior to each of the other listed options

As an intraoperative nurse, you know that the patient's emotional state can influence the outcome of his or her surgical procedure. How would you best reinforce the patient's ability to influence outcome? A) Teach the patient strategies for distraction. B) Pair the patient with another patient who has better coping strategies. C) Incorporate cultural and religious considerations, as appropriate. D) Give the patient antianxiety medication.

Ans: C Feedback: Because the patient's emotional state remains a concern, the care initiated by preoperative nurses is continued by the intraoperative nursing staff that provides the patient with information and reassurance. The nurse supports coping strategies and reinforces the patient's ability to influence outcomes by encouraging active participation in the plan of care incorporating cultural, ethnic, and religious considerations, as appropriate. "Buddying" a patient is normally inappropriate and distraction may or may not be effective. Nonpharmacologic measures should be prioritized.

A patient will be undergoing a total hip arthroplasty later in the day and it is anticipated that the patient may require blood transfusion during surgery. How can the nurse best ensure the patient's safety if a blood transfusion is required? A) Prime IV tubing with a unit of blood and keep it on hold. B) Check that the patient's electrolyte levels have been assessed preoperatively. C) Ensure that the patient has had a current cross-match. D) Keep the blood on standby and warmed to body temperature.

Ans: C Feedback: Few patients undergoing an elective procedure require blood transfusion, but those undergoing high-risk procedures may require an intraoperative transfusion. The circulating nurse anticipates this need, checks that blood has been cross-matched and held in reserve, and is prepared to administer blood. Storing the blood at body temperature or in IV tubing would result in spoilage and potential infection.

You are caring for a male patient who has had spinal anesthesia. The patient is under a physician's order to lie flat postoperatively. When the patient asks to go to the bathroom, you encourage him to adhere to the physician's order. What rationale for complying with this order should the nurse explain to the patient? A) Preventing the risk of hypotension B) Preventing respiratory depression C) Preventing the onset of a headache D) Preventing pain at the lumbar injection site

Ans: C Feedback: Lying flat reduces the risk of headache after spinal anesthesia. Hypotension and respiratory depression may be adverse effects of spinal anesthesia associated with the spread of the anesthetic, but lying flat does not help reduce these effects. Pain at the lumbar injection site typically is not a problem.

As an intraoperative nurse, you are the advocate for each of the patients who receives care in the surgical setting. How can you best exemplify the principles of patient advocacy? A) By encouraging the patient to perform deep breathing preoperatively B) By limiting the patient's contact with family members preoperatively C) By maintaining each of your patients' privacy D) By eliciting informed consent from patients

Ans: C Feedback: Patient advocacy in the OR entails maintaining the patient's physical and emotional comfort, privacy, rights, and dignity. Deep breathing is not necessary before surgery and obtaining informed consent is the purview of the physician. Family contact should not be limited.

Maintaining an aseptic environment in the OR is essential to patient safety and infection control. When moving around surgical areas, what distance must the nurse maintain from the sterile field? A) 2 feet B) 18 inches C) 1 foot D) 6 inches

Ans: C Feedback: Sterile areas must be kept in view during movement around the area. At least a 1-foot distance from the sterile field must be maintained to prevent inadvertent contamination.

The OR nurse is providing care for a 25-year-old major trauma patient who has been involved in a motorcycle accident. The nurse should know that the patient is at increased risk for what complication of surgery? A) Respiratory depression B) Hypothermia C) Anesthesia awareness D) Moderate sedation

Ans: C Feedback: The Joint Commission has issued an alert regarding the phenomenon of patients being partially awake while under general anesthesia (referred to as anesthesia awareness). Patients at greatest risk of anesthesia awareness are cardiac, obstetric, and major trauma patients. This patient does not likely face a heightened risk of respiratory depression or hypothermia. Moderate sedation is not a complication.

The perioperative nurse is constantly assessing the surgical patient for signs and symptoms of complications of surgery. Which symptom should first signal to the nurse the possibility that the patient is developing malignant hyperthermia? A) Increased temperature B) Oliguria C) Tachycardia D) Hypotension

Ans: C Feedback: The initial symptoms of malignant hyperthermia are related to cardiovascular and musculoskeletal activity. Tachycardia (heart rate greater than 150 beats per minute) is often the earliest sign. Oliguria, hypotension, and increased temperature are later signs of malignant hyperthermia.

What is a UA used for:

used as an overall screening test as well as an aid to diagnosing renal, hepatic, and other diseases. Requires a freshly voided sample. "dipstick".

A 68-year-old patient is scheduled for a bilateral mastectomy. The OR nurse has come out to the holding area to meet the patient and quickly realizes that the patient is profoundly anxious. What is the most appropriate intervention for the nurse to apply? A) Reassure the patient that modern surgery is free of significant risks. B) Describe the surgery to the patient in as much detail as possible. C) Clearly explain any information that the patient seeks. D) Remind the patient that the anesthetic will render her unconscious

Ans: C Feedback: The nurse can alleviate anxiety by supplying information as the patient requests it. The nurse should not assume that every patient wants as much detail as possible and false reassurance must be avoided. Reminding the patient that she will be unconscious is unlikely to reduce anxiety.

The circulating nurse will be participating in a 78-year-old patient's total hip replacement. Which of the following considerations should the nurse prioritize during the preparation of the patient in the OR? A) The patient should be placed in Trendelenburg position. B) The patient must be firmly restrained at all times. C) Pressure points should be assessed and well padded. D) The preoperative shave should be done by the circulating nurse.

Ans: C Feedback: The vascular supply should not be obstructed by an awkward position or undue pressure on a body part. During surgical procedures, the patient is at risk for impairment of skin integrity due to a stationary position and immobility. An elderly patient is at an increased risk of injury and impaired skin integrity. A Trendelenburg position is not indicated for this patient. Once anesthetized for a total hip replacement, the patient cannot move; restraints are not necessary. A preoperative shave is not performed; excess hair is removed by means of a clipper.

The nurse is performing wound care on a 68-year-old postsurgical patient. Which of the following practices violates the principles of surgical asepsis? A) Holding sterile objects above the level of the nurse's waist B) Considering a 1 inch (2.5 cm) edge around the sterile field as being contaminated C) Pouring solution onto a sterile field cloth D) Opening the outermost flap of a sterile package away from the body

Ans: C Feedback: Whenever a sterile barrier is breached, the area must be considered contaminated. Pouring solution onto a sterile field cloth violates surgical asepsis because moisture penetrating the cloth can carry microorganisms to the sterile field via capillary action. The other options are practices that help ensure surgical asepsis.

Prior to a patient's scheduled surgery, the nurse has described the way that members of diverse health disciplines will collaborate in the patient's care. What is the main rationale for organizing perioperative care in this collaborative manner? A) Historical precedence B) Patient requests C) Physicians' needs D) Evidence-based practice

Ans: D Feedback: Collaboration of the surgical team using evidence-based practice tailored to a specific case results in optimal patient care and improved outcomes. None of the other listed factors is the basis for the collaboration of the surgical team.

The nurse is caring for a patient who is scheduled to have a needle biopsy of the pleura. The patient has had a consultation with the anesthesiologist and a conduction block will be used. Which local conduction block can be used to block the nerves leading to the chest? A) Transsacral block B) Brachial plexus block C) Peudental block D) Paravertebral block

Ans: D Feedback: Examples of common local conduction blocks include paravertebral anesthesia, which produces anesthesia of the nerves supplying the chest, abdominal wall, and extremities; brachial plexus block, which produces anesthesia of the arm; and transsacral (caudal) block, which produces anesthesia of the perineum and, occasionally, the lower abdomen. A peudental block was used in obstetrics before the almost-routine use of epidural anesthesia.

A nurse is caring for a patient following knee surgery that was performed under a spinal anesthetic. What intervention should the nurse implement to prevent a spinal headache? A) Have the patient sit in a chair and perform deep breathing exercises. B) Ambulate the patient as early as possible. C) Limit the patient's fluid intake for the first 24 hours postoperatively. D) Keep the patient positioned supine.

Ans: D Feedback: Measures that increase cerebrospinal pressure are helpful in relieving headache. These include maintaining a quiet environment, keeping the patient lying flat, and keeping the patient well hydrated. Having the patient sit or stand up decreases cerebrospinal pressure and would not relieve a spinal headache. Limiting fluids is incorrect because it also decreases cerebrospinal pressure and would not relieve a spinal headache.

The nurse is packing a patient's abdominal wound with sterile, half-inch Iodoform gauze. During the procedure, the nurse drops some of the gauze onto the patient's abdomen 2 inches (5 cm) away from the wound. What should the nurse do? A) Apply povidone-iodine (Betadine) to that section of the gauze and continue packing the wound. B) Pick up the gauze and continue packing the wound after irrigating the abdominal wound with Betadine solution. C) Continue packing the wound and inform the physician that an antibiotic is needed. D) Discard the gauze packing and repack the wound with new Iodoform gauze.

Ans: D Feedback: Sterile surfaces or articles may touch other sterile surfaces or articles and remain sterile; contact with unsterile objects at any point renders a sterile area contaminated. The sterile gauze became contaminated when it was dropped on the patient's abdomen. It should be discarded and new Iodoform gauze should be used to pack the wound. Betadine should not be used in the wound unless ordered.

As a perioperative nurse, you know that the National Patient Safety Goals have the potential to improve patient outcomes in a wide variety of health care settings. Which of these Goals has the most direct relevance to the OR? A) Improve safety related to medication use B) Reduce the risk of patient harm resulting from falls C) Reduce the incidence of health care-associated infections D) Reduce the risk of fires

Ans: D Feedback: The National Patient Safety Goals all pertain to the perioperative areas, but the one with the most direct relevance to the OR is the reduction of the risk of surgical fires.

Which of the following nursing interventions is most likely to prevent respiratory complications such as pneumonia and atelectasis in a postsurgical client? a) Adequate pain control b) Control of anxiety and agitation c) Use of incentive spirometry d) Adequate nutrition and fluids

Ans: C Incentive spirometry improves lung expansion, helps expel anesthetic gases and mucus from the airway, and facilitates oxygenation of body tissues. Pain control and hydration may facilitate lung expansion and mobilization of secretions, but incentive spirometry directly increases lung volume and alveolar expansion.

The circulating nurse is admitting a patient prior to surgery and proceeds to greet the patient and discuss what the patient can expect in surgery. What aspect of therapeutic communication should the nurse implement? A) Wait for the patient to initiate dialogue. B) Use medically acceptable terms. C) Give preoperative medications prior to discussion. D) Use a tone that decreases the patient's anxiety.

Ans: D Feedback: When discussing what the patient can expect in surgery, the nurse uses basic communication skills, such as touch and eye contact, to reduce anxiety. The nurse should use language the patient can understand. The nurse should not withhold communication until the patient initiates dialogue; the nurse most often needs to initiate and guide dialogue, while still responding to patient leading. Giving medication is not a communication skill.

A 21-year-old patient is positioned on the OR bed prior to knee surgery to correct a sports-related injury. The anesthesiologist administers the appropriate anesthetic. The OR nurse should anticipate which of the following events as the team's next step in the care of this patient? A) Grounding B) Making the first incision C) Giving blood D) Intubating

Ans: D Feedback: When the patient arrives in the OR, the anesthesiologist or anesthetist reassesses the patient's physical condition immediately prior to initiating anesthesia. The anesthetic is administered, and the patient's airway is maintained through an intranasal intubation, oral intubation, or a laryngeal mask airway. Grounding or blood administration does not normally follow anesthetic administration immediately. An incision would not be made prior to intubation.

A client is undergoing surgery for an appendectomy. This would be considered what type of surgery? a) Emergency surgery b) Diagnostic surgery c) Elective surgery d) Palliative surgery

Ans: A An appendectomy is considered emergency or urgent surgery. Elective surgery can be scheduled in advance, and delay has no ill effects. Palliative surgery is done to relieve or reduce the intensity of an illness, and diagnostic surgery is done to make or confirm a diagnosis.

Which of the following clients most likely requires special pre‐operative assessment and treatment as a result of his or her existing medication regimen? a) A woman who takes daily anticoagulants to treat atrial fibrillation b) A woman who takes daily thyroid supplements to treat her longstanding hypothyroidism c) A man who regularly treats his rheumatoid arthritis with over ‐the ‐counter nonsteroidal anti‐inflammatory drugs (NSAIDs) d) A man who takes an angiotensin‐converting enzyme (ACE) inhibitor because he has hypertension

Ans: A Anticoagulants present a risk of hemorrhage. This risk supersedes that posed by thyroid supplements, ACE inhibitors, or most NSAIDs.

A nurse from the ambulatory surgical center is preparing discharge instructions for a client who has had pelvic surgery. Which criterion would the client need to demonstrate to ensure that she is ready for discharge? a) Void normally b) Exhibit no bleeding c) Eat without nausea d) Verbalize absence of pain

Ans: A Before discharge from an ambulatory surgical unit, the client should be able to void normally after a pelvic surgery. It is natural for the client to experience pain after surgery; however, the client should also have the comfort level to control it. The client may not be in a position to eliminate nausea and vomiting completely before discharge, but should suffer minimally from them. The client may have some bleeding or drainage, which should not be excessive at the time of discharge from an ambulatory surgical unit.

A pediatric nurse is preparing a child for cleft palate repair surgery. The nurse recognizes that this type of surgery is categorized as which of the following? a) Constructive surgery b) Palliative surgery c) Transplantation surgery d) Reconstructive surgery

Ans: A Cleft palate repair is considered constructive surgery because the goal is to restore function in congenital anomalies. Reconstructive surgery serves to restore function to traumatized or malfunctioning tissues and includes plastic surgery or skin grafting. Transplant surgery replaces organs or structures that are diseased or malfunctioning, such as a liver or kidney transplant. Palliative surgery is not curative and seeks to relieve or reduce the intensity of an illness, such as debridement of necrotic tissue.

A client, scheduled for open-heart surgery, tells the nurse he does not want to be "saved" if he dies during surgery. What should the nurse do next? A. Discuss with and document the wishes of the client and family B. Administer the ordered oral and intravenous preoperative medications C. Notify the physician after completion of the surgical procedure D. Verbally report the client's wishes to the operating room supervisor

Ans: A Feedback: Advance directives allow the client to specify instructions for health care treatment if unable to communicate these wishes during or after surgery. It is important for the nurse to discuss and document exact do not resuscitate (DNR) wishes of the client and family before surgery.

A nurse in an outpatient surgical center is teaching a client about what will be necessary for discharge to home. What information should the nurse include about transportation? A. The client is not allowed to drive a car home. B. If the client is not dizzy, driving a car is allowed. C. Only adults over the age of 25 may drive home. D. None; this is not necessary information.

Ans: A Feedback: After outpatient surgery, clients may go home when they are no longer dizzy or drowsy, have stable vital signs, and have voided. Clients are not allowed to drive a car home.

Which of the following nursing interventions occurs in the postoperative phase of the surgical experience? A. Airway/oxygen therapy/pulse oximetry B. Teaching deep breathing exercises C. Reviewing the meaning of p.r.n. orders for pain medications D. Putting in IV lines and administering fluids

Ans: A Feedback: Airway/oxygen therapy/pulse oximetry occur in the postanesthesia unit in the postoperative phase. Teaching deep-breathing exercises and reviewing the meaning of p.r.n. orders for medications occur in the preoperative phase. Putting in IV lines and administering fluids occurs in the intraoperative phase.

A nurse is educating a preoperative client on how to cough effectively. What can the nurse tell the client to do to facilitate coughing A. "Hold a pillow or folded bath blanket over the incision." B. "Get up and walk before you try to cough." C. "It would be best if you do not cough until you feel better." D. "When you cough, cover your nose and mouth with a tissue."

Ans: A Feedback: Because postoperative coughing is often painful, the client should be taught how to splint the incision by supporting it with a pillow or folded bath blanket.

Occurs when contraction of the detrusor muscle pushes stored urine through the relaxed internal urethral sphincter into the urethra and out of the body:

voiding/urination/micturition

Upon assessment, a client reports that he drinks five to six bottles of beer every evening after work. Based upon this information, the nurse is aware that the client may require which of the following? A. Larger doses of anesthetic agents and larger doses of postoperative analgesics B. Larger doses of anesthetic agents and lower doses of postoperative analgesics C. Lower doses of anesthetic agents and lower doses of postoperative analgesics D. Lower doses of anesthetic agents and larger doses of postoperative analgesics

Ans: A Feedback: Clients with a large habitual intake of alcohol require larger doses of anesthetic agents and postoperative analgesics, increasing the risk for drug-related complications.

A client scheduled for major surgery will receive general anesthesia. Why is inhalation anesthesia often used to provide the desired actions? A. Rapid excretion and reversal of effects B. Safe administration in the client's own room C. Involves only the respiratory system and skin D. Slow onset of action and maintains reflexes

Ans: A Feedback: General anesthesia involves the administration of drugs by inhalation and intravenous routes to produce central nervous system depression. Inhalation anesthesia is often used because it has the advantage of rapid excretion and reversal of effects.

The telemetry unit nurse is reviewing laboratory results for a client who is scheduled for an operative procedure later in the day. The nurse notes on the laboratory report that the client has a serum potassium level of 6.5 mEq/L, indicative of hyperkalemia. The nurse informs the physician of this laboratory result because the nurse recognizes hyperkalemia increases the client's operative risk for which of the following? A. Cardiac problems B. Infection C. Bleeding and anemia D. Fluid imbalances

Ans: A Feedback: Hyperkalemia or hypokalemia increases the client's risk for cardiac problems. A decrease in the hematocrit and hemoglobin level may indicate the presence of anemia or bleeding. An elevated white blood cell count occurs in the presence of infection. Abnormal urine constituents may indicate infection or fluid imbalances.

The nurse is providing education to a client regarding pain control after surgery. What time does the nurse inform the client is the best time to request pain medication? A. Before the pain becomes severe B. When the client experiences a pain rating of "10" on a 1-to-10 pain scale C. When there is no pain, but it is time for the medication to be administered D. After the pain becomes severe and relaxation techniques have failed

Ans: A Feedback: If a pain medication is ordered p.r.n., the client should be instructed to ask for the medication before the pain becomes severe.

In order to prevent the possibility of venous stasis, a nurse is teaching a surgical client how to perform leg exercises. Which of the client's following statements indicates a sound understanding of leg exercises? A. "I'll practice these now and try to start them as soon as I can after my surgery." B. "I'll try to do these lying on my stomach so that I can bend my knees more fully." C. "I'll make sure to do these, as long as my doctor doesn't tell me to stay on bed rest after my operation." D. "I'm pretty sure my stomach muscles are strong enough to lift both of my legs off the bed at the same time."

Ans: A Feedback: Leg exercises should be begun as soon as possible after surgery, unless contraindications exist. Bed rest does not preclude the performance of leg exercises and the legs should be lifted individually, not simultaneously. The client should perform leg exercises in a semi-Fowler's, not prone, position.

A client returning to the floor after orthopedic surgery is complaining of nausea. The nurse is aware that an appropriate intervention is to do which of the following? A. Avoid strong smelling foods. B. Provide clear liquids with a straw. C. Avoid oral hygiene until the nausea subsides. D. Hold all medications.

Ans: A Feedback: Nursing care for a xlient with nausea includes avoiding strong smelling foods, providing oral hygiene, administering prescribed medications (especially medications ordered for nausea and vomiting), and avoiding use of a straw.

The nurse is preparing to send a client to the operating room for an exploratory laparoscopy. The nurse recognizes that there is no informed consent for the procedure on the client's chart. The nurse informs the physician who is performing the procedure. The physician asks the nurse to obtain the informed consent signature from the client. What is the nurse's best action to the physician's request? A. Inform the physician that it is his or her responsibility to obtain the signature. B. Obtain the signature and ask another nurse to cosign the signature. C. Inform the physician that the nurse manager will need to obtain the signature. D. Call the house officer to obtain the signature.

Ans: A Feedback: The responsibility for securing informed consent from the client lies with the person who will perform the procedure. The nurse's best action is to inform the physician that it is his or her responsibility to obtain the signature.

A postoperative home care client has developed thrombophlebitis in her right leg. What category of medications will probably be prescribed for this cardiovascular complication? A. Anticoagulants B. Antibiotics C. Antihistamines D. Antigens

Ans: A Feedback: Thrombophlebitis is an inflammation of a vein associated with thrombus formation. Thrombophlebitis from venous stasis is most commonly seen in the legs of postoperative clients. Nursing interventions include administering ordered medications, most often anticoagulants.

The nurse is preparing a patient for a colonoscopy. The nurse is familiar with the colonoscopy procedures at the hospital and is aware that which type of anesthesia is commonly used for this procedure? a) Conscious sedation b) Spinal anesthesia c) Nerve block d) Epidural anesthesia

Ans: A Moderate sedation/analgesia is also known as conscious sedation or procedural sedation. It is used for short‐term and minimally invasive procedures such as endoscopy procedures (e.g., colonoscopy).

Which of the following statements, if made by an adolescent preparing for abdominal surgery, would indicate to the nurse that the client requires additional instruction? a) "I can have a hamburger and French fries as soon as I wake up." b) "When I can eat again, the best meal would be steak and orange juice." c) "I might be sick to my stomach and throw up after surgery." d) "The better I eat before surgery, the more likely I will heal."

Ans: A Oral fluid and food may be withheld until intestinal motility resumes.

The nurse recognizes the value of leg exercises in the prevention of postoperative thrombophlebitis. When should the nurse teach the correct technique for leg exercises to a client a) Prior to surgery b) When early signs of venous stasis are evident c) In postanesthetic recovery d) Upon transfer from postanesthetic care unit to the postsurgical unit

Ans: A Though leg exercises are begun after surgery, such preventative measures should ideally be taught to the patient during the preoperative period.

Upon admission for an appendectomy, the client provides the nurse with a document that specifies instructions his health care team should follow in the event he is unable to communicate these wishes postoperatively. What is the document best known as? a) An informed consent b) An advance directive c) An insurance card d) A Patient's Bill of Rights

Ans: B An advance directive, a legal document, allows the client to specify instructions for his or her health care treatment should he or she be unable to communicate these wishes postoperatively. The advance directive allows the client to discuss his or her wishes with the family members in advance of the surgery. Two common forms of advance directives include living wills and durable powers of attorney for health care.

A client states he has a latex allergy. What action should the nurse take? a) Inform the client to tell the anesthesiologist. b) Place an allergy identification band on the client. c) Have the client take a Benadryl before surgery. d) Send the client to the OR with epinephrine.

Ans: B Assist the client with allergies to medications, food, and latex before the surgical procedure, and clearly mark them on the client record, and on the client identification band.

A client has been taking aspirin since his heart attack in 1997. The client is at risk for what? a) Infection b) Hemorrhage c) Thrombophlebitis d) Blood clots

Ans: B Current medication use, especially use of medications that can affect coagulation status (warfarin, nonsteroidal anti‐inflammatory drugs, aspirin) is important and should be reported to the surgeon.

A female client is scheduled for liposuction surgery to reduce her weight. Based on urgency, how is this surgery classified? A. Urgent B. Elective C. Emergency D. Emergent

Ans: B Feedback: A liposuction procedure is classified as elective surgery, in which the procedure is preplanned and based on the client's choice. Other classifications are urgent (surgery is necessary for the client's health but not an emergency) and emergency (the surgery must be done immediately to preserve life, body part, or body function).

Which of the following interventions is of major importance during preoperative education? A. Performing skills necessary for gastrointestinal preparation B. Encouraging the client to identify and verbalize fears C. Discussing the site and extent of the surgical incision D. Telling the client not to worry or be afraid of surgery

Ans: B Feedback: A surgical procedure causes anxiety and fear. The nurse should encourage the client to identify and verbalize fears; often simply talking about fears helps to diminish their magnitude.

A diabetic client is undergoing surgery to amputate a gangrenous foot. This procedure would be considered which of the following categories of surgery based on purpose? A. Diagnostic B. Ablative C. Palliative D. Reconstructive

Ans: B Feedback: Ablative surgery is performed to remove a diseased body part. Diagnostic surgery is performed to make or confirm a diagnosis. Palliative surgery involves relieving or reducing intensity of an illness. Reconstructive surgery restores function to traumatized or malfunctioning tissue.

Which of the following interventions are recommended guidelines for meeting client postoperative elimination needs A. Assess abdominal distention, especially if bowel sounds are audible or are low pitched. B. Assess for the return of peristalsis by auscultating bowel sounds every four hours when the client is awake. C. Encourage food and fluid intake when ordered, especially dairy products and low-fiber foods. D. Assess for bladder distention by Palpating below the symphysis pubis if the client has not voided within eight hours after surgery.

Ans: B Feedback: Assess for the return of peristalsis by auscultating bowel sounds every four hours when the client is awake. Assess abdominal distention, especially if bowel sounds are inaudible or are high pitched. Encourage food and fluid intake when ordered, especially fruit juices and high-fiber foods. Assess for bladder distention by palpating above the symphysis pubis if the client has not voided within eight hours after surgery.

A nurse is assisting a physician during a cesarean section for a client. The client is administered epidural anesthesia. Which of the following is an advantage of epidural anesthesia? A. It counteracts the effects of conscious sedation. B. It decreases the risk of gastrointestinal complications. C. It prevents clients from remembering the initial recovery period. D. It acts on the central nervous system to produce loss of sensation.

Ans: B Feedback: Epidural anesthesia is a regional anesthesia administered to a client before surgery; it decreases the risk of gastrointestinal complications in clients. Reversal drugs are medications that counteract the effects of those used for conscious sedation. General anesthesia acts on the central nervous system to produce loss of sensation; it prevents clients from remembering their initial recovery period.

A nurse is providing ongoing postoperative care to a client who has had knee surgery. The nurse assesses the dressing and finds it saturated with blood. The client is restless and has a rapid pulse. What should the nurse do next? A. Document the data and apply a new dressing. B. Apply a pressure dressing and report findings. C. Reassure the family that this is a common problem. D. Make assessments every 15 minutes for four hours.

Ans: B Feedback: Hemorrhage is an excessive internal or external loss of blood. Common indications of hemorrhage include a rapid, thready pulse. If bleeding occurs, the nurse should apply a pressure dressing to the site, report findings to the physician, and be prepared to return the client to the operating room if bleeding cannot be stopped or is massive.

A young woman has been in an automobile crash that resulted in an amputation of her left lower leg. She verbalizes grief and loss. What knowledge by the nurse is used to provide interventions to help the client cope? A. The client should be grateful to be alive. B. This is a normal, appropriate response. C. This is an abnormal, inappropriate response. D. Tissue healing will help the client adapt.

Ans: B Feedback: Many surgical clients have the same reaction to loss of a body part as they would to a death. A surgical client's grief is a normal, appropriate response. The nurse must be aware of the client's needs and provide interventions to meet those needs in coping with change.

A nurse is educating a surgical client on postoperative p.r.n. pain control. Which of the following should be included? A. "We will bring you pain medications; you don't need to ask." B. "Even if you have pain, you may get addicted to the drugs." C. "You won't have much pain so just tough it out." D. "You need to ask for the medication before the pain becomes severe."

Ans: D Feedback: If medication for pain is ordered p.r.n., there is a time restriction between doses. The client needs to ask for the medication and should do so before the pain becomes severe.

A nurse is taking care of a client during the immediate post-operative period. Which of the following duties performed during the immediate post-operative period is most important? A. Ensure the safe recovery of surgical clients. B. Monitor the client for complications. C. Prepare a room for the client's return. D. Assess the client's health constantly.

Ans: B Feedback: The immediate post-operative period refers to the first 24 hours after surgery. During this time, the nurse monitors the client for complications as he or she recovers from anesthesia. Once the client is stable, the nurse prepares a room for the client's return and assesses the client to prevent or minimize potential complications. The nurse ensures the safe recovery of the client after the client has stabilized.

A nurse has been asked to ensure informed consent for a surgical procedure. What might be a role of the nurse A. Securing informed consent from the client B. Signing the consent form as a witness C. Ensuring the client does not refuse treatment D. Refusing to participate based on legal guidelines

Ans: B Feedback: The responsibility for securing informed consent from the client lies with the person who will perform the procedure, usually the physician. The nurse may sign as a witness, signifying that the client signed the consent form without coercion, and was alert and aware of the act.

What is the rationale for having the client void before surgery? a) To assess for pregnancy in women b) To prevent bladder distention c) To prevent electrolyte imbalance d) To assess for urinary tract infection

Ans: B Having the client void before surgery will assist in the prevention of bladder distention during or after the procedure.

The nurse has entered the room of a client who is postoperative day one and finds the client grimacing and guarding her incision. The client refuses the nurse's offer of PRN analgesia, and on discussion, states that this refusal is motivated by his fear of becoming addicted to pain medications. How should the nurse respond to the client's concerns? a) "The hospital has excellent resources for dealing with any addiction that might result from the medications you take to control your pain." b) "Research has shown that there is very little risk of clients becoming addicted to painkillers after they have surgery." c) "Actually, people who are not addicted to drugs before their surgery never develop a tolerance or addiction during their recovery." d) "You should remind yourself that treating your pain is important now, and that dealing with any resulting dependency can come later."

Ans: B There is little danger of addiction to pain medications used in the postoperative management of pain.

A nurse is assisting a physician in an emergency surgery for a client with intestinal perforation. Which of the following descriptions is most suitable to the type of surgery performed a) Surgery performed at the client's request b) Surgery required within one or two days c) Surgery required immediately for survival d) Surgery planned as per client's convenience

Ans: C An emergency surgery is a surgery required immediately for survival. Elective surgery is planned at the client's convenience; whereas, an optional surgery is performed at the client's request. When urgent surgery is required, it is necessary and done within one or two days.

A physician has ordered a nurse to administer conscious sedation to a client. Which of the following is possible after administering conscious sedation to a client? A. Client can respond verbally despite physical immobility. B. Client can tolerate long therapeutic surgical procedures. C. Client is relaxed, emotionally comfortable, and conscious. D. Client's consciousness level can be monitored by equipment.

Ans: C Feedback: Conscious sedation refers to a state in which the client is sedated in a state of relaxation and emotional comfort, but is not unconscious. The client is free of pain, fear, and anxiety and can tolerate unpleasant diagnostic and short therapeutic surgical procedures, such as an endoscopy or bone marrow aspiration. The client can respond verbally and physically. However, no equipment can replace a nurse's careful observations for monitoring clients.

A nurse is caring for a client who is scheduled to undergo a breast biopsy. Which of the following major tasks does the nurse perform immediately during the pre-operative period? A. Obtain a signature on the consent form. B. Review the surgical checklist. C. Conduct a nursing assessment. D. Reduce the dosage of toxic drugs.

Ans: C Feedback: During the immediate pre-operative period, the nurse conducts a nursing assessment. Nurses obtain the signature of the client, nearest blood relative, or someone with durable power of attorney before the administration of any pre-operative sedatives. They also administer medications as ordered by the physician regardless of their toxicity. They assist the client with psychosocial preparation and complete the surgical checklist, which is reviewed by the operating room personnel.

A nurse is reviewing results of preoperative screening tests and notes the client's potassium level is dangerously low. What should the nurse do next? A. Nothing; potassium levels have no influence on surgical outcome. B. Include the information in the postoperative end of shift report. C. Document the data and notify the physician who will do the surgery. D. Ask the client and family members why the potassium is low.

Ans: C Feedback: Either high or low levels of potassium put the surgical client at increased risk for cardiac problems during and after surgery. The nurse's role includes recording the data in the client's record and reporting abnormal findings.

A preoperative assessment finds a client to be 75 pounds overweight. The client is to have abdominal surgery. What nursing diagnosis would be appropriate based on the client's weight? A. Risk for Aspiration B. Risk for Imbalanced Body Temperature C. Risk for Infection D. Risk for Falls

Ans: C Feedback: Fatty tissue in obese clients has a poor blood supply and, therefore, has less resistance to infections. Postoperative complications of delayed wound healing, wound infection, and disruption of the wound are more common in obese clients.

An operating room nurse is preparing for a surgical procedure on an infant. The nurse's perioperative care is based on what physiologic factor that puts infants at greater risk from surgery than adults? A. Increased vascular rigidity B. Diminished chest expansion C. Lower total blood volume D. Decreased peripheral circulation

Ans: C Feedback: Infants are at a greater risk from surgery as a result of various physiologic factors. A major factor is that the infant has a lower total blood volume, making even a small loss of blood a serious consideration because of the risk for dehydration and the inability to respond to the need for increased oxygen during surgery.

The community health nurse is preparing for a family planning clinic. The nurse knows the following factors influence contraceptive choices

• Age • Marital status • Desire for future pregnancy • Religious beliefs • Level of education

A female client age 54 years has been scheduled for a bunionectomy (removal of bone tissue from the base of the great toe) which will be conducted on an ambulatory basis. Which of the following characteristics applies to this type of surgery? a) The surgery is classified as urgent rather than elective. b) The client must be previously healthy with low surgical risks. c) The client will be admitted the day of surgery and return home the same day. d) The surgery will be conducted using moderate sedation rather than general anesthesia.

Ans: C Outpatient surgeries, also known as ambulatory surgeries, are conducted with admission and discharge on the same day. Such surgeries have become increasingly common in recent years, and some surgeries of increasing complexity and risk are conducted on an outpatient basis. General anesthesia is possible, and common. This approach is more common for elective surgeries than urgent surgeries.

A nurse is reinforcing wound edges and applying a blinder to the separated incisions of a client after a surgery. Which of the following postoperative complications has the client developed a) Hypoxemia b) Evisceration c) Dehiscence d) Shock

Ans: C The nurse is taking care of a client with dehiscence. Hypoxemia develops when there is inadequate oxygenation of blood. Evisceration occurs when there is protrusion of abdominal organs through a separated wound. A client has shock when there is inadequate blood flow.

A nurse is caring for an older adult client who has been prescribed fluid restriction before surgery. Which of the following should the nurse check to assess the risks of fluid restriction in elderly clients? a) Anxiety level b) Self‐therapy c) Vital signs d) Cardiac status

Ans: C The nurse should assess the client's vital signs, weight, and sternal skin turgor prior to fluid restriction to serve as a baseline for comparison. The period of fluid restriction before surgery may be shortened for older adults to reduce their risk of dehydration and hypotension. Nurses check a client's self‐therapy practices and cardiac status to avoid any complications of bleeding and elimination of intravenous fluids given at a standard rate.

A client has arrived in the same‐day surgery suite. He states to the nurse, "I am so worried about being put to sleep and having the surgery." What would be the nurse's best response? a) "Have you ever had surgery before?" b) "I will have the anesthesiologist talk to you." c) "Tell me what you are most worried about." d) "You don't have to worry. It will be fine."

Ans: C The nurse should first assess what the client is most worried about, and then provide emotional support.

A nurse is preparing to receive a client in post‐anesthesia care unit (PACU). The client is diabetic and has undergone knee surgery. Which information would be most important for the receiving nurse to obtain to develop an appropriate plan of care for this client? a) Chronic disease history b) Environment of the operating room c) Amount of blood loss d) Information about allergic agents

Ans: C To plan care effectively in the postoperative period, the nurse needs to know about the amount of blood lost during the surgery, the type of surgery that was performed on the client, and whether there were any surgical or anesthetic complications. Information on chronic disease history and allergy history are done in the pre‐operative period, not in the postoperative period. Information on the environment in the operation room is checked by the circulatory nurse during the intra‐ operative care plan; it is not associated with the postoperative care plan.

In the postoperative phase of abdominal surgery, the client complains of severe abdominal pain. In the second postoperative day, the client's bowel sounds are absent. What does the nurse suspect? a) Abdominal infection b) Hernia development c) Normal response d) Paralytic ileus

Ans: D A potential complication after surgery is paralytic ileus, a condition in which there is decreased bowel functioning.

A nurse is preparing a client for endotracheal intubation. The anesthesiologist has ordered an anticholinergic medication for this client. Which of the following is an action of this medication? a) It promotes induction of anesthesia. b) It promotes sleep or conscious sedation. c) It decreases gastric acidity and volume. d) It decreases respiratory secretions.

Ans: D An anticholinergic medication decreases respiratory secretions and prevents vagal nerve stimulation during endotracheal intubation. Antianxiety drugs slow motor activity and promote the induction of anesthesia. Histamine‐2 receptor antagonists decrease gastric acidity and volume. Sedatives promote sleep or conscious sedation.

A nurse teaches deep breathing exercises to a client scheduled for surgery. In which of the following perioperative phases would this action occur? a) None of the above b) Intraoperative c) Postoperative d) Pre‐operative

Ans: D Exercises and physical activities occurring in the pre‐operative phase include deep‐breathing exercises, coughing, incentive spirometry, turning, leg exercises, and pneumatic compression stockings.

Which statement accurately represents a recommended guideline when providing postoperative care for the following clients? A. Force fluids for an adult client who has a urine output of less that 30 mL per hour. B. If client is febrile within 12 hours of surgery, notify the physician immediately. C. If the dressing was clean but now has a large amount of fresh blood, remove the dressing and reapply it. D. If vital signs are progressively increasing or decreasing from baseline, notify the physician of possible internal bleeding.

Ans: D Feedback: A continued decrease in blood pressure or an increase in heart rate could indicate internal bleeding, and the physician should be notified. If an adult client has a urine output of less than 30 mL per hour, the physician should be notified, unless this is expected. If the client is febrile within 12 hours of surgery, the nurse should assist the client with coughing and deep-breathing exercises. When large amounts of fresh blood are present, the dressing should be reinforced with more bandages and the physician notified.

A client with abdominal incisions experiences excruciating pain when he tries to cough. What should the nurse do to reduce the client's discomfort when coughing A. Administer prescribed pain medication just before coughing. B. Ask the client to drink plenty of water before coughing. C. Ask the client to lie in a lateral position when coughing. D. Administer prescribed pain medication 30 minutes before deliberately attempting to cough.

Ans: D Feedback: Coughing is painful for clients with abdominal or chest incisions. Administering pain medication approximately 30 minutes before coughing, or splinting the incision when coughing, can reduce discomfort. Making the client lie in a lateral position or asking the client to drink plenty of water is not helpful because it will make breathing and coughing even more difficult for the client.

A cleansing enema is ordered for a client who is scheduled to have colon surgery. What is the rationale for this procedure? A. Surgical clients routinely are given a cleansing enema. B. Cleansing enemas are given before surgery at the client's request. C. There will be less flatus and discomfort postoperatively. D. Peristalsis does not return for 24 to 48 hours after surgery.

Ans: D Feedback: If the client is scheduled for gastrointestinal tract surgery, a cleansing enema is usually ordered. Peristalsis does not return for 24 to 48 hours after the bowel is handled, so preoperative cleansing helps decrease postoperative constipation.

A nurse working in a PACU is responsible for conducting assessments on immediate postoperative clients. What is the purpose of these assessments? A. To determine the length of time to recover from anesthesia B. To use intraoperative data as a basis for comparison C. To focus on cardiovascular data and findings D. To prevent complications from anesthesia and surgery `

Ans: D Feedback: Immediate postoperative care in the PACU involves assessing the postoperative client with emphasis on preventing complications from the surgery.

A student is assessing a postoperative client who has developed pneumonia. The plan of care includes positioning the client in the Fowler's or semi-Fowler's position. What is the rationale for this position? A. It increases blood flow to the heart. B. The client will be more comfortable and have less pain. C. It facilitates nursing assessments of skin color and temperature. D. It promotes full aeration of the lungs.

Ans: D Feedback: Pneumonia may occur in the postoperative client from aspiration, immobilization, depressed cough reflex, infection, increased secretions from anesthesia, or dehydration. Nursing interventions include positioning the client in the Fowler or semi-Fowler position to promote full aeration of the lungs.

A nurse is educating a client about regional anesthesia. Which of the following statements is accurate about this type of anesthesia? A. "You will be asleep and won't be aware of the procedure." B. "You will be asleep but may feel some pain during the procedure." C. "You will be awake but will not be aware of the procedure." D. "You will be awake and will not have sensation of the procedure."

Ans: D Feedback: Regional anesthesia occurs when an anesthetic agent is injected near a nerve or nerve pathway in or around the operative site, inhibiting the transmission of sensory stimuli to central nervous system receptors. The client remains awake but loses sensation in a specific area or region of the body.

After conducting a preoperative health assessment, the nurse documents that the client has physical assessments supporting the medical diagnosis of emphysema. Based on this finding, what postoperative interventions would be included on the plan of care? A. Perform sterile dressing changes each morning. B. Administer pain medications as needed. C. Conduct a head-to-toe assessment each shift. D. Monitor respirations and breath sounds.

Ans: D Feedback: Respiratory disorders, including emphysema, increase the risk for respiratory depression from anesthesia as well as postoperative pneumonia and atelectasis.

A nurse is assisting a postoperative client with deep-breathing exercises. Which of the following is an accurate step for this procedure? A. Place the client in prone position, with the neck and shoulders supported. B. Ask the client to place the hands over the stomach, so he or she can feel the chest rise as the lungs expand. C. Ask the client to exhale rapidly and completely, and inhale through the nose rapidly and completely. D. Ask the client to hold his or her breath for three to five seconds and mentally count "one, one thousand, two, one thousand" and so forth.

Ans: D Feedback: The nurse should place the client in semi-Fowler's position, with the neck and shoulders supported, and ask the client to place the hands over the rib cage, so he or she can feel the chest rise as the lungs expand. Then, ask the patient to exhale gently and completely, inhale through the nose gently and completely, hold his or her breath for three to five seconds, and mentally count "one, one thousand, two, one thousand" etc., then exhale as completely as possible through the mouth with lips pursed (as if whistling).

The PACU nurse has received a semiconscious client from the operating room and reviews the chart for orders related to positioning of the client. There are no specific orders on the chart related to the client's position. In this situation, in what position will the nurse place the patient? a) Supine position b) Trendelenburg position c) Prone position d) Side‐lying position

Ans: D If the client is not fully conscious, place the client in the side‐lying position, unless there is an ordered position on the client's chart.

The operating room nurse is aware that which of the following clients is at a greater risk related to a surgical procedure? a) A man 48 years of age b) A boy 8 years of age c) A woman 34 years of age d) A woman 83 years of age

Ans: D Infants and older adults are at a greater risk from surgery than are children and young or middle‐age adults. Physiologic changes associated with aging increase the surgical risk for older clients.

A scrub nurse is assisting a surgeon with a kidney transplant. What are the patient responsibilities of this surgical team member? Select all that apply. a. Maintaining sterile technique b. Draping and handling instruments and supplies c. Identifying and assessing the patient on admission d. Integrating case management e. Preparing the skin at the surgical site f. Providing exposure of the operative area

Ans: a, b. The scrub nurse is a member of the sterile team who maintains sterile technique while draping and handling instruments and supplies. Two duties of the circulating nurse are to identify and assess the patient on admission to the operating room and prepare the skin at the surgical site. The RNFA actively assists the surgeon by providing exposure of the operative area. The APRN coordinates care activities, collaborates with physicians and nurses in all phases of perioperative and post anesthesia care, and integrates case management, critical paths, and research into care of the surgical patient.

While assessing a patient in the PACU, a nurse notes increased wound drainage, restlessness, a decreasing blood pressure, and an increase in the pulse rate. The nurse interprets these findings as most likely indicating: a. Thrombophlebitis b. Atelectasis c. Infection d. Hemorrhage

Ans: d. Increased wound drainage, restlessness, decreasing blood pressure, and increasing pulse rate are assessment findings that indicate hemorrhage. Manifestations of thrombophlebitis are pain and cramping in the calf or thigh of the involved extremity, redness and swelling in the affected area, elevated temperature, and an increase in the diameter of the involved extremity. Manifestations of atelectasis include decreased lung sounds over the affected area, dyspnea, cyanosis, crackles, restlessness, and apprehension. Signs of infection include elevated white blood count and fever.

A responsibility of the nurse is the administration of preoperative medications to patients. Which statements describe the action of these medications? Select all that apply. a. Diazepam is given to alleviate anxiety. b. Ranitidine is given to facilitate patient sedation. c. Atropine is given to decrease oral secretions. d. Morphine is given to depress respiratory function. e. Cimetidine is given to prevent laryngospasm. f. Fentanyl citrate-droperidol is given to facilitate a sense of calm

Ans: a, c, f. Sedatives, such as diazepam (Valium), midazolam (Versed), or lorazepam (Ativan) are given to alleviate anxiety and decrease recall of events related to surgery. Anticholinergics, such as atropine and glycopyrrolate (Robinul) are given to decrease pulmonary and oral secretions and to prevent laryngospasm. Neuroleptanalgesic agents, such as fentanyl citrate-droperidol (Innovar) are given to cause a general state of calm and sleepiness. Histamine-2 receptor blockers, such as cimetidine (Tagamet) and ranitidine (Zantac) are given to decrease gastric acidity and volume. Narcotic analgesics, such as morphine, are given to facilitate patient sedation and relaxation and to decrease the amount of anesthetic agent needed.

A patient had a surgical procedure that necessitated a thoracic incision. The nurse anticipates that he will have a higher risk for postoperative complications involving which body system? a. Respiratory system b. Circulatory system c. Digestive system d. Nervous system

Ans: a. A thoracic incision makes it more painful for the patient to take deep breaths or cough. Shallow respirations and ineffective coughing increase the risk for respiratory complications.

A 72-year-old woman who is scheduled for a hip replacement is taking several medications on a regular basis. Which drug category might create a surgical risk for this patient? a. Anticoagulants b. Antacids c. Laxatives d. Sedatives

Ans: a. Anticoagulant drug therapy would increase the risk for hemorrhage during surgery

A nurse has been asked to witness a patient signature on an informed consent form for surgery. For which of these patients would the document be valid? Select all that apply. a. A 92-year-old patient who is severely confused b. A 45-year-old patient who is oriented and alert c. A 10-year-old patient who is oriented and alert d. A 36-year-old patient who has had a narcotic premedication e. A 45-year-old mentally ill patient who has been ruled incompetent f. A 22-year old patient having an abortion against her partner's wishes

Ans: b, f. A consent form is not legal if the patient signing the form is confused, sedated, unconscious, or a minor.

A nurse is caring for an obese patient who has had surgery. The nurse monitors this patient for what postoperative complication? a. Hunger b. Impaired wound healing c. Hemorrhage d. Gas pains

Ans: b. Fatty tissue is less vascular and, therefore, less resistant to infection and more prone to delayed wound healing.

A patient tells the nurse she is having pain in her right lower leg. How does the nurse assess for the presence of thrombophlebitis? a. By palpating the skin over the tibia and fibula b. By documenting daily calf circumference measurements c. By recording vital signs obtained four times a day d. By noting difficulty with ambulation

Ans: b. Inflammation from thrombophlebitis increases the size of the affected extremity and can be assessed by measuring circumference on a regular basis.

Older adults often have reduced vital capacity as a result of normal physiologic changes. Which nursing intervention would be most important for the postoperative care of an older surgical patient specific to this change? a. Take and record vital signs every shift. b. Turn, cough, and deep breathe every 4 hours. c. Encourage increased intake of oral fluids. d. Assess bowel sounds daily.

Ans: b. Reduced vital capacity in older adults increases the risk for respiratory complications, including pneumonia and atelectasis. Having the patient turn, cough, and deep breathe every 4 hours maintains respiratory function and helps to prevent complications.

A 70-year-old male is scheduled for surgery. He says to the nurse, "I am so frightened—what if I don't wake up?" What would be the nurse's best response? a. "You have a wonderful doctor." b. "Let's talk about how you are feeling." c. "Everyone wakes up from surgery!" d. "Don't worry, you will be just fine."

Ans: b. This answer allows the patient to talk about his feelings and fears, and is therapeutic.

A nurse is applying a pneumatic compression device to a client. What is the purpose of a pneumatic compression device? a) Enables the client to void b) Reduces swelling and pain c) Promotes circulation of venous blood d) Pumps liquid diet to the client

Ans: c A pneumatic compression device promotes the circulation of venous blood and relocation of excess fluid into the lymphatic vessels. The device does not pump a liquid diet to the client, help the client to void, or reduce swelling and pain.

A nurse is preparing a patient for a cesarean section and teaches her the effects of the regional anesthesia she will be receiving. Which effects would the nurse expect? Select all that apply. a. Loss of consciousness b. Relaxation of skeletal muscles c. Reduction or loss of reflex action d. Localized loss of sensation e. Prolonged pain relief after other anesthesia wears off f. Infltrates the underlying tissues in an operative area

Ans: c, d. A localized loss of sensation and possible loss of reflexes occurs with a regional anesthetic. Loss of consciousness and relaxation of skeletal muscles occurs with general anesthesia. Prolonged pain relief after other anesthesia wears off and infiltration of the underlying tissues in an operative area occur with topical anesthesia.

A perioperative nurse is preparing a patient for surgery for treatment of a ruptured spleen as the result of an automobile crash. The nurse knows that this type of surgery belongs in what category? a. Minor, diagnostic b. Minor, elective c. Major, emergency d. Major, palliative

Ans: c. This surgery would involve a major body organ, has the potential for postoperative complications, requires hospitalization, and must be done immediately to save the patient's life. Elective surgery is a procedure that is preplanned by essentially healthy people. Diagnostic surgery is performed to confirm a diagnosis. Palliative surgery is not curative, rather it is done to relieve or reduce the intensity of an illness.

A nurse is explaining the rationale for performing leg exercises after surgery. Which reason would the nurse include in the explanation? a. Promote respiratory function b. Maintain functional abilities c. Provide diversional activities d. Increase venous return

Ans: d. Leg exercises in the postoperative period increase venous return. As a result, the patient has a decreased risk for thrombophlebitis and emboli.

A nurse is explaining pain control methods to a patient undergoing a bowel resection. The patient is interested in the PCA pump and asks the nurse to explain how it works. What would be the nurse's correct response? a. "The pump allows the patient to be completely free of pain during the postoperative period." b. "The pump allows the patient to take unlimited amounts of medication as needed." c. "The pump allows the patient to choose the type of medication given postoperatively." d. "The pump allows the patient to self-administer limited doses of pain medication."

Ans: d. PCA infusion pumps allow patients to self-administer doses of pain-relieving medication within physician-prescribed time and dose limits. Patients activate the delivery of the medication by pressing a button on a cord connected to the pump or a button directly on the pump.

A nurse is teaching a man scheduled to have same-day surgery. Which teaching method would be most effective in preoperative teaching for ambulatory surgery? a. Lecture b. Discussion c. Audiovisuals d. Written instructions

Ans: d. Written instructions are most effective in providing information for same-day surgery.

A male client reports having impotence. The nurse examines the client's medication regimen and determines that a contributing factor to impotence could be:

Antihypertensives

Which statement is true of gender identity? A) It is the opposite of biologic identity. B) It may be the same as or different from biologic identity. C) It is determined by male (XY) or female (XX) chromosomes. D) It guides ones gender role behavior as male or female.

B) It may be the same as or different from biologic identity.

What is the most significant difficulty regarding sexuality faced by people taking medications for hypertension? A) Medications result in increased desire for sex. B) Medications change sexual functioning. C) Patients experience a growth of body hair. D) Patients experience increased body odors.

B) Medications change sexual functioning.

A male patient tells the nurse that he does not understand why he feels the way he does when he is sexually excited. What would the nurse teach the patient? A) I don't know, but I will ask my boyfriend if he can describe his feelings to me. B) The sexual response cycle includes excitement, plateau, orgasm, and resolution. C) That is something that just happens and nobody knows why. D) Isn't sex wonderful? I think it has different parts to the experience.

B) The sexual response cycle includes excitement, plateau, orgasm, and resolution.

A heterosexual couple enjoys anal intercourse. What may result from this type of intercourse? A) feelings of guilt and shame B) vaginal infections C) damage to the vagina D) penile infections

B) vaginal infections

A nurse is providing ongoing postoperative care to a client who has had knee surgery. The nurse assesses the dressing and finds it saturated with blood. The client is restless and has a rapid pulse. What should the nurse do next? A. Document the data and apply a new dressing. B. Apply a pressure dressing and report findings. C. Reassure the family that this is a common problem. D. Make assessments every 15 minutes for four hours.

B. Apply a pressure dressing and report findings. Feedback: Hemorrhage is an excessive internal or external loss of blood. Common indications of hemorrhage include a rapid, thready pulse. If bleeding occurs, the nurse should apply a pressure dressing to the site, report findings to the physician, and be prepared to return the client to the operating room if bleeding cannot be stopped or is massive.

Which of the following interventions are recommended guidelines for meeting client postoperative elimination needs? A. Assess abdominal distention, especially if bowel sounds are audible or are low pitched. B. Assess for the return of peristalsis by auscultating bowel sounds every four hours when the client is awake. C. Encourage food and fluid intake when ordered, especially dairy products and low-fiber foods. D. Assess for bladder distention by Palpating below the symphysis pubis if the client has not voided within eight hours after surgery.

B. Assess for the return of peristalsis by auscultating bowel sounds every four hours when the client is awake. Feedback: Assess for the return of peristalsis by auscultating bowel sounds every four hours when the client is awake. Assess abdominal distention, especially if bowel sounds are inaudible or are high pitched. Encourage food and fluid intake when ordered, especially fruit juices and high-fiber foods. Assess for bladder distention by palpating above the symphysis pubis if the client has not voided within eight hours after surgery.

A female client is scheduled for liposuction surgery to reduce her weight. Based on urgency, how is this surgery classified? A. Urgent B. Elective C. Emergency D. Emergent

B. Elective Feedback: A liposuction procedure is classified as elective surgery, in which the procedure is preplanned and based on the client's choice. Other classifications are urgent (surgery is necessary for the client's health but not an emergency) and emergency (the surgery must be done immediately to preserve life, body part, or body function).

Which of the following interventions is of major importance during preoperative education? A. Performing skills necessary for gastrointestinal preparation B. Encouraging the client to identify and verbalize fears C. Discussing the site and extent of the surgical incision D. Telling the client not to worry or be afraid of surgery

B. Encouraging the client to identify and verbalize fears Feedback: A surgical procedure causes anxiety and fear. The nurse should encourage the client to identify and verbalize fears; often simply talking about fears helps to diminish their magnitude.

A nurse is assisting a physician during a cesarean section for a client. The client is administered epidural anesthesia. Which of the following is an advantage of epidural anesthesia? A. It counteracts the effects of conscious sedation. B. It decreases the risk of gastrointestinal complications. C. It prevents clients from remembering the initial recovery period. D. It acts on the central nervous system to produce loss of sensation

B. It decreases the risk of gastrointestinal complications. Feedback: Epidural anesthesia is a regional anesthesia administered to a client before surgery; it decreases the risk of gastrointestinal complications in clients. Reversal drugs are medications that counteract the effects of those used for conscious sedation. General anesthesia acts on the central nervous system to produce loss of sensation; it prevents clients from remembering their initial recovery period.

A nurse is taking care of a client during the immediate post-operative period. Which of the following duties performed during the immediate post-operative period is most important? A. Ensure the safe recovery of surgical clients. B. Monitor the client for complications. C. Prepare a room for the client's return. D. Assess the client's health constantly.

B. Monitor the client for complications. Feedback: The immediate post-operative period refers to the first 24 hours after surgery. During this time, the nurse monitors the client for complications as he or she recovers from anesthesia. Once the client is stable, the nurse prepares a room for the client's return and assesses the client to prevent or minimize potential complications. The nurse ensures the safe recovery of the client after the client has stabilized.

The nurse is teaching a class about sexuality at a public health clinic. After class the nurse is approached by John, a 22-year-old male, who eventually discloses to the nurse that, while he enjoys having sex with his girlfriend, he also has experienced pleasurable sex with males. The term for this behavior is which of the following?

Bisexual

A nurse is caring for an adolescent girl experiencing menstruation for the first time. What are the changes that occur in the body at this stage that the nurse should share with the adolescent girl?

Breast development

Which of the following questions or statements would be most useful in eliciting information about a patients sexual history? A) I know this is hard to talk about, but it is important. B) Why did you have unprotected sex? C) How would you describe your problem? D) I need to know sex partners numbers.

C) How would you describe your problem?

A nurse is teaching a student nurse how STIs affect the health of their patients. Which of the following statements accurately describes an effect of an STI? A) STIs are most common in young to middle adulthood populations. B) The incidence of STIs is decreasing due to health promotion efforts. C) Most of the time STIs cause no symptoms, especially in women. D) Health problems caused by STIs are more severe and frequent in men.

C) Most of the time STIs cause no symptoms, especially in women.

While answering questions posed by a nurse during a health history, a young woman says, Before my period I get headaches, am moody, and my breasts hurt. What is the patient experiencing? A) perimenopause B) menarche C) PMS D) menses

C) PMS

A nurse is responding to sexual harassment from a patient at work. Which of the following are recommended guidelines for dealing with this behavior? A) If confronted by management, deny any feelings about being harassed. B) Do not confront the person harassing you in person. C) Set and enforce limits to the behavior and maintain boundaries. D) Document the incident but do not report it to the supervisor unless harassment continues.

C) Set and enforce limits to the behavior and maintain boundaries.

A 70-year-old woman tells the nurse that she is still sexually active. How would the nurse respond? A) You are too old for that kind of behavior. B) Tell me what you enjoy the most. C) You can be sexually active as long as you want to be. D) There comes a time in life when this is no longer important.

C) You can be sexually active as long as you want to be.

As a result of loss of estrogen, postmenopausal women have decreased vaginal lubrication. What may occur as a result? A) frequent urination B) excess fatigue C) painful intercourse D) painful menses

C) painful intercourse

What are the primary nursing considerations when assisting with or conducting a physical assessment of the genitalia? A) ensuring sterility of all equipment and supplies B) respecting the patients privacy and modesty C) providing a means for cleansing the area D) leaving the room during the assessment

C) providing a means for cleansing the area

2.The nurse is collecting a stool specimen. Arrange the following steps in the order in which the nurse should perform them. Label the steps from A to D, with A being the first step to perform. A. Have the patient defecate into a special container placed under the toilet seat. B. Put on gloves and place the specimen in a specimen container. C. Ask the patient to void to empty the bladder. D. Place a label on the specimen container.

C, A, B, D The nurse should ask the patient to void and then have him defecate into a special container placed under the toilet seat. Next, the nurse should put on gloves and, using a tongue blade, place the specimen into the container. Finally, she should label the specimen and send it to the laboratory for analysis. PTS: 1 DIF: Difficult REF: pp. 989-990 KEY: Nursing process: Interventions | Client need: SECE | Cognitive level: Analysis

Which outcome is appropriate for the patient who underwent urinary diversion surgery and creation of an ileal conduit for invasive bladder cancer? 1) Patient will resume his normal urination pattern by (target date). 2) Patient will perform urostomy self-care by (target date). 3) Patient will perform self-catheterization by (target date). 4) Patients urine will remain clear with sufficient volume.

Patient will perform urostomy self care by (target date)

A physician has ordered a nurse to administer conscious sedation to a client. Which of the following is possible after administering conscious sedation to a client? A. Client can respond verbally despite physical immobility. B. Client can tolerate long therapeutic surgical procedures. C. Client is relaxed, emotionally comfortable, and conscious. D. Client's consciousness level can be monitored by equipment.

C. Client is relaxed, emotionally comfortable, and conscious. Feedback: Conscious sedation refers to a state in which the client is sedated in a state of relaxation and emotional comfort, but is not unconscious. The client is free of pain, fear, and anxiety and can tolerate unpleasant diagnostic and short therapeutic surgical procedures, such as an endoscopy or bone marrow aspiration. The client can respond verbally and physically. However, no equipment can replace a nurse's careful observations for monitoring clients.

A nurse is caring for a client who is scheduled to undergo a breast biopsy. Which of the following major tasks does the nurse perform immediately during the pre-operative period? A. Obtain a signature on the consent form. B. Review the surgical checklist. C. Conduct a nursing assessment. D. Reduce the dosage of toxic drugs

C. Conduct a nursing assessment. Feedback: During the immediate pre-operative period, the nurse conducts a nursing assessment. Nurses obtain the signature of the client, nearest blood relative, or someone with durable power of attorney before the administration of any pre-operative sedatives. They also administer medications as ordered by the physician regardless of their toxicity. They assist the client with psychosocial preparation and complete the surgical checklist, which is reviewed by the operating room personnel.

A preoperative assessment finds a client to be 75 pounds overweight. The client is to have abdominal surgery. What nursing diagnosis would be appropriate based on the client's weight? A. Risk for Aspiration B. Risk for Imbalanced Body Temperature C. Risk for Infection D. Risk for Falls

C. Risk for Infection Feedback: Fatty tissue in obese clients has a poor blood supply and, therefore, has less resistance to infections. Postoperative complications of delayed wound healing, wound infection, and disruption of the wound are more common in obese clients.

What can arteriosclerosis (which is common in older adults) cause for kidney function?

Can reduce blood flow and impair renal function

A teenage female is in the community health clinic seeking contraception. The community health nurse demonstrates effective health promotion and prevention with the client when including which of the following in the health education teaching plan?

Contraception plans prior to initiation of sexual activity.

1.When performing an abdominal assessment, what sequence of assessment techniques should the nurse use? Label the steps from A to D, with A being the first step to perform. A. Auscultation B. Palpation C. Percussion D. Inspection

D, A, C, B When performing an abdominal assessment, the nurse should follow the sequence: inspection, auscultation, percussion, and palpation. Percussion and palpation may stimulate peristalsis, so the techniques with the least contact should be done first. PTS:1DIF:ModerateREF:p. 972 KEY: Nursing process: Assessment | Client need: PHSI | Cognitive level: Recall

When administering an enema, list the following steps in the order in which they should be performed. Label the steps from A to F, with A being the first step to perform. A. Document the results of the procedure. B. Assess the patient for cramping. C. Insert the tubing about 3 to 4 inches into the rectum. D. Lubricate the tip of the enema tubing generously. E. Raise the container to the correct height and instill the solution at a slow rate. F. Encourage the patient to hold the solution for 3 to 15 minutes, depending on the type of enema.

D, C, E, B, F, A You must lubricate the tip before inserting the tubing. You would then insert the tubing and begin instilling the solution before assessing for cramping that the instillation might produce. Only after the solution is instilled would you ask the patient to hold the solution. The last action is to document the results of the procedure, after the procedure is finished. PTS:1DIF:DifficultREF:pp. 993-995 KEY: Nursing process: Interventions | Client need: SECE | Cognitive level: Application

A nurse is educating a surgical client on postoperative p.r.n. pain control. Which of the following should be included? A. "We will bring you pain medications; you don't need to ask." B. "Even if you have pain, you may get addicted to the drugs." C. "You won't have much pain so just tough it out." D. "You need to ask for the medication before the pain becomes severe."

D. "You need to ask for the medication before the pain becomes severe."

A client with abdominal incisions experiences excruciating pain when he tries to cough. What should the nurse do to reduce the client's discomfort when coughing? A. Administer prescribed pain medication just before coughing. B. Ask the client to drink plenty of water before coughing. C. Ask the client to lie in a lateral position when coughing. D. Administer prescribed pain medication 30 minutes before deliberately attempting to cough.

D. Administer prescribed pain medication 30 minutes before deliberately attempting to cough. Feedback: Coughing is painful for clients with abdominal or chest incisions. Administering pain medication approximately 30 minutes before coughing, or splinting the incision when coughing, can reduce discomfort. Making the client lie in a lateral position or asking the client to drink plenty of water is not helpful because it will make breathing and coughing even more difficult for the client.

A nurse is assisting a postoperative client with deep-breathing exercises. Which of the following is an accurate step for this procedure? A. Place the client in prone position, with the neck and shoulders supported. B. Ask the client to place the hands over the stomach, so he or she can feel the chest rise as the lungs expand. C. Ask the client to exhale rapidly and completely, and inhale through the nose rapidly and completely. D. Ask the client to hold his or her breath for three to five seconds and mentally count "one, one thousand, two, one thousand" and so forth.

D. Ask the client to hold his or her breath for three to five seconds and mentally count "one, one thousand, two, one thousand" and so forth. Feedback: The nurse should place the client in semi-Fowler's position, with the neck and shoulders supported, and ask the client to place the hands over the rib cage, so he or she can feel the chest rise as the lungs expand. Then, ask the patient to exhale gently and completely, inhale through the nose gently and completely, hold his or her breath for three to five seconds, and mentally count "one, one thousand, two, one thousand" etc., then exhale as completely as possible through the mouth with lips pursed (as if whistling).

A student is assessing a postoperative client who has developed pneumonia. The plan of care includes positioning the client in the Fowler's or semi-Fowler's position. What is the rationale for this position? A. It increases blood flow to the heart. B. The client will be more comfortable and have less pain. C. It facilitates nursing assessments of skin color and temperature. D. It promotes full aeration of the lungs.

D. It promotes full aeration of the lungs. Feedback: Pneumonia may occur in the postoperative client from aspiration, immobilization, depressed cough reflex, infection, increased secretions from anesthesia, or dehydration. Nursing interventions include positioning the client in the Fowler or semi-Fowler position to promote full aeration of the lungs.

After conducting a preoperative health assessment, the nurse documents that the client has physical assessments supporting the medical diagnosis of emphysema. Based on this finding, what postoperative interventions would be included on the plan of care? A. Perform sterile dressing changes each morning. B. Administer pain medications as needed. C. Conduct a head-to-toe assessment each shift. D. Monitor respirations and breath sounds.

D. Monitor respirations and breath sounds.

A cleansing enema is ordered for a client who is scheduled to have colon surgery. What is the rationale for this procedure? A. Surgical clients routinely are given a cleansing enema. B. Cleansing enemas are given before surgery at the client's request. C. There will be less flatus and discomfort postoperatively. D. Peristalsis does not return for 24 to 48 hours after surgery.

D. Peristalsis does not return for 24 to 48 hours after surgery. Feedback: If the client is scheduled for gastrointestinal tract surgery, a cleansing enema is usually ordered. Peristalsis does not return for 24 to 48 hours after the bowel is handled, so preoperative cleansing helps decrease postoperative constipation.

A nurse working in a PACU is responsible for conducting assessments on immediate postoperative clients. What is the purpose of these assessments? A. To determine the length of time to recover from anesthesia B. To use intraoperative data as a basis for comparison C. To focus on cardiovascular data and findings D. To prevent complications from anesthesia and surgery

D. To prevent complications from anesthesia and surgery Feedback: Immediate postoperative care in the PACU involves assessing the postoperative client with emphasis on preventing complications from the surgery.

The mother of a 4-year-old boy reports her son has voiced curiosity about her breasts. She asks the nurse what she should do. What information is best for the nurse to give the parent?

Encourage the parent to determine what the child's specific questions are and answers them briefly

A 50-year-old man has a long history of type 1 diabetes, which is poorly controlled. What does diabetes greatly increase the man's risk of experiencing?

Erectile dysfunction

After teaching a group of students about erectile dysfunction, the instructor determines that the teaching was successful when the students identify which of the following as true?

Erectile dysfunction may be due to testosterone insufficiency.

A 60-year-old female client with a history of osteoarthritis in her hip confides to the nurse that "intercourse hurts my hip." Which of the following would be the best option for the nurse to suggest?

Exploring alternate positions for intercourse

A patient who underwent a bowel resection to correct diverticula suffered irreparable nerve damage. During the case review, the team is determining if incorrect positioning may have contributed to the patient's nerve damage. What surgical position places the patient at highest risk for nerve damage? A) Trendelenburg B) Prone C) Dorsal recumbent D) Lithotomy

Feedback: Shoulder braces must be well padded to prevent irreparable nerve injury, especially when the Trendelenburg position is necessary. The other listed positions are less likely to cause nerve injury.

Diabetes Mellitus:

Glucose in the urine provides nutrients for bacteria to multiply thus making it a risk factor for a UTI

A patient who sustained a spinal cord injury will perform intermittent self-catheterization after discharge. After discharge teaching, which statement by the patient would indicate correct understanding of the procedure? 1) I will need to replace the catheter weekly. 2) I can use clean, rather than sterile, technique at home. 3) I will remember to inflate the catheter balloon after insertion. 4) I will dispose of the catheter after use and get a new one each time.

I can use clean, rather than sterile, technique at home

A menopausal woman is taking hormone replacement therapy. The nurse teaches the client that a warning sign for endometrial cancer that needs to be reported is:

Irregular vaginal bleeding

Protein concentration of urine:

Less than 20mg/dL

High fever can contribute to high or low output?

Low

What do standardized nursing care plans and individualized care plans have in common? They both 1) Reflect critical thinking for a specific patient 2) Are preprinted to apply to needs common to a group of patients 3) Address a patient's individual needs 4) Provide detailed nursing interventions

NS: 4 They both provide detailed nursing interventions, although the individualized care plan is more specific to the patient's needs and reflects critical thinking, whereas standardized plans do not. It is not true of individual nursing care plans that they are preprinted and apply to a group. PTS: 1 DIF: Moderate REF: pp. 87 KEY: Nursing process: Planning | Client need: SECE | Cognitive level: Analysis

Is incontinence normal in the aging adult?

No, it is common but not a normal change

What should a nurse consider when assessing older clients with a sexually transmitted infection (STI)?

Older adults can still be sexually active.

What are the secondary functions of the kidneys?

Produce erythropoiietin, secrete enzyme renin, and activate vitamin D (calcitrol)

Which task can the nurse safely delegate to the nursing assistive personnel? 1) Palpating the bladder of a patient who is unable to void 2) Administering a continuous bladder irrigation 3) Providing indwelling urinary catheter care 4) Obtaining the patients history and physical assessment

Providing indwelling urinary catheter care

A perimenopausal woman informs the nurse that she is having irregular vaginal bleeding. What should the nurse encourage the patient to do?

See her gynecologist as soon as possible.

A nurse is caring for an obese patient who has had surgery. The nurse monitors this patient for what postoperative complication? a. Hunger b. Impaired wound healing c. Hemorrhage d. Gas pains

b. Impaired wound healing Feedback: Fatty tissue is less vascular and, therefore, less resistant to infection and more prone to delayed wound healing.

Why is the female more prone to UTI's?

because their urethra is about 3-4 cm where a male is 20cm, so because it is much shorter, microorganisms residing in the vagina and rectum can reach bladder easier.

The structures that act as a microscopic filter, controlling the excretion and retention of fluids and solutes:

bowman's capsule and the glomerulus

Other than the normal functioning properties of the bladder and urethra, what else must be functioning properly?

brain, spinal cord, and nerves supplying the bladder and urethra be intact.

The school nurse is educating a group of 5th and 6th grade girls on menstruation. One girl asks if there is a name for having her first menstrual period. The nurse replies with which of the following terms? a) Menses b) Menopause c) Menarche d) Follicular phase

c) Menarche Explanation: The first menstrual period, called menarche, is experienced at about 12 years of age. Menstruation is often referred to as a woman's "period" or "menses." Menopause is the cessation of a woman's menstrual activity. In the ovaries, in a typical 28-day cycle, the phase from day 4 to 14 is called the follicular phase.

A nurse is applying a pneumatic compression device to a client. What is the purpose of a pneumatic compression device? a) Enables the client to void b) Reduces swelling and pain c) Promotes circulation of venous blood d) Pumps liquid diet to the client

c) Promotes circulation of venous blood

What is the most common cause in men for obstruction:

enlarged prostate

nighttime loss of bladder control or bedwetting:

enuresis

Monitor I&O at least?

every 8 hours, more often if patient is experiencing problems or in ICU

How often should you empty the collection bag for a catheter:

every 8 hours, when sediment occurs,urine does not drain well, sandy particles build up inside cannot be freed by rolling the tubing between your hands

Physical assessment for urinary elimination includes:

examination of the kidneys, bladder, urethra, and skin surrounding the genitals.

Hypertrophy?

excessive growth of the prostate gland due to benign or cancerous lesions, which interferes with flow of urine from bladder into urethra

Which position should you use for male and female when inserting a catheter:

female- dorsal recumbent men- supine

Which medication will the physician will most likely prescribe to increase urine output in the patient admitted with congestive heart failure? 1) Digoxin 2) Furosemide 3) Lovastatin 4) Atorvastatin

furosemide

A nurse is planning strategies to address factors that affect sexual dysfunction in older adults. Which interventions would be appropriate? Select all that apply.

• Educate older adult clients, intimate partners, and family about the sexual side effects of specific medications. • Assist in attitude and value clarification about substance use, sexuality, and sexual behavior. • Encourage older adult clients to have a thorough physical evaluation by a health care provider. • Provide an open, nonjudgmental response when older clients display a need for warmth, close contact, and companionship.

There should be no____ detected in urine:

glucose, ketones, hemoglobin, bilirubin, nitrite, renal cells.

A nurse reads a client's chart and sees that she may have dyspareunia. Which of the following would be priority assessments?

• History of hormonal imbalance • Client's ability to use vaginal lubricants during sexual act • Physical assessment of internal and external genitalia

What are nurses responsible for during a diagnostic procedure?

preparing the client, assisting with specimen collection, delivering aftercare, and sometimes assisting the physician

A couple is considering vasectomy as a contraception option. However, the husband is nervous about how such a procedure would affect his sexual functioning. Which information should the nurse mention to the man?

The man will still have full erection capacity

Functioning of kidneys in older adults:

The size and functioning of the kidneys begin to decrease at about age 50 and by age 80 only about 2/3 of the functioning nephrons remain

What happens to the bladder volume in older adults?

The volume of the bladder decreases because of a loss of elasticity in the bladder wall. Loss of elasticity and muscle tone also decreases the ability of the bladder to empty completely

During an internal vaginal examination, the nurse practitioner notes a frothy and malodorous discharge. The nurse suspects the odor is caused by which bacteria?

Trichomonas

ANS: C When a pressure ulcer has been staged and is beginning to heal, the ulcer keeps the same stage and is labeled with the words "healing stage." Once an ulcer has been staged, the stage endures even as the ulcer heals. This ulcer was labeled a stage III, it cannot return to a previous stage such as stage I or II. This ulcer is healing, so it is no longer labeled a stage III.

The wound care nurse visits a patient in the long-term care unit. The nurse is monitoring a patient with a stage III pressure ulcer. The wound seems to be healing, and healthy tissue is observed. How would the nurse stage this ulcer? a. Stage I pressure ulcer b. Healing stage II pressure ulcer c. Healing stage III pressure ulcer d. Stage III pressure ulcer

The student nurse asks the provider if she will prescribe an indwelling urinary catheter for a hospitalized patient who is incontinent. The provider explains that catheters should be utilized only when absolutely necessary because: 1) They are the leading cause of nosocomial infection. 2) They are too expensive for routine use. 3) They contain latex, increasing the risk for allergies. 4) Insertion is painful for most patients.

They are the leading cause of nosocomial infection

Which of the following clients most likely requires special pre‐operative assessment and treatment as a result of his or her existing medication regimen? a) A woman who takes daily anticoagulants to treat atrial fibrillation b) A woman who takes daily thyroid supplements to treat her longstanding hypothyroidism c) A man who regularly treats his rheumatoid arthritis with over ‐the ‐counter nonsteroidal anti‐inflammatory drugs (NSAIDs) d) A man who takes an angiotensin‐converting enzyme (ACE) inhibitor because he has hypertension

a) A woman who takes daily anticoagulants to treat atrial fibrillation Feedback: Anticoagulants present a risk of hemorrhage. This risk supersedes that posed by thyroid supplements, ACE inhibitors, or most NSAIDs.

A client tells a nurse that her menstrual cycle has become irregular. Which of the following does the nurse consider as a cause of the irregularity? Select all that apply. a) Anorexia nervosa b) Oral contraception taken every day year round c) Primary ovarian insufficiency d) Infection of the reproductive organs e) Diabetes

a) Anorexia nervosa b) Oral contraception taken every day year round c) Primary ovarian insufficiency d) Infection of the reproductive organs Explanation: Menstrual cycle irregularities can have many different causes, including pregnancy or breast feeding, eating disorders (anorexia nervosa), polycystic ovary syndrome, premature ovarian failure (also known as primary ovarian insufficiency), pelvic inflammatory disease (infection of the reproductive organs), and, uterine fibroids. Diabetes does not cause irregular menstrual periods.

The nurse is preparing a patient for a colonoscopy. The nurse is familiar with the colonoscopy procedures at the hospital and is aware that which type of anesthesia is commonly used for this procedure? a) Conscious sedation b) Spinal anesthesia c) Nerve block d) Epidural anesthesia

a) Conscious sedation Feedback: Moderate sedation/analgesia is also known as conscious sedation or procedural sedation. It is used for short‐term and minimally invasive procedures such as endoscopy procedures (e.g., colonoscopy).

A pediatric nurse is preparing a child for cleft palate repair surgery. The nurse recognizes that this type of surgery is categorized as which of the following? a) Constructive surgery b) Palliative surgery c) Transplantation surgery d) Reconstructive surgery

a) Constructive surgery Feedback: Cleft palate repair is considered constructive surgery because the goal is to restore function in congenital anomalies. Reconstructive surgery serves to restore function to traumatized or malfunctioning tissues and includes plastic surgery or skin grafting. Transplant surgery replaces organs or structures that are diseased or malfunctioning, such as a liver or kidney transplant. Palliative surgery is not curative and seeks to relieve or reduce the intensity of an illness, such as debridement of necrotic tissue.

The nurse is advising an adolescent male about sexual myths that have him concerned. Which statements describe accurate client education regarding these concerns? Select all that apply. a) No male or female should feel pressured into sexual activity at any age. b) A larger penis allows for a more satisfying sexual experience. c) Nocturnal emissions are normal in men of all ages. d) Masturbation or self-stimulation is a natural and healthy outlet for sexual urges. e) Nocturnal emissions are signs of a sexually transmitted infection. f) Nocturnal emissions indicate the existence of a sexual disorder.

a) No male or female should feel pressured into sexual activity at any age. c) Nocturnal emissions are normal in men of all ages. d) Masturbation or self-stimulation is a natural and healthy outlet for sexual urges. Explanation: Masturbation or self-stimulation is natural. No one should ever feel pressured into having sex. Nocturnal emissions are normal and occur in about 85% of men. Penis size has no effect on the satisfaction of the sexual experience

The nurse recognizes the value of leg exercises in the prevention of postoperative thrombophlebitis. When should the nurse teach the correct technique for leg exercises to a client a) Prior to surgery b) When early signs of venous stasis are evident c) In postanesthetic recovery d) Upon transfer from postanesthetic care unit to the postsurgical unit

a) Prior to surgery Feedback: Though leg exercises are begun after surgery, such preventative measures should ideally be taught to the patient during the preoperative period.

The nurse is teaching a class about sexuality at a public health clinic. After class the nurse is approached by Cathleen, a 54-year-old female, who eventually discloses to the nurse that she is having a sexual affair with a female partner. The term for an individual's preference for a partner of a particular gender is which of the following? a) Sexual orientation b) Transvestite c) Transsexual d) Gender role behavior

a) Sexual orientation Explanation: Sexual orientation refers to the preferred gender of the partner of an individual. A transvestite is an individual who desires to take on the role or wear the clothes of the opposite sex. Gender role behavior is the behavior a person conveys about being male or female. A transsexual is a person of a certain biologic gender who has the feelings of the opposite sex.

A female client tells the nurse that she and her husband have selected periodic abstinence as their choice of contraception. Which of the following is the best method for the nurse to recommend to predict ovulation? a) Symptothermal method b) Calendar method c) Temperature method d) Cervical mucus method

a) Symptothermal method Explanation: The best approach to monitoring fertility is a combination of all three methods (temperature, cervical mucus, and calendar), called the symptothermal method.

A nurse from the ambulatory surgical center is preparing discharge instructions for a client who has had pelvic surgery. Which criterion would the client need to demonstrate to ensure that she is ready for discharge? a) Void normally b) Exhibit no bleeding c) Eat without nausea d) Verbalize absence of pain.

a) Void normally Feedback: Before discharge from an ambulatory surgical unit, the client should be able to void normally after a pelvic surgery. It is natural for the client to experience pain after surgery; however, the client should also have the comfort level to control it. The client may not be in a position to eliminate nausea and vomiting completely before discharge, but should suffer minimally from them. The client may have some bleeding or drainage, which should not be excessive at the time of discharge from an ambulatory surgical unit.

A scrub nurse is assisting a surgeon with a kidney transplant. What are the patient responsibilities of this surgical team member? Select all that apply. a. Maintaining sterile technique b. Draping and handling instruments and supplies c. Identifying and assessing the patient on admission d. Integrating case management e. Preparing the skin at the surgical site f. Providing exposure of the operative area

a. Maintaining sterile technique b. Draping and handling instruments and supplies Feedback: The scrub nurse is a member of the sterile team who maintains sterile technique while draping and handling instruments and supplies. Two duties of the circulating nurse are to identify and assess the patient on admission to the operating room and prepare the skin at the surgical site. The RNFA actively assists the surgeon by providing exposure of the operative area. The APRN coordinates care activities, collaborates with physicians and nurses in all phases of perioperative and post anesthesia care, and integrates case management, critical paths, and research into care of the surgical patient.

A patient had a surgical procedure that necessitated a thoracic incision. The nurse anticipates that he will have a higher risk for postoperative complications involving which body system? a. Respiratory system b. Circulatory system c. Digestive system d. Nervous system

a. Respiratory system Feedback: A thoracic incision makes it more painful for the patient to take deep breaths or cough. Shallow respirations and ineffective coughing increase the risk for respiratory complications.

Urinary output fluctuates depending on factors such as:

ability of heart to circulate the blood, adequate kidney functioning, ability of the patient to void the urine

Having a systemic infection:

accompanied by a high fever causes the kidneys to reabsorb and retain water

By promoting normal urine production....

adequate urine production flushes pathogens out of the bladder, provides natural irrigation of the tubing, and prevents stasis of urine.

Medications that are nephrotoxic:

antibiotics- gentamicin and amphotercin and high doses or long term doses of aspirin and ibuprofen

Anticholinergic effects (medication):

antidiuretics- inhibit the free flow of urine, causing urinary retention

Which medications are antidiuretics (urinary retention)?

antihistamines, anticholinergics, tricyclic antidepressents

When the kidneys are not producing urine:

anuria

A nurse practitioner is teaching the student about sexual health and is evaluating the student's knowledge of sexual identity. The practitioner knows that the student understands when the student says which of the following? a) "Biologic sex is the inner sense a person has of being male or female." b) "Gender role behavior is the behavior a person conveys about being male or female." c) "Gender identity refers to the preferred gender of a person's partner." d) "Sexual orientation is the term used to denote chromosomal sexual development."

b) "Gender role behavior is the behavior a person conveys about being male or female." Explanation: Sexual identity encompasses a person's self-identity, biologic sex, gender identity, gender role behavior or orientation, and sexual orientation or preference. Gender role behavior is the behavior a person conveys about being male or female. Biologic sex is the term used to denote chromosomal sexual development. Gender identity is the inner sense a person has of being male or female. Sexual orientation refers to the preferred gender of a person's partner.

A gerontology nurse is discussing sexual dysfunction with an older adult male client. The nurse determines that the education has been effective when the client states: a) "I need to be able to walk a mile in order for sexual intercourse to not be dangerous." b) "Malnutrition may cause impotence." c) "Erectile dysfunction, caused by diabetes, has no treatment." d) "Antihypertensive medication will not affect sexual function."

b) "Malnutrition may cause impotence." Explanation: Factors that contribute to sexual dysfunction in the older adult are as follows: use of medications, especially antihypertensive medications; age-related metabolic disorders such as anemia, diabetes, malnutrition, and fatigue (may cause impotence). Sexual intercourse and similar forms of sexual expression are not considered dangerous for anyone able to walk around a room. Erectile dysfunction has treatments such as a penile prosthesis or pharmacologic management.

The nurse has entered the room of a client who is postoperative day one and finds the client grimacing and guarding her incision. The client refuses the nurse's offer of PRN analgesia, and on discussion, states that this refusal is motivated by his fear of becoming addicted to pain medications. How should the nurse respond to the client's concerns? a) "The hospital has excellent resources for dealing with any addiction that might result from the medications you take to control your pain." b) "Research has shown that there is very little risk of clients becoming addicted to painkillers after they have surgery." c) "Actually, people who are not addicted to drugs before their surgery never develop a tolerance or addiction during their recovery." d) "You should remind yourself that treating your pain is important now, and that dealing with any resulting dependency can come later."

b) "Research has shown that there is very little risk of clients becoming addicted to painkillers after they have surgery." Feedback: There is little danger of addiction to pain medications used in the postoperative management of pain.

A female client has recently had surgery for cervical cancer and asks the nurse about other ways she can have sex with her husband. Which of the following statements by the nurse is most accurate? a) "Complete abstinence would satisfy your sexual needs." b) "Stimulation of the genitals by the mouth and tongue may be used to reach orgasm." c) "Fellatio is stimulation of the female genitals." d) "Anal intercourse is not practiced by heterosexuals."

b) "Stimulation of the genitals by the mouth and tongue may be used to reach orgasm." Explanation: Stimulation of the genitals by the mouth and tongue may be used as a way to reach orgasm. Abstinence is not having sex and would not meet the couple's needs. Anal intercourse, commonly practiced by gay men, is also used by heterosexual couples. Fellatio is stimulation of the male genitals; cunnilingus is stimulation of the female genitals.

Upon admission for an appendectomy, the client provides the nurse with a document that specifies instructions his health care team should follow in the event he is unable to communicate these wishes postoperatively. What is the document best known as? a) An informed consent b) An advance directive c) An insurance card d) A Patient's Bill of Rights

b) An advance directive Feedback: An advance directive, a legal document, allows the client to specify instructions for his or her health care treatment should he or she be unable to communicate these wishes postoperatively. The advance directive allows the client to discuss his or her wishes with the family members in advance of the surgery. Two common forms of advance directives include living wills and durable powers of attorney for health care.

A woman age 49 years has sought care from her primary care provider because of "intimacy problems." Upon questioning, the woman reveals that she is experiencing sexual desire, but that intercourse causes her significant pain. In the absence of sexual activity, the woman states that she does not have any significant vaginal discomfort. What would the clinician recognize that this client is most likely experiencing? a) A sexually transmitted infection (STI) b) Dyspareunia c) Vaginismus d) Vulvodynia

b) Dyspareunia Explanation: Dyspareunia is painful intercourse. Vaginismus is characterized by difficult penetration rather than acute pain during intercourse, and vulvodynia is associated with pain that is not limited to intercourse. An STI may or may not be contributing to the woman's problem, though most cases of dyspareunia are unrelated to infections.

A client has been taking aspirin since his heart attack in 1997. The client is at risk for what? a) Infection b) Hemorrhage c) Thrombophlebitis d) Blood clots

b) Hemorrhage Feedback: Current medication use, especially use of medications that can affect coagulation status (warfarin, nonsteroidal anti‐inflammatory drugs, aspirin) is important and should be reported to the surgeon.

The forensic nurse is caring for a client who was raped four days ago. Which of the following could the nurse teach the client is an effective contraceptive? a) Depo-Provera injection b) Insertion of a copper intrauterine device (IUD) c) Transdermal contraceptive patch d) Use of the Norplant system

b) Insertion of a copper intrauterine device (IUD) Explanation: Emergency contraception is provided in two ways: increased doses of oral contractptive pills (usually within 72 hours) or insertion of a copper IUD within 5-7 days after unprotected intercourse. The other options are used for routine contraception

Risks associated with urinary diversions:

primarily infection and permanent kidney damage, which can occur from hydronephrosis (distention of the kidneys with urine, resulting from obstruction of the ureter)

What color may urine be after surgery or invasive procedure:

red or pink tinged is normal

Kidney stones, or tumors, which obstruct the normal flow or urine:

renal calculi

The nurse is preparing a talk on health issues in the LGBT population. Which statistics would the nurse include? a) Lesbians are more likely to get preventive services for cancer. b) LGBT youth are 2 to 3 times more likely to attempt suicide. c) LGBT populations have lower rates of tobacco, alcohol, and other drug use. d) Lesbians and bisexual females are more likely to be underweight or anorexic.

b) LGBT youth are 2 to 3 times more likely to attempt suicide. Explanation: Healthy People 2020 found that LGBT youth are 2 to 3 times more likely to attempt suicide. Lesbians and bisexual females are more likely to be overweight or obese. LGBT populations have higher rates of tobacco, alcohol, and other drug use than other populations. Lesbians are more likely to get preventive services for cancer

A client states he has a latex allergy. What action should the nurse take? a) Inform the client to tell the anesthesiologist. b) Place an allergy identification band on the client. c) Have the client take a Benadryl before surgery. d) Send the client to the OR with epinephrine.

b) Place an allergy identification band on the client. Feedback: Assist the client with allergies to medications, food, and latex before the surgical procedure, and clearly mark them on the client record, and on the client identification band.

In which of these situations does the nursing action NOT advocate for patient sexual needs? a) The nurse refers to the patient as Mr., Mrs., Miss, or Ms., according to the patient's preference. b) The nurse ensures the patient wears a hospital gown to protect personal clothing. c) The nurse anticipates potentially shaming situations for the patient. d) The nurse interfaces with the physician to obtain information for the patient

b) The nurse ensures the patient wears a hospital gown to protect personal clothing. Explanation: Patient's should be given the option to wear their own sleepwear to promote sexual identity rather than hospital gowns. The nurse can anticipate potentially shaming situations by informing the patient about embarrassing procedures and that is not unusual for patients to be embarrassed in these situations. The nurse can be the patient's spokesperson when obtaining information from the physician. Referring to patients by their title shows respect to the patient.

What is the rationale for having the client void before surgery? a) To assess for pregnancy in women b) To prevent bladder distention c) To prevent electrolyte imbalance d) To assess for urinary tract infection

b) To prevent bladder distention Feedback: Having the client void before surgery will assist in the prevention of bladder distention during or after the procedure.

A male client comes to the clinic because he thinks that he has syphilis. Which test would the nurse expect the client to have done? a) HPV b) VDRL c) Pap smear d) Wet preparation

b) VDRL Explanation: VDRL is a blood test done to detect syphilis in males and females. HPV is a test done to detect the human papilloma virus. Pap smear is performed on females to evaluate for changes in the cervix. A wet preparation is a sample of vaginal secretions done to detect Candida, Gardnerella and Trichomona

The nurse is assessing the sexual health of a female client age 20 years. During the interview, the client says, "I feel that my vaginal opening constricts whenever I am about to have intercourse. I seem to have no control over it." What would the nurse use when documenting the client's report? a) Orgasmic dysfunction b) Vaginismus c) Celibacy d) Dyspareunia

b) Vaginismus Explanation: The client is experiencing an involuntary contraction of the muscles surrounding the vaginal orifice; this should be documented as vaginismus. Dyspareunia is painful intercourse. Difficulty achieving orgasm is documented as orgasmic dysfunction. Abstention from sexual intercourse is documented as celibacy.

The nurse is providing education about sexuality to a group of college freshman. One of the terms he discusses is gender identity. Which of the following describes gender identity? a) behavior conveyed about being male or female b) inner sense of being male or female c) desire to wear the clothes of the opposite sex d) preferred gender of the partner of an individual

b) inner sense of being male or female Explanation: Gender identity is the inner sense a person has of being male or female. Gender role behavior is the behavior a person conveys about being male or female. Sexual orientation refers to the preferred gender of the partner of an individual. A transvestite is an individual who desires to take on the role (or wear the clothes) of the opposite sex

A 70-year-old male is scheduled for surgery. He says to the nurse, "I am so frightened—what if I don't wake up?" What would be the nurse's best response? a. "You have a wonderful doctor." b. "Let's talk about how you are feeling." c. "Everyone wakes up from surgery!" d. "Don't worry, you will be just fine."

b. "Let's talk about how you are feeling." Feedback: This answer allows the patient to talk about his feelings and fears, and is therapeutic.

A nurse has been asked to witness a patient signature on an informed consent form for surgery. For which of these patients would the document be valid? Select all that apply. a. A 92-year-old patient who is severely confused b. A 45-year-old patient who is oriented and alert c. A 10-year-old patient who is oriented and alert d. A 36-year-old patient who has had a narcotic premedication e. A 45-year-old mentally ill patient who has been ruled incompetent f. A 22-year old patient having an abortion against her partner's wishes

b. A 45-year-old patient who is oriented and alert f. A 22-year old patient having an abortion against her partner's wishes Feedback: A consent form is not legal if the patient signing the form is confused, sedated, unconscious, or a minor.

A 16-year-old girl tells the nurse that her friend has genital warts and asks the nurse how to make sure that she does not get them. Which of the following should the nurse recommend? a) "Have the Norplant system implanted in your arm." b) "Douching will help to prevent them." c) "Get the human papillomavirus vaccine." d) "If your male partner doesn't have genital warts, you will not get them."

c) "Get the human papillomavirus vaccine." Explanation: Human papillomavirus (HPV) is a DNA virus also called genital warts and is a sexually transmitted infection (STI). A vaccine is now available to prevent this disease. Male partners with HPV may or may not have lesions. Women should avoid douching to prevent STIs, as they remove normal protective bacteria in the vagina and increases the risk of getting some STIs. The Norplant system is a contracteptive method and does not prevent STIs.

A client has arrived in the same‐day surgery suite. He states to the nurse, "I am so worried about being put to sleep and having the surgery." What would be the nurse's best response? a) "Have you ever had surgery before?" b) "I will have the anesthesiologist talk to you." c) "Tell me what you are most worried about." d) "You don't have to worry. It will be fine."

c) "Tell me what you are most worried about." Feedback: The nurse should first assess what the client is most worried about, and then provide emotional support.

The nurse is teaching a male adolescent how to use a condom. Which teaching points would the nurse include? (Select all that apply.) a) "Roll the condom onto the penis before it becomes erect." b) "Do not use a spermicide with the condom." c) "Use a condom with every act of intercourse." d) "If using a lambskin condom, the condom may be reused a second time." e) "If the condom does not have a nipple receptacle, leave a small space at the end for semen to collect." f) "Immediately after ejaculation remove the condom and discard it."

c) "Use a condom with every act of intercourse." e) "If the condom does not have a nipple receptacle, leave a small space at the end for semen to collect." f) "Immediately after ejaculation remove the condom and discard it." Explanation: If the condom does not have a nipple receptacle end, a small space should be left at the end of condom to collect the semen in order to provide a pocket to collect semen and prevent breakage. A condom should be used with every act of intercourse to help prevent STI transmission and pregnancy. Immediately after ejaculation, the penis and condom should be removed from the vagina by holding onto the base of the condom. This prevents spillage of semen into the vagina. The condom should be rolled onto the penis as soon as it becomes erect; this protects against sperm from secretions from Cowper's glands. Spermicide can be used with condoms to increase contraception properties. Condoms can never be used a second time

A nurse is preparing to receive a client in post‐anesthesia care unit (PACU). The client is diabetic and has undergone knee surgery. Which information would be most important for the receiving nurse to obtain to develop an appropriate plan of care for this client? a) Chronic disease history b) Environment of the operating room c) Amount of blood loss d) Information about allergic agents

c) Amount of blood loss Feedback: To plan care effectively in the postoperative period, the nurse needs to know about the amount of blood lost during the surgery, the type of surgery that was performed on the client, and whether there were any surgical or anesthetic complications. Information on chronic disease history and allergy history are done in the pre‐operative period, not in the postoperative period. Information on the environment in the operation room is checked by the circulatory nurse during the intra‐ operative care plan; it is not associated with the postoperative care plan.

The nurse is justified in assessing for sexual dysfunction among male clients who are receiving which of the following? a) Non-steroidal anti-inflammatory drugs (NSAIDs) b) Bronchodilators c) Antihypertensive medication d) Antibiotics

c) Antihypertensive medication Explanation: Antihypertensives are among the drugs implicated in sexual dysfunction. Antibiotics, bronchodilators, and NSAIDs do not typically have this effect.

The nurse recommends a barrier method of contraception for a client who is concerned about the side effects of hormonal contraception. Which method might the nurse suggest? a) Abstinence b) Sterilization c) Cervical cap d) Norplant system

c) Cervical cap Explanation: Barrier methods of birth control help in preventing pregnancy by using a device to block sperm from fertilizing an egg. Barrier methods include the cervical cap, condom, diaphragm, and vaginal sponge used in combination with a spermicidal agent. Abstinence means having no sexual intercourse. Norplant is a hormonal method of birth control. Sterilization is a surgical method of birth control.

A nurse is teaching a female client about the sexual response cycle. The nurse determines that the education was successful when the client identifies which female structure as comparable to the male penis? a) Skene glands b) Mons pubis c) Clitoris d) Ovaries

c) Clitoris Explanation: The clitoris corresponds to the penis in the male in that both organs contain erectile tissue and respond to stimulation that can result in orgasm. Skene or paraurethral glands lie inside of, and on, the posterior of the urethra. The Skene and the Bartholin glands produce a small amount of lubricant. The mons pubis is a pad of fatty tissue over the bony prominence called the symphysis pubis. The ovaries contain the female sex cells and female hormones.

A nurse is reinforcing wound edges and applying a blinder to the separated incisions of a client after a surgery. Which of the following postoperative complications has the client developed? a) Hypoxemia b) Evisceration c) Dehiscence d) Shock

c) Dehiscence Feedback: The nurse is taking care of a client with dehiscence. Hypoxemia develops when there is inadequate oxygenation of blood. Evisceration occurs when there is protrusion of abdominal organs through a separated wound. A client has shock when there is inadequate blood flow.

What is associated with the resolution phase of the male sexual response cycle? a) The ability to begin the excitement phase again b) Increased blood flow to the penis c) Feelings of relaxation and fulfillment d) Intense physical pleasure

c) Feelings of relaxation and fulfillment Explanation: The resolution phase of the sexual response cycle is associated with feelings of relaxation, fatigue, and fulfillment. The most intense pleasure of a physical nature takes place during orgasm, and the male experiences a refractory period during which he is incapable of sexual response. Blood flow to the penis returns to normal levels during the resolution phase.

A school nurse is providing a class on sexually transmitted infections (STIs). Which statement is correct regarding STIs? a) STIs disproportionately affect people with a lower socioeconomic status and education. b) The incidence of STIs is decreasing due to limited sex partners. c) STIs are most prevalent among teenagers and young adults. d) The signs and symptoms of an STI are obvious.

c) STIs are most prevalent among teenagers and young adults. Explanation: STIs are most prevalent among teenagers and young adults, and nearly two- thirds of all STIs occur in people younger than 25 years of age. The incidence of STIs is increasing due to multiple sex partners and sexual activity at a younger age. STIs affect men and women of all backgrounds and economic levels

The community health nurse is aware that many factors influence a client's decision for contraception. Which of the following is the most likely reason a client would select permanent contraception? a) To reduce the risk of sexually transmitted diseases. b) The desire to have multiple sex partners. c) The choice not to start a family. d) Religious beliefs which prevent reproduction.

c) The choice not to start a family. Explanation: Some people choose a permanent method to prevent pregnancy from ever occurring. Factors that affect a person's choice of a contraceptive method include age, marital status, desire for future pregnancy, religious beliefs, level of education, cost, and ease of use. Permanent contraception offers no protection against sexually transmitted infections.

Which sexually transmitted infection has the following characteristics: thin, foamy, greenish vaginal discharge that causes itching of the vulva and vagina? a) Herpes simplex 1 b) Nonspecific vaginitis c) Trichomoniasis d) Herpes simplex 2

c) Trichomoniasis Explanation: Trichomoniasis is characterized by thin, foamy, greenish vaginal discharge that causes itching of the vulva and vagina; it is often asymptomatic in males. Nonspecific vaginitis is characterized by foul-smelling, thin, grayish-white vaginal discharge. Herpes simplex virus type 1 and 2 are characterized by lesions

A nurse is caring for an older adult client who has been prescribed fluid restriction before surgery. Which of the following should the nurse check to assess the risks of fluid restriction in elderly clients? a) Anxiety level b) Self‐therapy c) Vital signs d) Cardiac status

c) Vital signs Feedback: The nurse should assess the client's vital signs, weight, and sternal skin turgor prior to fluid restriction to serve as a baseline for comparison. The period of fluid restriction before surgery may be shortened for older adults to reduce their risk of dehydration and hypotension. Nurses check a client's self‐therapy practices and cardiac status to avoid any complications of bleeding and elimination of intravenous fluids given at a standard rate.

When a man cannot achieve an erection, the phase of the sexual response in which the man is experiencing difficulty is: a) orgasmic phase. b) obligatory phase. c) excitement phase. d) refractory phase.

c) excitement phase. Explanation: The excitement phase is characterized by rapid erection of the penis with tensing and thickening of the scrotal skin and elevation of the scrotal sac.

A perioperative nurse is preparing a patient for surgery for treatment of a ruptured spleen as the result of an automobile crash. The nurse knows that this type of surgery belongs in what category? a. Minor, diagnostic b. Minor, elective c. Major, emergency d. Major, palliative

c. Major, emergency Feedback: This surgery would involve a major body organ, has the potential for postoperative complications, requires hospitalization, and must be done immediately to save the patient's life. Elective surgery is a procedure that is preplanned by essentially healthy people. Diagnostic surgery is performed to confirm a diagnosis. Palliative surgery is not curative, rather it is done to relieve or reduce the intensity of an illness.

A nurse is preparing a patient for a cesarean section and teaches her the effects of the regional anesthesia she will be receiving. Which effects would the nurse expect? Select all that apply. a. Loss of consciousness b. Relaxation of skeletal muscles c. Reduction or loss of reflex action d. Localized loss of sensation e. Prolonged pain relief after other anesthesia wears off f. Infiltrates the underlying tissues in an operative area

c. Reduction or loss of reflex action d. Localized loss of sensation Feedback: A localized loss of sensation and possible loss of reflexes occurs with a regional anesthetic. Loss of consciousness and relaxation of skeletal muscles occurs with general anesthesia. Prolonged pain relief after other anesthesia wears off and infiltration of the underlying tissues in an operative area occur with topical anesthesia.

What can anesthetic agents do to urine production?

can decrease blood pressure and glomerular filtration thus decreasing urine formation

What is urinary incontinenct most often related to in women?

childbirth, vaginal delivery, periomenopausal status, high BMI, diabetes, cigs

During a routine physical exam, a male client informs the nurse that he frequently participates in anal intercourse with his new girlfriend. The nurse discusses this practice with the client by informing him that

condoms are recommended for anal intercourse.

Which example best supports the diagnosis of Sexual Dysfunction: Dyspareunia? a) A 39-year-old alcoholic woman is no longer interested in having sex with her partner. b) A 50-year-old woman with a history of stroke is afraid to have sex with her partner for fear it will elevate her blood pressure. c) A client with a colostomy believes she cannot have a sexual relationship with her husband because he will be repulsed by her stoma. d) A 50-year-old woman in the process of menopause has pain and burning during intercourse.

d) A 50-year-old woman in the process of menopause has pain and burning during intercourse. Explanation: Dyspareunia refers to pain and burning during intercourse. This is a common cause of sexual dysfunction, especially during menopause. A colostomy, fear of blood pressure elevation, and lack of interest in sex may lead to the nursing diagnosis of Sexual Dysfunction, but not related to dyspareunia.

The operating room nurse is aware that which of the following clients is at a greater risk related to a surgical procedure? a) A man 48 years of age b) A boy 8 years of age c) A woman 34 years of age d) A woman 83 years of age

d) A woman 83 years of age Ans: D Infants and older adults are at a greater risk from surgery than are children and young or middle‐age adults. Physiologic changes associated with aging increase the surgical risk for older clients.

A nurse is educating a male client about testicular self-examination. The nurse would encourage the client to perform the exam at which frequency? a) Once a week b) Once every 3 months c) Once every 6 months d) Once a month

d) Once a month Explanation: Testicular self-examination should be performed once a month.

In the postoperative phase of abdominal surgery, the client complains of severe abdominal pain. In the second postoperative day, the client's bowel sounds are absent. What does the nurse suspect? a) Abdominal infection b) Hernia development c) Normal response d) Paralytic ileus

d) Paralytic ileus Feedback: A potential complication after surgery is paralytic ileus, a condition in which there is decreased bowel functioning.

A nurse teaches deep breathing exercises to a client scheduled for surgery. In which of the following perioperative phases would this action occur? a) None of the above b) Intraoperative c) Postoperative d) Pre‐operative

d) Pre‐operative Feedback: Exercises and physical activities occurring in the pre‐operative phase include deep‐breathing exercises, coughing, incentive spirometry, turning, leg exercises, and pneumatic compression stockings.

The nurse working at a family planning clinic evaluates a client's risk for complications from oral contraceptives. Which of the following places the client at highest risk? a) Drinks 1 glass of wine daily b) Age less than 35 c) History of diabetes d) Smokes 1 pack cigerettes per week

d) Smokes 1 pack cigarettes per week Explanation: Complications for oral contraceptives include an increased incidence of blood clots, heart attacks, and stroke, especially in women who smoke.

The term that is used to describe the female client who states that she's a man trapped in a women's body is: a) homosexual. b) bisexual. c) heterosexual. d) transsexual.

d) transsexual. Explanation: A transsexual woman views herself as a man trapped in woman's body. Some people are heterosexual, attracted sexually to members of the opposite gender. Others are bisexual, attracted to both men and women sexually. Others are homosexual, attracted sexually to members of the same gender.

A nurse is explaining pain control methods to a patient undergoing a bowel resection. The patient is interested in the PCA pump and asks the nurse to explain how it works. What would be the nurse's correct response? a. "The pump allows the patient to be completely free of pain during the postoperative period." b. "The pump allows the patient to take unlimited amounts of medication as needed." c. "The pump allows the patient to choose the type of medication given postoperatively." d. "The pump allows the patient to self-administer limited doses of pain medication."

d. "The pump allows the patient to self-administer limited doses of pain medication." Feedback: PCA infusion pumps allow patients to self-administer doses of pain-relieving medication within physician-prescribed time and dose limits. Patients activate the delivery of the medication by pressing a button on a cord connected to the pump or a button directly on the pump.

A nurse is teaching a man scheduled to have same-day surgery. Which teaching method would be most effective in preoperative teaching for ambulatory surgery? a. Lecture b. Discussion c. Audiovisuals d. Written instructions

d. Written instructions Feedback: Written instructions are most effective in providing information for same-day surgery.

Nephrotoxic?

damaging to the kidneys

Cardiovascular and metabolic disorders:

decrease blood flow through the glomeruli and thus impair filtration and urine production

Impacted stool?

distends the rectum and causes forward pressure on the urethra, interfering with the flow of urine out of the bladder

What position should the patient assume before the nurse inserts an indwelling urinary catheter? 1) Modified Trendelenburg 2) Prone 3) Dorsal recumbent 4) Semi-Fowlers

dorsal recumbent

What sound does a distended bladder produce?

dull sound

discomfort when urinating:

dysuria

blood in the urine:

hematuria

The nurse knows that women may choose periodic abstinence for several reasons. In order to assist the client in her choice to return to sexual activity, while preventing pregnancy, the nurse should make sure to discuss which of the following with the client?

how to chart her fertility pattern

The surgeon orders hourly urine output measurement for a patient after abdominal surgery. The patients urine output has been greater than 60 mL/hour for the past 2 hours. Suddenly the patients urine output drops to almost nothing. What should the nurse do first? 1) Irrigate the catheter with 30 mL of sterile solution. 2) Replace the patients indwelling urinary catheter. 3) Infuse 500 mL of normal saline solution IV over 1 hour. 4) Notify the surgeon immediately.

irrigate the catheter with 30 mL of sterile solution

When the mother of a 2-year-old tells the pediatric nurse that the child masturbates, the nurse informs the mother that the child:

is exhibiting normal behavior.

If intake is high and output is low>

kidneys are not working well OR kidneys are producing urine but he has urinary retention because of something ELSE

Which medication class will the primary care provider most likely prescribe to increase urine output in the patient admitted with congestive heart failure? 1) Thiazide diuretic 2) Loop diuretic 3) MAO inhibitor 4) Anticholinergic

loop diuretic

What is a 24-hour urine collection used for:

may be prescribed to evaluate some renal disorders by showing kidney function at different times of the day and night

What are BUN and creatinine levels used for

measured to assess renal function and hydration

As a nurse you will do what to help with urinary incontinence:

monitor and assist clients with urinary elimination, teach them about bodily functions, work collaboratively with the healthcare team to facilitate normal urinary function

Nervous system conditions:

neurogenic bladder- impaired neurological functions. The person cannot perceive bladder fullness or control the urinary sphincters. Ex:stroke or spinal cord injury

Inflammation and swelling may cause?

obstruction

Etiologies for urinary retention:

obstruction, inflammation, swelling, neurological problems, medications and anxiety

Functional incontinence:

occurs d/t physical disability, immobility, pain, exteral obstacles, or problems of thinking/communicating that prevents a person from reaching a toilet. (confusion/disorientation)

Which action should the nurse take when beginning bladder training using scheduled voiding? 1) Offer the patient a bedpan every 2 hours while she is awake. 2) Increase the voiding interval by 30 to 60 minutes each week. 3) Frequently ask the patient if she has the urge to void. 4) Increase the frequency between voiding even if urine leakage occurs

offer the patient a bedpan every 2 hours while she is awake

when urine output is below normal:

oliguria

While performing a physical assessment, the student nurse tells her instructor that she cannot palpate her patients bladder. Which statement by the instructor is best? You should: 1) Try to palpate it again; it takes practice but you will locate it. 2) Palpate the patients bladder only when it is distended by urine. 3) Document this abnormal finding on the patients chart. 4) Immediately notify the nurse assigned to the care of your patient.

palpate the patients bladder only when it is distended by urine

How to collect urine:

place a specimen hat under the toilet seat to collect urine, use a bed pan or urinal for a client with mobility problems

a kidney infection/inflammation of the kidneys:

polynephritis

Which intervention should the nurse take first to promote micturition in a patient who is having difficulty voiding? 1) Insert an indwelling urinary catheter. 2) Notify the provider immediately. 3) Insert an intermittent, straight catheter. 4) Pour warm water over the patients perineum.

pour warm water over the patients perineum


Related study sets

Andrew Carnegie, J.P. Morgan, and John D. Rockefeller

View Set

Purines and pyrimidines.........

View Set

Which of the following defines business ethics? An approach toward decision making in which a person evaluates moral principles Professional standards of conduct, based on moral principles and values, guiding decision-making and actions Choices made ba

View Set

CISC 310 Ch 1, 2, 3, 7, 8, 10, 12, 13, 14, 15, 16, 17

View Set